Manual: Kyiv National Aviation University NAU-druk Publishing 2009

You might also like

Download as pdf or txt
Download as pdf or txt
You are on page 1of 272

Part 1

Manual

Kyiv
National Aviation University
«NAU-druk» Publishing
2009
UDC 513.123:517.2=111(075.8)
ББК В10я7
Н 65

All rights reserved. No part of this book may be reproduced


in any form without the prior written permission of the publisher

Authors:
V. P. Denisiuk, L. I. Grishina, O. V. Karupu,
T. A. Oleshko, V. V. Pakhnenko, V. K. Repeta

Reviewers:
O. P. Besklinska — candidate of science (physics and mathematics), associate professor
(Kyiv National Linguistic University)
P. T. Kachanov — candidate of science (engineering), associate professor
(State University of Information and Communication Technologies)

English language adviser


O. Yu. Kravchuk, associate professor

Approved by the Methodical


and Editorial Board of the National Aviation University
(Minutes № 8/09 of 17.12.2009)

The manual fully corresponds to the educational program of higher mathe-


matics for engineering universities. It contains basic theoretical information,
examples of problems solution, class, home and self-test assignments.
The manual is intended for the first course students of all specialities.

© Denisiuk V. P., Grishina L. I.,


ISBN 978–966–598–612–6 Karupu O. V. and fo on, 2009
ISBN 978–966–598–614–0 (Part 1) © NAU, 2009
INTRODUCTION

This book offers the modular technology of studying higher mathematics


course by the students of engineering specialties in the first term.
The module represents the logically compiled sections of studied subject.
The material of the first term is divided into three modules:
1. Elements of linear and vector algebra.
2. Elements of analytical geometry.
3. Introduction to mathematical analysis. Derivatives and differentials of a
function of one variable. Applications of derivatives.
Each module contains several micromodules.
Every module begins with the general statements, which represent the
themes of the sections, basic concepts, and main tasks.
Each micromodule covers:
1) basic theoretical information,
2) examples of problem solutions,
3) class and home assignments,
4) self-test assignments.
The theoretical part gives all necessary material for mastering the theme in
questions (lecture summary). The references to the literature are applied to all
the themes, which enable the students to master their theoretical material.
The practical part contains examples of problem solutions, that illustrate the
theoretical material and problems with answers for class and home work.
Each micromodule is provided with self-test assignments which contain
several problems to be done by the students in a written form.

3
Моdule
1 ELEMENTS OF LINEAR
AND VECTOR ALGEBRA

MODULE STRUCTURE
Micromodule 1. Determinants. Determinants of 2nd and 3rd orders.
Properties of determinants. Minors and cofactors. General definition of a
determinant of order n. Calculation of determinants.
Micromodule 2. Matrices. Definition of a matrix, operations with them.
Inverse matrix. Matrix equations. Rank of a matrix and its property.
Micromodule 3. Systems of linear algebraic equations. Investigation of
the behaviour of a linear algebraic equations System. Consistence (compa-
tibility), investigation of consistence by matrix rank. Solution of the systems
by Cramer’s rule, matrix method, Gauss’ method. Indefinite systems and
their solutions. Homogeneous systems of linear algebraic equations. Eigen-
values and eigenvectors of matrix.
Micromodule 4. Vectors. Vectors, linear operations with vectors. Pro-
jection of vector on axis. Linear dependence and independence of vectors.
Basis and coordinate system. Vectors in Cartesian coordinate system.
Division of line segment according to a given ratio.
Micromodule 5. Dot Product. Definition of the dot product, it’s
properties and coordinate form. Two vectors perpendicularity condition.
Micromodule 6. Cross and Triple Products. Cross product, it’s algebraic and
geometrical properties. Coordinate form. Triple product, it’s algebraic and
geometrical properties. Coordinate form. Three vectors complanarity condition.

Basic concepts. 1. Determinant. 2. Matrix. 3. System of Linear Algebraic


Equations (SLAE). 4. Vector. 5. Projection of vector. 6. Basis. 7. Cartesian
Coordinate System. 8. Vector coordinates. 9. Dot product. 10. Cross product.
11. Triple product.

Key word: determinant — визначник, matrix — матриця, minor — мінор,


cofactor — алгебраїчне доповнення, permutation of elements — перестановка
елементів, inverse matrix — обернена матриця, rank — ранг, homogeneous
systems — однорідні системи, vector — вектор, eigenvalues — власні
значення, eigenvectors — власні вектори, linear dependence — лінійна
залежність, linear independence — лінійна незалежність, basis — базис,
expansion of a vector in terms of base vectors — розклад вектора по базисним
векторам, the Cartesian rectangular system of coordinates — прямокутна сис-
тема координат, dot product (scalar product) — скалярний добуток, cross

4
product (vector product) — векторний добуток, triple product (mixed product) —
мішаний добуток, collinear vectors — колінеарні вектори, coplanar vectors —
компланарні вектори, orthogonal vectors — ортогональні вектора

Main tasks. 1. Calculation of determinants. 2. Solution of Linear algebraic


equations system. 3. Matrix equation solution. 4. Investigation of the linear
algebraic equations systems. 5. Operations with vectors. 6. Mutual disposition of
vectors. 7. Dot, cross and triple product usage in solving geometrical problems.

A STUDENT MUST BE READY TO DO


THE FOLLOWING ASSIGNMENTS
1. Concepts, definitions, formulations:
• Determinants of the 2nd, the 3rd and the n-th orders.
• Matrices. Linear operations with matrices. Multiplication of matrices.
• Inverse matrix.
• Minors, rank of matrix.
• Definite, indefinite, consistent, inconsistent SLAE.
• Matrix form of SLAE.
• Gauss’ method of SLAE solution.
• Kronecker-Capellі theorem usage in SLAE investigation.
• Eigenvalues and eigenvectors of matrix.
• Geometrical vector. Vector addition and subtraction operations, multipli-
cation by scalar.
• Linear dependence and independence of vectors.
• Basis on a plane and in space.
• Cartesian coordinate system (CCS).
• Dot product of two vectors.
• Cross product of two vectors.
• Triple product.
2. Proofs and conclusions
• Properties of determinants (2nd and 3rd orders).
• Matrix addition and multiplication properties.
• Existence of an inverse matrix.
• Inverse matrix method of SLAE solution.
• Cramer’s Theorem.
• Kronecker-Capellі Theorem.
• Projection of vector on axis.
• Representation of a vector in terms of base vectors.
• Properties of a dot product; calculation by coordinates.
• Properties of a cross product; calculation by coordinates.
• Properties of a triple product; calculation by coordinates.
5
3. Assignments
• Calculate the determinants of order 2, 3 and n, to be able to lay out a
determinant by the elements of any row or column, to reduce determinant to the
triangle form.
• Find the matrix sum, difference and product.
• Find the matrix rank.
• Find an inverse matrix.
• Solve the square systems by Cramer’s method, through inverse matrix.
• Solve the arbitrary SLAE by Gauss’ method.
• Analyse SLAE on the consistence (compatibility) according to Kronecker-
Capellі Theorem.
• Find the eigenvalues and eigenvectors of matrix.
• Find the vector coordinates, it’s length, unit vector. Find the angle
between vectors.
• Find the vector sum, difference, dot and cross products.
• Calculate the area of the triangle, volume of pyramid.
• Be able to represent the vector in terms of base vectors .
• Be able to use the condition of two vectors perpendicularity.

Micromodule 1
BASIC THEORETICAL INFORMATION. DETERMINANTS

Determinants of the 2nd and the 3rd orders. Properties of determinants.


Minor and cofactor. General definition of a determinant of the n-th order.
Calculations of determinants.

Literature: [1, chapter 1], [4, part 2, p.2.3], [6, chapter 1 §1], [7, chapter
2 §7], [10, chapter 1 §2] [11, chapter 1 §1].

1.1. Determinants of the 2nd and the 3rd orders

Definition 1.1. A number


a11 a12
Δ= = a11 a22 − a12 a21
a21 a22
is the determinant of the 2nd order.
Such determinants appear to be true, for example, for the solution of such
linear systems as

⎧a11 x1 + a12 x2 = b1 ,

⎩a21 x1 + a22 x2 = b2 .

6
Let’s find x . We multiply both parts of the first equation by a22 , and of the
2nd equation by ( −a12 ), whereupon, by adding the left-hand and right-hand
sides of the given equations, we get the equality
a22 (a11 x1 + a12 x2 ) − a12 (a21 x1 + a22 x2 ) = a22 b1 − a12 b2 ,
which, after simplifications, we obtain
(a22 a11 − a12 a21 ) x1 = a22 b1 − a12 b2 .

In the case of condition a22 a11 − a12 a21 ≠ 0 , we get


b1 a12
a22 b1 − a12 b2 b2 a22
x1 = = .
a22 a11 − a12 a21 a11 a12
a21 a22
By eliminating variable x1 , we get
a11 b1
a11b2 − a21b1 a21 b2
x2 = = .
a22 a11 − a12 a21 a11 a12
a21 a22
So, the solutions of the given system are expressed through the determinants
of the 2nd order.
Definition 1.2. A number
a11 a12 a13
Δ = a21 a22 a23 = a11 a22 a33 + a12 a23 a31 + a13 a21 a32 −
a31 a32 a33
−a13 a22 a31 − a11 a23 a32 − a12 a21 a33 .
is called the determinant of the 3rd order.
The order of determinant equals to the number of its rows or columns.
The numbers aij are elements of a determinant thus the first index i indicates a
number of row, and the 2nd index j is a number of column, in which the element lies.
Elements a11 , a22 in the determinant of the 2nd order and a11 , a22 , a33 of the
determinant of the 3rd order, all together, form a principal (leading) diagonal of a
determinant and elements a12 , a21 and a13 , a22 , a31 form a secondary diagonal.
For the calculation of determinant of the 2nd order it is necessary to subtract
the product of all elements of a secondary diagonal from the product of elements
of a principal diagonal.

7
The determinant of the 3rd order is calculated by the rule of triangle. First
three multipliers with a sign plus are the product of all elements of a principal
diagonal and in the tops of two triangles in which one side is parallel to the
principal diagonal. Next three multipliers with a sign minus are the product of
elements of a secondary diagonal and in the tops of two triangles in which one
side is parallel to secondary diagonal.
To remember this formula it is advisable to use the scheme represented in
Fig. 1.1.

+ –
Fig. 1.1

Remark. It is important to know the following formula. If a determinant is


upper (lower) triangular form, then this determinant is equal to the product of all
elements of the principal diagonal, for example

a11 a12 a13


Δ= 0 a22 a23 = a11a22 a33 .
0 0 a33

1.2. Principal properties of determinants


We shall formulate the properties for the third-order determinants, although
they are true for determinant of any order.
1. The value of a determinant does not vary if its rows and columns are
interchanged.
a11 a12 a13 a11 a21 a31
a21 a22 a23 = a12 a22 a32 .
a31 a32 a33 a13 a23 a33

Property 1 establishes the equivalence of the rows and columns of a deter-


minant.
2. Interchanging of two rows or of two columns of a determinant is
equivalent to its multiplication by –1.

8
For example,

a11 a12 a13 a31 a32 a33


a21 a22 a23 = − a21 a22 a23 .
a31 a32 a33 a11 a12 a13

3. If all elements of a row (or a column) of a determinant are zero, then the
determinant itself is zero.
4. If a determinant has two identical rows (or columns), then it is equal to zero.
5. The product of all elements of one column or one row of a determinant by
any number k is equivalent to the multiplication of the determinant by this number.
For example,

a11 ka12 a13 a11 a12 a13


a21 ka22 a23 = k a21 a22 a23 .
a31 ka32 a33 a31 a32 a33

6. If the elements of two rows (or columns) of a determinant are proportional


then the determinant is zero.
7. If every element of the i-th row (or j-th column) of a determinant is the
sum of two terms, then the determinant can be represented as the sum of two
determinants.
For example,

′ + a11
a11 ′′ ′ + a12
a12 ′′ ′ + a13
a13 ′′ ′
a11 ′
a12 ′
a13 ′′
a11 ′′
a12 ′′
a13
a21 a22 a23 = a21 a22 a23 + a21 a22 a23 .
a31 a32 a33 a31 a32 a33 a31 a32 a33

8. If the elements of a row (or a column) of a determinant are summed up


with the elements of another row (column) multiplied by any common factor k,
then the value of the determinant remains unchanged.
For example,
a11 + ka21 a12 + ka22 a13 + ka23
a21 a22 a23 =
a31 a32 a33

a11 a12 a13 ka21 ka22 ka23 a11 a12 a13 a21 a22 a23
= a21 a22 a23 + a21 a22 a23 = a21 a22 a23 + k a21 a22 a23 =
a31 a32 a33 a31 a32 a33 a31 a32 a33 a31 a32 a33

9
a11 a12 a13
= a21 a22 a23 .
a31 a32 a33

All properties of determinants of order 3 are true for the determinants of any
order.

1.3. Expansion of a determinant by the elements


of the row or the column

Let’s consider a determinant of order 3


a11 a12 a13
Δ = a21 a22 a23 .
a31 a32 a33

Definition 1.3. The Minor М ij of an element aij of a determinant of order n


is a determinant of order (n – 1) obtained from the given determinant by striking
out a row i and a column j in which the element lies.
For instance, the minor of the element a23 of the determinant ∆ is the
a11 a12
second-order determinant M 23 = . The minor of the element a31 is the
a31 a32
a12 a13
second-order determinant M 31 = .
a22 a23
Definition 1.4. The Cofactor, or algebraic adjunct, Aij of an element aij of
a determinant is the minor М ij of that element multiplied by (−1)i + j :

i+ j
Aij = ( −1) M ij .

Laplas’ Theorem 1.1. A determinant is equal to the sum of the products


of all elements of a row or of a column by their cofactors.

For example, for the determinant of the 3rd order such equalities are executed:
Δ = a11 A11 + a12 A12 + a13 A13 (expansion by the elements of the first row);
Δ = a12 A12 + a22 A22 + a32 A32 (expansion by the elements of the second
column).

10
Theorem 1. 2. The sum of the products of the elements of a row or of a
column of a determinant by the cofactors of the respective elements of another
row or column is zero.

For example, for the determinant of the third order such equalities are
executed:
a11 A21 + a12 A22 + a13 A23 = 0 , a12 A13 + a22 A23 + a23 A33 = 0 .

Remark. The theorems 1.1. and 1.2. hold true for the determinants of any order.

1.4. Concept of determinants of random order

According to theorem 1.1., determinant of the n-th order equals the sum of
products of the elements of a row (a column) by their cofactors.
For example, there is the expansion by the elements of the first row for a
determinant of order n:

a11 a12 ... a1n


a21 a22 ... a2 n
Δ= = a11 A11 + a12 A12 + ... + a1n A1n .
... ... ... ...
an1 an 2 ... ann

As a result of application of this formula instead of calculation of one


determinant of the n-th order it is necessary to calculate n determinants of order
(n – 1). But such method of calculation is too bulky.
In practice it is convenient when the part of elements of a row (or a column)
is equal to zero (and the more zero elements, the less calculations). For this
reason it is better to transform determinant so that all elements of some row
(column), except one, were zero. It is necessary to apply properties 1—8 to the
determinant (property 8 plays here main role). Then the expansion of a
determinant by elements of this row (column) contains only one factor.
We will give formal determination of determinant of the n-th order now.
Lets there is a permutation j = ( j1 , j2 , ..., jn ) , where j1 , j2 , ..., jn are
numbers 1, 2 ,..., п formed in a definite order.
The pair of elements ( ji , jk ) , i < k forms the inversion, if ji > jk .
Number of all such inversions pairs is called as number of inversions in a
permutation j = ( j1 , j2 , ..., jn ) and designate as inv( j1 , j2 , ..., jn ) .
The permutation is called even, if the number of all inversions is even and
otherwise it is odd. For example inv(3, 2, 1) = 3.

11
Definition 1.5. The determinant of the n-th order
a11 a12 ... a1n
a21 a22 ... a2 n
... ... ... ...
an1 an 2 ... ann
is the algebraic sum of all possible products which contain one element from
every row and every column. The sign of every addend is equal to (−1)t , where
t is a number of inversions in other indexes, if the elements (factors aij ) of the
addend are in ascending order of first indexes. So,

a11 a12 ... a1n


a21 a22 ... a2 n
= ∑ (−1)inv ( j1 , j2 , ..., jn )
a1 j1 a2 j2 ...anjn .
... ... ... ... j
an1 an 2 ... ann
The number of such products is n! Half of them is taken with a sign plus, and
other half is taken with a sign minus.
Remark. The determinant of order n in the triangular form is equal to the
product of all elements of the principal diagonal

a11 a12 ... a1n


0 a22 ... a2 n
Δ= = a11a22 … ann .
... ... ... ...
0 0 ... ann

Micromodule 1
EXAMPLES OF PROBLEMS SOLUTION

Example 1. Calculate determinants:


0 −2 sin α cos α
a) ; b) .
8 7 sin β cos β
Solution.
0 −2
а) = 0 ⋅ 7 − 8 ⋅ (−2) = 16 ;
8 7

12
sin α cos α
b) = sin α cos β − sin β cos α = sin(α − β).
sin β cos β

3 2 −4
Example 2. 3 −1 1 .
1 1 3

Solution. The first method. By the rule of triangles we have

3 2 −4
3 −1 1 = 3 ⋅ (−1) ⋅ 3 + 2 ⋅1 ⋅1 + 3 ⋅1 ⋅ (−4) −
1 1 3
−1 ⋅ (−1) ⋅ (−4) − 2 ⋅ 3 ⋅ 3 − 1 ⋅1 ⋅ 3 = −9 + 2 − 12 − 4 − 18 − 3 = −44.
The second method. By the theorem 1.1 (expansion by the elements of the
first row) we get
3 2 −4
−1 1 3 1
3 −1 1 = 3 ⋅ (−1)1+1 + 2 ⋅ (−1)1+ 2 +
1 3 1 3
1 1 3
3 −1
+(−4) ⋅ (−1)1+3 = 3 ⋅ (−4) − 2 ⋅ 8 − 4 ⋅ 4 = −44.
1 1

The third method. Using properties of a determinant we obtain

3 2 −4 + III (−3) 0 −1 −13


3 −1 1 + III (−3) = 0 −4 −8 = a31 A31 =
1 1 3 1 1 3
−1 −13
= 1 ⋅ (−1)3+1 = 8 − 52 = −44.
−4 −8

1 0 −1 2
1 2 −2 0
Example 3. .
−1 3 0 2
2 1 −1 3
Solution. The determinant has some zero elements, but it is better, when the
zero elements are in one row or a column. For example, let’s make zero all
elements of the first column, except for the first element. For this purpose to a
column 3 we shall add a column 1, and to a column 4 we shall add a column 1,
multiplied by(-2), we get

13
1 0 −1 2
1 2 −2 0
Δ= =
−1 3 0 2
2 1 −1 3
+ I + I (−2)
1 0 −1 + 1 2 − 2 1 0 0 0
1 2 −2 + 1 0 − 2 1 2 −1 −2
= = .
−1 3 0 − 1 2 + 2 −1 3 −1 4
2 1 −1 + 2 3 − 4 2 1 1 −1

Using the theorem 1.1. we obtain

2 −1 −2
1+1
Δ = a11 A11 = 1 ⋅ (−1) M 11 = 3 −1 4 .
1 1 −1

Let’s make two zeros in the second column. We have

2 −1 −2 + III 3 0 −3
Δ = 3 −1 4 + III = 4 0 3 .
1 1 −1 1 1 −1

We expand a determinant by the elements of the second column

3 −3
Δ = a32 A32 = 1 ⋅ (−1)3+ 2 M 32 = − = −(9 + 12) = −21 .
4 3

Answer. Δ = –21.
Example 4. Solve equation

−2 − k 2 0
2 4−k 6 =0.
1 2 3− k

Solution. We expand a determinant by the elements of the first row


−2 − k 2 0
4−k 6 2 6
2 4−k 6 = (−2 − k )(−1)1+1 + 2(−1)1+ 2 +0 =
2 3− k 1 3− k
1 2 3− k

14
= (−2 − k )((4 − k )(3 − k ) − 12) − 2(2(3 − k ) − 6) = −(2 + k )(k 2 − 7 k ) + 4k =
= −(k 3 + 2k 2 − 7 k 2 − 14k ) + 4k = −k 3 + 5k 2 + 18k .
So, the given equation is equivalent to the equation
−k 3 + 5k 2 + 18k = 0 .
We have
−k (k 2 − 5k − 18) = 0 ;

5 ± 97
k = 0 or k 2 − 5k − 18 = 0 , from here k = .
2
5 ± 97
Answer: k = 0; .
2

Micromodule 1
CLASS AND HOME ASSIGMENTS

Calculate determinants:
6 −4 5 0 121 110
1. . 2. . 3. .
9 5 11 −22 132 121
2 4
sin α − cos α log 2 5 − log 9 16 3 5
4. . 5. . 6. .
cos α sin α log8 3 log5 2 1 3
2 2
1 4 −1 −3
1 5 −161
1
2 3
7. . 8. −2 −1 7 . 9. −4 −2 13 .
2 2
4 −5 −10 8 −4 −23
7 3
−1 1 −2 2 1 −3 −1 3
1 −2 −2
−4 3 −1 −1 −1 4 −1 −1
10. −6 3 22 . 11. . 12. .
11 −8 34 16 1 0 27 20
4 −11 −3
10 −8 8 −1 2 −11 10 −3

Answers
1. 66. 2. –110. 3. 121. 4. 1. 5. 5/3. 6. 0,6. 7. 13/21. 8. –5. 9. –10. 10. –15. 11.
11. 12. 12.

15
Micromodule 1
SELF-TEST ASSIGNMENTS

1.1. Calculate determinants, using:


а) method of transformation to a triangular form;
b) expansion of a determinant by the elements of some row (column).
c) rule of triangles.
2 3 4 3 4 5 4 5 6
1.1.1. 4 6 7 . 1.1.2. 6 7 8 . 1.1.3. 7 8 9 .
8 9 10 9 10 11 10 11 12

1 4 1 1 5 1 1 6 1
1.1.4. 2 1 5 . 1.1.5. 3 1 6 . 1.1.6. 4 1 7 .
1 6 −1 1 7 −1 1 8 −1

1 1 1 1 1 1 1 1 1
1.1.7. 3 1 4 . 1.1.8. 4 1 5 . 1.1.9. 5 1 6 .
5 2 6 6 3 7 7 4 8

1 −1 1 1 −1 1 1 −1 1
1.1.10. 1 6 −1 . 1.1.11. 1 7 −1 . 1.1.12. 1 8 −1 .
4 3 7 5 4 8 6 5 9

2 3 1 3 4 1 4 5 1
1.1.13. 5 4 1 . 1.1.14. 6 5 1 . 1.1.15. 7 6 1 .
6 7 0 7 8 0 8 9 0

4 3 7 5 4 8 6 5 9
1.1.16. 2 0 5 . 1.1.17. 3 0 6 . 1.1.18. 4 0 7 .
1 −1 1 1 −1 1 1 −1 1

7 2 4 8 3 5 9 4 6
1.1.19. 3 1 5 . 1.1.20. 4 1 6 . 1.1.21. 5 1 7 .
1 −1 1 1 −1 1 1 −1 1

2 1 4 3 1 5 4 1 6
1.1.22. 1 3 1 . 1.1.23. 1 4 1 . 1.1.24. 1 5 1 .
5 1 6 6 1 7 7 1 8

16
−1 4 3 −1 5 4 −1 6 5
1.1.25. 1 2 −1 . 1.1.26. 1 3 −1 . 1.1.27. 1 4 −1 .
−1 5 2 −1 6 3 −1 7 4

−1 6 1 −1 7 1 −1 8 1
1.1.28. 3 1 4 . 1.1.29. 4 1 5 . 1.1.30. 5 1 6 .
−1 5 1 −1 6 1 −1 7 1

1.2. Find the real roots of the given equation:

1− k 2 3 2−k 1 3
1.2.1. 3 4−k 7 =0. 1.2.2. 3 3− k 6 =0.
−1 2 1− k −2 1 −1 − k

1− k 4 5 −1 − k 2 1
1.2.3. 2 1− k 3 =0. 1.2.4. 1 4−k 5 =0
−4 3 −1 − k 3 2 5−k

1− k 1 2 −2 − k 2 0
1.2.5. −2 3−k 1 = 0. 1.2.6. 2 4−k 6 =0.
4 2 6−k 1 2 3− k

2−k −1 1 −3 − k 3 0
1.2.7. 5 1− k 6 =0. 1.2.8. −1 3− k 2 =0.
−2 2 −k 2 1 3− k

4−k −2 2 −3 − k 4 1
1.2.9. −1 3− k 2 =0. 1.2.10. −3 1− k −2 =0.
1 2 3− k 1 −2 −1 − k

1− k 0 1 −3 − k −1 −4
1.2.11. 4 −2 − k 2 =0. 1.2.12. 1 2−k 3 =0.
3 −2 1− k −1 3 2−k

2−k 1 3 1− k 3 4
1.2.13. −3 2−k −1 = 0 . 1.2.14. 0 2−k 2 =0.
0 2 2−k 3 −2 1 − k

17
1− k 2 3 −1 − k −2 −3
1.2.15. −1 − k −1 = 0 . 1.2.16. 3 2−k 5 =0.
3 −2 1 − k 2 0 2−k
1− k −1 0 −2 − k 2 0
1.2.17. −1 2−k 1 = 0. 1.2.18. 2 4−k 6 =0.
5 −2 3 − k 1 2 3− k
3− k −1 2 −2 − k 2 0
1.2.19. −1 3−k 2 =0. 1.2.20. 2 4−k 6 =0.
2 −2 −k 1 2 3− k
2−k 0 2 −1 − k 2 1
1.2.21. −1 2−k 1 =0. 1.2.22. 3 −4 − k −1 = 0 .
3 1 4−k 2 −2 −k
1 − k −3 −2 −1 − k 2 1
1.2.23. −1 3 − k 2 =0. 1.2.24. 1 4−k 5 = 0.
−3 2 −1 − k 1 −2 −1 − k
2−k 1 3 −k 2 2
1.2.25. 0 3− k 3 = 0. 1.2.26. −2 4 − k 2 =0.
3 −2 1 − k 1 2 3− k
2−k 3 5 −1 − k 3 2
1.2.27. −1 3− k 2 = 0. 1.2.28. 4 −k 4 =0.
−1 2 1− k 1 2 3− k
1− k 3 4 −2 − k 2 0
1.2.29. −1 2−k 1 =0. 1.2.30. 2 4−k 6 =0.
3 −2 1− k 1 2 3− k

1.3. Calculate:
1 −2 −1 3 1 −2 −1 3
−1 3 −1 −1 −1 3 −1 −1
1.3.1. . 1.3.2. .
3 −8 1 5 3 −8 4 6
2 3 −14 21 2 2 −12 19

18
1 −2 −1 3 1 −2 −1 3
−1 3 −1 −1 −1 3 −1 −1
1.3.3. . 1.3.4. .
3 −8 7 7 3 −8 10 8
2 1 −10 17 2 0 −8 15
1 −2 −1 3 1 −2 −1 3
−1 3 −1 −1 −1 3 −1 −1
1.3.5. . 1.3.6. .
3 −8 13 9 3 −8 16 10
2 −1 −6 13 2 −2 −4 11
1 −2 −1 3 1 −2 −1 3
−1 3 −1 −1 −1 3 −1 −1
1.3.7. . 1.3.8. .
3 −8 19 11 3 −8 22 12
2 −3 −2 9 2 −4 −10 7
1 −2 −1 3 1 −2 −1 3
−1 3 −1 −1 −1 3 −1 −1
1.3.9. . 1.3.10. .
3 −8 25 13 3 −8 28 14
2 −5 2 5 2 −6 4 3
1 1 0 −2 1 2 0 −2
1 2 1 −4 2 5 1 −6
1.3.11. . 1.3.12. .
3 13 11 −18 3 15 11 −16
−2 2 5 1 −2 0 6 1
1 3 0 −2 1 4 0 −2
3 10 1 −8 4 17 1 −10
1.3.13. . 1.3.14. .
3 17 11 −14 3 19 11 −12
−2 −2 7 1 −2 −4 8 1
1 5 0 −2 1 6 0 −2
5 26 1 −12 6 37 1 −14
1.3.15. . 1.3.16. .
3 21 11 −10 3 23 11 −8
−2 −6 9 1 −2 −8 10 1
1 7 0 −2 1 8 0 −2
7 50 1 −16 8 65 1 −18
1.3.17. . 1.3.18. .
3 25 11 −6 3 27 11 −4
−2 −10 11 1 −2 −12 12 1

19
1 −2 2 4 1 9 0 −2
2 −3 3 5 9 82 1 −20
1.3.19. . 1.3.20. .
−1 10 2 −27 3 29 11 −2
2 0 4 −2 −2 −14 13 1

1 −2 2 1 1 −2 2 2
2 −3 3 −1 2 −3 3 1
1.3.21. . 1.3.22. .
−1 4 −1 −6 −1 6 0 −13
2 0 1 −8 2 0 2 −6

1 −2 2 3 1 10 0 −2
2 −3 3 3 9 91 1 −20
1.3.23. . 1.3.24. .
−1 8 1 −20 3 31 11 0
2 0 3 −4 1 15 25 1

1 −2 2 5 1 −2 2 6
2 −3 3 7 2 −3 3 9
1.3.25. . 1.3.26. .
−1 12 3 −34 −1 14 4 −41
2 0 5 0 2 0 6 2

1 −2 2 7 1 −2 2 8
2 −3 3 11 2 −3 3 13
1.3.27. . 1.3.28. .
−1 16 5 −48 −1 18 6 −55
2 0 7 4 2 0 8 6

1 −2 2 9 1 −2 2 10
2 −3 3 15 2 −3 3 17
1.3.29. . 1.3.30. .
−1 20 7 −62 −1 22 8 −69
2 0 9 8 2 0 10 10

1.4. Calculate determinants, using:


а) method of transformation to a triangular form;
b) expansion of a determinant by the elements of some row (column).
1 2 −1 −1 2 1 −1 −1 2 1 −3 0
0 −1 1 2 0 −2 2 2 0 −2 2 1
1.4.1. . 1.4.2. . 1.4.3. .
−1 1 1 −1 −3 0 1 −1 −3 4 1 −1
2 −1 0 0 1 −1 2 0 2 −1 0 2

20
0 3 −1 −1 −1 0 −2 0 1 3 −2 −1
2 −2 1 2 2 −1 3 1 2 −2 4 3
1.4.4. . 1.4.5. . 1.4.6. .
−1 3 1 −1 3 2 0 −1 −2 0 2 −1
3 −1 2 0 4 −1 1 1 4 −1 2 0

0 1 −2 5 0 4 −1 1 3 0 1 0
0 −3 2 1 2 −1 1 2 2 −1 2 −3
1.4.7. . 1.4.8. . 1.4.9. .
−4 2 1 −1 −3 1 1 −1 1 −2 3 −1
1 1 3 2 0 −1 1 4 0 −3 0 2

0 3 −1 −1 1 1 −3 0 −1 2 −4 1
2 −2 1 2 3 −1 2 2 0 −1 2 1
1.4.10. . 1.4.11. . 1.4.12. .
−1 3 1 −1 5 3 0 −1 3 0 −1 −1
3 −1 2 0 0 −1 3 3 3 1 0 2

1 1 −1 0 1 4 −1 −1 0 −3 −1 −1
1.4.13. 2 0 2 1 1 −2 2 2 1 −1 1 2
−3 4 1 −1 1.4.14. . 1.4.15. .
−1 2 1 0 0 2 2 1
4 −2 1 1
2 0 −2 0 3 1 2 0

0 3 −1 −1 4 0 −2 0 0 −3 −1 −1
1.4.16. 2 −2 1 2 . 1 −1 2 1 2 −2 4 2
−1 3 1 −1 1.4.17. . 1.4.18. .
−1 2 1 −1 −1 3 1 −1
3 −1 2 0 2 −1 1 3 3 −1 0 0

1 1 −3 4 0 3 −2 −1 −2 1 −1 0
2 0 0 3 1 0 1 2 1 −2 2 0
1.4.19. . 1.4.20. . 1.4.21. .
3 4 1 −1 −1 2 0 −1 −2 2 1 −1
1 −1 4 2 3 −1 2 0 3 −1 3 −2

0 3 −4 0 1 1 −3 0 2 3 −1 −1
3 −2 1 2 0 −2 2 1 2 −2 1 2
1.4.22. 1.4.23. . 1.4.24. .
−1 0 1 −1 −1 2 0 −1 −1 0 1 −1
1 −1 2 0 4 −2 0 1 2 0 2 1

1 1 −1 1 −1 2 −1 −1 −2 1 −3 2
1.4.25. 0 −2 2 0 . −2 −1 1 2 0 −2 0 1
−3 4 2 −1 1.4.26. . 1.4.27. .
−1 2 1 −1 −2 1 1 −1
2 −1 0 3
0 0 2 1 4 −2 0 2

21
1 2 −1 −1 2 1 −2 4 1 3 0 0
1.4.28. 2 −2 1 2 1 −2 2 1 2 −2 1 2
−1 3 0 0 1.4.29. . 1.4.30. .
−1 0 0 −3 −1 3 1 −1
2 −1 2 4 2 −1 1 2 2 −1 2 1

Micromodule 2
BASIC THEORETICAL INFORMATION. MATRICES

The definition of a matrix, operations with matrices. Inverse matrix. Matrix


equations. Rank of a matrix and its properties.

Literature: [1, chapter 2—3], [4, part 2, p.p.2.2—2.4], [6, chapter 1, §2],
[7, chapter 2, §7], [10, chapter 1, §2], [11, chapter 1, §1].

2.1. The basic concepts

Definition 1.6. A rectangular table of numbers written in the form


⎛ a11 ... a1n ⎞
a12
⎜ ⎟
a a22
... a2 n ⎟
A = ⎜ 21 ,
⎜ ...... ... ... ⎟
⎜⎜ ⎟
am 2 ... amn ⎟⎠
⎝ am1
contains m × n numbers aij ( i = 1, 2,..., m, j = 1, 2,...n) ) is called a matrix. In a
brief form, the matrix can be written as
A = (aij ) (i = 1, 2, ..., m, j = 1, 2, ..., n) ,
where aij are elements of this matrix.
Elements of a matrix form rows and columns. The first index i shows the
number of the row and the second index j shows the number of the column
whose intersection is occupied by the element aij .
For example, we have a matrix as

⎛ a11 a12 a13 ⎞


⎜ ⎟
B = ⎜ a21 a22 a23 ⎟ .
⎜a a32 a33 ⎟⎠
⎝ 31
This matrix has three rows and three columns.
The product m × n is called the matrix dimension.

22
Definition 1.7. If the number of rows in a matrix is equal to the number of
columns ( m = n ) , then it is a square matrix, otherwise it is called a rectangular
matrix. Thus matrix B is square, of order, or dimension, three.
The characteristic of a square matrix is a determinant. Only a square matrix
has a determinant.
The elements a11 , a22 , ..., ann form a principal (or a leading) diagonal of a
square matrix, and the elements a1n , a2 n −1 , ..., an1 form a secondary diagonal.
Definition 1.8. Any matrix in which all elements are equal to zero is called a
matrix zero.
Definition 1.9. If m = 1 and n > 1, then we get a single-row matrix

A = (a1 , a2 , ..., an )

which is known as a row vector, or row matrix, and if m > 1 and n = 1, we get a
single-column matrix
⎛ a1 ⎞
⎜a ⎟
A=⎜ 2 ⎟
⎜ ... ⎟
⎜a ⎟
⎝ m⎠

which is known as a column vector, or column matrix.


Definition 1.10. A square matrix is called a triangle matrix if all elements
placed upon (under) the principal diagonal are equal to zero and there are non
zero elements among the other ones.
Definition 1.11. A square matrix is called a diagonal matrix if all its non-
diagonal elements are equal to zero.
Definition 1.12. A square matrix is called a unit matrix if all diagonal
elements are equal to 1, and non-diagonal elements are equal to 0. It is de-
noted as:

⎛1 0 0 ... 0 ⎞
⎜0 1 0 ... 0 ⎟
⎜ ⎟
I =⎜0 0 1 ... 0 ⎟ .
⎜ ... ... ... ... ⎟
⎜ ⎟
⎝0 0 0 ... 1 ⎠

Definition 1.13. If the rows of a matrix AT are the columns of matrix A and
the columns of a matrix AT are the rows of matrix A then this matrix A T is
called a transpose matrix for matrix A.

23
For any square matrix
⎛ a11 a12 ... a1n ⎞
⎜ ⎟
a a22 ... a2 n ⎟
A = ⎜ 21
⎜ ... ... ... ... ⎟
⎜⎜ ⎟
⎝ an1 an 2 ... ann ⎟⎠
we can correspondently set a determinant det( A) (or Δ( A) ):
a11 a12 ... a1n
a21 a22 ... a2 n
det( A) = .
... ... ... ...
an1 an 2 ... ann

Definition 1.14. A square matrix for which det( A) ≠ 0 is called a non-


singular matrix.

If the det( A) = 0, then matrix is called a singular matrix.


Definition 1.15. Two matrices A = (aij ) and B = (bij ) are equal if the
elements occupying the same places are equal, i.e. if aij = bij for all i and j (in
this case the number of rows (and, similarly, columns) of the matrices A and B
must be equal).

2.2. Operations with matrices

Let matrix A and matrix B be the same order.


1. The Sum of two matrix A = (aij ) and B = (bij ) is a matrix
С = (сij ) whose elements are defined by the relation
aij + bij = cij (i = 1, 2, ..., m, j = 1, 2, ..., n).
The notation is A + B = C.
2. The product of the matrix A = ( aij ) by any real number λ is a matrix
whose every element is equal to the product of the respective element of the
matrix A by the number λ, i.e.
λA = λ (aij ) = (λaij ) (i = 1, 2, ..., m, j = 1, 2, ..., n).
3. The product of the matrix A = (aij ) , which has m rows and k columns
by the matrix B = (bij ) which has k rows and n columns is a matrix С = (сij )

24
which has m rows and n columns and whose element сij is equal to the sum of
the products of the elements of the i-th row of the matrix A by the j-th column of
the matrix B, i.e.
Am×k • Bk ×n = Cm×n

must be the same

size of product

where сij = ai1b1 j + ai 2b2 j + ... + aik bkj

(i = 1, 2, ..., m, j = 1, 2, ..., n) .
In this the number k of columns of the matrix A must be equal to the
number of rows of the matrix B, otherwise the product is not defined.
Remark. In general case AB ≠ BA . The multiplication of matrices is not
commutative.
Properties of operations with matrices

1. A + B = B + A . 6. (αA) B = A(αB).
2. A + ( B + C ) = ( A + B ) + C . 7. ( A + B )C = AC + BC .
3. (α + β) A = αA + β A . 8. ( AB)C = A( BC ) .
4. α ( A + B ) = αA + αB . 9. ( AT )T = A .
5. α(βA) = (αβ) A . 10. ( A + B )T = AT + BT .

2.3. Inverse matrix


An inverse matrix A−1 exists if and only if matrix A is non-singular А
( det( A) ≠ 0 ).

−1
Definition 1.16. The inverse matrix A is a matrix which satisfies the
conditions
A−1 ⋅ A = A ⋅ A−1 = I ,

were I is an identity matrix

25
The inverse matrix can be calculated by the formula

⎛ A11 A21 ... An1 ⎞


⎜ ⎟
1 A A22 ... An 2 ⎟
−1
A = ⋅ ⎜ 12 ,

det( A) ... ... ... ... ⎟
⎜ ⎟
⎝ A1n A2 n ... Ann ⎠

where Aij is the cofactor of the element aij .


Remark. Pay the special attention to the order of indexes.

2.4. Matrices equations

Let’s consider the system of n linear equations with n unknowns

⎧ a11 x1 + a12 x2 + ... + a1n xn = b1 ,


⎪a x + a x + ... + a x = b ,
⎪ 21 1 22 2 2n n 2

⎪ − − − − − − − − − − − − − − −
⎪⎩ an1 x1 + an 2 x2 + ... + ann xn = bn .
We write the above equations in the matrix form. We introduce the following
designations
⎛ a11 a12 ... a1n ⎞ ⎛ x1 ⎞ ⎛ b1 ⎞
⎜ ⎟ ⎜ ⎟ ⎜ ⎟
⎜ a21 a22 ... a2 n ⎟ ⎜ x2 ⎟ ⎜ b2 ⎟
A=⎜ ⎟ , X = ⎜ ⎟ and B = ⎜ ⎟ .
... .... .... ... ... ...
⎜ ⎟ ⎜ ⎟ ⎜ ⎟
⎜a ⎟ ⎜ ⎟ ⎜ ⎟
⎝ n1 an 2 ... ann ⎠ ⎝ xn ⎠ ⎝ bn ⎠
The matrix A is called the matrix of system. X is the matrix of unknowns. B
is the matrix of constant terms.
Then applying the rule of multiplication of matrices the given system can be
written by one matrix equation
A ⋅ X = B.
Solution.
Let the matrix A is non-singular matrix ( det A ≠ 0 ) . Therefore the inverse
matrix A−1 exists. Multiply both parts of the matrix equation by A−1 we get
A−1 ⋅ A ⋅ X = A−1 ⋅ B, (property A−1 ⋅ A = I );

26
I ⋅ X = A−1 ⋅ B, (property I ⋅ X = X ).
We have
X = A−1 ⋅ B.

Consequence. To decide a system it is sufficient to find the inverse matrix of


system and multiply it by the matrix of constant terms B on the right.

2.5. Rank of a matrix

Let’s consider a matrix A of m × n


⎛ a11 a12 ... a1n ⎞
⎜a a22 ... a2 n ⎟
A = ⎜ 21 ⎟.
⎜ ... .... .... ... ⎟
⎜a am 2 ... amn ⎟⎠
⎝ m1
We choose in the given matrix A k rows and k columns, where number
k ≤ m and k ≤ n. The determinant of order k , which consists of the cross elements
of chosen rows and columns, is called a minor of order k of the matrix A.

Definition 1.17. Rank of matrix A, designate as r ( A) = rank ( A), is called


the greatest order of the minor if this minor isn’t equal to zero

1. The rank of a matrix exists for any matrix and

0 ≤ r ( A) ≤ min(m, n).

2. r ( A) = 0 if and only if A is zero matrix.


3. r ( A) = n for square matrix A of order n if and only if the matrix A is non-
singular ( det A ≠ 0 ) .
For example, the matrix
⎛ 1 3 −1⎞
⎜ ⎟
A = ⎜4 2 1 ⎟
⎜ 2 −4 0 ⎟
⎝ ⎠
1 3 1 −1
has nine minors of order 2, among which there are also such: , ,
4 2 4 1
3 −1
. The minor of the third order of the given matrix is its determinant.
−4 0

27
We can calculate a rank of a matrix in the following way. If there is a minor
of order k is not equal to zero and all minors of order (k+1) are equal to zero,
then r(A) = k.
On the practice to find a rank of the highest orders it is useful to apply
another method:

A rank of a matrix will not change if we use elementary transformations:


1) Interchanging of two rows (columns);
2) Multiplication of each element of a row (column) by a number k;
3) Addition elements of a row (column) and corresponding elements of
another row (column) multiplying by a number k.
4) Elimination of a zero row from the matrix

Using elementary transformations a matrix can be transformed to the form


when all elements except only a11 , a22 ,..., arr , where r ≤ min( m, n) , are
equal to zero. Then a rank of matrix is equal to r.

Micromodule 2
EXAMPLES OF PROBLEMS SOLUTION

Example 1. Find the product of matrices AB , if

⎛ −2 0 1 ⎞
⎛ 1 3 2⎞ ⎜ ⎟
A=⎜ ⎟ , B = ⎜ 5 1 −1⎟ .
⎝ 4 1 3 ⎠ ⎜ 4 3 2⎟
⎝ ⎠
Solution. We have A2×3 and B3×3 . As the number of columns of a matrix A
is equal to the number of rows of a matrix B. The operation of multiplication
A ⋅ B has the sense and we calculate the product of matrices in the following way:
⎛1 ⋅ ( −2 ) + 3 ⋅ 5 + 2 ⋅ 4 1 ⋅ 0 + 3 ⋅1 + 2 ⋅ 3 1 ⋅1 + 3 ⋅ ( −1) + 2 ⋅ 2 ⎞ ⎛ 21 9 2 ⎞
AB = ⎜ =
⎜ 4 ⋅ ( −2 ) + 1 ⋅ 5 + 3 ⋅ 4 4 ⋅ 0 + 1 ⋅1 + 3 ⋅ 3 4 ⋅1 + 1 ⋅ ( −1) + 3 ⋅ 2 ⎟⎟ ⎜ 9 10 9 ⎟
.
⎝ ⎠ ⎝ ⎠
⎛ 2 −3 ⎞ 2
Example 2. Find f ( A) , if A = ⎜ ⎟ , f ( x) = ( x − 3 x )(3 x + 2) .
⎝ 4 5 ⎠
Solution. It is necessary to find the value
f ( A) = ( A2 − 3 A) ⋅ (3 A + 2 E ) .
We obtain
2
⎛ 2 −3 ⎞ ⎛ 2 −3 ⎞ ⎛ 2 −3 ⎞ ⎛ 4 − 12 −6 − 15 ⎞ ⎛ −8 −21⎞
A2 = ⎜ ⎟ =⎜ ⎟⎜ ⎟=⎜ ⎟=⎜ ⎟;
⎝4 5 ⎠ ⎝ 4 5 ⎠ ⎝ 4 5 ⎠ ⎝ 8 + 20 −12 + 25 ⎠ ⎝ 28 13 ⎠

28
⎛ −8 −21⎞ ⎛ 2 −3 ⎞ ⎛ −8 −21⎞ ⎛ 6 −9 ⎞ ⎛ −14 −12 ⎞
A2 − 3 A = ⎜ ⎟ − 3⎜ ⎟=⎜ ⎟ −⎜ ⎟=⎜ ;
⎝ 28 13 ⎠ ⎝ 4 5 ⎠ ⎝ 28 13 ⎠ ⎝ 12 15 ⎠ ⎝ 16 −2 ⎟⎠
⎛ 2 −3 ⎞ ⎛ 1 0 ⎞ ⎛ 6 −9 ⎞ ⎛ 2 0 ⎞ ⎛ 8 −9 ⎞
3A + 2I = 3 ⎜ ⎟ + 2 ⎜ 0 1 ⎟ = ⎜ 12 15 ⎟ + ⎜ 0 2 ⎟ = ⎜ 12 17 ⎟ ;
⎝ 4 5 ⎠ ⎝ ⎠ ⎝ ⎠ ⎝ ⎠ ⎝ ⎠
⎛ −14 −12 ⎞ ⎛ 8 −9 ⎞ ⎛ −14 ⋅ 8 + (−12) ⋅12 −14 ⋅ (−9) + (−12) ⋅17 ⎞
f ( A) = ⎜ = =
⎝ 16 −2 ⎠⎟ ⎝⎜ 12 17 ⎠⎟ ⎝⎜ 16 ⋅ 8 + ( −2) ⋅12 16 ⋅ ( −9) + ( −2) ⋅17 ⎠⎟
⎛ − 256 − 78 ⎞
= ⎜⎜ ⎟⎟ .
⎝ 104 − 178 ⎠

⎛ − 256 − 78 ⎞
Answer: f ( A) = ⎜ ⎟⎟ .

⎝ 104 − 178 ⎠
Remark. If f ( x ) = a0 x n + a1 x n −1 + an −1 x + an and A is any square matrix
then
f ( A) = a0 An + a1 An −1 + an −1 A + an E .
Example 3. Find the inverse matrix, if
⎛ 1 2 −1⎞
⎜ ⎟
A = ⎜ −1 3 1 ⎟ .
⎜ 2 −1 0 ⎟
⎝ ⎠
Solution. Firstly we calculate the determinant of matrix А:
1 2 −1 1 2 −1
det( A) = −1 3 1 = 0 5 0 = 10 ≠ 0 .
2 −1 0 2 −1 0
Then the inverse matrix exists because А is a non-singular matrix. We find
cofactors:
3 1 2 −1 2 −1
A11 = (−1)1+1 = 1; A21 = (−1) 2+1 = 1; A31 = (−1)3+1 = 5;
−1 0 −1 0 3 1
−1 1 1 −1 1 −1
A12 = − = 2; A22 = = 2; A32 = − = 0;
2 0 2 0 −1 1
−1 3 1 2 1 2
A13 = = −5; A23 = − = 5; A33 = = 5.
2 −1 2 −1 −1 3

29
So, the inverse matrix can be written in the form

⎛ 1 1 5 ⎞ ⎛ 0,1 0,1 0, 5 ⎞
−11 ⎜ ⎟ ⎜ ⎟
A = ⎜ 2 2 0 ⎟ = ⎜ 0, 2 0, 2 0 ⎟ .
10 ⎜ ⎟ ⎜ ⎟
⎝ −5 5 5 ⎠ ⎝ −0, 5 0, 5 0, 5 ⎠
⎛ 2 2⎞ ⎛ −1 4 ⎞
Example 4. Solve the matrix equation X ⋅ A ⋅ B = C , if A = ⎜ ⎟ , B =⎜ ⎟,
⎝ 3 1⎠ ⎝ 2 −7⎠
С = (1 −2 ) .
Solution. Consequently we have
X ⋅ A ⋅ B = C , X ⋅ A ⋅ BB −1 = CB −1 , X ⋅ A ⋅ E = CB −1 , X ⋅ A = CB −1 ,
X ⋅ AA−1 = CB −1 A−1 , X ⋅ E = CB −1 A−1 , X = CB −1 A−1 .
Find the inverse matrices A−1 and B −1 :
2 2
Δ ( A) = = −4 ≠ 0 , A11 = 1 , A21 = −2 , A12 = −3 , A22 = 2 .
3 1
1 ⎛ 1 −2 ⎞
A−1 = − ⎜ ⎟;
4 ⎝ −3 2 ⎠
−1 4
Δ( B) = = −1 , B11 = −7 , B21 = −4 , B12 = −2 , B22 = −1 .
2 −7
⎛ −7 −4 ⎞ ⎛ 7 4 ⎞
B −1 = − ⎜ ⎟=⎜ ⎟.
⎝ −2 −1 ⎠ ⎝ 2 1 ⎠
Then
⎛ 7 4 ⎞ ⎛ 1 ⎞ ⎛ 1 −2 ⎞
X = CB −1 A−1 = (1 −2 ) ⎜ ⎟⎜− ⎟⎜ ⎟=
⎝ 2 1 ⎠ ⎝ 4 ⎠ ⎝ −3 2 ⎠
1 ⎛ 1 −2 ⎞ 1 ⎛ 1 −2 ⎞
= − (1 ⋅ 7 − 2 ⋅ 2 1 ⋅ 4 − 2 ⋅1) ⎜ ⎟ = − (3 2) ⎜ ⎟=
4 ⎝ −3 2 ⎠ 4 ⎝ −3 2 ⎠
1 1 ⎛3 1⎞
=− (3 ⋅1 + 2 ⋅ (−3) 3 ⋅ (−2) + 2 ⋅ 2) = − (−3 − 2) = ⎜ .
4 4 ⎝4 2 ⎟⎠
⎛3 1⎞
Answer: X = ⎜ ⎟.
⎝ 4 2⎠
Example 5. Find the rank of a matrix
⎛ 1 −2 0 4 ⎞
⎜ ⎟
Α = ⎜ −1 3 −1 2 ⎟ .
⎜ 0 1 −1 6 ⎟
⎝ ⎠

30
Solution. We take a minor of the second order, for example,

1 −2
Μ( ) =
2
= 1 ≠ 0.
−1 3

The bordering minors of the third order for its are

1 −2 0 1 −2 4
−1 3 −1 and −1 3 2 .
0 1 −1 0 1 6

( 2)
If both minors of the third order are equal to zero and the minor M of the
second order isn’t equal to zero, then by the definition r ( A) = 2.

⎛ 2 −2 1 −1 3 ⎞
⎜ ⎟
−4 −4 2 −4 −2 ⎟
Example 6. Find the rank of matrix A = ⎜ .
⎜ 3 0 0 2 3 ⎟
⎜⎜ ⎟
⎝ −2 −6 3 −5 1 ⎟⎠

Solution. Let’s consider the elementary transformations to the given matrix

⎛ 2 −2 1 −1 3⎞ ⎛ 2 0 1 −1 3 ⎞ ⎛1 0 3 −1 3⎞
⎜ ⎟ ⎜ ⎟ ⎜ ⎟
⎜ −4 −4 2 −4 −2 ⎟ ⎜ −4
~
0 2 −4 −2 ⎟ ⎜ 2
~
0 −2 −4 −2 ⎟
~
⎜ 3 0 0 2 3 ⎟ ⎜ 3 0 0 2 3 ⎟ ⎜0 0 3 2 3 ⎟
⎜⎜ ⎟⎟ ⎜⎜ ⎟⎟ ⎜⎜ ⎟⎟
⎝ −2 −6 3 −5 1 ⎠ ⎝ −2 0 3 −5 1 ⎠ ⎝3 0 1 −5 1⎠

⎛ 1 −1 3 0 0 ⎞ ⎛ 1 −1 3 0 0⎞
⎜ ⎟ ⎜ ⎟
2 −4 −2 0 0 ⎟ ⎜ 0 −2 −8 0 0⎟
~⎜ ~ ~
⎜0 2 3 0 0⎟ ⎜ 0 2 3 0 0⎟
⎜⎜ ⎟⎟ ⎜⎜ ⎟⎟
⎝ 3 −5 1 0 0 ⎠ ⎝ 0 −2 −8 0 0⎠

⎛ 1 −1 3 0 0 ⎞ ⎛ 1 −1 3 0 0⎞
⎜ ⎟ ⎜ ⎟
0 −2 −8 0 0 ⎟ ⎜ 0 −2 −8 0 0⎟
~⎜ ~ .
⎜0 2 3 0 0 ⎟ ⎜ 0 0 −5 0 0⎟
⎜⎜ ⎟⎟ ⎜⎜ ⎟⎟
⎝0 0 0 0 0⎠ ⎝ 0 0 0 0 0⎠

The determinant of the third order, which consists of the cross elements of
the first 3 rows and columns of the last matrix, is not equal to zero, and all the
minors of the forth order are equal to zero. So, r ( A) = 3 .

31
Micromodule 2
CLASS AND HOME ASSIGNMENT

Calculate the product of the matrices:


⎛ 4 2⎞
⎛ −2 3 1 ⎞ ⎜ ⎟
1. AB and BA , if A = ⎜ ⎟ , B = ⎜7 3⎟ .
⎝ 5 0 6 ⎠ ⎜ −1 1 ⎟
⎝ ⎠

⎛ 1 2 −1⎞ ⎛6⎞
⎜ ⎟ ⎜ ⎟
2. AB , if A = ⎜ 3 0 1 ⎟ , B = ⎜ −3 ⎟ .
⎜ −2 1 4 ⎟ ⎜9⎟
⎝ ⎠ ⎝ ⎠

⎛ 2 3 −1⎞
⎛ 4 3⎞ ⎛ −4 −2 4 ⎞ ⎜ ⎟
3. ABC , if A = ⎜ ⎟, B =⎜ ⎟, C =⎜ 3 1 0 ⎟ .
⎝ 2 1 ⎠ ⎝ 1 0 3 ⎠ ⎜ −2 0 0 ⎟
⎝ ⎠
Find f ( A) if:

⎛ 2 −3 ⎞
⎟ , f ( x ) = ( x − 3x) ⋅ (3x + 2) .
2
4. A = ⎜
⎝ 4 5 ⎠

⎛1 0 4⎞
⎜ ⎟
5. A = ⎜ 1 3 −2 ⎟ , f ( x ) = 2 x 2 + x − 4 .
⎜ −2 1 2 ⎟
⎝ ⎠
Find an inverse matrix, if:

⎛ 1 1 −1 ⎞ ⎛ 1 3 −1 ⎞
⎜ ⎟ ⎜ ⎟
6. A = ⎜ 0 −2 −3 ⎟ . 7. A = ⎜ 1 1 −5 ⎟ .
⎜ 2 3 −1 ⎟ ⎜ −1 −2 2 ⎟
⎝ ⎠ ⎝ ⎠
Find the rank of matrix:
⎛1 −2 3 ⎞ ⎛ 1 −2 1 −1 3 ⎞
⎜ ⎟ ⎜ ⎟
8. ⎜
3 2 4 ⎟
. 9. ⎜ −2 −4 2 −4 0 ⎟
.
⎜ −2 −4 −1⎟ ⎜ −5 −4 5 −2 3 ⎟
⎜⎜ ⎟ ⎜⎜ ⎟
⎝4 0 7 ⎟⎠ ⎝ −6 −10 8 −7 6 ⎟⎠

32
Answers

⎛ 2 12 16 ⎞ ⎛ −9 ⎞
⎛12 6 ⎞ ⎜ ⎟ ⎛ −242 −66 ⎞
1. AB = ⎜ ⎟ , BA = ⎜⎜ 1 21 25⎟⎟ . 2. AB = ⎜ 27 ⎟ . 4. f ( A) = ⎜ ⎟.
⎝14 16 ⎠ ⎜ 7 −3 5 ⎟ ⎜ 21 ⎟ ⎝ 132 −176 ⎠
⎝ ⎠ ⎝ ⎠

⎛ 11 −2 −5 ⎞
−1 ⎜ ⎟
6. A = ⎜ −6 1 3 ⎟.
⎜ 4 −1 −2 ⎟
⎝ ⎠
Micromodule 2
SELF-TEST ASSIGNMENT
2.1. Find f ( A) if:
⎛ 1 2 −1⎞
⎜ ⎟
2.1.1. A = ⎜ 3 0 1 ⎟ , f ( x) = 2 x 2 + 3 x + 1 .
⎜0 1 0 ⎟
⎝ ⎠
⎛3 4 0⎞
⎜ ⎟
2.1.2 . A = ⎜ 1 2 3 ⎟ , f ( x) = x 2 + 5 x + 2 .
⎜ −3 1 −1⎟
⎝ ⎠
⎛2 1 ⎞ 3 2
2.1.3. A = ⎜ ⎟ , f ( x) = x + x − 7 x + 3 .
⎝ 0 −1 ⎠
⎛ 1 −2 3 ⎞
⎜ ⎟
2.1.4. A = ⎜ 2 −4 1 ⎟ , f ( x) = 3x 2 − 2 x + 8 .
⎜ 3 −5 2 ⎟
⎝ ⎠
⎛ −2 2 −1⎞
⎜ ⎟
2.1.5. A = ⎜ 1 0 5 ⎟ , f ( x) = x 2 − 3x − 1 .
⎜ 4 3 0⎟
⎝ ⎠
⎛ 2 1 1⎞
⎜ ⎟
2.1.6. A = ⎜ 3 1 2 ⎟ , f ( x) = x 2 − x − 1 .
⎜ 1 −1 0 ⎟
⎝ ⎠
⎛ −1 3 0 ⎞
⎜ ⎟
2.1.7. A = ⎜ −2 1 1 ⎟ , f ( x) = x 2 − 2 x + 1 .
⎜ 3 0 −2 ⎟
⎝ ⎠

33
⎛2 1 3⎞
⎜ ⎟
2.1.8. A = ⎜ −3 0 −1⎟ , f ( x) = −2 x 2 + 8 x − 6 .
⎜ 4 2 −1⎟
⎝ ⎠
⎛ −3 1⎞ 3
2.1.9. A = ⎜ ⎟ , f ( x) = x − 4 x .
⎝ 2 1 ⎠
⎛ 2 1 4⎞
⎜ ⎟
2.1.10. A = ⎜ −1 3 0 ⎟ , f ( x) = 2 x 2 + 6 x − 3 .
⎜ 0 −2 3 ⎟
⎝ ⎠
⎛ −4 8 1 ⎞
⎜ ⎟
2.1.11. A = ⎜ 0 2 3 ⎟ , f ( x) = x 2 − 5 x + 3 .
⎜ 1 −3 −1⎟
⎝ ⎠
⎛ 1 −3 −1⎞
⎜ ⎟
2.1.12. A = ⎜ 2 2 3 ⎟ , f ( x) = 5 x 2 + 2 x − 8.
⎜3 1 2 ⎟
⎝ ⎠
⎛ 1 2⎞ 3 2
2.1.13. A = ⎜ ⎟ , f ( x) = x − 4 x + x − 1 .
⎝ −2 1 ⎠
⎛ 3 −1⎞ 3
2.1.14. A = ⎜ ⎟ , f ( x) = 2 x − 8 x + 6 .
⎝ 4 2 ⎠
⎛ 0 3 1⎞
⎜ ⎟
2.1.15. A = ⎜ −2 1 0 ⎟ , f ( x) = x 2 − 6 x + 9.
⎜ −1 5 −4 ⎟
⎝ ⎠
⎛4 3 0 ⎞
⎜ ⎟
2.1.16. A = ⎜ 2 −1 5 ⎟ , f ( x) = 9 x 2 − 4 .
⎜ 1 2 −1⎟
⎝ ⎠
⎛ 5 −2 1 ⎞
⎜ ⎟
2.1.17. A = ⎜ −1 3 6 ⎟ , f ( x) = 9 − x 2 .
⎜ 0 4 −5 ⎟
⎝ ⎠
⎛ 1 0 4⎞
⎜ ⎟
2.1.18. A = ⎜ 2 3 0 ⎟ , f ( x) = 2 x 2 + 8 x + 8 .
⎜ 1 −1 2 ⎟
⎝ ⎠

34
⎛1 3 2⎞
⎜ ⎟
2.1.19. A = ⎜ −4 0 5 ⎟ , f ( x) = 5 − 4 x − x 2 .
⎜ 2 4 −7 ⎟
⎝ ⎠
⎛ 5 4 0⎞
⎜ ⎟
2.1.20. A = ⎜ 1 3 1 ⎟ , f ( x) = 3x 2 − 7 x + 5 .
⎜ −2 1 −4 ⎟
⎝ ⎠
⎛ 6 2 5⎞
⎜ ⎟
2.1.21. A = ⎜ −1 3 1 ⎟ , f ( x) = ( x + 3) .
2

⎜ 0 4 2⎟
⎝ ⎠
⎛ 3 2 1⎞
⎜ ⎟
2.1.22. A = ⎜ 4 1 2 ⎟ , f ( x) = ( x − 2 ) ⋅ ( 2 x + 3) .
⎜ 1 −5 3 ⎟
⎝ ⎠
⎛8 0 2⎞
⎜ ⎟
2.1.23. A = ⎜ −3 2 −3 ⎟ , f ( x) = 4 − 2 x − x 2 .
⎜ 1 1 −1 ⎟
⎝ ⎠
⎛ 3 −1 4 ⎞
⎜ ⎟
2.1.24. A = ⎜ 0 2 5 ⎟ , f ( x) = 5 x 2 − 8 x + 6 .
⎜ 5 −1 8 ⎟
⎝ ⎠
⎛ 1 −5 ⎞ 3 2
2.1.25. A = ⎜ ⎟ , f ( x) = 3x + 4 x − 2 .
⎝ 6 2 ⎠
⎛ 2 −3 ⎞ 2
2.1.26. A = ⎜ ⎟ , f ( x) = ( x − 2 x)(2 x − 1) .
⎝ 4 5 ⎠
⎛0 3 5⎞
⎜ ⎟
2.1.27. A = ⎜ −1 4 −2 ⎟ , f ( x) = (4 − 2 x)( x + 6) .
⎜2 1 3⎟
⎝ ⎠
⎛ 7 3 2⎞
⎜ ⎟
2.1.28. A = ⎜ −1 4 5 ⎟ , f ( x) = 2 x 2 − 6 x + 7 .
⎜ 2 1 1⎟
⎝ ⎠
⎛ 2 −3 ⎞ 2
2.1.29. A = ⎜ ⎟ , f ( x) = ( x − 2 x)(2 x + 1) .
⎝ 4 5 ⎠

35
⎛1 2 4⎞
⎜ ⎟
2.1.30. A = ⎜ 1 3 −2 ⎟ , f ( x) = 5 x 2 + 3 x − 4 .
⎜ −1 0 5 ⎟
⎝ ⎠

2.2. Solve the matrix equations.


⎛ 3 2⎞ ⎛1 2 7⎞
2.2.1. A ⋅ X = B , A = ⎜ ⎟ , B=⎜ ⎟.
⎝1 1⎠ ⎝0 4 8⎠
⎛2 8⎞
⎛ 4 3⎞ ⎜ ⎟
2.2.2. X ⋅ A = B , A = ⎜ ⎟ , B = ⎜1 1 ⎟ .
⎝ 1 1 ⎠ ⎜ 0 −1⎟
⎝ ⎠
⎛5 4 1⎞
⎛ 2 3⎞ ⎜ ⎟ ⎛ 1 1 0⎞
2.2.3. A ⋅ X ⋅ B = C , A = ⎜ ⎟ , B = ⎜1 1 7⎟ , C = ⎜ ⎟.
⎝ 5 8⎠ ⎜6 5 9 ⎟⎠ ⎝ 0 1 1⎠

⎛ 2 1⎞ ⎛1 0 7⎞
2.2.4. A ⋅ X = B , A = ⎜ ⎟ , B=⎜ ⎟.
⎝ 1 1⎠ ⎝8 1 2⎠
⎛ 4 7⎞
⎛ 3 2⎞ ⎜ ⎟
2.2.5. X ⋅ A = B , A = ⎜ ⎟ , B = ⎜0 1 ⎟ .
⎝ 1 1 ⎠ ⎜2 3⎟
⎝ ⎠
⎛4 3⎞ ⎛ 6 8⎞ ⎛ 5 4⎞
2.2.6. A ⋅ X ⋅ B = C , A = ⎜ ⎟, B =⎜ ⎟,С = ⎜ ⎟.
⎝1 1⎠ ⎝ 2 1⎠ ⎝ −2 0 ⎠
⎛1 3 2⎞ ⎛1 0 ⎞
⎜ ⎟ ⎜ ⎟
2.2.7. A ⋅ X = B , A = ⎜ 3 8 5⎟ , B = ⎜3 1 ⎟ .
⎜2 7 6 ⎟⎠ ⎜ 3 −4 ⎟
⎝ ⎝ ⎠
⎛ 4 3 6⎞
⎜ ⎟ ⎛2 1 5 ⎞
2.2.8. X ⋅ A = B , A = ⎜ 5 4 7 ⎟ , B = ⎜ ⎟.
⎜ 6 5 7⎟ ⎝ 0 0 −1⎠
⎝ ⎠
⎛7 8 7⎞
⎛ 5 4⎞ ⎜ ⎟ ⎛ −1 −1 0 ⎞
2.2.9. A ⋅ X ⋅ B = C , A = ⎜ ⎟ , B = ⎜ 8 9 6⎟ , C = ⎜ ⎟.
⎝ 9 7 ⎠ ⎜ 5 6 8⎟ ⎝ 1 1 0⎠
⎝ ⎠
⎛ 1 1 2⎞ ⎛1 5 ⎞
⎜ ⎟ ⎜ ⎟
2.2.10. A ⋅ X = B , A = ⎜ 2 −1 2 ⎟ , B = ⎜ 4 −1⎟ .
⎜ 4 1 2⎟ ⎜2 7 ⎟
⎝ ⎠ ⎝ ⎠

36
⎛ 5 5 3⎞
⎜ ⎟ ⎛6 4 6 ⎞
2.2.11. X ⋅ A = B , A = ⎜ 3 4 2 ⎟ , B = ⎜ ⎟.
⎜ 4 3 5⎟ ⎝ 0 3 −5 ⎠
⎝ ⎠
⎛ 3 3 2⎞ ⎛ 1 2⎞
⎜ ⎟ ⎛2 3⎞ ⎜ ⎟
2.2.12. A ⋅ X ⋅ B = C , A = ⎜ 4 5 3 ⎟ , B = ⎜ ⎟ , C = ⎜ 4 6⎟ .
⎜ 5 4 4⎟ ⎝ −3 −4 ⎠ ⎜ 4 7⎟
⎝ ⎠ ⎝ ⎠
⎛ 1 2 −3 ⎞ ⎛ 1 1 1⎞
⎜ ⎟ ⎜ ⎟
2.2.13. A ⋅ X = B , A = ⎜ 0 1 2 ⎟ , B = ⎜ 2 −3 1 ⎟ .
⎜1 0 4 ⎟ ⎜ 4 1 5⎟
⎝ ⎠ ⎝ ⎠
⎛ 2 1 3⎞ ⎛1 −1 3 ⎞
⎜ ⎟ ⎜ ⎟
2.2.14. X ⋅ A = B , A = ⎜ 0 2 0 ⎟ , B = ⎜ 4 2 0⎟ .
⎜ 1 4 5⎟ ⎜1 4 5 ⎟⎠
⎝ ⎠ ⎝
⎛ 3 4⎞ ⎛ −1 2⎞ ⎛ 1 0⎞
2.2.15. A ⋅ X ⋅ B = C , A = ⎜ ⎟, B =⎜ ⎟, С =⎜ ⎟.
⎝ 1 1 ⎠ ⎝3 −5 ⎠ ⎝ 0 1⎠
⎛ 3 −2 ⎞ ⎛3 2⎞ ⎛2 1 ⎞
2.2.16. X ⋅ A ⋅ B = C , A = ⎜ ⎟,B=⎜ ⎟, С =⎜ ⎟.
⎝ 1 −1 ⎠ ⎝1 4⎠ ⎝ 4 −3 ⎠
⎛ 5 3 4⎞
⎜ ⎟
2.2.17. X ⋅ A = B , A = ⎜ −6 −3 −5 ⎟ , B = ( 3 2 1) .
⎜ 4 2 2⎟
⎝ ⎠
⎛ 4 5⎞ ⎛ 5 8⎞ ⎛2 4⎞
2.2.18. A ⋅ X ⋅ B = C , A = ⎜ ⎟, B=⎜ ⎟, С =⎜ ⎟.
⎝ −5 − 6 ⎠ ⎝ −2 − 3 ⎠ ⎝ −3 −4 ⎠
⎛ 2 4 5⎞ ⎛1 5 1⎞
⎜ ⎟ ⎜ ⎟
2.2.19. X ⋅ A = B , A = ⎜ 3 −1 1 ⎟ , B = ⎜ −3 −1 −1⎟ .
⎜ 1 1 1⎟ ⎜2 5 3⎟
⎝ ⎠ ⎝ ⎠
⎛ 3 −1 ⎞ ⎛ 5 6⎞ ⎛14 16 ⎞
2.2.20. A ⋅ X ⋅ B = C , A = ⎜ ⎟, B=⎜ ⎟, С =⎜ ⎟.
⎝ 5 −2 ⎠ ⎝ 7 8⎠ ⎝ 9 10 ⎠
⎛ 2 1⎞ ⎛ −3 2 ⎞ ⎛ −2 4 ⎞
2.2.21. A ⋅ X ⋅ B = C , A = ⎜ ⎟, B=⎜ ⎟, С =⎜ ⎟.
⎝ 3 2⎠ ⎝ 3 −1⎠ ⎝ 3 − 1⎠
⎛ 5 3 1⎞ ⎛ −8 3 0 ⎞
⎜ ⎟ ⎜ ⎟
2.2.22. X ⋅ A = B , A = ⎜ 1 −3 −2 ⎟ , B = ⎜ −5 9 0 ⎟ .
⎜ −5 2 1 ⎟ ⎜ −2 15 0 ⎟
⎝ ⎠ ⎝ ⎠

37
⎛1 2 −3 ⎞ ⎛ 1 0 −3 ⎞
⎜ ⎟ ⎜ ⎟
2.2.23. A ⋅ X = B , A = ⎜ 3 2 −4 ⎟ , B = ⎜10 2 7 ⎟ .
⎜2 −1 0 ⎟⎠ ⎜10 7 8 ⎟
⎝ ⎝ ⎠
⎛1 1 1⎞ ⎛1 1 2 ⎞
⎜ ⎟ ⎜ ⎟
2.2.24. A ⋅ X = B , A = ⎜ 1 2 3 ⎟ , B = ⎜1 1 2 ⎟ .
⎜1 3 4 ⎟⎠ ⎜1 1 2 ⎟
⎝ ⎝ ⎠
⎛ 2 3 1⎞
⎜ ⎟
2.2.25. X ⋅ A = B , A = ⎜ 3 7 2 ⎟ , B = (10 3 3) .
⎜ 5 4 2⎟
⎝ ⎠
⎛1 2⎞ ⎛1 3⎞ ⎛ 2 4 ⎞
2.2.26. A ⋅ X ⋅ B = C , A = ⎜ ⎟, B = ⎜ ⎟, С =⎜ ⎟.
⎝ − 3 − 5 ⎠ ⎝ − 1 −2 ⎠ ⎝ −6 −11⎠
⎛ 2 2 3⎞ ⎛1 0⎞
⎜ ⎟ ⎜ ⎟
2.2.27. A ⋅ X = B , A = ⎜ 1 −1 0 ⎟ , B = ⎜ 3 1 ⎟ .
⎜ −1 2 1 ⎟ ⎜ −3 −4 ⎟
⎝ ⎠ ⎝ ⎠
⎛ 1 1 − 1⎞ ⎛ 1 −1 3 ⎞
⎜ ⎟ ⎜ ⎟
2.2.28. X ⋅ A = B , A = ⎜ 2 1 0 ⎟ , B = ⎜ 4 3 2 ⎟ .
⎜ 1 −1 1 ⎟ ⎜ 1 −2 5 ⎟
⎝ ⎠ ⎝ ⎠
⎛ 2 2⎞ ⎛ −1 4 ⎞
2.2.29. X ⋅ A ⋅ B = C , A = ⎜ ⎟, B=⎜ ⎟ , С = (1 −2 ) .
⎝ 3 1⎠ ⎝ 2 −7 ⎠
⎛ 5 −6 4 ⎞ ⎛ 3⎞
⎜ ⎟ ⎜ ⎟
2.2.30. A ⋅ X = B , A = ⎜ 3 −3 2 ⎟ , B = ⎜ 2 ⎟ .
⎜ 4 −5 2 ⎟ ⎜1⎟
⎝ ⎠ ⎝ ⎠
2.3. Find the inverse matrix, if:
⎛1 3 −1 ⎞ ⎛1 5 −1⎞
⎜ ⎟ ⎜ ⎟
2.3.1. A = ⎜ 0 −6 −5 ⎟ . 2.3.2. A = ⎜ −1 4 −3 ⎟ .
⎜1 2 −2 ⎟⎠ ⎜ −1 0 −1⎟⎠
⎝ ⎝
⎛ −1 2 −2 ⎞ ⎛2 −2 1 ⎞
⎜ ⎟ ⎜ ⎟
2.3.3. A = ⎜ −3 5 −4 ⎟ . 2.3.4. A = ⎜ 0 −5 −3 ⎟ .
⎜1 0 1 ⎟⎠ ⎜2 −1 2 ⎟⎠
⎝ ⎝
⎛ 1 −1 −2 ⎞ ⎛2 1 −3 ⎞
⎜ ⎟ ⎜ ⎟
2.3.5. A = ⎜ 0 −5 −4 ⎟ . 2.3.6. A = ⎜ 0 −3 −2 ⎟ .
⎜ 2 −7 −9 ⎟ ⎜ 2 −2 −4 ⎟
⎝ ⎠ ⎝ ⎠

38
⎛ 1 1 −1 ⎞ ⎛ 3 2 −1 ⎞
⎜ ⎟ ⎜ ⎟
2.3.7. A = ⎜ 0 −7 −3 ⎟ . 2.3.8. A = ⎜ −1 −1 −1 ⎟ .
⎜ 3 3 −4 ⎟ ⎜ −2 0 −2 ⎟
⎝ ⎠ ⎝ ⎠

⎛ 2 2 1⎞ ⎛1 2 1⎞
⎜ ⎟ ⎜ ⎟
2.3.9. A = ⎜ −3 −3 3 ⎟ . 2.3.10. A = ⎜ 3 5 1 ⎟ .
⎜ −1 0 1 ⎟ ⎜ 2 −2 0 ⎟
⎝ ⎠ ⎝ ⎠

⎛1 1 −3 ⎞ ⎛ 1 1 4⎞
⎜ ⎟ ⎜ ⎟
2.3.11. A = ⎜ 4 7 −11⎟ . 2.3.12. A = ⎜ 4 7 3 ⎟ .
⎜ 1 −1 0 ⎟ ⎜ 2 2 0⎟
⎝ ⎠ ⎝ ⎠

⎛ 1 2 −1⎞ ⎛2 1 1⎞
⎜ ⎟ ⎜ ⎟
2.3.13. A = ⎜ 6 1 −1⎟ . 2.3.14. A = ⎜ 5 8 1⎟ .
⎜8 1 0 ⎟⎠ ⎜ −1 −7 0 ⎟
⎝ ⎝ ⎠

⎛ 1 −1 3 ⎞ ⎛ 1 −1 4 ⎞
⎜ ⎟ ⎜ ⎟
2.3.15. A = ⎜ 3 −6 3 ⎟ . 2.3.16. A = ⎜ 2 −3 4 ⎟ .
⎜1 5 0 ⎟⎠ ⎜ 4 −4 0 ⎟
⎝ ⎝ ⎠

⎛ 1 2 11 ⎞ ⎛ 3 2 3⎞
⎜ ⎟ ⎜ ⎟
2.3.17. A = ⎜ 4 −3 −2 ⎟ . 2.3.18. A = ⎜ −5 6 1 ⎟ .
⎜ 1 −1 0 ⎟ ⎜ −3 3 0 ⎟
⎝ ⎠ ⎝ ⎠

⎛ 1 −2 1 ⎞ ⎛ 1 1 −3 ⎞
⎜ ⎟ ⎜ ⎟
2.3.19. A = ⎜ 2 −3 0 ⎟ . 2.3.20. A = ⎜ 4 −8 4 ⎟ .
⎜ 2 6 1⎟ ⎜ 5 −5 0 ⎟
⎝ ⎠ ⎝ ⎠

⎛ −1 7 0 ⎞ ⎛ 4 11 0⎞
⎜ ⎟ ⎜ ⎟
2.3.21. A = ⎜ 1 9 3⎟ . 2.3.22. A = ⎜ 1 13 −1⎟ .
⎜ 3 2 3⎟ ⎜ 1 −2 1 ⎟⎠
⎝ ⎠ ⎝

⎛ −1 0 3⎞ ⎛ 5 1 2⎞
⎜ ⎟ ⎜ ⎟
2.3.23. A = ⎜ 3 −1 9 ⎟ . 2.3.24. A = ⎜ 1 4 0 ⎟ .
⎜4 −5 11⎟⎠ ⎜ 2 1 2⎟
⎝ ⎝ ⎠

⎛ 9 −2 1 ⎞ ⎛1 0 1⎞
⎜ ⎟ ⎜ ⎟
2.3.25. A = ⎜ 2 −5 0 ⎟ . 2.3.26. A = ⎜ 4 −13 4 ⎟ .
⎜ 4 −2 1 ⎟ ⎜5 −3 3 ⎟⎠
⎝ ⎠ ⎝

39
⎛ 1 0 1⎞ ⎛ 2 −2 13 ⎞
⎜ ⎟ ⎜ ⎟
2.3.27. A = ⎜ −3 9 3 ⎟ . 2.3.28. A = ⎜ 0 −1 4 ⎟ .
⎜ 0 2 3⎟ ⎜ 2 −2 1 ⎟
⎝ ⎠ ⎝ ⎠
⎛ 2 −10 7 ⎞ ⎛ 1 2 1⎞
⎜ ⎟ ⎜ ⎟
2.3.29. A = ⎜ 0 29 7 ⎟ . 2.3.30. A = ⎜ 0 5 1 ⎟ .
⎜ 1 −5 4 ⎟ ⎜ 1 2 3⎟
⎝ ⎠ ⎝ ⎠

2.4. Find the rank of matrix.


⎛ 0 1 −1 2 2⎞ ⎛1 1 −2 1 3 ⎞
⎜ ⎟ ⎜ ⎟
−2 1 3 −1 1⎟ −1 2 1 −1 −2 ⎟
2.4.1. ⎜ . 2.4.2. ⎜ .
⎜ 3 −2 1 1 −2 ⎟ ⎜3 0 2 1 −1 ⎟
⎜⎜ ⎟ ⎜⎜ ⎟
⎝1 0 3 2 1 ⎟⎠ ⎝3 3 1 1 −1 ⎟⎠
⎛ −2 2 1 −1 −3 ⎞ ⎛ −1 −1 1 −1 0 ⎞
⎜ ⎟ ⎜ ⎟
3 1 0 1 0⎟ 2 1 −1 3 1 ⎟
2.4.3. ⎜ . 2.4.4. ⎜ .
⎜ −1 1 2 2 1⎟ ⎜3 −1 −1 2 −1⎟
⎜⎜ ⎟ ⎜⎜ ⎟
⎝ 0 4 3 2 −2 ⎟⎠ ⎝4 −1 −1 4 0 ⎟⎠
⎛3 1 3 2 5⎞ ⎛1 1 1 −1 2 ⎞
⎜ ⎟ ⎜ ⎟
−1 2 3 1 0⎟ −1 1 −1 −1 0 ⎟
2.4.5. ⎜ 2.4.6. ⎜ .
⎜0 1 −1 2 2 ⎟ ⎜1 −1 2 3 0 ⎟
⎜⎜ ⎟ ⎜⎜ ⎟⎟
⎝2 4 5 5 7 ⎟⎠ ⎝1 1 2 1 2⎠
⎛2 1 −2 1 3 ⎞ ⎛ 3 −1 2 3 5 ⎞
⎜ ⎟ ⎜ ⎟
1 −1 0 2 0 ⎟ −2 1 −1 −2 −3 ⎟
2.4.7. ⎜ . 2.4.8. ⎜ .
⎜ −1 2 1 −1 1 ⎟ ⎜1 2 1 −1 2 ⎟
⎜⎜ ⎟ ⎜⎜ ⎟
⎝2 2 −1 2 4 ⎟⎠ ⎝ 2 2 2 0 4 ⎟⎠
⎛ −1 0 1 −1 0 ⎞ ⎛1 −2 2 −1 4 ⎞
⎜ ⎟ ⎜ ⎟
−2 1 −2 1 −4 ⎟ 3 1 −1 1 −2 ⎟
2.4.9. ⎜ . 2.4.10. ⎜ .
⎜ 2 3 −1 1 1 ⎟ ⎜2 0 3 1 3⎟
⎜⎜ ⎟⎟ ⎜⎜ ⎟⎟
⎝ −1 4 −2 1 −3 ⎠ ⎝6 −1 4 1 5 ⎠
⎛4 1 2 −3 −5 ⎞ ⎛1 −1 2 −1 −3 ⎞
⎜ ⎟ ⎜ ⎟
1 −2 1 0 −1 ⎟ 3 2 −1 1 2 ⎟
2.4.11. ⎜ . 2.4.12. ⎜ .
⎜ −3 −1 −2 1 3 ⎟ ⎜ −2 1 1 2 1⎟
⎜⎜ ⎟ ⎜⎜ ⎟
⎝2 −2 1 −2 −3 ⎟⎠ ⎝ 2 2 2 2 0 ⎟⎠

40
⎛ −2 2 1 −1 −1⎞ ⎛ −3 1 2 −1 3 ⎞
⎜ ⎟ ⎜ ⎟
3 0 −1 1 −1⎟ 2 3 −1 1 2 ⎟
2.4.13. ⎜ 2.4.14. ⎜ .
⎜1 −1 3 2 4 ⎟ ⎜1 −2 3 2 1⎟
⎜⎜ ⎟ ⎜⎜ ⎟
⎝ 2 1 3 2 2 ⎟⎠ ⎝0 2 4 2 6 ⎟⎠
⎛1 3 −1 2 3 ⎞ ⎛1 0 1 −1 2 ⎞
⎜ ⎟ ⎜ ⎟
−2 1 3 1 −1 ⎟ −2 1 0 1 −2 ⎟
2.4.15. ⎜ . 2.4.16. ⎜ .
⎜ 3 −2 −1 −2 1 ⎟ ⎜ 2 1 1 2 3⎟
⎜⎜ ⎟ ⎜⎜ ⎟
⎝ 2 2 1 1 3 ⎟⎠ ⎝1 2 2 2 3 ⎟⎠
⎛3 2 −1 2 3 ⎞ ⎛ −2 3 2 1 3⎞
⎜ ⎟ ⎜ ⎟
−1 0 1 3 2⎟ 1 −2 1 −1 0 ⎟
2.4.17. ⎜ . 2.4.18. ⎜ .
⎜0 −1 2 −4 −6 ⎟ ⎜ 3 1 −1 0 −1⎟
⎜⎜ ⎟ ⎜⎜ ⎟
⎝2 1 2 1 −1 ⎟⎠ ⎝ 2 2 2 0 2 ⎟⎠
⎛1 1 2 1 3⎞ ⎛2 3 2 −1 2 ⎞
⎜ ⎟ ⎜ ⎟
0 1 3 −1 2 ⎟ −1 1 0 2 3⎟
2.4.19. ⎜ . 2.4.20. ⎜ .
⎜ −2 −3 −1 1 0 ⎟ ⎜3 −2 −1 0 −2 ⎟
⎜⎜ −1 ⎟
−1 4 1 5 ⎟⎠ ⎜⎜ 4 2 1 1 3 ⎟⎠

⎝ ⎝
⎛ −3 2 −2 2 4 ⎞ ⎛ 4 −2 3 1 3⎞
⎜ ⎟ ⎜ ⎟
2 −1 1 −1 −2 ⎟ −2 −1 −2 1 −3 ⎟
2.4.21. ⎜ 2.4.22. ⎜
⎜ −1 0 −1 1 2 ⎟ ⎜ 0 1 1 −1 1 ⎟
⎜⎜ ⎟ ⎜⎜ ⎟
⎝ −2 1 −2 2 4 ⎟⎠ ⎝ 2 −2 2 1 1 ⎟⎠
⎛0 2 −1 2 3 ⎞ ⎛ 3 0 2 2 5⎞
⎜ ⎟ ⎜ ⎟
1 −1 2 −1 −3 ⎟ −2 1 −1 0 −2 ⎟
2.4.23. ⎜ 2.4.24. ⎜ .
⎜3 0 1 1 0⎟ ⎜ 3 −2 1 1 4⎟
⎜⎜ ⎟ ⎜⎜ ⎟
⎝4 1 2 2 0 ⎟⎠ ⎝ 4 −1 2 3 7 ⎟⎠
⎛2 2 2 2 8⎞ ⎛ −1 1 2 −2 −4 ⎞
⎜ ⎟ ⎜ ⎟
2 −1 2 −1 2 ⎟ 3 1 −1 3 4 ⎟
2.4.25. ⎜ . 2.4.26. ⎜ .
⎜ −3 1 −2 2 −2 ⎟ ⎜ −1 −1 1 0 −1 ⎟
⎜⎜ 1 2 2 3 8 ⎟⎟ ⎜⎜ 1 1 2 1 −1 ⎟⎟
⎝ ⎠ ⎝ ⎠
⎛ 3 3 0 −1 5 ⎞ ⎛1 1 1 1 4⎞
⎜ ⎟ ⎜ ⎟
−2 1 1 2 2⎟ −2 −1 2 1 0⎟
2.4.27. ⎜ 2.4.28. ⎜ .
⎜ 1 −2 0 −3 −4 ⎟ ⎜ 2 0 −1 1 2⎟
⎜⎜ ⎟ ⎜⎜ ⎟
⎝ 2 2 1 −2 3 ⎟⎠ ⎝1 0 2 3 6 ⎟⎠

41
⎛ −4 2 0 −1 −3 ⎞ ⎛ 1 −1 2 1 3⎞
⎜ ⎟ ⎜ ⎟
3 −1 1 2 5 ⎟ 0 3 1 2 6⎟
2.4.29. ⎜ . 2.4.30. ⎜ .
⎜ 2 2 −2 1 3 ⎟ ⎜ −2 1 −1 −2 −4 ⎟
⎜⎜ ⎟⎟ ⎜⎜ ⎟⎟
⎝ 1 3 −1 2 5 ⎠ ⎝ −1 0 1 −1 −1 ⎠

Micromodule 3
BASIC THEORETICAL INFORMATION.
SYSTEMS OF LINEAR ALGEBRAIC EQUATIONS

System of linear algebraic equations, its consistency, investigation of the


system consistency using matrix rank. Solving the system by the Cra-
mer’s formulas, matrix method, Gauss’ method. Undetermined systems
and their solutions. Homogeneous systems of linear algebraic equations.
Eigenvalues numbers and eigenvectors of matrix.

Literature: [1,chapter 1], [4, part 2, §2.3], [6, chapter 1, §3], [7, chapter
2, §7], [10, chapter 1, §3], [11, chapter 1, §1].

3.1. Basic definitions

Definition 1.18. System of m equations with n unknowns of the form

⎧a11 x1 + a12 x2 + ... + a1n xn = b1 ,



⎪a21 x1 + a22 x2 + ... + a2 n xn = b2 ,
⎨ (1.1)
⎪..................................................
⎪⎩am1 x1 + am 2 x2 + ... + amn xn = bm

is called the system of linear algebraic equations (SLAE), where x1 , x2 ,..., xn


are unknowns; aij ( i = 1, 2,..., m , j = 1, 2, ..., n ) are given coefficients, b1 , b2 , ..., bn
are absolute terms of the system.
To solve the system means to find such values of unknowns x j , which after
substituting them into given system transform all of its equations into identities.
Definition 1.19. SLAE is called homogeneous, if all absolute terms are
equal to zero and is called non-homogeneous, if at least one of them isn’t equal
to zero.
The homogeneous system always has a zero solution.
Definition 1.20. The system of equations is called consistent, if it has at least
one solution, and it is called inconsistent, if it has no solutions at all.

42
Definition 1.21. A consistent system is called determined, if it has only one
solution ( x10 , x20 , … xn0 ), and it is called undetermined, if it has more than one
solution.
Definition 1.20. Matrices
⎛ a11 a12 ... a1n ⎞ ⎛ a11 a12 ... a1n b1 ⎞
⎜ ⎟ ⎜ ⎟
a a22 ... a2 n ⎟ a a22 ... a2 n b2 ⎟
A = ⎜ 21 and B = ⎜ 21
⎜ ... ... ... ... ⎟ ⎜ ... ... ... ... …⎟
⎜⎜ ⎟ ⎜ ⎟
⎝ am1 am 2 ... amn ⎟⎠ ⎜ am1
⎝ am 2 ... amn bm ⎟⎠
are called basic and extended matrices of the system (1.1) correspondingly.

3.2. Methods of solution of systems


of linear algebraic equations

3.2.1. Cramer’s method. The given system be

⎧a11 x1 + a12 x2 + ... + a1n xn = b1 ,



⎪a21 x1 + a22 x2 + ... + a2 n xn = b2 ,
⎨ (1.2)
⎪................................................
⎪⎩an1 x1 + an 2 x2 + ... + ann xn = bn .

This system can be reduced to the form

⎧Δ ⋅ x1 = Δ1 ,

⎪Δ ⋅ x2 = Δ 2 ,

⎪................
⎪⎩Δ ⋅ xn = Δ n ,
where

a11 a12 ... a1n b1 a12 ... a1n


a21 a22 ... a2 n b2 a22 ... a2 n
Δ= , Δ1 = ,
... ... ... ... ... ... ... ...
an1 an 2 ... ann bn an 2 ... ann

a11 b1 ... a1n a11 a12 ... b1


a21 b2 ... a2 n a21 a22 ... b2
Δ2 = , … , Δn = .
... ... ... ... ... ... ... ...
an1 bn ... ann an1 an 2 ... bn

43
Definition 1.23. If Δ ≠ 0 , then SLAE (1.2) has a unique solution, which
can be found by the Cramer’ formulas:
Δ Δ Δ
x1 = 1 , x2 = 2 , … , xn = n .
Δ Δ Δ

Definition 1.24.
If Δ = 0 , and at least one of the determinants Δi ≠ 0 , then the system is
(1.2) inconsistent.
If Δ = 0 and all the determinants Δ 1 , Δ 2 , ...., Δ n are equal to zero, then
the system (1.2) has infinitely many solutions.

3.2.2. Matrix method. Using the concept of matrix product, the system (1.1)
can be written in the form
AX = B,
where
⎛ a11 a12 ... a1n ⎞ ⎛ x1 ⎞ ⎛ b1 ⎞
⎜ ⎟ ⎜ ⎟ ⎜ ⎟
a a22 ... a2 n ⎟ x b
A = ⎜ 21 , X =⎜ 2⎟, B=⎜ 2⎟.
⎜ ... ... ... ... ⎟ ⎜ ... ⎟ ⎜ ... ⎟
⎜⎜ a ⎟
⎟ ⎜ ⎟⎟
⎜ ⎜ ⎟⎟

⎝ m1 am 2 ... amn ⎠ ⎝ xn ⎠ ⎝ bn ⎠
If in SLAE п = т and the determinant of the system Δ( A) ≠ 0 , then unique
solution of the system can be determined by the formula

X = A−1 ⋅ B. (1.3)
3.2.3. Gauss method is used for solving SLAE of an arbitrary form. This
method is based on the elementary transformation of rows of the system, i.e. the
system remains equivalent to the given system, if:
1) interchange two equations;
2) multiply both sides of the equation by the non-zero multiplier;
3) add elements of one equation to the corresponding elements of another
equation multiplied by the same number.
With help of such transformations the system (1.1) can be reduced to the
trapezium form

There can be only two cases when r < n:


1. if at least one of the numbers br +1 , ..., bm isn’t equal to zero. Then the
system (3.1) has no solutions;
2. if all the numbers br +1 , ..., bm are equal to zero. Then the system (3.1)
has infinitely many solutions.

44
⎧ a11 x1 + a12 x2 + ... + a1n xn = b1 ,

⎪ a22 x2 + ... + a2 n xn = b2 ,
⎪ a33 x3 + ... + a3n xn = b3 ,

⎪− − − − − − − − − − − − − − − − − − −
⎨ (1.4)
⎪ ars xs + ... + arn xn = br ,
⎪ 0 = br +1 ,

⎪ − − −−−
⎪⎩ 0 = bm .

If r = n , then the system (1.4) acquires triangular form


⎧a11 x1 + c12 x2 + ... + a1n xn = b1 ,

⎪ a22 x2 + ... + a2 n xn = b2 ,

⎪ ..................................
⎪ amn xn = bm .

The obtained system and then the given system (1.1), has the unique
solution, which is determined in such way. At first let’s find xn from the last
equation, after that from the next to last equation let’s find xn −1 ; climbing onto
the system from the last equation to the first one, we’ll find all the unknowns.
Remark. Since extended matrix one-to-one corresponds to SLAE, then
elementary transformations of the equations are equivalent to the transforma-
tions of rows of the extended matrix. Therefore, further on when solving SLAE
by the Gauss method we’ll operate only with extended matrix.

3.3. Criteria of SLAE consistency


Let we have a system in the form
⎧a11 x1 + a12 x2 + ... + a1n xn = b1 ,

⎪a21 x1 + a22 x2 + ... + a2 n xn = b2 ,

⎪................................................
⎪⎩am1 x1 + am 2 x2 + ... + amn xn = bm

Let’s construct the basic and the extended matrices of the given system
⎛ a11 a12 ... a1n ⎞ ⎛ a11 a12 ... a1n b1 ⎞
⎜ ⎟ ⎜ ⎟
⎜ a21 a22 ... a2 n ⎟ ⎜ a21 a22 ... a2 n b2 ⎟
A= , B= .
⎜ ... ... ... ... ⎟ ⎜ ... ... ... ... … ⎟
⎜⎜ ⎟⎟ ⎜ ⎟
⎜ am1 am 2 ... amn bm ⎟
⎝ am1 am 2 ... amn ⎠ ⎝ ⎠

45
Theorem 1.3. (Kronecker-Capelli) In order to make SLAE consistent, it
is necessary and sufficient, the rank r ( A) of the basic matrix А be equal to
the rank r ( B ) of the extended matrix В.
If rank of the basic matrix equals the rank of the extended matrix and is
equal to the number of unknowns, i.e. r ( A) = r ( B ) then the system has a
unique solution.
If rank of the basic matrix equals the rank of the extended matrix, but it is
less than the number of unknowns, i.e. r ( A) = r ( B ) < n then the system
has infinitely many solutions.
If r ( B ) > r ( A) , then the system is inconsistent

3.4. Homogeneous system of linear algebraic equations

Let’s consider homogeneous system of the n–th order


⎧a11 x1 + a12 x2 + ... + a1n xn = 0;

⎪a21 x1 + a22 x2 + ... + a2 n xn = 0;
⎨ (1.5)
⎪..........................................
⎪⎩an1 x1 + an 2 x2 + ... + ann xn = 0.

Since the extended matrix В of homogeneous system has no additional non-


zero minors (in comparison with matrix Α ), then r ( A) = r ( B ) , i.e. homogeneous
system is always consistent.
If r ( A) = n or, that is the same, Δ( A) ≠ 0 , then the system has a unique
trivial solution x1 = x2 = ... = xn = 0 , of what it is easily to check it, for instance,
applying Cramer’s rule to the system.
If r ( A) = k < n , then the homogeneous system (1.5) has infinitely many
solutions, which are determined, for example, by the Gauss` method.
Remark. Universal method of solving systems of linear algebraic equations
both determined and undetermined, homogeneous and non- homogeneous is the
Gauss method.

3.5. Eigenvalues and eigenvectors of matrix


Definition 1.25. Any non-zero column vector X , which satisfies the con-
dition
A⋅ X = λ ⋅ X ,
where λ is a real number, is called eigenvector of the matrix A , and number λ
is the egenvalue of the matrix A , which corresponds to the vector X . The
vector X is ambiguously determined (accurate to non-zero scalar multiplier).

46
Egenvalues of matrix A are the roots of its characteristic equation
Δ( A − λE ) = 0 , or in the unfolded form (for example of the third order matrix)

a11 − λ a12 a13


a21 a22 − λ a23 = 0 .
a31 a32 a33 − λ

⎛ x1 ⎞
⎜ ⎟
Eigenvector X = ⎜ x2 ⎟ , which corresponds to the egenvalue λ , is deter-
⎜x ⎟
⎝ 3⎠
mined from the system of equations

⎧(a11 − λ) x1 + a12 x2 + a13 x3 = 0,



⎨a21 x1 + (a22 − λ ) x2 + a23 x3 = 0,
⎪a x + a x + (a − λ) x = 0.
⎩ 31 1 32 2 33 3

When λ is multiple root of the characteristic equation, the system can define
several eigenvectors.

Micromodule 3
EXAMPLES OF PROBLEMS SOLUTION

Example 1. Solve the system of equations

⎧2 x1 + x2 − 3 x3 = 3,

⎨ x1 + x2 + 2 x3 = 2,
⎪− x + x + x = 0.
⎩ 1 2 3

by the matrix method and by the Cramer’s formulas.


⎛ 2 1 −3 ⎞
⎜ ⎟
Solution. The Matrix method. A = ⎜ 1 1 2 ⎟ is basic matrix of the
⎜ −1 1 1 ⎟
⎝ ⎠
system. Using properties of determinants, we’ll calculate the determinant of the
basic matrix:
2 1 −3 2 3 −1 0 3 −1
Δ ( A) = 1 1 2 = 1 2 3 = 0 2 3 =
−1 1 1 −1 0 0 −1 0 0
= 0 − 9 + 0 − (2 + 0 + 0) = −11 ≠ 0

47
As Δ( A) ≠ 0 , then there exists an inverse matrix. We find it
1 2 1 −3 1 −3
A11 = = −1 , A21 = − = −4 , A31 = = 5,
1 1 1 1 1 2
1 2 2 −3 2 −3
A12 = − = −3 , A22 = = −1 , A32 = − = −7 ,
−1 1 −1 1 1 2
1 1 2 1 2 1
A13 = = 2 , A23 = − = −3 , A33 = =1.
−1 1 −1 1 1 1
The inverse matrix has such form
⎛ −1 −4 5 ⎞
−1 −1 ⎜ ⎟
A = −3 −1 −7 ⎟ .
11 ⎜⎜ ⎟
⎝ 2 −3 1 ⎠
Then using the formula (1.3) we obtain

⎛ x1 ⎞ ⎛ −1 −4 5 ⎞ ⎛ 3 ⎞ ⎛ −1 ⋅ 3 − 4 ⋅ 2 + 5 ⋅ 0 ⎞
⎜ ⎟ 1 ⎜ ⎟⎜ ⎟ 1⎜ ⎟
X = ⎜ x2 ⎟ = − ⎜ −3 −1 −7 ⎟ ⎜ 2 ⎟ = − ⎜ −3 ⋅ 3 − 1 ⋅ 2 − 7 ⋅ 0 ⎟ =
⎜x ⎟ 11 ⎜ 2 −3 1 ⎟ ⎜ 0 ⎟ 11 ⎜ 2 ⋅ 3 − 3 ⋅ 2 + 1⋅ 0 ⎟
⎝ 3⎠ ⎝ ⎠⎝ ⎠ ⎝ ⎠
⎛ −11⎞ ⎛ 1 ⎞
1 ⎜ ⎟ ⎜ ⎟
=− −11⎟ = ⎜ 1 ⎟ .
11 ⎜⎜ ⎟ ⎜ ⎟
⎝ 0 ⎠ ⎝ 0⎠
Thus, x1 = x2 = 1, x3 = 0 is a solution of the given system.
Cramer’s method. Now let’s use Cramer’s formulas:

3 1 −3 2 3 −3
Δ = −11, Δ1 = 2 1 2 = −11 , Δ 2 = 1 2 2 = −11 ,
0 1 1 −1 0 1
2 1 3
Δ Δ Δ
Δ3 = 1 1 2 = 0 ; x1 = 1 = 1 , x2 = 2 = 1 , x3 = 3 = 0.
Δ Δ Δ
−1 1 0
Example 2. Solve the system of equations
⎧ x1 + 2 x2 + 3 x3 = 12,

⎨ 2 x1 + x2 + 2 x3 = 9,
⎪⎩ −3x1 + x2 + 4 x3 = 10,

using Gauss` method.

48
Solution. We write down the extended matrix of the given system

⎛ 1 2 3 12 ⎞
B=⎜ 2 1 2 9⎟
⎜⎜ ⎟⎟
⎝ −3 1 4 10 ⎠

and, performing elementary transformations of the rows, we reduce it to the


trapezium form:

⎛ 1 2 3 12 ⎞ ⎛ 1 2 3 12 ⎞ ⎛ 1 2 3 12 ⎞
⎜ 2 1 2 9 ⎟ ~ ⎜ 0 −3 −4 −15 ⎟ ~ ⎜ 0 −3 −4 −15 ⎟ ~
⎜⎜ ⎟⎟ ⎜⎜ ⎟⎟ ⎜⎜ ⎟
⎝ −3 1 4 10 ⎠ ⎝ 0 7 13 46 ⎠ ⎝ 0 1 5 16 ⎟⎠
⎛ 1 2 3 12 ⎞ ⎛ 1 2 3 12 ⎞
~ ⎜ 0 1 5 16 ⎟ ~ ⎜ 0 1 5 16 ⎟ .
⎜⎜ ⎟⎟ ⎜⎜ ⎟
⎝ 0 −3 −4 −15 ⎠ ⎝ 0 0 11 33 ⎟⎠

The last row corresponds to the equation 11x3 = 33 , from which x3 = 3 .


Then we write down the equations:
x2 + 5 x3 = 16 , thus x2 = 16 − 5 x3 = 16 − 15 = 1 , x2 = 1 .
x1 + 2 x2 + 3x3 = 12 , thus x1 = 12 − 2 x2 − 3 x3 = 12 − 2 − 9 = 1 .

Hence, the solution of the system is: x1 = 1 , x2 = 1 , x3 = 3 .


Example 3. Investigate the system of equations

⎧ x1 + 2 x2 + 3x3 − x4 = 1,

⎪3x1 + 2 x2 + x3 − x4 = 1,

⎪2 x1 + 3 x2 + x3 + x4 = 1,
⎪⎩2 x1 + 2 x2 + 2 x3 − x4 = 1.

on consistency and in the case of consistency find the solution.


Solution. We write down the extended matrix of the given system

⎛1 2 3 −1 1⎞
⎜ ⎟
3 2 1 −1 1⎟
B=⎜ .
⎜2 3 1 1 1⎟
⎜ ⎟
⎜2 2 2 −1 1⎟⎠

We find the rank of this matrix (and simultaneously the rank of the basic
matrix), performing the elementary transformations of the rows:

49
⎛1 2 3 −1 1⎞ ⎛ 1 2 3 −1 1 ⎞ ⎛ 1 2 3 −1 1⎞
⎜ ⎟ ⎜ ⎟ ⎜ ⎟
⎜3 2 1 −1 1⎟ ⎜ 0 −4 −8 2 −2 ⎟ ⎜ 0 −2 −4 1 −1⎟
~ ~ ~
⎜2 3 1 1 1⎟ ⎜ 0 −1 −5 3 −1 ⎟ ⎜ 0 −1 −5 3 −1⎟
⎜ ⎟ ⎜ ⎟ ⎜ ⎟
⎜2 2 2 −1 1⎟⎠ ⎜⎝ 0 −1 1 −2 0 ⎟⎠ ⎜⎝ 0 −1 1 −2 0 ⎟⎠

⎛ 1 2 3 −1 1 ⎞ ⎛ 1 2 3 −1 1 ⎞
⎜ ⎟ ⎜ ⎟
0 0 6 −5 1 ⎟ ⎜ 0 −1 −5 3 −1⎟
~⎜ ~ .
⎜ 0 −1 −5 3 −1⎟ ⎜ 0 0 6 −5 1 ⎟
⎜ ⎟ ⎜ ⎟
⎜ 0 0 6 −5 1 ⎟⎠ ⎜⎝ 0 0 0 0 0 ⎟⎠

From the form of the last matrix we make the conclusion that the rank of a
basic matrix equals 3. The rank of an extended matrix also equals 3, since the
rank of an extended matrix isn’t less than rank of a basic matrix, and the last
row, which contains only zero elements, doesn’t increase a rank (all minors of
the fourth order are equal to zero).
Conclusion. The given system has infinitely many solutions. We find these
solutions the following way. According to the form of the last matrix we write
down the system
⎧ x1 + 2 x2 + 3x3 − x4 = 1,

⎨ − x2 − 5 x3 + 3 x4 = −1,
⎪ 6 x3 − 5 x4 = 1,

which is equivalent to the given system.
Moving step by step from the last equation to the first one, we find:

1 + 5 x4 1 + 5 x4 1 − 7 x4
x3 = ; x2 = −5 x3 + 3 x4 + 1 = −5 ⋅ + 3 x4 + 1 = ;
6 6 6

1 − 7 x4 1 + 5 x4 1 + 5 x4
x1 = 1 − 2 x2 − 3x3 + x4 = 1 − 2 ⋅ −3 + x4 = .
6 6 6
1 + 5t 1 − 7t 1 + 5t
We can write the equation in such form: x1 = , x2 = , x3 = ,
6 6 6
x4 = t , where t ∈ R .
Example 4. Find all solutions of the system

⎧3x1 + 2 x2 − x3 = 0,

⎨2 x1 − x2 + 3 x3 = 0,
⎪ x + 3x − 4 x = 0.
⎩ 1 2 3

50
Solution. We write down the extended matrix of the given system

⎛ 3 2 −1 0 ⎞
⎜ ⎟
B = ⎜ 2 −1 3 0 ⎟ .
⎜ 1 3 −4 0 ⎟
⎝ ⎠
We find the rank of this matrix (and simultaneously the rank of a basic
matrix), performing the elementary transformations of rows:

⎛ 3 2 −1 0 ⎞ ⎛ 1 3 −4 0 ⎞ ⎛ 1 3 −4 0 ⎞ ⎛1 3 −4 0 ⎞
⎜ ⎟~⎜ ⎟ ~⎜ ⎟~ .
⎜ 2 −1 3 0 ⎟ ⎜ 2 −1 3 0 ⎟ ⎜ 0 −7 11 0 ⎟ ⎜⎝ 0 −7 11 0 ⎟⎠
⎜ 1 3 − 4 0 ⎟ ⎜ 1 3 −4 0 ⎟ ⎜ 0 0 0 0 ⎟
⎝ ⎠ ⎝ ⎠ ⎝ ⎠

1 3
As the angular minor is not equal to zero, i.e. Μ ( ) =
2
= −7 ≠ 0 , the
0 −7
rank equals 2 (2<3). Thus the system has infinitely many solutions.
By the last matrix we write down the equivalent system

⎧ x1 +3x2 − 4 x3 = 0,

⎩ -7x2 + 11x3 = 0.

The values x3 = 7t , x2 = 11t , where t ∈ R , satisfy the second equation.


And now from the first equation we obtain x1 = 4 x3 − 3x2 = 28t − 33t = −5t .
Answer: x1 = −5t , x2 = 11t , x3 = 7t , where t ∈ R .
Example 5. Find egenvalues and eigenvectors of the matrix

⎛ 3 −1 1 ⎞
A = ⎜ −1 5 −1⎟ .
⎜ ⎟
⎜ 1 −1 3 ⎟
⎝ ⎠
Solution. Let’s solve the characteristic equation for the given matrix
3−λ −1 1
−1 5 − λ −1 = 0 ,
1 −1 3 − λ
(3 − λ )(5 − λ)(3 − λ) + 1 + 1 − (5 − λ ) − (3 − λ ) − (3 − λ) = 0,
(λ − 3)(λ − 2)(λ − 6) = 0, λ1 = 2, λ 2 = 3, λ3 = 6.
Thus, the matrix A has three egenvalues. Now let’s find eigenvectors,
substituting in turn the values λ1 , λ 2 , λ3 into the system

51
⎧(3 − λ) x1 − x2 + x3 = 0,

⎨− x1 + (5 − λ ) x2 − x3 = 0,
⎪ x − x + (3 − λ ) x = 0.
⎩ 1 2 3

The eigenvector x1 corresponds to the egenvalue λ1 = 2 , which coordinates


satisfy the system

⎧(3 − 2) x1 − x2 + x3 = 0, ⎧ x1 − x2 + x3 = 0,
⎪ ⎪
⎨− x1 + (5 − 2) x2 − x3 = 0, or ⎨− x1 + 3x2 − x3 = 0,
⎪ x − x + (3 − 2) x = 0, ⎪ x − x + x = 0.
⎩ 1 2 3 ⎩ 1 2 3
⎛1⎞
⎧2 x2 = 0, ⎜ ⎟
Hence ⎨ and the eigenvector X 1 = C1 ⎜ 0 ⎟ .
x
⎩ 1 = − x3 ⎜ −1⎟
⎝ ⎠
⎛ 1⎞ ⎛1⎞
⎜ ⎟ ⎜ ⎟
By the analogy the eigenvectors X 2 = C2 ⎜ 1⎟ and X 3 = C3 ⎜ −2 ⎟ are found,
⎜ 1⎟ ⎜1⎟
⎝ ⎠ ⎝ ⎠
where C1 , C2 , C3 are non-zero real numbers.

Micromodule 3
CLASS AND HOME ASSIGNMENT

Solve the system of equations:


а) by the matrixм method; b) by the Cramer’s formulas.

⎧ x1 + 3 x2 − x3 = 4, ⎧2 x1 + x2 + x3 = 2,
⎪ ⎪
1. ⎨− x1 − x2 + 3x3 = 6, 2. ⎨3x1 + 3 x2 + 2 x3 = 5,
⎪2 x + 6 x − x = 11. ⎪5 x + 3x + 4 x = 3.
⎩ 1 2 3 ⎩ 1 2 3

Solve the system of equations using Gauss` method.


⎧ x1 + 2 x2 + x3 = 5, ⎧2 x1 − x2 + x3 − x4 = 1,
⎪ ⎪
3. ⎨ 2 x1 + x2 + 3x3 = 9, 4. ⎪3 x1 + 2 x2 − x3 − x4 = 0,
⎪−3 x + x + 4 x = 6. ⎨
⎩ 1 2 3 ⎪− x1 − 2 x2 + 2 x4 = 1,
⎪⎩ x1 − 3 x2 + x3 + x4 = 2.
⎧ x1 + 2 x2 + 2 x3 + x4 = 0,

5. ⎨2 x1 + 5 x2 + 5 x3 + 2 x4 = 1,
⎪3x + 7 x + 7 x + 3x = 2.
⎩ 1 2 3 4

52
Answers
1. x1 = 1, x2 = 2, x3 = 3. 2. x1 = 1 , x2 = 2 , x3 = −2 . 3. x1 = 1 , x2 = 1 ,
x3 = 2 . 4. x1 = 3 + 2t , x2 = −6 + 8t , x3 = −3 + 13t , x4 = t , where t ∈ R .
9 9 9

Micromodule 3
SELF-TEST ASSIGNMENTS

3.1. Solve the system of: а) by the matrix method; b) by the Cramer’s
formulas.
⎧ x1 + x2 − x3 = 0, ⎧2 x1 + x2 + x3 = 2,
⎪ ⎪
3.1.1. ⎨2 x1 + x2 + 2 x3 = 10, 3.1.2. ⎨ x1 + 2 x2 + x3 = 3,
⎪ x − 3 x + x = −2. ⎪ x + x + 2 x = −1.
⎩ 1 2 3 ⎩ 1 2 3

⎧ x1 + 2 x2 + 3 x3 = −1, ⎧3x1 + 4 x2 − x3 = −2,


⎪ ⎪
3.1.3 ⎨2 x1 + x2 − 4 x3 = −3, 3.1.4. ⎨5 x1 + 3 x2 − 4 x3 = −2,
⎪3x + 2 x + x = 1. ⎪4 x + 2 x + 3x = 5.
⎩ 1 2 3 ⎩ 1 2 3

⎧ 2 x1 − x2 + 3x3 = −4, ⎧3 x1 − 2 x2 + 4 x3 = −17,


⎪ ⎪
3.1.5. ⎨ x1 + 2 x2 − 4 x3 = 19, 3.1.6. ⎨4 x1 + 3 x2 − 2 x3 = 18,
⎪−3x + 4 x + 2 x = 3. ⎪3 x + x + 3x = −7.
⎩ 1 2 3 ⎩ 1 2 3

⎧ x1 + x2 − x3 = 4, ⎧ x1 + 2 x2 − 2 x3 = −3,
⎪ ⎪
3.1.7. ⎨2 x1 + x2 + 2 x3 = 2, 3.1.8. ⎨2 x1 + x2 + 3x3 = 8,
⎪3x − x + x = 0. ⎪3x − 4 x + x = 5.
⎩ 1 2 3 ⎩ 1 2 3

⎧ x1 + 3x2 − x3 = 0, ⎧2 x1 + 3x2 − 3x3 = 2,


⎪ ⎪
3.1.9. ⎨2 x1 − 4 x2 + 4 x3 = 6, 3.1.10. ⎨ x1 − 4 x2 + 5 x3 = 2,
⎪3x + 2 x + x = 4. ⎪3x + 4 x − 3x = 4.
⎩ 1 2 3 ⎩ 1 2 3

⎧3x1 + 4 x2 + 3 x3 = 3, ⎧ x1 + 2 x3 = 7,
⎪ ⎪
3.1.11. ⎨4 x1 + 5 x2 − 3 x3 = 4, 3.1.12. ⎨ x1 + 3 x2 = 7,
⎪2 x + 3x − 4 x = 2. ⎪2 x − 2 x + x = 1.
⎩ 1 2 3 ⎩ 1 2 3

⎧3x1 + x2 + 2 x3 = −2, ⎧3 x1 − 2 x2 + x3 = 0,
⎪ ⎪
3.1.13. ⎨ x1 + 3x2 + 2 x3 = 2, 3.1.14. ⎨7 x1 + x2 + 4 x3 = 0,
⎪2 x + x − x = −1. ⎪ x2 + 5 x3 = 4.
⎩ 1 2 3 ⎩

53
⎧ x1 − x2 + x3 = 1, ⎧2 x1 + x2 + x3 = 2,
⎪ ⎪
3.1.15. ⎨2 x1 + 2 x3 = 4, 3.1.16. ⎨3x1 + 3 x2 + 2 x3 = 5,
⎪3x + 2 x = 5. ⎪ x − x + x = −3.
⎩ 1 2 ⎩ 1 2 3

⎧ x1 + 2 x2 + x3 = 2, ⎧3 x1 + x2 + x3 = 4,
⎪ ⎪
3.1.17. ⎨2 x1 + 3 x3 = −13, 3.1.18. ⎨ x1 + 3 x2 + x3 = 6,
⎪3x − 3x − 4 x = 0. ⎪ x + x + 3 x = 0.
⎩ 1 2 3 ⎩ 1 2 3

⎧ x1 + 3x2 = 6, ⎧3x1 + 3 x2 + 4 x3 = 4,
⎪ ⎪
3.1.19. ⎨ x1 − x2 − x3 = 0, 3.1.20. ⎨5 x1 − 7 x2 + 8 x3 = 20,
⎪2 x + 2 x + 3x = −2. ⎪4 x + 5 x − 7 x = −8.
⎩ 1 2 3 ⎩ 1 2 3

⎧ x1 + 2 x2 + 3 x3 = 4, ⎧ x1 − 2 x2 + x3 = 4,
⎪ ⎪
3.1.21. ⎨2 x1 − x2 + 2 x3 = 1, 3.1.22. ⎨3 x1 + x2 + 2 x3 = 3,
⎪ x + 3 x + x = 4. ⎪3 x + 8 x − 3x = 8.
⎩ 1 2 3 ⎩ 1 2 3

⎧2 x1 − 3x2 + x3 = 8, ⎧ x1 − 2 x2 + 3 x3 = 3,
⎪ ⎪
3.1.23. ⎨3 x1 + x2 − 2 x3 = 7, 3.1.24. ⎨2 x1 + 3 x2 − x3 = 13,
⎪2 x + 5 x − 3 x = −12. ⎪3x − x − 2 x = 8.
⎩ 1 2 3 ⎩ 1 2 3

⎧3x1 + x2 − 2 x3 = −2, ⎧3 x1 + 2 x2 + x3 = 7,
⎪ ⎪
3.1.25. ⎨ x1 + 2 x2 + 3 x3 = 7, 3.1.26. ⎨5 x1 + x2 − x3 = 4,
⎪2 x + 3x + x = 1. ⎪2 x − 7 x − 3x = −11.
⎩ 1 2 3 ⎩ 1 2 3

⎧ x1 + 2 x2 + x3 = 4, ⎧2 x1 + 3x2 + x3 = 1,
⎪ ⎪
3.1.27. ⎨3x1 − x2 + 4 x3 = 3, 3.1.28. ⎨5 x1 − 2 x2 − 3x3 = 5,
⎪2 x + 5 x + 6 x = 6. ⎪3x + x + x = 7.
⎩ 1 2 3 ⎩ 1 2 3

⎧ x1 + x2 + x3 = 2, ⎧ x1 − 2 x2 + 2 x3 = −2,
⎪ ⎪
3.1.29. ⎨2 x1 + x2 − 2 x3 = −3, 3.1.30. ⎨2 x1 + x2 + 3x3 = 10,
⎪3x − 2 x + x = 7. ⎪3 x − 4 x + 6 x = 0.
⎩ 1 2 3 ⎩ 1 2 3

3.2. Investigate SLAE given in the extended matrix on consistency and in the
case of consistency find its general solution.
⎛ 1 1 −3 0 2 ⎞ ⎛ 1 −2 0 1 −3 ⎞
⎜ ⎟ ⎜ ⎟
3.2.1. ⎜
1 −1 1 2 −1⎟
. 3.2.2. ⎜ 3 −1 −2 0 1⎟
.
⎜ 3 3 −9 0 6 ⎟ ⎜ 2 1 −2 −2 4⎟
⎜ ⎟ ⎜ ⎟
⎜ 1 2 −5 −1 3 ⎟ ⎜ 1 3 −2 −2 7 ⎟⎠
⎝ ⎠ ⎝

54
⎛1 1 1 1 1 1⎞ ⎛2 3 2 7⎞
⎜ ⎟ ⎜ ⎟
1 1 4 3 5 2⎟ 5 4 3 12 ⎟
3.2.3. ⎜ . 3.2.4. ⎜ .
⎜2 1 9 2 7 3⎟ ⎜1 1 1 3⎟
⎜ ⎟ ⎜ ⎟
⎜4 4 7 6 8 5 ⎟⎠ ⎜7 1 1 9 ⎟⎠
⎝ ⎝
⎛1 2 3 4 0⎞ ⎛1 2 3 14 ⎞
⎜ ⎟ ⎜ ⎟
7 14 20 27 0⎟ 3 2 1 10 ⎟
3.2.5. ⎜ . 3.2.6. ⎜ .
⎜ 5 10 16 19 −2 ⎟ ⎜1 1 1 6⎟
⎜ ⎟ ⎜ ⎟
⎜ 3 5 6 13 5 ⎟⎠ ⎜1 1 0 3 ⎟⎠
⎝ ⎝
⎛ 2 −1 2 −1 0 ⎞ ⎛1 2 1 0 8⎞
⎜ ⎟ ⎜ ⎟
3.2.7. ⎜ 1 5 4 3 1 ⎟ . 3.2.8. ⎜ 0 1 3 1 15 ⎟ .
⎜4 0 1 1 11 ⎟
⎜ 5 3 8 1 1⎟ ⎜ ⎟
⎝ ⎠ ⎜1 1 0 5 23 ⎟⎠

⎛ 1 −1 1 −1 −2 ⎞ ⎛2 7 3 1 5 ⎞
⎜ ⎟ ⎜ ⎟
3.2.9. ⎜ 1 2 −2 −1 −5 ⎟ . 3.2.10. ⎜ 1 3 5 −2 3 ⎟ .
⎜ 2 −1 −3 2 −1 ⎟ ⎜ 1 5 −9 8 1 ⎟
⎜ ⎟ ⎜ ⎟
⎜ 1 2 3 −6 −10 ⎟ ⎜ 5 18 4 5 12 ⎟
⎝ ⎠ ⎝ ⎠
⎛3 2 4 4 5 2⎞ ⎛ 5 1 −3 −6 ⎞
⎜ ⎟ ⎜ ⎟
3.2.11. ⎜
7 5 9 8 9 3⎟
. 3.2.12. ⎜ 2 −5 7 9 ⎟ .
⎜5 3 7 9 4 3⎟ ⎜ 4 2 −4 −7 ⎟
⎜ ⎟ ⎜ ⎟
⎜6 5 7 5 −5 −3 ⎟⎠ ⎜ 5 −2 2 1 ⎟
⎝ ⎝ ⎠
⎛ 7 8 −5 6 − 4 5 ⎞ ⎛1 3 3 5 −1⎞
⎜ ⎟
3.2.13. ⎜⎜ 6 7 −4 4 3 5 ⎟⎟
3.2.14. ⎜ 2 6 5 6 1 ⎟.
⎜ 5 9 7 5 7 3⎟ ⎜3 7 4 8 2⎟
⎝ ⎠ ⎜ ⎟
⎜3 5 1 9 1 ⎟⎠

⎛2 3 4 3 0 ⎞ ⎛ 2 −1 1 −1 1 ⎞
⎜ ⎟ ⎜ ⎟
3.2.15. ⎜ 4 6 9 8 −3 ⎟ . 3.2.16. ⎜ 2 −1 0 −3 2 ⎟ .
⎜6 9 9 4 8 ⎟ ⎜ 3 0 −1 1 −3 ⎟
⎝ ⎠ ⎜ ⎟
⎜ 2 2 −2 5 −6 ⎟
⎝ ⎠
⎛5 3 4 −2 3 1⎞ ⎛ 1 −2 3 4 4 ⎞
⎜ ⎟ ⎜ ⎟
3.2.17. ⎜
8 5 5 −4 4 2⎟ 3.2.18. ⎜ 0 1 −1 1 −3 ⎟ .
⎜7 4 7 −3 7 −1⎟ ⎜ 1 3 0 −3 1 ⎟
⎜ ⎟ ⎜ ⎟
⎜4 3 −1 −3 −2 4 ⎟⎠ ⎜ 0 −7 3 1 −3 ⎟
⎝ ⎝ ⎠

55
⎛1 2 3 −2 4 ⎞ ⎛ 1 −3 2 −1 11 ⎞
⎜ ⎟ ⎜ ⎟
2 −1 2 −3 1 ⎟ 3.2.20. ⎜ 2 1 −3 4 −5 ⎟ .
3.2.19. ⎜ . ⎜ 2 −13 11 −8 49 ⎟
⎜1 3 1 2 4⎟ ⎜ ⎟
⎜ ⎟ ⎜ 4 9 −13 14 −37 ⎟
⎜4 4 6 −3 9 ⎟⎠ ⎝ ⎠

⎛ 1 2 −3 4 −1 2 ⎞ ⎛3 2 1 1 7 ⎞
⎜ ⎟ ⎜ ⎟
3.2.21. ⎜ 5 1 3 4 2 12 ⎟ . 3.2.22. ⎜ 5 1 −1 2 4 ⎟ .
⎜ 7 5 −3 12 0 16 ⎟ ⎜ 1 −3 −2 −5 2 ⎟
⎜ ⎟ ⎜ ⎟
⎜ 2 −5 12 8 5 6 ⎟ ⎜ 2 −7 −3 −4 −11⎟
⎝ ⎠ ⎝ ⎠
⎛1 1 1 1 0 ⎞ ⎛ 1 −2 3 2 −2 ⎞
⎜ ⎟ ⎜ ⎟
3.2.23. ⎜ 2 1 −2 −2 1 ⎟ 3.2.24. ⎜
2 1 −2 3 10 ⎟
.
⎜ 3 −2 1 −3 13 ⎟ ⎜ 3 −4 1 6 0⎟
⎜ ⎟ ⎜ ⎟
⎜ 1 −2 −2 1 −3 ⎟ ⎜1 2 2 1 9 ⎟⎠
⎝ ⎠ ⎝
⎛1 −2 1 1 1 ⎞ ⎛ 1 −2 3 −4 4 ⎞
⎜ ⎟ ⎜ ⎟
3.2.25. ⎜1 −2 1 −1 −1⎟ . 3.2.26. ⎜ 0 1 −1 1 −3 ⎟ .
⎜ 1 3 0 −3 1 ⎟
⎜1 −2 1 5 5 ⎟ ⎜ ⎟
⎝ ⎠ ⎜ 0 −7 3 1 −3 ⎟
⎝ ⎠
⎛1 2 3 −1 1⎞ ⎛ 2 1 −1 −1 1 1 ⎞
⎜ ⎟ ⎜ ⎟
3.2.27. ⎜
3 2 1 −1 1⎟
. 3.2.28. ⎜ 1 −1 1 1 −2 0 ⎟ .
⎜2 ⎜ 3 3 −3 −3 4 2 ⎟
3 1 1 1⎟ ⎜ ⎟
⎜ ⎟ ⎜ 4 5 −5 −5 7 3 ⎟
⎜2 2 2 −1 1⎟⎠ ⎝ ⎠

⎛ 3 4 −5 7 1⎞ ⎛ 3 1 1 −2 3 4 ⎞
⎜ ⎟ ⎜ ⎟
3.2.29. ⎜ 2 −3 3 −2 −1⎟ .
3.2.30. ⎜ 2 3 −2 1 −4 5 ⎟ .
⎜ 4 11 −13 16 2⎟ ⎜ 1 2 3 4 1 3⎟
⎜ ⎟ ⎜ ⎟
⎜ 7 −2 1 3 3 ⎟⎠ ⎜ 1 −2 3 −3 7 1 ⎟
⎝ ⎝ ⎠

3.3. Solve the homogeneous system of linear algebraic equations.

⎧ x2 − x3 + 2 x4 = 0, ⎧ x1 + x2 − 2 x3 + x4 = 0,
⎪ ⎪
−2 x + x + 3x3 − x4 = 0, − x + 2 x2 + x3 − x4 = 0,
3.3.1. ⎪⎨ 1 2 3.3.2. ⎪⎨ 1
⎪3x1 − 2 x2 + x3 + x4 = 0, ⎪3 x1 + x3 + x4 = 0,
⎪⎩ x1 + 3x3 + 2 x4 = 0. ⎪⎩3 x1 + 3 x2 + x3 + x4 = 0.

56
⎧−2 x1 + 2 x2 + x3 − x4 = 0, ⎧ x1 − x2 + 2 x3 + x4 = 0,
⎪ ⎪
3.3.3. ⎪⎨ 3x1 + x2 + x4 = 0, 5 x − 2 x2 + 5 x3 + 4 x4 = 0,
3.3.4. ⎪⎨ 2
− + + + 2 x4 = 0,
⎪ 1 x x 2 2 x3 ⎪−2 x1 + x2 − x3 − 2 x4 = 0,
⎪⎩ 4 x2 + 3x3 + 2 x4 = 0. ⎪−2 x1 + x2 − x3 − 2 x4 = 0.

⎧3 x1 + x2 + 3 x3 + 2 x4 = 0, ⎧ x1 + x2 + x3 − x4 = 0,
⎪ ⎪
− x + 2 x2 + 3 x3 + x4 = 0, − x + x − x − x = 0,
3.3.5. ⎪⎨ 1 3.3.6. ⎪⎨ 1 2 3 4
⎪ x2 − x3 + 2 x4 = 0, ⎪ x1 − x2 + 2 x3 + 3 x4 = 0,
⎪2 x1 + 4 x2 + 5 x3 + 5 x4 = 0. ⎪⎩ x1 + x2 + 2 x3 + x4 = 0.

⎧2 x1 + x2 − 2 x3 + x4 = 0, ⎧3 x1 − x2 + 2 x3 + 3 x4 = 0,
⎪ ⎪
x −x + x4 = 0, −2 x + x − x − 2 x4 = 0,
3.3.7. ⎪⎨ 1 2 3.3.8. ⎪⎨ 1 2 3

⎪ 1 x + 2 x2 + x3 − x4 = 0, ⎪ x1 + 2 x2 + x3 − x4 = 0,
⎪⎩2 x1 + 2 x2 − x3 + 2 x4 = 0. ⎪⎩2 x1 + 2 x2 + 2 x3 = 0.
⎧ − x1 + x3 − x4 = 0, ⎧ x1 − 2 x2 + 2 x3 − x4 = 0,
⎪ ⎪
3.3.9. ⎪⎨−2 x1 + x2 − 2 x3 + x4 = 0, 3 x + x − x + x = 0,
3.3.10. ⎪⎨ 1 2 3 4
⎪ 2 x1 + 3 x2 − x3 + x4 = 0, ⎪2 x1 + 3x3 + x4 = 0,
⎪⎩ − x1 + 4 x2 − 2 x3 + x4 = 0. ⎪6 x1 − x2 + 4 x3 + x4 = 0.

⎧ 4 x1 + x2 + 2 x3 − 3 x4 = 0, ⎧ x1 − x2 + 2 x3 − x4 = 0,
⎪ ⎪
x − 2 x2 + x3 = 0, 3x + 2 x2 − x3 + x4 = 0,
3.3.11. ⎪⎨ 1 3.3.12. ⎪⎨ 1
⎪ − 3 x1 − x2 − 2 x3 + x4 = 0, ⎪−2 x1 + x2 + x3 + 2 x4 = 0,
⎪⎩ 2 x1 − 2 x2 + x3 − 2 x4 = 0. ⎪ x1 + x2 + x3 + x4 = 0.

⎧−2 x1 + 2 x2 + x3 − x4 = 0, ⎧ −3 x1 + x2 + 2 x3 − x4 = 0,
⎪ ⎪
3 x − x + x = 0, 2 x + 3 x2 − x3 + x4 = 0,
3.3.13. ⎪⎨ 1 3 4 3.3.14. ⎪⎨ 1
⎪ x1 − x2 + 3 x3 + 2 x4 = 0, ⎪ x1 − 2 x2 + 3 x3 + 2 x4 = 0,
⎪⎩ 2 x1 + x2 + 3 x3 + 2 x4 = 0. ⎪ x2 + 2 x3 + x4 = 0.

⎧ x1 + 3 x2 − x3 + 2 x4 = 0, ⎧ x1 + 2 x3 − x4 = 0,
⎪ ⎪
−2 x + x + 3 x3 + x4 = 0, −2 x + x + x = 0,
3.3.15. ⎪⎨ 1 2 3.3.16. ⎪⎨ 1 2 4
⎪3x1 − 2 x2 − x3 − 2 x4 = 0, ⎪2 x1 + x2 + x3 + 2 x4 = 0,
⎪⎩ 2 x1 + 2 x2 + x3 + x4 = 0. ⎪⎩ x1 + 2 x2 + 2 x3 + 2 x4 = 0.

⎧3 x1 + 2 x2 − x3 + 2 x4 = 0, ⎧−2 x1 + 3x2 + 2 x3 + x4 = 0,
⎪ ⎪
3.3.17. ⎪⎨− x1 + x3 + 3 x4 = 0, 3.3.18. ⎪⎨ 1
x − 2 x2 + x3 − x4 = 0,
⎪− x2 + 2 x3 − 4 x4 = 0, ⎪ x1 + x2 − x3 = 0,
3
⎪2 x1 + x2 + 2 x3 + x4 = 0. ⎪⎩ x1 + x2 + x3 = 0.

57
⎧ x1 + x2 + 2 x3 + x4 = 0, ⎧2 x1 + 3 x2 + 2 x3 − x4 = 0,
⎪ ⎪
2 x + 3 x3 − x4 = 0,
3.3.19. ⎪⎨ 2 3.3.20. ⎪⎨− x1 + x2 + 2 x4 = 0,
⎪−2 x1 − 3 x2 − x3 + x4 = 0, ⎪3 x1 − 2 x2 − x3 = 0,
⎪⎩− x1 − x2 + 4 x3 + x4 = 0. ⎪⎩4 x1 + 2 x2 + x3 + x4 = 0.

⎧−3x1 + 2 x2 − 2 x3 + 2 x4 = 0, ⎧ 4 x1 − 2 x2 + 3 x3 + x4 = 0,
⎪ ⎪
2 x − x + x − x = 0, −2 x − x − 2 x3 + x4 = 0,
3.3.21. ⎪⎨ 1 2 3 4 3.3.22. ⎪⎨ 1 2
⎪− x1 − x3 + x4 = 0, ⎪ x2 + x3 − x4 = 0,
⎪⎩−2 x1 + x2 − 2 x3 + 2 x4 = 0. ⎪ 2 x1 − 2 x2 + 2 x3 + x4 = 0.

⎧2 x2 − x3 + 2 x4 = 0, ⎧3 x1 + 2 x3 + 2 x4 = 0,
⎪ ⎪
x − x + 2 x3 − x4 = 0,
3.3.23. ⎪⎨ 1 2 3.3.24. ⎪⎨−2 x1 + x2 − x3 = 0,
⎪ x1 + x3 + x4 = 0,
3 ⎪3 x1 − 2 x2 + x3 + x4 = 0,
⎪⎩4 x1 + x2 + 2 x3 + 2 x4 = 0. ⎪⎩4 x1 − x2 + 2 x3 + 3 x4 = 0.

⎧ x1 + x2 + x3 + x4 = 0, ⎧ − x1 + x2 + 2 x3 − 2 x4 = 0,
⎪ ⎪
2 x − x + 2 x3 − x4 = 0, 3 x + x − x + 3 x4 = 0,
3.3.25. ⎪⎨ 1 2 3.3.26. ⎪⎨ 1 2 3
⎪−3 x1 + x2 − 2 x3 + 2 x4 = 0, ⎪ − x1 − x2 + x3 = 0,
⎪ x1 + 2 x2 + 2 x3 + 3 x4 = 0. ⎪⎩ x1 + x2 + 2 x3 + x4 = 0.

⎧3 x1 + 3x2 − x4 = 0, ⎧ x1 + x2 + x3 + x4 = 0,
⎪ ⎪
3.3.27. ⎪⎨−2 x1 + x2 + x3 + 2 x4 = 0, −2 x − x + 2 x3 + x4 = 0,
3.3.28. ⎪⎨ 1 2
⎪ x1 − 2 x2 − 3 x4 = 0, ⎪2 x1 − x3 + x4 = 0,
⎪⎩2 x1 + 2 x2 + x3 − 2 x4 = 0. ⎪ x1 + x3 + 3 x4 = 0.

⎧ −4 x1 + 2 x2 − x4 = 0, ⎧ x1 − x2 + 2 x3 + x4 = 0,
⎪ ⎪
3x − x + x + 2 x4 = 0, 3 x + x + 2 x4 = 0,
3.3.29. ⎪⎨ 1 2 3 3.3.30. ⎪⎨ 2 3
⎪ 2 x1 + 2 x2 − 2 x3 + x4 = 0, ⎪−2 x1 + x2 − x3 − 2 x4 = 0,
⎪⎩ x1 + 3 x2 − x3 + 2 x4 = 0. ⎪− x1 + x3 − x4 = 0.

Micromodule 4
BASIC THEORETICAL INFORMATION VECTORS

Vectors, linear operations with vectors. Projection of a vector on an axis.


Linear dependence and independence of vectors. Basis and system of
coordinates. Vectors in CCS.
Literature: [1, chapter 4], [4, part 3, item 3.2], [6, chapter 2, §§ 1—3],
[7, chapter 1, § 3], [10, chapter 1, § 2], [11, chapter 1, § 2].

58
4.1. Vectors. Basic concepts

The vector is defined by the numerical value and a direction (speed,


acceleration, force, etc.)
Geometrically the vector is a directed line segment (Fig. 1.2, а) and is
designated as a or AB , where the point A is the beginning or initial point of the
vector, and В is its end or terminal point.
Definition 1.24. A distance between the beginning of a vector and its end is
called the length (or the module) of a vector and is designated as | a | or | AB | .

Definition 1.25. Vectors which lie on one straight line or parallel straight
lines are called collinear.

Vectors a and b are equal if they are collinear, have identical modules and
identical directions (Fig. 1.2, b).
Two vectors are opposite if they are collinear, have identical modules and
opposite directions.
A vector whose beginning and end coincide is called a zero
vector. Its direction is not determined.
The vector whose length is equal to a unit is called a unit vector.

Definition 1.26. A unit vector whose direction coincides with a direction


of a vector a is called an ort of the vector a and is designated as a0 .

Definition 1.27. Three vectors are called coplanar if they lie on the same
or parallel planes

Three vectors are coplanar if two of them or all three are collinear or if one
of them is a zero vector.

4.2. Linear operations on vectors

1) Addition (subtraction) of vectors.


2) Multiplication of a vector by a number (scalar).
We define the sum a + b of two vectors a and b as follows: we position
vectors a and b so that the terminal point of a coincides with the initial point
of b . Then the sum a + b is represented by an arrow directed from the initial
point of a to the terminal point of b (Fig. 1.2, c) (a rule of a triangle).
The sum of two vectors can also be constructed with help of the rule of
parallelogram (Fig. 1.2, d).

59
Subtraction of vectors is defined as an operation opposite to addition. We
define a difference a - b of two vectors a and b as follows: we position vectors
a and b so that the initial point of a coincides with the initial point of b .Then
the difference a - b is represented by a vector directed from the terminal point of b
to the terminal point of a . (Fig. 1.2, e).

В b
a a a+b a–b
a a
b a +b
a
A b b

a
a b c

Fig. 1.2
d e

Definition 1.28. If a is a vector and λ is a scalar then the product λ a is a


vector having a magnitude equal to the product | λa |=| λ || a | and having the same
direction as a if λ > 0 and the direction opposite to a if λ < 0 .
From definition of multiplication of a vector on number it follows that when
vectors are collinear there is a unique number λ such that b = λ a , and on the
contrary, if b = λ a , a and b are collinear.

Properties of operations on vectors

1. a + b = b + a . 2. (a + b ) + c = a + (b + c ) .
3. λ(a + b ) = λa + λb . 4. a + (− a ) = 0 .
5. a + 0 = a . 6. a (λ + μ) = λa + μa.
7. λ(μa ) = λμa.

4.3. Projection of a vector on axis

Definition 1.29. A directed straight line with a given initial point and a unit
of length is called an axis.
A basis of perpendicular AA1 dropped from the point A on an axis l is called
a projection of the point A on the axis l (Fig. 1.3, а).
Suppose A1 is the projection of the point A onto the axis l, B1 is the
projection of a point B onto the axis l. We suppose a = AB .

60
Definition 1.30. Projection of a vector a on an axis l is called a positive
number | A1 B1 | , if the axis l and the vector A1 B1 are equally directed (Fig.
1.3, а), and a negative number − | A1 B1 | , if the axis l and the vector A1 B1
are opposite directed (Fig. 1.3, b).

Projection of a vector a to an axis l is designated as: Prl a .


If φ is an angle between a direction of the axis l and a direction of the vector
a , then

Prl a = a ⋅ cos ϕ.

В А
a a

φ φ
А
В

А1 В1 l А1 В1 l
а b

Fig. 1.3

π π
Thus Prl a > 0 , if an angle 0 ≤ ϕ < , Prl a < 0 if <ϕ ≤π .
2 2

4.4. Linear dependence and independence of vectors. Basis

Definition 1.31. An expression of such a kind as

x1 a1 + x2 a2 + ... + xn an

is called a linear combination of vectors a1 , a2 , ... , an .

Definition 1.32. Vectors a1 , a2 , ... , an are called linearly dependent if


there are exist such numbers c1 , c2 ,..., cn not all equal to zero, that their
linear combination c1 a1 + c2 a2 + ... + cn an = 0 , and linearly independent if
this equality is carried out only if all numbers c1 , c2 , ..., cn equal to zero.

61
A set of linearly independent vectors a1 , a2 , ... , an is called a basis of space
n
R if for each vector b in R n there exist such real numbers x1 , x2 , ..., xn ,
that b = x1 a1 + x2 a2 + ... + xn an .
This equality is called an expansion of the vector b in the basis
a1 , a2 , ... , an .

Definition 1.33. Arbitrary nonzero vector on a straight line is called a


basis on this straight line.

If a vector a is a basis on a straight line then there is a unique expansion of a


vector b such that b = λa , where λ is the coordinate of the vector b in the
basis a .

Definition 1.34. An arbitrary ordered pair of noncollinear vectors is


called a basis on a plane.

Definition 1.35. An arbitrary ordered triple of noncomplanar vectors is


called a basis in space.

If vectors a , b and c are basis in space and a vector d is d = αa + βb + γc ,


then α, β, γ are coordinates of the vector d in the given basis.

4.5. Cartesian coordinate system


Any coordinate system in space is given by a point O and three non-coplanar
ordered vectors e1 , e2 , e3 (basis).
The point O is called an origin of coordinate system and straight lines
passing through the origin in a direction of basic vectors are called axes of
coordinates.
They are usually labeled as x-axis, y-axis, z-axis.
We consider basis i , j , k such that: | i |= 1, | j |= 1, | k |= 1 and
i ⊥ j , j ⊥ k , i ⊥ k . Such basis is called orthonormal basis.
Coordinate system in space with orthonormal basis is called the Cartesian
coordinate system.
If to connect any point M in space with the origin O we can consider a vector
r = OM called a radius-vector of the point M relative to the point O. Then there
are three unique numbers (x, y, z) such that

62
r = xi + yj + zk .

Coordinates x, y, z of a radius-vector OM are called coordinates of a point


M and are designated as M (x, y, z) (Fig. 1.4).

z
z М(х, у, z) аz
G
a
k γ
j у β ау
i
α
у ах у
х
х х

Fig. 1.4 Fig. 1.5

4.6. Vectors in CCS

Let a vector a be given in CCS (Fig. 1.5). In the basis i , j , k the vector a
may be decomposed as a = axi + a y j + az k . It is denoted as a = { a x , a y , a z }.
Here ax , a y , az are coordinates of the vector a in this basis. These
coordinates are projections of the vector a on coordinate axes, i.e.

a x = Prx a = a ⋅ cos α,
a y = Pry a = a ⋅ cos β,
a z = Prz a = a ⋅ cos γ ,

where α, β, γ are angles formed by the vector a with coordinate axes


Οx , Οy , Οz respectively.
The length (module) of the vector a can be found by the formula

| a |= ax2 + a 2y + az2 .

Then
ax ay a
cos α = , cos β = , cos γ = z .
|a| |a| |a|

63
Definition 1.36. cos α , cos β , cos γ are called directing cosines of the
vector a . They define the direction of the vector a and satisfy the condition

cos 2 α + cos 2 β + cos 2 γ = 1.

If A( x1 , y1 , z1 ) and B ( x2 , y2 , z2 ) , then

AB = ( x2 − x1 , y2 − y1 , z2 − z1 ).

The length of the vector AB is written down as:

| AB |= ( x2 − x1 ) 2 + ( y2 − y1 ) 2 + ( z2 − z1 ) 2 .

Let vectors be given by their coordinates, i.e.

a = ( xa , ya , za ) , b = ( xb , yb , zb ),
then
λ a = (λ xa , λ ya , λ za )

a + b = ( xa + xb , y a + yb , z a + zb ).

Definition 1.37. Vectors a and b are equal if their coordinates are equal:
xa = xb , ya = yb , za = zb .

Vectors a and b are collinear if their coordinates are proportional:


xa y z
= a = a
xb yb zb

4.7. Division of line segment according to a given ratio


Let a line segment АВ be defined by the points A ( x1 , y1 , z1 ) and
B ( x2 , y2 , z2 ) . Then coordinates of a point M(x, y, z) dividing this segment in
the ratio | AM | : | MB |= λ may be found by the formulas:

x1 + λx2 y + λ y2 z + λz2
x= y= 1 z= 1
1+ λ 1+ λ 1+ λ

64
Coordinates of the point dividing a segment in half ( λ = 1) , are:

x1 + x2 y + y2 z +z
x= y= 1 z= 1 2 .
2 2 2

Micromodule 4
EXAMPLES OF PROBLEMS SOLUTION
Example 1. The points M 1 (3 ; 3; −2) M 2 (0 ; 1; 4) are given. Find
а) coordinates, the length, directing cosines and ort of the vector M 1 M 2 ;
b) coordinates of a the point M dividing a segment M 1 M 2 in the ratio
| M1 M | : | MM 2 |= 2 : 3
Solution. а) M 1 M 2 = (0 − 3 ; 1 − 3; 4 − (−2)) = (−3; −2; 6) ;

−3 −2 6
| M1 M 2 |= 9 + 4 + 36 = 7 ; cos α = , cos β = , cos γ = .
7 7 7
An ort of the vector M 1 M 2 is the following:
e = {cos α, cos β, cos γ} = {−3 / 7; −2 / 7; 6 / 7} ;
2
b) λ = , then
3
2 2 2
3+ ⋅0 3 + ⋅1 −2 + ⋅ 4
3 9 3 11 3 =2.
xM = = , yM = = , zM =
2 5 2 5 2 5
1+ 1+ 1+
3 3 3
Example 2. Find a vector a = {ax ; a y ; az } forming identical angles with
coordinate axes provided a = 2 3.
Solution. Taking into account the equalities
ax =| a | cos α , a y =| a | cos β , az =| a | cos γ

and a condition α = β = γ we write down that


cos 2 α + cos 2 α + cos 2 α = 1 .
1 1 1
We get cos 2 α = , cos α = ± , ax = a y = az = 2 3 ⋅ = 2 or ax =
3 3 3
= a y = a z = −2 . The final answer is a = {2; 2; 2} or a = {−2; −2; −2} .

65
Example 3. Given vectors a = {1; −2; 3} and b = {4; 2; −1} . Define
whether the vectors c1 = 2a − 5b and c2 = a − 2b are collinear.
Solution. We get

c1 = 2a − 5b = {2; −4; 6} − {20; 10; −5} = {−18; −14; 11} ,


c2 = a − 2b = {1; −2; 3} − {8; 4; −2} = {−7; −6; 5} .

As the coordinates of the vectors c1 and c2 are not proportional, vectors are
not collinear.

Micromodule 4
CLASS AND HOME ASSIGNMENTS
1. The points M 1 (−4; 5; −6), M 2 (5; −7; 2) are given. Find:
a) coordinates, length, directing cosines and an ort of the vector M 1 M 2 ;
b) coordinates of a point M which divides a segment M 1 M 2 in the ratio
| M1M |:| MM 2 |= 3 : 5 .
2. Find the vector a = {a x ; a y ; a z } if it forms with axes Ox and Oy angles
π π
α= and β = respectively, and a = 6 .
4 3
3. Define whether the vectors c1 = − a + 4b and c2 = 3a − 2b , constructed on
vectors a = {2; −2; 3} and b = {3; 1; −1} are collinear.

Answers
9 12 8
1. а) M1M 2 = {9; − 12; 8}; cos α = , cos β = − , cos γ = ; b) M (−5 / 8;
17 17 17
0,5; −3). 2. a = {3 2; 3; ± 3} . 3. No.

Micromodule 4
SELF–TEST ASSIGNMENTS
4.1. Two points M 1 and M 2 are given. Find:
а) coordinates, the length, directing cosines, an ort of a vector M 1 M 2 ;
b) coordinates of the point M, if M 1 M : MM 2 = m : n ;
c) coordinates of the point M 2 , if M 1M 3 = λ M 1M 2 .

66
4.1.1. M 1 (1; 2; − 1) , M 2 (3; 4; − 2) , m : n = 2 : 5 , λ = 3 .
4.1.2. M 1 (−2; 0; − 4), M 2 (−4; 1; − 2) , m : n = 3 :1, λ = 2 .
4.1.3. M 1 (−5; 1; 4), M 2 (1; 3; 1) , m : n = 3 : 2 , λ = 4 .
4.1.4. M 1 (5; − 1; − 4), M 2 (11; 1; − 1) , m : n = 2 :1, λ = −2 .
4.1.5. M 1 (−3; − 1; 8), M 2 (−7; − 5; 6) , m : n = 1: 4, λ = −3 .
4.1.6. M 1 (15; − 2; − 14), M 2 (11; 0; 10) , m : n = 2 : 3, λ = 4 .
4.1.7. M 1 (−8; − 12; 3), M 2 (0; − 3; 15) , m : n=1:5, λ = –2.
4.1.8. M 1 (10; − 5; − 4), M 2 (1; 7; 5) , m : n=3:5, λ = –3.
4.1.9. М1(5; 2; –6), М2(25; –10; 3), m : n = 4:5, λ = 3.
4.1.10. М1(–3; –2; 16), М2(9; 18; 7), m : n = 2:3, λ = –2.
4.1.11. М1(–1; 8; 26), М2(23; 0; 20), m : n = 3:2, λ = –4.
4.1.12. М1(–7; 7; 15), М2(–1; –1; –9), m : n = 2:7, λ =2.
4.1.13. М1(–4; 5; 22), М2(4; –1; –2), m : n = 6:5, λ = 4.
4.1.14. М1(1; –8; 12), М2(25; –2; 4), m : n = 1:2, λ = –2.
4.1.15. М1(4; 9; 14), М2(–2; –15; 22), m : n = 1:3, λ = –3.
4.1.16. М1(–5; 17; 21), М2(4; 5; 1), m : n = 4:3, λ = –5.
4.1.17. М1(2; 11; 33), М2(22; –1; 24), m : n = 4:1, λ = 4.
4.1.18. М1(–7; 4; 13), М2(1; –5; 1), m : n=5:3, λ = –6.
4.1.19. М1(3; –8; 14), М2(–9; 1; 6), m : n = 5:2, λ = 5.
4.1.20. М1(–9; 3; 5), М2(0; 15; 13), m : n = 4:7, λ = –1.
4.1.21. М1(–1; 4; 12), М2(3; 0; 10), m : n = 6:7, λ = –2.
4.1.22. М1(2; 6; 4), М2(6; 4; 8), m : n = 2:1, λ = –3.
4.1.23. М1(–11; 16; 1), М2(–5; 10; 4), m : n = 3:4, λ = 2.
4.1.24. М1(–14; –3; 2), М2(–8; 3; –1), m : n = 4:5, λ = –3.
4.1.25. М1(2; 4; 7), М2(4; 7; 1), m : n = 2:3, λ = 4.
4.1.26. М1(–11; 18; 36), М2(1; 14; 30), m : n = 4:5, λ = –2.
4.1.27. М1(–4; –3; 0), М2(2; 1; 12), m : n = 1:6, λ = –4.
4.1.28. М1(9; 4; 16), М2(49; 28; –2), m : n = 4:3, λ = 2.
4.1.29. М1(0; 5; 21), М2(18; 11; 12), m : n = 6:5, λ = 3.
4.1.30. М1(–3; 5; 20), М2(3; 14; 2), m : n = 1:4, λ = –5.

67
4.2. Define whether the vectors c1 and c2 , constructed on vectors a and b
are collinear.
4.2.1. a = {1; −2; 3} , b = {3; 0; −1} , c1 = 2a + 4b , c2 = 3b − a .
4.2.2. a = {1; 0; 1} , b = {−2; 3; 5} , c1 = a + 2b , c2 = 3a − b .
4.2.3. a = {−2; 4; 1} , b = {1; −2; 7} , c1 = 5a + 3b , c2 = 2a − b .
4.2.4. a = {1; 2; −3} , b = {2; −1; −1} , c1 = 4a + 3b , c2 = 8a − b .
4.2.5. a = {3; 5; 4} , b = {5; 9; 7} , c1 = −2a + b , c2 = 3a − 2b .
4.2.6. a = {1; 4; −2} , b = {1; 1; −1} , c1 = a + b , c2 = 4a + 2b .
4.2.7. a = {1; −2; 5} , b = {3; −1; 0} , c1 = 4a − 2b , c2 = b − 2a .
4.2.8. a = {3; 4; −1} , b = {2; −1; 1} , c1 = 6a − 3b , c2 = b − 2a .
4.2.9. a = {−2; −3; −2} , b = {1; 0; 5} , c1 = 3a − 9b , c2 = −3b − a .
4.2.10. a = {−1; 4; 2} , b = {3; −2; 6} , c1 = 2a − b , c2 = 3b − 6a .
4.2.11. a = {5; 0; −1} , b = {7; 2; 3} , c1 = 2a − b , c2 = 3b + 6a .
4.2.12. a = {0; 3; −2} , b = {1; −2; 1} , c1 = 5a − 2b , c2 = 5b + 3a .
4.2.13. a = {−2; 7; −1} , b = {−3; 5; 2} , c1 = 2a + 3b , c2 = 2b + 3a .
4.2.14. a = {3; 7; 0} , b = {1; −3; 4} , c1 = 4a − 2b , c2 = b − 2a .
4.2.15. a = {3; 7; 0} , b = {1; −3; 4} , c1 = 4a − 2b , c2 = b + 2a .
4.2.16. a = {3; −2; 0} , b = {3; 0; −4} , c1 = 2a + 3b , c2 = 4b − a .
4.2.17. a = {2; 0; 1} , b = {−2; 3; −5} , c1 = a − 2b , c2 = 4b − 3a .
4.2.18. a = {−3; 4; −1} , b = {1; −2; 6} , c1 = 4a + 3b , c2 = 3a − b .
4.2.19. a = {1; 4; −3} , b = {2; 1; −1} , c1 = 4a + 3b , c2 = 6a − b .
4.2.20. a = {3; 5; 2} , b = {5; 0; 7} , c1 = −2a + 3b , c2 = 3a − 2b .
4.2.21. a = {0; 4; −3} , b = {4; 1; −1} , c1 = a + b , c2 = 5a + 2b .
4.2.22. a = {1; −2; 3} , b = {3; −1; 1} , c1 = 4a − b , c2 = b − 3a .
4.2.23. a = {4; 4; −1} , b = {2; −1; 3} , c1 = 5a − 3a , c2 = b − 4a .
4.2.24. a = {2; 3; −2} , b = {1; 0; 3} , c1 = 3a − 7b , c2 = −2b − a .
4.2.25. a = {−1; 4; 3} , b = {3; −2; 1} , c1 = 2a − b , c2 = 3b − 4a .
4.2.26. a = {3; 0; −1} , b = {7; 2; 3} , c1 = 2a − b , c2 = 3b + 2a .

68
4.2.27. a = {1; 3; −2} , b = {1; −2; 2} , c1 = 5a − 2b , c2 = 4b + 3a .
4.2.28. a = {−2; 4; −1} , b = {−3; 1; 2} , c1 = 2a + 3b , c2 = 2b + a .
4.2.29. a = {3; 3; 0} , b = {1; −3; 1} , c1 = 4a − 2b , c2 = b − 2a .
4.2.30. a = {3; 2; 1} , b = {1; −3; 2} , c1 = 4a − 2b , c2 = 2b + a .

Micromodule 5
BASIC THEORETICAL INFORMATION.
DOT PRODUCT OF TWO VECTORS

Definition of dot product of two vectors, its properties and the coordinate
form. Condition of perpendicularity of two vectors.

Literature: [1, chapter 4], [4, section 3, item 3.2], [6, chapter 2,
§ 4], [7, chapter 1, § 3], [10, chapter 1, § 2], [11, chapter 1, § 2].

5.1. Dot product of two vectors

Definition 1.38. Dot (scalar) product of two vectors a and b is the number
a ⋅ b (or (a , b ) ) equal to the product of lengths of these
two vectors and a cosine of the angle between them:

a ⋅ b =| a | ⋅ | b | cos ϕ.

If one of vectors a or b is zero, then according to the definition


a ⋅ b = 0.

If equalities | a | cos ϕ = Prb a , | b | cos ϕ = Pra b are true then

a ⋅ b =| b | ⋅ Prb a =| a | ⋅ Pra b .

The geometrical significance of the scalar product is next. The scalar product
of two vectors is equal to the product of the length of one vector by a projection
on it of the other vector.
Then
a ⋅b
Prb a = .
|b |

69
5.2. Properties of dot product

Algebraic properties of a dot product are


1) a ⋅ b = b ⋅ a ; 2) (λa ) ⋅ b = λ(a ⋅ b ) ;
3) a (b + c ) = a ⋅ b + a ⋅ c .
Geometrical properties of a dot product are next.
1) If a ≠ 0 and b ≠ 0, then a ⋅ b > 0 if the angle ϕ < 900, and a ⋅ b < 0 if
the angle ϕ ≥ 900.
2) A dot product of two nonzero vectors is equal to zero if and only if these
vectors are perpendicular.
3) The scalar square of a vector is equal to a square of its length, i.e.
a ⋅ a = a 2 =| a |2 .
Thus, | a |= a ⋅ a .

Condition of perpendicularity of two vectors


Two nonzero vectors a and b are perpendicular if and only if their scalar
product is equal to zero:

a ⊥ b ⇔ a ⋅ b = 0.

5.3. Representation of a dot product in coordinate form.


An angle between two vectors

Let vectors a and b be a set determined by coordinates


a = (ax , a y , a z ) , b = (bx , by , bz ) .

Then a ⋅ b = ax bx + a y by + az bz .
Conclusions from this formula are the following:
1) condition of perpendicularity of two vectors a and b :

ax bx + a y by + az bz = 0 ;

2) the length of the vector a is | a |= ax 2 + a y 2 + az 2 ;


3) a cosine of the angle between the vectors a and b is
a ⋅b ax b x + a y by + az bz
cos ϕ = = .
| a || b | ax + a y 2 + az 2 bx 2 + by 2 + bz 2
2

70
Micromodule 5
EXAMPLES OF PROBLEMS SOLUTION

Example 1. The angle between vectors a and b is ϕ = 120°. Knowing that


a = 3 | b |= 4, calculate:
а) (3a − 2b )(a + 2b ) ; b) | a − b | .
Solution.
а) A = (3a − 2b )(a + 2b ) = 3a 2 − 2ba + 6ab − 4b 2 = 3a 2 + 4ab − 4b 2 .
2 2
If equalities a 2 = a = 9 , b 2 = b = 16 ,

ab = a b cos120° = 3 ⋅ 4 ⋅ (−0, 5) = −6 are true, then

A = 3 ⋅ 9 + 4 ⋅ (−6) − 4 ⋅16 = −61 .

b) | a − b | = (a − b ) 2 = a 2 − 2ab + b 2 = 9 − 2 ⋅ (−6) + 16 = 37 .

Example 2. The given vectors are a = {1; 2; −2} and b = {3; 3; −4} . Find:
а) scalar product (4a + 3b )(a − 2b ) ;
b) angle between vectors a + b and a − b .
Solution.
а) 4a + 3b = {4; 8; −8} +{9; 9; −12} = {13; 17; −20} ,
a − 2b = {1; 2; −2} −{6; 6; −8} = {−5; −4; 6} ,
(4a + 3b )(a − 2b ) = 13 ⋅ (−5) + 17 ⋅ (−4) − 20 ⋅ 6 = −233 ;
b) c = a + b = {4; 5; −6} , d = a − b = {−2; −1; 2} ,
c ⋅d 4 ⋅ (−2) + 5 ⋅ (−1) − 6 ⋅ 2 −25
cos ϕ = = = ,
| c || d |
4 +5 +6 2 2 2 2 2
2 +1 + 2 2 3 77
25
Therefore ϕ = π − arccos .
3 77

Example 3. The given vectors are a = i − j + 4k , b = 6i + 5 j + 4k , c =


= −2i − 3 j + 4k . Find vector x satisfying the equalities x ⋅ a = 8 , x ⋅ b = −3 and
x ⋅ c = 13 .
Solution. Let x = {x1 , x2 , x3 } , then the condition x ⋅ a = 8 equals to the
equation x1 − x2 + 4 x3 = 8. Similarly we receive two other equations 6 x1 + 5 x2 +

71
+ 4 x3 = −3 and −2 x1 − 3 x2 + 4 x3 = 13. By solving the system of linear
equations
⎧ x1 − x2 + 4 x3 = 8,

⎨6 x1 + 5 x2 + 4 x3 = −3,
⎪−2 x − 3x + 4 x = 13,
⎩ 1 2 3

we get values: x1 = −1, x2 = −1, x3 = 2.


Answer: x = {−1; −1; 2).
Example 4. Points Α(−1; − 2; 4) , Β (−4; −2; 0) , C (3; − 2; 1) are vertices of a
triangle АBС. Find ∠Β and a projection of a vector AB on a vector BC .
Solution. We find coordinates of the vectors BA and BC that coincide with
the corresponding sides of a triangle:
BA = {3; 0; 4} , BC = {7; 0;1} .

We find cosines of the angle ϕ between vectors BA and BC according to


the formula:
BA ⋅ BC 3 ⋅ 7 + 0 ⋅ 0 + 4 ⋅1 25 1
cos ϕ = = = = ,
| BA |⋅ | BC | 9 + 0 + 4 49 + 0 + 1 25 2 2
whence ϕ = 45 . So ∠Β = 45 .
We find a projection of the vector AB on the vector BC according to the
formula:
AB ⋅ BC = −3 ⋅ 7 + 0 + ( −4) ⋅1 = −25 = − 5 2 .
PrBC AB =
BC 50 5 2 2

Micromodule 5
CLASS AND HOME ASSIGNMENT

1. Vectors a and b form an angle ϕ = 60°. Knowing that | a |= 2 | b |= 5 ,


calculate: а) (4a − b )(2a + 3b ) ; b) | a − 2b | .
2. The given vectors are a = {1; 4; −1} and b = {0; 3; −2} . Find:
а) a scalar product (a + 3b )(3a − 2b ) ;
b) an angle between vectors 2a + b and a − b ;
c) a projection of a vector −3a + 2b on a vector a − b .

72
3. Vectors a = 2i − j + 3k , b = i + 4k , c = i − j + 4k are given. Find a
vector x , if x ⋅ a = 8 , x ⋅ b = 10 , x ⋅ c = 8 .

Answers
3
1. а) 57; b) 2 21. 2. а)79; b) arccos . 3. x = (2; 2; 2).
47

Micromodule 5
SELF-TEST ASSIGNMENTS

5.1. Evaluate:
5.1.1. а) (4a + 7b )(a − 2b ) ; b) | 2a − 3b | , if | a |= 2 , | b |= 5 , ϕ = 60°.

5.1.2. а) (2a + 5b )(3a − 2b ) ; b) | a − 3b | , if | a |= 3 , | b |= 4 , ϕ = .
3
π
5.1.3. а) (3a + b )(2a + 3b ) ; b) | 2a − 3b | , if | a |= 2 , | b |= 3 , ϕ = .
4
π
5.1.4. а) (4a + 3b )(a − 4b ) ; b) | 2a + 3b | , if | a |= 1 , | b |= 6 , ϕ = .
3
π
5.1.5. а) (4a + 5b )(a − 2b ) ; b) | 2a − b | , if | a |= 3, | b |= 1, ϕ = .
6
π
5.1.6. а) (5a + 3b )(a + 2b ) ; b) | a − b | , if | a |= 3, | b |= 4, ϕ = .
3

5.1.7. а) (2a + 4b )(−3a − b ) ; b) | a + 2b | , if | a |= 3, | b |= 2, ϕ = .
3
π
5.1.8. а) (3a + 2b)(−a + 3b ) ; b) | 2a + 3b | , if | a |= 4 2, | b |= 3, ϕ = .
4
π
5.1.9. а) (4a + b )(3a − b ) ; b) | 2a + b | , if | a |= 1, | b |= 4, ϕ = .
3
π
5.1.10. а) (6a + 5b )(a + b ) ; b) | 4a − b | , if | a |= 2 3, | b |= 1, ϕ = .
6
π
5.1.11. а) (5a + b )(a + b ) ; b) | a + b | , if | a |= 3, | b |= 2, ϕ = .
3

5.1.12. а) (3a + 4b )(−3a − b ) ; b) | a + 2b | , if | a |= 5, | b |= 2, ϕ = .
3
π
5.1.13. а) (5a + 2b)(−a + 3b ) ; b) | a + 4b | , if | a |= 2, | b |= 3, ϕ = .
4

73
π
5.1.14. а) (4a + 3b )(3a − 2b ) ; b) | 2a + 5b | , if | a |= 2, | b |= 4, ϕ = .
3
π
5.1.15. а) (2a + 5b )(a + b ) ; b) | 2a − b | , if | a |= 3, | b |= 1, ϕ = .
6
π
5.1.16. а) (a + b )(a − 2b ) ; b) | a − b | , if | a |= 3, | b |= 4, ϕ = .
3

5.1.17. а) (3a + 5b )(3a − b ) ; b) | a + 2b | , if | a |= 3, | b |= 2, ϕ = .
3
π
5.1.18. а) (3a + 2b)(− a + 3b ) ; b) | a + 3b | , if | a |= 6 2, | b |= 3, ϕ = .
4
π
5.1.19. а) (−4a + 3b )(3a + 2b ) ; b) | 2a + b | , if | a |= 2, | b |= 6, ϕ = .
3
π
5.1.20. а) (2a − 5b )(a − b ) ; b) | 2a − 3b | , if | a |= 4 3, | b |= 1, ϕ = .
6
π
5.1.21. а) (a + b )(3a − 2b ) ; b) | a − b | , if | a |= 5, | b |= 4, ϕ = .
3

5.1.22. а) (3a + b )(4a − b ) ; b) | a + 2b | , if | a |= 3, | b |= 5, ϕ = .
3
π
5.1.23. а) (3a + 5b)(− a − b ) ; b) | a + b | , if | a |= 3 2, | b |= 2, ϕ = .
4
π
5.1.24. а) (−a + 3b )(3a + b ) ; b) | 2a + b | , if | a |= 2, | b |= 3, ϕ = .
3
π
5.1.25. а) (2a − 3b )(a − b ) ; b) | 2a − b | , if | a |= 3 3, | b |= 1, ϕ = .
6
π
5.1.26. а) (a + b )(a − 2b ) ; b) | 3a + b | , if | a |= 3, | b |= 4, ϕ = .
3

5.1.27. а) (2a + b )(4a − b ) ; b) | 5a + 2b | , if | a |= 3, | b |= 2, ϕ = .
3
π
5.1.28. а) (3a + 5b)(− a + b ) ; b) | a + b | , if | a |= 5 2, | b |= 2, ϕ = .
4
π
5.1.29. а) (a + 3b )(a + b ) ; b) | 2a + b | , if | a |= 4, | b |= 3, ϕ = .
3
π
5.1.30. а) (2a − b )(a − b ) ; b) | 2a − b | , if | a |= 5 3, | b |= 2, ϕ = .
6

74
5.2. Find a dot product pq , an angle between vectors p and q and a
projection of the vector p onto the vector q if:
5.2.1. p = 2a + 4b , q = 3a − b , a = {−1; 3; 4} , b = {−5; 1; 2} .
5.2.2. p = 5a + 2b , q = 2a − 3b , a = {−2; 1; 2} , b = {−2; 4; 3} .
5.2.3. p = −2a + 7b , q = 3a − 2b , a = {2; − 3; 4} , b = {−1; − 1; 3} .

5.2.4. p = 3a + 4b , q = 2a − b , a = {−4; 3; 1} , b = {2; 2; 1} .


5.2.5. p = a − 4b , q = 2a − 3b , a = {−4; 3; 2} , b = {−2; 4; 5} .
5.2.6. p = −a + 3b , q = 2a + b , a = {3; 3; 1} , b = {−2; − 3; − 2} .
5.2.7. p = 3a + 2b , q = 2a − 6b , a = {4; 3; 2} , b = {2; 1; 4} .
5.2.8. p = −3a + 7b , q = 3a − b , a = {−1; 0; 4} , b = {−3; 1; 2} .
5.2.9. p = −3a − 2b , q = 2a + b , a = {3; 2; 1} , b = {−1; − 2; − 2} .
5.2.10. p = 5a − 4b , q = 2a − b , a = {0; − 4; 4} , b = {−2; − 3; 3} .
5.2.11. p = − a + 4b , q = a + b , a = {−1; 4; 4} , b = {3; 1; − 2} .
5.2.12. p = 3a − 2b , q = 2a − 3b , a = {−5; 1; 2} , b = {−3; 4; 3} .
5.2.13. p = −3a − 4b , q = a − b , a = {0; − 2; 2} , b = {−2; − 3; 0} .
5.2.14. p = −3a + 2b , q = 2a + 9b , a = {2; 3; 2} , b = {2; − 1; − 4} .
5.2.15. p = 5a − 3b , q = a − 3b , a = {−2; 4; 2} , b = {−3; 0; 3} .
5.2.16. p = −a + 3b , q = a + 2b , a = {3; − 2; 1} , b = {−1; − 2; 2} .
5.2.17. p = 3a − 2b , q = 3a + 2b , a = {2; 1; 2} , b = {2; − 1; − 2} .
5.2.18. p = 3a − b , q = 4a − b , a = {−1; 2; 3} , b = {4; 1; − 3} .
5.2.19. p = −3a + b , q = 2a + b , a = {3; − 2; 0} , b = {1; − 2; 2} .
5.2.20. p = 2a − 3b , q = a − 2b , a = {0; − 2; 1} , b = {−2; − 4; 0} .
5.2.21. p = − a − 2b , q = − a + 2b , a = {−1; 2; 1} , b = {4; 1; − 2} .
5.2.22. p = 3a − 2b , q = a + 2b , a = {−2; 4; 3} , b = {−3; 0; 2} .
5.2.23. p = 2a − b , q = 3a + b , a = {0; − 2; 2} , b = {−2; − 3; 0} .
5.2.24. p = −3a + 2b , q = 4a + b , a = {2; 1; 4} , b = {2; − 1; − 3} .

75
5.2.25. p = 2a − b , q = 5a − 2b , a = {−2; 4; 0} , b = {−4; 0; 2} .
5.2.26. p = − a − 2b , q = −a − 3b , a = {4; − 1; 0} , b = {1; − 2; 3} .
5.2.27. p = 3a − b , q = a + b , a = {2; 0; 4} , b = {3; − 1; − 2} .
5.2.28. p = a − b , q = 3a + 4b , a = {−1; 4; 1} , b = {3; 1; − 2} .
5.2.29. p = 3a + 2b , q = 4a + b , a = {3; − 2; 0} , b = {1; − 4; 3} .
5.2.30. p = 2a − 3b , q = 2a − b , a = {0; − 1; 1} , b = {−2; − 1; 0} .

5.3. Find a vector x provided


5.3.1. x ⋅ (i − j + k ) = 1 , x ⋅ (i + 3k ) = 5 , x ⋅ (i − 3 j + 6k ) = 2 .
5.3.2. x ⋅ (i − j + 2k ) = 2 , x ⋅ (i − j + 5k ) = 8 , x ⋅ (i − 4 j + 8k ) = 2 .
5.3.3. x ⋅ (i − j + 3k ) = 5 , x ⋅ (i − 2 j + 7k ) = 11 , x ⋅ (i − 5 j + 10k ) = 2.
5.3.4. x ⋅ (i − j + 4k ) = 10 , x ⋅ (i − 3 j + 9k ) = 14 , x ⋅ (i − 6 j + 12k ) = 2.
5.3.5. x ⋅ (i − j + 5k ) = 17 , x ⋅ (i − 4 j + 11k ) = 17 , x ⋅ (i − 7 j + 14k ) = 2 .
5.3.6. x ⋅ (i − j + 6k ) = 26 , x ⋅ (i − 5 j + 13k ) = 20 , x ⋅ (i − 8 j + 16k ) = 2 .
5.3.7. x ⋅ (i − j + 7k ) = 37 , x ⋅ (i − 6 j + 15k ) = 23 , x ⋅ (i − 9 j + 18k ) = 2 .
5.3.8. x ⋅ (i − j − k ) = 5 , x ⋅ (i + 2 j − k ) = −1 , x ⋅ (i − j + 2k ) = 2 .
5.3.9. x ⋅ (i − j − 3k ) = 17 , x ⋅ (i + 4 j − 5k ) = −7 , x ⋅ (i + j − 2k ) = 2 .
5.3.10. x ⋅ (i − j − 4k ) = 26 , x ⋅ (i + 5 j − 7k ) = −10 , x ⋅ (i + 2 j − 4k ) = 2 .
5.3.11. x ⋅ (i − j − 5k ) = 37 , x ⋅ (i + 6 j − 9k ) = −13 , x ⋅ (i + 3 j − 6k ) = 2 .
5.3.12. x ⋅ (i − j + k ) = 3 , x ⋅ (i − j + 2k ) = 1 , x ⋅ (3i + j + 10k ) = −1 .
5.3.13. x ⋅ (3i − 2 j + 3k ) = 10 , x ⋅ (2i − j + 3k ) = 4 , x ⋅ (5i + j + 12k ) = 0 .
5.3.14. x ⋅ (2i − j + 2k ) = 7 , x ⋅ (3i − j + 4k ) = 7 , x ⋅ (7i + j + 14k ) = 1 .
5.3.15. x ⋅ (5i − 2 j + 5k ) = 18 , x ⋅ (4i − j + 5k ) = 10 , x ⋅ (9i + j + 16k ) = 2.
5.3.16. x ⋅ (3i − 2 j + 3k ) = 11 , x ⋅ (5i − j + 6k ) = 13 , x ⋅ (11i + j + 18k ) = 3.
5.3.17. x ⋅ (i − j ) = 10 , x ⋅ (6i − j + 7 k ) = 16 , x ⋅ (13i + j + 20k ) = 4.
5.3.18. x ⋅ (i + 2 j + k ) = 6 , x ⋅ (2i + j + 2k ) = 9 , x ⋅ (−3i + j + 4k ) = −4.
5.3.19. x ⋅ (i + j + k ) = 5 , x ⋅ (3i + j + 2k ) = 11 , x ⋅ (−5i + j + 2k ) = −5 .

76
5.3.20. x ⋅ (3i + 2 j + 3k ) = 14 , x ⋅ (4i + j + 3k ) = 14 , x ⋅ (−7i + j ) = −6 .
5.3.21. x ⋅ (2i + j + 2k ) = 9 , x ⋅ (5i + j + 4k ) = 17 , x ⋅ (9i − j + 2k ) = 7 .
5.3.22. x ⋅ (5i + 2 j + 5k ) = 22 , x ⋅ (6i + j + 5k ) = 20 , x ⋅ (11i − j + 4k ) = 8 .
5.3.23. x ⋅ (3i + j + 3k ) = 13 , x ⋅ (7i + j + 6k ) = 23 , x ⋅ (13i − j + 6k ) = 9 .
5.3.24. x ⋅ (5i + 2 j − 3k ) = −2 , x ⋅ (i + j + k ) = 3 , x ⋅ (i − j + 4k ) = 0 .
5.3.25. x ⋅ (5i + j − 2k ) = 1 , x ⋅ (2i + j ) = 3 , x ⋅ (i − 2 j + 5k ) = −1 .
5.3.26. x ⋅ (5i − k ) = 4 , x ⋅ (3i + j − k ) = 5 , x ⋅ (i − 3 j + 6k ) = −2 .
5.3.27. x ⋅ (5i − j ) = 7 , x ⋅ (4i + j − 2k ) = 9 , x ⋅ (i − 4 j + 7 k ) = −3 .
5.3.28. x ⋅ (5i − 2 j + k ) = 10 , x ⋅ (5i + j − 3k ) = 15 , x ⋅ (i − 5 j + 8k ) = −4.
5.3.29. x ⋅ (5i + 4 j − 5k ) = −8 , x ⋅ (−i + j + 3k ) = 9 , x ⋅ (i + j + 2k ) = 2 .
5.3.30. x ⋅ (5i + 5 j − 6k ) = −11, x ⋅ (−2i + j + 4k ) = 15, x ⋅ (i + 2 j + k ) = 3.

Micromodule 6
BASIC THEORETICAL INFORMATION
CROSS AND TRIPLE PRODUCTS

Cross product of two vectors, its algebraic and geometrical properties. The
coordinate form. Triple product of three vectors, its algebraic and geometrical
properties. The coordinate form. The condition of complanarity of three vectors.

Literature: [1, chapter 4], [4, part 3, p. 3.2], [6, chapter 2, §§ 5,6], [7, chap-
ter1, § 4], [10, chapter1, § 2], [11, chapter1, § 2].

6.1. Cross product

Definition 1.39. The cross product of two vectors a and b is said to be a


vector c satisfying the following conditions:
1) a module of the vector c is determined according to the formula:
| c |=| a || b | sin ϕ, where φ is an angle between vectors a and b ;
2) the vector c is perpendicular to each of vectors a and b ;
3) the vectors a , b and c form a right-hand triple (that is, if we look from
the end of the resulting vector c then the shortest turn from the first vector a to
the second vector b can be seen counterclockwise (Fig. 1.6)).

77
A cross product designation is a × b.

Fig. 1.6 Fig. 1.7

6.2. Cross product properties

Geometrical interpretation of a cross product. The module of cross product is


equal to the area of a parallelogram built on vectors a and b applied to the common
beginning (Fig.1.7).
1) anticommutativity of multiplication:

a × b = −b × a ;

(
2) (λ a) × b = λ (a × b) ; a × (λb) = λ a × b ; )
( )
3) a × b + c = a × b + a × c .
Note. If the coordinates of a triangle ABC are known, then its area can be
determined according to the formula:

1
S ΔABC = AB × AC .
2

6.3. Cross product of two vectors in coordinate form

Let vectors a = {ax , a y , az } , b = {bx , by , bz } be given by their coordinates in


Cartesian coordinate system. Then their cross product is determined according to the
formula:

i j k
a × b = ax ay az
bx by bz

78
or
a × b = (a y bz − az by )i − (ax bz − az bz ) j + (ax by − a y bx )k .

6.4. Triple product


Definition 1.40. A number abc equal to the dot product of the vector a × b
by the vector c is called the mixed (triple) product of three vectors a , b and c .

abc = (a × b) ⋅ c .
Properties:
1) If any two multipliers are interchanged in a triple product, then the triple
product changes its sign. For example,
abc = −cba.
2) In case of a cycle interchange of multipliers a triple product is not changed.
3) Signs of cross and dot products can be interchanged in a mixed product:
(a × b) ⋅ c = a ⋅ (b × c ) = (b × c) ⋅ a.
4) Geometrical content of a mixed product. A module of a mixed product abc is
equal to the volume of parallelepiped, built on vectors a , b and c applied to the
common beginning, that is V =| abc | .
Note. The volume of a pyramid, built on vectors a , b and c is equal to 1/6 of
the parallelepiped volume.
5) If abc > 0, then vectors a , b and c form a right-hand triple and if
abc < 0, then they form a left-hand triple.
6) Condition of three vectors complanarity.

Vectors a , b and c are complanar if and only if abc = 0.

6.5. Triple product of three vectors in coordinate form


Let vectors a = {ax , a y , az }, b = {bx , by , bz }, c = {cx , c y , cz } be given by
their coordinates in Cartesian coordinate system.Then their triple product is determi-
ned according to the formula
ax ay az
abc = bx by bz .
cx cy cz

79
Micromodule 6
EXAMPLES OF PROBLEMS SOLUTION

Example 1. Vectors a = {2; 3; 1} and b = {−2; 4; 0} are given. Find the vector
(2a − b) × (3a + 2b).
Solution. Firstly we find
2a − b = {4; 6; 2} − {−2; 4; 0} = {6; 2; 2},
3a + 2b = {6; 9; 3} + {−4; 8; 0} = {2; 17; 3}.
Then
i j k
2 2 6 2 6 2
(2a − b) × (3a + 2b) = 6 2 2 = i+ j+ k=
17 3 2 3 2 17
2 17 3
= −28i − 14 j + 98k .
Answer: −28i − 14 j + 98k .
Example 2. Find the area of Δ ABC and volume of a pyramid, whose verti-
ces are at the points A(2; –1; 1). B(5; 5; 4), C(3; 2; –1), D(4; 1; 3).
Solution. Let us find the coordinates of vectors AB, AC and AD, on which the
pyramid is built:
AB = {3; 6; 3}, AC = {1; 3; − 2}, AD = {2; 2; 2}.
The area of Δ ABC is determined according to the formula
1
SΔABC = AB × AC .
2
Therefore
i j k
AB × AC = 3 6 3 = (−12 − 9)i − (−6 − 3) j + (9 − 6)k =
1 3 −2
= −21i + 9 j + 3k = 3(−7i + 3 j + k );
1 1 3
SΔABC = AB × AC = ⋅ 3 ⋅ (−7) 2 + 32 + 12 = 59.
2 2 2
Volume of a pyramid is equal to 1/6 of parallelepiped volume, built on vectors
AB, AC and AD, that is,
3 6 3
1 1
VABCD = mod 1 3 −2 = mod(−18) = 3.
6 6
2 1 2

80
3
Answer: SΔABC = 59 square units, VABCD = 3 cubic units.
2
Example 3. Prove that vectors a = {2; 1; 3}, b = {2; − 3; 1} and c = {1; 2; 1}
form a basis and decompose the vector p = {0; 11; 3} according to this basis.
Solution. We remember that any ordered triple of noncoplanar vectors is the
basis in the space. Consequently, the given vectors form a basis if their mixed
product is not equal to zero. Let us verify this:
2 1 3
abc = 2 −3 1 = 10 ≠ 0.
1 2 1

Therefore the vectors a , b and c are the basis.


Any vector p is decomposed with respect to the basis a , b and c as
p = αa + βb + γc , where α, β, γ are unknown numbers (coordinates of the
vector p in the given basis). The vector equation is equivalent to the system of
equations (vectors are equal if their corresponding coordinates are also equal):

⎧0 = 2α + 2β + γ ,

⎨11 = α − 3β + 2 γ ,
⎪⎩3 = 3α + β + γ.

From the last system we can find that α = 1, β = −2, γ = 2. Therefore,


p = a − 2b + 2c .

Micromodule 6
CLASS AND HOME ASSIGNMENT

1. Vectors a = {−2; 0; 1} and b = {−1; 4; 5} are given. Find:


a) (2a − 5b) × (a + 2b); b) | (a − b) × (a + 2b) | .
2. Determine the area of Δ ABC and the volume of a pyramid, whose
vertices are at points A(2; 4; 5), B(–4; 4; –5), C (5; 0; 3), D(1; 2; 0).
3. Prove that vectors a = {−2; 2; − 1}, b = {−2; 1; − 1} and c = {−1; 0; 1} form
a basis and decompose the vector p = {0; 11; 3} with respect to this basis.

Answers

3
1. a) {–36; 81; –72}; b) 2 161 . 2. S = 377, V = 6.
2

81
Micromodule 6
SELF-TEST ASSIGNMENT

6.1. Find a cross product:


6.1.1. (2a + 4b) × (3a − b) , if a = {−2; 1; 2}, b = {−2; 4; 3}.
6.1.2. (5a + 2b) × (2a − 3b) , if a = {−1; 3; 4} , b = {−5; 1; 2}.
6.1.3. (−2a + 7b) × (3a − 2b), if a = {−4; 3; 1}, b = {2; 2; 1}.
6.1.4. (3a + 4b) × (2a − b), if a = {2; − 3; 4}, b = {−1; − 1; 3}.
6.1.5. (a − 4b) × (2a − 3b), if a = {3; 3; 1}, b = {−2; − 3; − 2}.
6.1.6. (− a + 3b) × (2a + b), if a = {−4; 3; 2}, b = {−2; 4; 5}.
6.1.7. (3a + 2b) × (2a − 6b), if a = {−1; 0; 4}, b = {−3; 1; 2}.
6.1.8. (−3a + 7b) × (3a − b), if a = {4; 3; 2}, b = {2; 1; 4}.
6.1.9. (−3a − 2b) × (2a + b), if a = {0; − 4; 4}, b = {−2; − 3; 3}.
6.1.10. (5a − 4b) × (2a − b), if a = {3; 2; 1}, b = {−1; − 2; − 2}.
6.1.11. (− a + 4b) × (a + b), if a = {−5; 2; 1}, b = {−3; 4; 3}.
6.1.12. (3a − 2b) × (2a − 3b), if a = {−1; 4; 4}, b = {3; 1; − 2}.
6.1.13. (−3a − 4b) × (a − b), if a = {2; 3; 2}, b = {2; − 1; − 4}.
6.1.14. (−3a + 2b) × (2a + 9b), if a = {0; − 2; 2}, b = {−2; − 3; 0}.
6.1.15. (5a − 3b) × (a − 3b), if a = {3; − 2; 1}, b = {−1; − 2; 2}.
6.1.16. (− a + 3b) × (a + 2b), if a = {−2; 4; 2}, b = {−3; 0; 3}.
6.1.17. (3a − 2b) × (3a + 2b), if a = {−1; 2; 3}, b = {4; 1; − 3}.
6.1.18. (3a − b) × (4a − b), if a = {2; 1; 2}, b = {2; − 1; − 2}.
6.1.19. (−3a + b) × (2a + b), if a = {0; − 2; 1}, b = {−2; − 4; 0}.
6.1.20. (2a − 3b) × (a − 2b), if a = {3; − 2; 0}, b = {1; − 2; 2}.
6.1.21. (−a − 2b) × (−a + 2b), if a = {−2; 4; 3}, b = {−3; 0; 2}.
6.1.22. (3a − 2b) × (a + 2b), if a = {−1; 2; 1}, b = {4; 1; − 2}.
6.1.23. (2a − b) × (3a + b), if a = {2; 1; 4}, b = {2; − 1; − 3}.
6.1.24. (−3a + 2b) × (4a + b), if a = {0; − 2; 2}, b = {−2; 3; 0}.
6.1.25. (2a − b) × (5a − 2b), if a = {4; − 1; 0}, b = {1; − 2; 3}.
6.1.26. (−a − 2b) × (−a − 3b), if a = {−2; 4; 0}, b = {−4; 0; 2}.
6.1.27. (3a − b) × (a + b), if a = {−1; 4; 1}, b = {3; 1; − 2}.

82
6.1.28. (a − b) × (3a + 4b), if a = {2; 0; 4}, b = {3; − 1; − 2}.
6.1.29. (3a + 2b) × (4a + b), if a = {0; − 1; 1}, b = {−2; − 1; 0}.
6.1.30. (2a − 3b) × (2a − b), if a = {3; − 2; 0}, b = {1; − 4; 3}.

6.2. Determine the area of the Δ ABC and the volume of a pyramid ABCD,
whose vertices are at the points:
6.2.1. A(1; –2; 3), B(2; 4; 7), C(–3; –4; 0), D(1; 0; 5).
6.2.2. A(–3; 5; 4), B(0; 0; 8), C(–1; 3; –2), D(2; 6; 1).
6.2.3. A(0; –5; 4), B(3; 5; 1), C(–4; –4; 1), D(3; 1; 6).
6.2.4. A(–2; 0; 2), B(1; 0; 6), C(–5; 4; –1), D(0; 4; 2).
6.2.5. A(2; 1; 7), B(–1; 3; 5), C(5; –4; 1), D(2; 5; 1).
6.2.6. A(3; –3; 0), B(4; 4; 2), C(–5; –3; 0), D(1; 1; 4).
6.2.7. A(–4; 6; 4), B(3; 10; 8), C(1; 4; –2), D(–2; 3; 1).
6.2.8. A(0; –3; 5), B(–3; –1; 1), C(2; –5; 2), D(4; 3; 6).
6.2.9. A(–5; 0; 3), B(2; 1; 5), C(–4; 2; –1), D(0; 0; 3).
6.2.10. A(2; 3; 5), B(–1; –3; 4), C(4; –3; 2), D(1; 6; 1).
6.2.11. A(2; –2; 0), B(5; 3; 2), C(–3; –2; 0), D(1; 2; 3).
6.2.12. A(–3; 5; 4), B(2; 8; 7), C(1; 3; –2), D(–1; 4; 1).
6.2.13. A(0; –2; 4), B(–2; –2; 1), C(3; –3; 2), D(3; 3; 4).
6.2.14. A(–3; 2; 3), B(3; 1; –5), C(4; –2; –1), D(4; 0; 3).
6.2.15. A(–2; 3; –5), B(1; 3; 4), C(4; 3; 2), D(1; –6; 1).
6.2.16. A(1; –3; 0), B(4; 3; 1), C(–4; –3; 0), D(–1; –2; 3).
6.2.17. A(3; –5; 4), B(2; 6; –7), C(–1; 3; 2), D(–1; –4; 1).
6.2.18. A(1; 0; 4), B(2; –2; –1), C(3; –1; 0), D(3; 2; 5).
6.2.19. A(–4; 1; 3), B(3; 2; –7), C(2; –1; –1), D(5; 1; 3).
6.2.20. A(–1; 5; –3), B(0; 3; 2), C(1; 3; 4), D(2; –3; 0).
6.2.21. A(–1; –2; 0), B(–4; 3; –1), C(4; –4; 0), D(1; –2; 4).
6.2.22. A(2; –1; 4), B(2; 3; –5), C(–2; 2; 3), D(–2; –3; 1).
6.2.23. A(1; 2; 5), B(2; –3; 1), C(4; –2; 0), D(3; 3; 6).
6.2.24. A(–1; 2; 4), B(0; 2; –6), C(2; –3; 2), D(6; –1; 0).
6.2.25. A(1; –5; –3), B(1; 3; 0), C(–1; 3; –4), D(3; –3; 2).
6.2.26. A(–2; –4; 0), B(4; –3; –1), C(3; –2; 0) D(2; –1; 5).
6.2.27. A(2; –3; 5), B(1; 3; –4), C(–1; 2; 4), D(–1; –5; 2).
6.2.28. A(2; 2; 2), B(3; –3; 4), C(4; –5; 1), D(3; 2; 1).
6.2.29. A(–2; 3; 5), B(1; 2; –4), C(3; –4; 2), D(3; –2; 5).
6.2.30. A(2; –4; –4), B(2; 3; 1), C(–3; 4; –4), D(4; –2; 1).

83
6.3. Prove that vectors p, q and r form a basis and decompose a vector x
with respect to this basis:
6.3.1. x ={–2; 11; 14}, p ={1; 1; 3}, q ={1; 2; 1}, r ={–4; 1; 1}.
6.3.2. x ={2; –6; –3}, p ={2; 1; 8}, q ={2; –3; 1}, r ={1; –1; 2}.
6.3.3. x ={–12; 13; –4}, p ={1; 2; 3}, q ={1; 4; 1}, r ={5; 1; –3}.
6.3.4. x ={11; 14; 12}, p ={2; 4; 3}, q ={1; 2; –1}, r ={4; 4; 5}.
6.3.5. x ={–3; –2; 2}, p ={1; 2; 1}, q ={2; 2; –3}, r ={–1; 1; 4}.
6.3.6. x ={–12; –2; –15}, p ={1; 2; –1}, q ={–4; 3; 1}, r ={2; 4; 5}.
6.3.7. x ={–4; 13; 16}, p ={1; 3; 1}, q ={7; –1; 4}, r ={–1; 2; 3}.
6.3.8. x ={4; 5; –7}, p ={1; 2; 1}, q ={2; –1; 1}, r ={–1; 1–3; }.
6.3.9. x ={0; 0; 2}, p ={2; 2; 3}, q ={4; –3; –1}, r ={–6; 1; –1}.
6.3.10. x ={–1; 13; 10}, p ={1; 2; 3}, q ={3; –3; 1}, r ={–2; 4; 1}.
6.3.11. x ={–1; 9; 12}, p ={1; 3; 2}, q ={–2; –1; 3}, r ={3; 2; –1}.
6.3.12. x ={5; –6; 2}, p ={1; 2; 3}, q ={–4; 1; –2}, r ={2; –3; –1}.
6.3.13. x ={16; 2; 10}, p ={2; 1; 1}, q ={–6; 1; –1}, r ={8; –1; 4}.
6.3.14. x ={13; –3; 6}, p ={3; 1; 4}, q ={2; –2; –1}, r ={1; 1; 1}.
6.3.15. x ={13; 16; –1}, p ={1; 2; 1}, q ={2; 3; 1}, r ={–1; 1; 4}.
6.3.16. x ={11; 11; 27}, p ={1; 1; 5}, q ={5; 1; 1}, r ={–1; –5; –1}.
6.3.17. x ={–1; –1; 2}, p ={2; 1; 3}, q ={2; –3; 1}, r ={1; 2; 1}.
6.3.18. x ={1; 2; 6}, p ={1; 2; 3}, q ={1; –3; –1}, r ={–4; 2; –1}.
6.3.19. x ={4; 11; 11}, p ={2; 3; 3}, q ={–1; 4; –2}, r ={–1; –2; 4}.
6.3.20. x ={8; 6; –4}, p ={2; 1; 3}, q ={–1; –3; 1}, r ={1; –5; –7}.
6.3.21. x ={1; 2; 3}, p ={3; 1; 2}, q ={2; 1; 2}, r ={–1; 2; 5}.
6.3.22. x ={10; 8; –2}, p ={1; 6; 3}, q ={6; 3; 1}, r ={3; –1; –6}.
6.3.23. x ={–1; 7; 1}, p ={2; 7; 1}, q ={6; –1; 1}, r ={3; –1; 1}.
6.3.24. x ={–4; 6; 4}, p ={2; 1; 3}, q ={–3; 2; –1}, r ={–3; 4; 2}.
6.3.25. x ={1; 1; 1}, p ={7; 2; –5}, q ={–3; 5; –2}, r ={–3; –6; 8}.
6.3.26. x ={8; 9; 3}, p ={–1; 4; 6}, q ={4; 2; –1}, r ={5; 3; –2}.
6.3.27. x ={0; –9; –3}, p ={3; –2; 6}, q ={4; –3; 1}, r ={5; 5; –1}.
6.3.28. x ={–2; –4; 3}, p ={1; 2; 4}, q ={–4; –3; 1}, r ={2; –1; 2}.
6.3.29. x ={7; 8; 5}, p ={2; 2; 3}, q ={3; 4; –1}, r ={–1; 1; 1}.
6.3.30. x ={–6; 4; –3}, p ={1; 3; 3}, q ={–3; 1; –2}, r ={–3; 3; 2}.

84
Моdule
2 ELEMENTS OF ANALYTICAL
GEOMETRY

MODULE STRUCTURE
Micromodule 7. Straight line on a plane. General equation of a straight
line, incomplete equations. Symmetric and parametric equations of straight
line. Straight line passing through two given points. Equation of a straight
line in segments on axes, a slope — intercept form of a straight line. Angle
between two straight lines, conditions of parallelism and perpendicularity of
two straight lines. Distance from a point to a straight line.
Micromodule 8. Plane in space. General equation of a plane, incomplete
equations. Equation of a plane passing through three points. The equation of
a plane in segments on axes. Normal equation of a plane, distance from point
to the plane. Angle between two planes, conditions of parallelism and
perpendicularity of two planes.
Micromodule 9. Straight line in space. Mutual location of a straight line
and a plane.
Micromodule 10. Curves of the second order. Circle, an ellipse, a hyper-
bola, a parabola: definitions, the canonic equations, the basic characteristics.
Micromodule 11. Surfaces of the second order. Cylindrical surfaces. Conic
surfaces. Sphere. An ellipsoid. A hyperboloid of one sheet and two sheets.
An elliptic and hyperbolic paraboloid. The canonic equations.

Basic concepts. 1. Cartesian rectangular coordinate system. 2. Coordinates


of a point. 3. An equation of a line on a plane and in space. 4. An equation of a
surface in space. 5. Lines and surfaces of the second order.

Key words: axis — вісь, area — площа, angle — кут, distance — відстань,
straight line — пряма лінія, plane — площина, in segments on axes — у від-
різках на осях, parallelism — паралельність, perpendicularity — перпенди-
кулярність, a slope — intercept form of a straight line — рівняння прямої з
кутовим коефіціентом, curve — крива, circle — коло, ellipse — еліпс,
hyperbola — гіпербола, parabola — парабола, Cartesian rectangular coordinate
system — декартова прямокутна система координат, surface — поверхня,
sphere — сфера, cylinder— циліндр, ellipsoid — еліпсоїд, cone — конус,
hyperboloid of one sheet — однопорожнинний гіперболоїд, hyperboloid of
two sheets — двопорожнинний гіперболоїд.

85
Main tasks: 1. Construction of equations of straight lines and planes with
different given elements. 2. Construction of equations of the second order curves
and surfaces. 3. Studying of mutual location of straight lines and planes.

STUDENT MUST BE READY TO DO


THE FOLLOWING ASSIGNMENTS

1. Concepts, definitions, formulations:


• Different equations of a straight line (typical problems of finding equa-
tions of a straight line).
• Curves of the second order: a circle, an ellipse, a hyperbola, a parabola
(their standard equations).*
• A plane. Different equations of a plane (typical problems on finding of
equations of a plane).
• Cylindrical, conic surfaces.
• Surfaces of revolution.
• Method of sections.

2. Proofs and conclusions


• Different forms of the equation of a straight line on a plane (general,
symmetric, parametric, passing through two points, in slope — intercept form, in
«segments», normal).
• Mutual location of two straight lines. An angle between two straight lines.
Conditions of parallelism and perpendicularity.
• Distance from a point to a straight line.
• Equation of a plane passing through a point perpendicularly to a given
vector.
• Equation of a plane passing through three given points.
• Equation of a circle, an ellipse, a parabola.
• Symmetric equations of a straight line in space.

3. Assignments
• To work out the equation of a straight line passing through two points,
through one point in the given direction.
• To work out equations of a plane passing through a point perpendicularly
to a vector, through three points.
• To find angles between straight lines and planes.
• To find an intersection point of a straight line and a plane.
• To reduce equations of the second order to the standard form and to sketch
their graphs.

86
Micromodule 7
BASIC THEORETICAL INFORMATION
STRAIGHT LINE ON A PLANE

General equation of a straight line, incomplete equations. Symmetric and


parametric equations of a straight line. A straight line passing through
two given points. An equation of a straight line in segments on axes, the
slope — intercept form. An angle between two straight lines, conditions
of parallelism and perpendicularity of two straight lines. Distance from a
point to a straight line.

Literature: [1, chapter 6], [4, part 3, item 3.3], [6, chapter 3, § 3], [7,
chapter 2, § 5], [9, chapter 2, § 2].

7.1. General equation of a straight line

Let us consider equations of a straight G line L passing through the point G


Μ 1 ( x1 , y1 ) perpendicularly to the vector n = { Α, Β} (Fig. 2.1, a). The vector n
is called the normal vector of the straight line. Let usG take an arbitrary point
JJJJJJ
Μ ( x, y ) on the line L and consider the vector Μ1 Μ = { x − x1 , y − y1 } . It is
G JJJJJJG
obvious that vectors n = { Α, Β} and Μ 1 Μ are perpendicular. According to
G JJJJJJG
condition of perpendicularity of two vectors their dot product is n ⋅ Μ 1 Μ = 0 ,
that is,

Α( x − x1 ) + Β ( y − y1 ) = 0. (2.1)

The equation (2.1) is called the equation of a straight line passing through
G
the point Μ 1 ( x1 , y1 ) perpendicularly to the normal n = { Α, Β} .
Removing brackets in the equation (2.1) and denoting − Αx1 − Βy1 = C we
receive the equation of the straight line L

Αx + Βy + C = 0, (2.2)

called a general equation of a straight line on a plane.


Let us consider special cases of location of a straight line depending on
values of coefficients A, B, C (Table 7.1):

87
Table 7.1
Equation
Condition Position of a straight line
of a straight line

А = 0, B ≠ 0 By + C = 0 It is parallel to the axis Ох

В =0, A ≠ 0 Ах + C = 0 It is parallel to the axis Оу

It passes through the origin of coor-


С=0 Ах + Ву = 0
dinates

А = 0, С = 0, B ≠ 0 у=0 It passes through the axis Ох

В = 0, С = 0, A ≠ 0 х=0 It passes through the axis Оу

7.2. Symmetric equation of a straight line

Let a straight line L pass through the point Μ 1 ( x1 , y1 ) and be parallel to the
G
vector a = {l , m} called a direction vector of the straight line L (Fig. 2.1, b). We
take any point Μ ( x, y ) on the straight line L.
JJJJJJG
Then two vectors Μ 1 Μ = { x − x1 , y − y1 } and a = {l , m} are collinear so
their coordinates must be proportional:

x − x1 y − y1
= . (2.3)
l m

The equation (2.3) is called the symmetric equation of the straight line.

7.3. Equation of a straight line passing through two given points

Let a straight line L pass through two points Μ 1 ( x1 , y1 ) and Μ 2 ( x2 , y2 )


JJJJJJJJG
(Fig. 2.1, c). Having chosen a vector Μ 1 Μ 2 = { x2 − x1 ; y2 − y1 } as a direction
vector of the straight line L in the equation (2.3) we get the following equation:

x − x1 y − y1
= , (2.4)
x2 − x1 y 2 − y1

that is, an equation of a straight line passing through two given points.

88
y y G L
G a = {l , m} y
n = { Α, Β}
L M2(x2, y2)
M (x, y) M1(x1, y1)
M (x, y) M1 (x1, y1)
O x O x O x
M1(x1, y1)
a b c
Fig. 2.1

7.4. Parametric equations of a straight line

x − x1 y − y1
Let us take = = t . From here we get
l m

⎧ x = x1 + lt ,
⎨ t∈R .
⎩ y = y1 + mt ,

These equations of a straight line are called parametric.

7.5. Equation of a straight line in segments on axes

Let a straight line L pass through two points Α(a, 0) and Β (0, b) , and cut
off segments on coordinate axes whose lengths are | a | and | b | (Fig. 2.2, а).
Having substituted coordinates of points Α(a, 0) and Β (0, b) in the equation
(2.4) we get the equation

x y
+ = 1, (2.5)
a b

called the equation of a straight line in segments on axes.


The general equation of a straight line (2.2) may be reduced to such form
(2.5) only if all its coefficients are not zeros. Then

c c
where − = a, − = b .
Α Β

89
Remark. The straight line Αx + Βy + C = 0 in case A ≠ 0 , B ≠ 0 and C ≠ 0
crosses coordinate axes at points (−C / A; 0) and (0; −C / B) . To find coor-
dinates of these points it is enough to solve the equation (2.2) provided х= 0 and
y = 0 in turn.

7.6. Equation of a straight line in slope–intercept form


If a straight line L passes through a point Μ 0 ( x0 , y0 ) and forms an angle α
with the positive direction of the abscissa axis (Fig. 2.2, b) then the number
k = tg α is called a slope of the straight line.
Having taken any point Μ ( x, y ) on the straight line, we get from a triangle
Μ 0 ΜK
ΜK y − y0
k = tg α = = ,
Μ 0 K x − x0
whence y − y0 = k ( x − x0 ).
It is the equation of the straight line L passing through the point Μ 0 ( x0 , y0 )
and having the slope k. If we take a point B (0, b) as the point Μ 0 we shall get
the equation
y = kx + b, (2.6)
called the equation of a straight line in the slope — intercept form and b is the
ordinate of the intersection point of the straight line with the axis Оу.

y y L
L y M (x,y)
B (0, b) M0 α
y0 K
│b│
А(а, 0) α
О x0 x x
О x B(0,b)
│a│

а b
Fig. 2.2

7.7. Angle between two straight lines.


Conditions of parallelism and perpendicularity of straight lines
For the given straight lines whose equations are y = k1 x + b1 and y = k2 x + b2
(Fig. 2.3) an acute angle between these straight lines is ϕ =|α 2 − α1 | , therefore

90
tg α 2 − tg α1 k −k
tg ϕ =|tg(α 2 − α1 ) |= = 2 1 . (2.7)
1 + tg α1 tg α 2 1 + k1 k2

y
L2 φ
L1 G
n
M0
d
α1 α2 M1 L
О x
Fig. 2.3 Fig. 2.4

If two straight lines L1 і L2 are parellel then ϕ = 0, tg ϕ = 0, so k2 − k1 = 0.


Therefore the condition of parallelism of two straight lines is
k1 = k2 . (2.8)
π
If two straight lines L1 і L2 are perpendicular then ϕ = , tg ϕ does not
2
exists, so 1 + k1 k2 = 0 by the formula (2.7). Therefore the condition of per-
pendiculariry is:
1
k2 = − . (2.9)
k1

Let two straight lines L1 і L2 be given by their general equations


Α1 x + Β1 y + C1 = 0 and Α2 x + Β2 y + C2 = 0. Then
1) the angle between these straight lines is equal to the angle between their
G G
normal vectors n1 = { Α1 , Β1 } аnd n2 = { Α2 , Β2 } . This angle is determined from
the equality:
Α1 A2 + B1 Β2
cos ϕ = ;
Α12 + Α22 Β12 + Β22

2) the condition of parallelism of straight lines is:

Α1 Β
= 1;
Α2 Β2

3) the condition of perpendicularity is:

Α1 Α2 + Β1 Β2 = 0.

91
7.8. Distance from a point to a straight line

Let a straight line L be given by the equation Αx + Βy + C = 0 and a point


M0 ( x0 ; y0 ) be given too (Fig. 2.4).
A distance d from the point M0 to the straight line L equals the module of a
JJJJJJJJG
projection of the vector Μ 1 Μ 0 where M1 ( x1 ; y1 ) is any point of the straight line
G
L onto the direction of a normal vector n = { Α; Β} . So
JJJJJJJJG
JJJJJJJJG | Μ1 Μ 0 ⋅ nG | | ( x0 − x1 ) ⋅ A + ( y0 − y1 ) ⋅ B |
d =| пр n Μ1 Μ 0 |=
G G = =
|n| A2 + B 2
| Ax0 + By0 − ( Ax1 + By1 ) | | Ax0 + By0 + C |
= = .
2 2
A +B A2 + B 2
So a distance from a point to a straight line may be found by the formula
Αx0 + By0 + C
d= . (2.10)
Α2 + B 2

Micromodule 7
EXAMPLES OF PROBLEMS SOLUTION

Example 1. Given points A(3; 1) , B (2; −3), C(−1; 2) are vertices of a


triangle ABC. Form:
а) a general equation of the side AB;
b) a symmetric equation of the height AD;
c) parametric equations of the median ВМ;
d) an equation of a straight line passing through the point C (−1; 2) and paral-
lel to the side AB.
Solution. Let us construct a triangle ABC (Fig. 2.5) and consider the fol-
lowing cases.
а) As we know coordinates of the points A and B we can get the equation of
the side AB according to the formula (2.4):
x − 3 y −1 x − 3 y −1
= , or = , or y −1 = 4( x − 3) .
2 − 3 −3 − 1 −1 −4
Whence 4 x − y − 11 = 0. The latter is a general equation of the straight line
containing the side AB.
b) To write down symmetric equations of a straight line it is necessary to
know a point lying on the straight line, and its direction vector. The vector

92
JJJG
BC = {−3; 5} for the height AD is a normal vector then the vector a = {5; 3} will
JJJG
be perpendicular to the vector BC (as their dot product BC ⋅ a = 0 ). Now we
x − 3 y −1
can write down the symmetric equation of the straight line AD: = .
5 3
c) As the point M is a midpoint of the line segment АС then
3 −1 1+ 2
xM = = 1, yM = = 1,5 .
2 2
JJJJG
The vector BM = {1 − 2; 1,5 + 3} = {−1; 4,5} is a direction vector of the
straight
JJJJline
G BM . For the direction vector it is also possible to take a vector
a = 2 BM = {−2; 9} . So l = −2 , m = 9 and the parametric equations of the
median may be written as:
⎧ x = 2 − 2t ,
⎨ y = −3 + 9t , t ∈ R.

d) So as a straight line passes through the point C (−1; 2) and is parallel to
the side AB we can take a normal vector n = {4; − 1} of the straight line АВ
for the normal vector of the required straight line. Then the required equation
looks like
4( x + 1) − ( y − 2) = 0 , or 4 x − y + 6 = 0 .
Example 2. Calculate the area of a triangle bounded by a straight line
passing through two points Μ 1 (−3; −4) and Μ 2 (6; 2) , and axes of coordinates
(Fig. 2.6).
Solution. Using the formula (2.4), we shall make an equation of a straight
line passing through the points Μ 1 (−3; −4) and Μ 2 (6; 2) :

x+3 y+4 x+3 y+4 x+3 y+4


= , = , = ,
6+3 2+4 9 6 3 2
2( x + 3) = 3( y + 4), 2x − 3y − 6 = 0 .

The latter formula is the general equation of straight line Μ 1 Μ 2 .


Let us find coordinates of intersection points of the straight line with the
coordinate axes.
Let х = 0, then 3b + 6 = 0, or b = −2. If y = 0, then 2a − 6 = 0, or a = 3.
So the straight line intersects the axis Ox at the point A (3; 0), and the axis
Оу at the point B(0; −2). Lengths of legs in the triangle AOB are OΑ = 3, OB = 2
respectively, then the area of the triangle is:
S ΑΟΒ = 3 sq. units.

93
у
у
M2
С M 2
А
–3 О А 6 х
D х
В
M1
В –4

Fig. 2.5 Fig. 2.6

Example 3. Find distance between straight lines 4 x − 3 y + 2 = 0 and 8x −


− 6 y −13 = 0 .
Solution. So as the given straight lines are parallel, distance between them is
constant and equal to distance from any point of the second straight line to the
first. We take any point on the straight line 8 x − 6 y −13 = 0. Let x = 0 , then
13
y=− . Distance from the point M (0; − 13/ 6) to the straight line 4 x − 3 y + 2 = 0
6
may be found by the formula (2.9):

13
4 ⋅ 0 − 3 ⋅ (− )+2
d= 6 = 1, 7.
16 + 9
Example 4. Find m and n for which straight lines mx + 8 y + n = 0 and
2 x + my − 1 = 0 а) are parallel; b) coincide; c)are perpendicular.
m 8
Solution. а) Condition of parallelism is: = , whence m 2 = 16 , m = ±4 ;
2 m
m 8 n
b) The straight lines coincide if the condition = = holds.
2 m −1
From this we receive two pairs of values m = 4, n = −2 or m = −4, n = 2 .
c) Condition of perpendicularity of straight lines is m × 3 + 8 × m = 0, that
is m = 0 .
Answers: а) m = ±4 , n ∈ R ; б) m = 4, n = −2 ; m = −4, n = 2 ;
c) m = 0 , n ∈ R .

Example 5. Find a point symmetric to the point P (−8; 12) relative to the
straight line 4 x + 7 y + 13 = 0 .

94
Solution. Let Q ( xQ , yQ ) be the required point (Fig. 2.7). The problem may
be solved in such a sequence:

Р(–8; 12)

n ={4; 7}
O
L
Q

Fig. 2.7

1) we make equations of a straight line РО passing through the point Р and


perpendicular to the straight line L;
2) we find an intersection point O of the straight lines L and РО. It is a
projection of the point Р onto the straight line L;
3) we determine coordinates of the point Q, taking into account that the point
O is a midpoint of PQ.
The normal vector of the straight line L at the same time is a direction vector
of the perpendicular РО, therefore the symmetric equation of the straight line
РО has such a form:
x + 8 y −12
= ,
4 7
whence we receive 7( x + 8) = 4( y −12), 7 x − 4 y + 104 = 0.
Having solved the system of equations
⎧7 x − 4 x + 104 = 0,
⎨4 x + 7 x + 13 = 0,

we can find coordinates of the intersection point of straight lines:
xO = −12, yO = 5.
Connection between coordinates of points P, O and Q is well known:
xP + xQ yP + yQ
xO = , yO = .
2 2
Then
−8 + xQ 12 + yQ
−12 = , 5= ,
2 2
whence we receive coordinates of the symmetric point:
xQ = −16, yQ = −2.

95
Micromodule 7
CLASS AND HOME ASSIGNMENTS

1. Sketch graphs of straight lines y = 3x − 2 and x + 2 y − 3 = 0 , and find a


point of their intersection.
2. The area of a triangle АВС is 8, two its vertices are points A (0; –3) and B
(1; 2). Find coordinates of the vertex C if it lies on the straight line y = 3 .
3. Let the points A(0; –4), B (3; 2), C(–5; 1) be vertices of a triangle. Find:
а) a general equation of the side AB;
b) a symmetric equation of the height AD;
c) a parametric equation of a median BM;
d) distance of a vertex B to the median AM.
4. Find projection of a point Р (–7; 11) onto a straight line passing through
points A (3;−4) and B(−4; 0).
5. Show that straight lines 5 x −12 y + 13, 5 = 0 and 10 x − 24 y = 25 are
parallel and find distance between them.
6. Let a point A (3; 4) be a vertex of a rectangular triangle. ∠ A = 45° and
x − y + 2 = 0 is an equation of the leg opposite to the vertex A. Find equations
of two other sides of the triangle.
7. An equation of a straight line 3x + 2 y + 7 = 0 is given. Find equations of
straight lines passing through the point M (–2; 0), and:
а) parallel to the given straight line;
b) perpendicular to the given straight line.

Answers
x y+4
2. С (–2; 3) or (4,4; 3). 3. а) 2 x − y − 4 = 0 ; b) = . 4. (–11; 4).
1 −8
5. 2. 7. а) 3 x + 2 y + 6 = 0 ; b) 2 x − 3 y + 4 = 0 .

Micromodule 7
SELF-TEST ASSIGNMENTS

7.1. Given points M 1 , M 2 , M 3 are vertices of a triangle M1 M 2 M 3 . Find


a) a general equation of the side M 1 M 2 ;
b) a symmetric equation of the height M 1 D ;
c) a parametric equations of the median M 2 M ;
d) an equation of a straight line passing through the point M 3 and parallel to
the side M 1 M 2 .
e) a projection of the point M 1 onto the straight line M 2 M 3 .

96
7.1.1. M 1 (0; 1) , M 2 (6; 3) , M 3 (−1; 0) .
7.1.2. M 1 (0; 6) , M 2 (1; − 3) , M 3 (−2; 3) .
7.1.3. M 1 (6; − 2) , M 2 (−4; −1) , M 3 (0; − 2) .
7.1.4. M 1 (2; 2) , M 2 (−1; 7) , M 3 (1; 4) .
7.1.5. M 1 (1; 8) , M 2 (0; − 3) , M 3 (−1; 2) .
7.1.6. M 1 (−5; −1) , M 2 (−3; 0) , M 3 (1; −2) .
7.1.7. M 1 (0; −2) , M 2 (−3; 6) , M 3 (5; 3) .
7.1.8. M 1 (3; −2) , M 2 (2; 7) , M 3 (−2; 1) .
7.1.9. M 1 (0; 5) , M 2 (11; −5) , M 3 (−1; −1) .
7.1.10. M 1 (1; 0) , M 2 (6; 1) , M 3 (3; −2) .
7.1.11. M 1 (−1; 4) , M 2 (11; 5) , M 3 (0; 1) .
7.1.12. M 1 (4; 6) , M 2 (−6; 3) , M 3 (−2; 0) .
7.1.13. M 1 (2; −1) , M 2 (5; 0) , M 3 (−2; −2) .
7.1.14. M 1 (−4; 5) , M 2 (2; 0) , M 3 (−2; −2) .
7.1.15. M 1 (7; 5) , M 2 (−1; 2) , M 3 (1; −1) .
7.1.16. M 1 (3; 2) , M 2 (3; 5) , M 3 (1; −1) .
7.1.17. M 1 (−3; 9) , M 2 (7; −2) , M 3 (3; 3) .
7.1.18. M 1 (0; 1) , M 2 (6; 4) , M 3 (−1; 0) .
7.1.19. M 1 (2; −4) , M 2 (4; −2) , M 3 (0; 2) .
7.1.20. M 1 (5; −7) , M 2 (−5; 1) , M 3 (−1; −1) .
7.1.21. M 1 (1; 3) , M 2 (3; 2) , M 3 (5; 0) .
7.1.22. M 1 (−1; −2) , M 2 (2; −1) , M 3 (0; −1) .
7.1.23. M 1 (7; 1) , M 2 (−1; 0) , M 3 (−2; 3) .
7.1.24. M 1 (0; 1) , M 2 (−1; 4) , M 3 (6; 0) .
7.1.25. M 1 (−4; −9) , M 2 (6; −1) , M 3 (2; 1) .
7.1.26. M1 (−1; 1) , M 2 (2; −6) , M 3 (1; 2) .
7.1.27. M 1 (1; 2) , M 2 (−2; 1) , M 3 (−4; 3) .
7.1.28. M 1 (−1; 4) , M 2 (−5; 1) , M 3 (1; −1) .
7.1.29. M 1 (3; −4) , M 2 (−2; 5) , M 3 (0; 4) .
7.1.30. M 1 (13; − 3), M 2 (2; 5) , M 3 (−2; 1) .

7.2. Find the distance between straight lines


7.2.1. 2 x + 5 y −12 = 0 and 4 x + 10 y −11 = 0 .
7.2.2. 3x − 4 y − 7 = 0 and −6 x + 8 y −15 = 0 .
7.2.3. 2 x − 5 y −11 = 0 and −6 x + 15 y −17 = 0 .
7.2.4. 3x − 7 y − 7 = 0 and 12 x − 28 y − 25 = 0 .

97
7.2.5. 5 x + 6 y + 22 = 0 and 10 x + 12 y − 31 = 0 .
7.2.6. x − 7 y − 32 = 0 and 2 x −14 y −13 = 0 .
7.2.7. 3 x + 5 y + 5 = 0 and 9 x + 15 y −17 = 0 .
7.2.8. 3x − 8 y − 27 = 0 and −6 x + 16 y + 11 = 0 .
7.2.9. 2 x − 9 y − 37 = 0 and −6 x + 27 y −10 = 0 .
7.2.10. 3x − 4 y −18 = 0 and 15 x − 20 y − 41 = 0 .
7.2.11. x + 6 y −14 = 0 and 4 x + 24 y − 23 = 0 .
7.2.12. 3 x − 7 y − 8 = 0 and 9 x − 21 y −16 = 0 .
7.2.13. 3x − 5 y −19 = 0 and 6 x −10 y − 21 = 0 .
7.2.14. −3 x − 4 y − 28 = 0 and 9 x + 12 y + 7 = 0 .
7.2.15. 4 x − 3 y + 7 = 0 and 8 x − 6 y −11 = 0 .
7.2.16. 5 x − 4 y − 48 = 0 and 15 x −12 y − 5 = 0 .
7.2.17. 5 x + 3 y − 43 = 0 and 20 x + 15 y + 22 = 0 .
7.2.18. 7 x − 2 y −15 = 0 and 14 x − 4 y − 5 = 0 .
7.2.19. 3x − y − 6 = 0 and 15 x − 5 y − 32 = 0 .
7.2.20. 3 x + 7 y + 42 = 0 and 12 x + 28 y − 61 = 0 .
7.2.21. 6 x + 7 y + 16 = 0 and 12 x + 14 y − 21 = 0 .
7.2.22. 8 x − 5 y + 32 = 0 and 16 x −10 y −17 = 0 .
7.2.23. 6 x + 11y + 22 = 0 and 18 x + 33 y − 43 = 0 .
7.2.24. 3x + 10 y + 27 = 0 and 9 x + 30 y −11 = 0 .
7.2.25. 4 x − 7 y + 35 = 0 and 12 x − 21 y − 62 = 0 .
7.2.26. −3 x + 5 y + 4 = 0 and 18 x − 30 y −1 = 0 .
7.2.27. 2 x + 5 y + 12 = 0 and 10 x + 25 y −13 = 0 .
7.2.28. −4 x + 3 y + 17 = 0 and 12 x − 9 y − 22 = 0 .
7.2.29. 5 x + 7 y + 16 = 0 and 15 x + 21 y − 44 = 0 .
7.2.30. 11x + 5 y + 15 = 0 and 22 x + 10 y − 7 = 0 .
7.3. Find the area of a triangle cut off from the coordinate angle by a straight line:
7.3.1. 4 x + 5 y − 40 = 0 . 7.3.2. 4 x − 5 y + 25 = 0 .
7.3.3. 5 x + 8 y − 35 = 0 . 7.3.4. 6 x −11 y −128 = 0 .
7.3.5. 3 x + 13 y −195 = 0 . 7.3.6. 2 x + 7 y −140 = 0 .
7.3.7. 7 x −12 y −168 = 0 . 7.3.8. 7 x + 5 y −140 = 0 .
7.3.9. 13x + 5 y − 260 = 0 . 7.3.10. 4 x + 7 y − 560 = 0 .
7.3.11. 3x + 7 y − 210 = 0 . 7.3.12. 8 x + 13 y − 208 = 0 .
7.3.13. 6 x − 7 y − 210 = 0 . 7.3.14. 9 x + 4 y −180 = 0 .

98
7.3.15. 2 x + 7 y − 35 = 0 . 7.3.16. 9 x + 7 y −126 = 0 .
7.3.17. 4 x − 7 y − 420 = 0 . 7.3.18. 3x + 11y + 132 = 0 .
7.3.19. 4 x + 5 y + 45 = 0 . 7.3.20. 6 x + 7 y + 21 = 0 .
7.3.21. 5 x − 7 y −175 = 0 . 7.3.22. 8 x − 5 y + 50 = 0 .
7.3.23. 6 x + 13 y + 130 = 0 . 7.3.24. 5 x + 8 y + 75 = 0 .
7.3.25. 3x + 5 y + 80 = 0 . 7.3.26. 4 x + 9 y + 81 = 0 .
7.3.27. 3x − 13 y − 169 = 0 . 7.3.28. 9 x + 5 y + 75 = 0 .
7.3.29. 4 x + 11 y + 121 = 0 . 7.3.30. 5 x − 8 y − 64 = 0 .

7.4. Determine values m and n for which straight lines are:


а) parallel; b) coincident; c) perpendicular.
7.4.1. mx + 6y + n = 0 and 3x + my-2 = 0.
7.4.2. mx + 7y + n = 0 and 2x + my – 5 = 0.
7.4.3. (m + 1)x + 5y + 2n = 0 and 3x + my – 4 = 0.
7.4.4. (m – 2)x + 3y + n + 2 = 0 and x – my – 2 = 0.
7.4.5. (m + 4)x + 2y + n – 1 = 0 and 2x + my – 6 = 0.
7.4.6. (m – 4)x – y + n – 2 = 0 and 3x + my – 5 = 0.
7.4.7. (m + 2)x + 4y + n – 3 = 0 and 3x – my + 4 = 0.
7.4.8. (m + 5)x + 4y + n + 1 = 0 and x + 2my – 3 = 0.
7.4.9. m x + 3 y + n – 3 = 0 and 3x + (m + 1)y – n = 0.
7.4.10. (m – 2)x + 5y + n – 4 = 0 and x – (m + 1)y – 6n = 0.
7.4.11.. (m – 3)x + y + n – 4 = 0 and x – (m + 1)y – 6n = 0.
7.4.12. (m + 3)x + 2y + n – 5 = 0 and 2x – (m + 2)y – 4n = 0.
7.4.13. (m – 1)x + y + n + 2 = 0 and 3x – (m – 1)y – 2n = 0.
7.4.14. (m – 2)x + 3y + n – 3 = 0 and x + (m + 1)y + 2n = 0.
7.4.15. (m + 3)x + 2y + 2n – 1 = 0 and x – my – 6 + n = 0.
7.4.16. (m – 1)x + 4y + 3n – 4 = 0 and x – (m + 2)y – n = 0.
7.4.17. (m – 3)x + y + n – 4 = 0 and x – (m + 1)y – 6n = 0.
7.4.18. ( m – 5)x + 2y + 2n – 3 = 0 and x – (m + 2)y – n – 1 = 0.
7.4.19. ( m + 2)x + y – n – 4 = 0 and x + (m + 1)y + 2n = 0.
7.4.20. ( m + 1)x – y + n – 3 = 0 and x – (m – 1)y – 3n = 0.
7.4.21. ( m – 6)x – y + n – 2 = 0 and x + (m + 2)y – n – 1 = 0.
7.4.22. mx + 2y + 3n – 2 = 0 and x – (m + 4)y – 2n = 0.
7.4.23. mx – y + 2n – 5 = 0 and x + (m + 2)y + n = 0.
7.4.24. (m – 1)x – y + 2n – 1 = 0 and 2x – my – n + 1 = 0.
7.4.25. (2m – 1)x + y-n – 3 = 0 and x – (m + 1)y – n = 0.
7.4.26. mx – y + 3n – 1 = 0 and x – (m – 1)y – n – 1 = 0.

99
7.4.27. (m – 3)x – y + 2n – 4 = 0 and x + my – n + 1 = 0.
7.4.28. mx + 2y + n – 2 = 0 and x + (m + 2)y + 2n – 1 = 0.
7.4.29. (m – 1)x + 2y + n = 0 and x – (m – 1)y – n – 2 = 0.
7.4.30. mx + 3y + 2n – 1 = 0 and x + (m + 3)y + n = 0.

Micromodule 8
BASIC THEORETICAL INFORMATION. PLANE

General equation of a plane, incomplete equations. An equation of a


plane passing through three points. An equation of a plane in segments on
axes. A normal equation of a plane, the distance from a point to a plane.
An angle between two planes, conditions of parallelism and perpendicu-
larity of planes.

Literature: [1, chapter 8, item 8.2, 8.3], [4, item 3, item 3.5], [6, chapter 3,
§ 4], [7, chapter 2, § 6], [10, chapter 2, § 2], [11, chapter 2, § 2].

8.1. General equation of a plane

Only one plane passes through the point Μ 0 ( x0 , y0 , z0 ) perpendicularly to


G G
the vector n = { A; B; C} (Fig. 2.8). The vector n is called a normal vector of
that plane.
G
n ={A, B, C}

М
М0

Fig. 2.8

Let us find an equation of this plane. Let Μ ( x, y, z ) be any point of the


G JJJJJG
plane. Then two vectors n = { A; B; C} and Μ 0 Μ = {x − x0 ; y − y0 ; z − z0 } are
perpendicular, so their dot product is equal to zero:
Α( x − x0 ) + Β ( y − y0 ) + C ( z − z0 ) = 0. (2.11)
The equation (2.11) is an equation of a plane passing through the point Μ 0
G
and having a normal vector n . Removing brackets and denoting − Αx0 −
− Βy0 − Cz0 = D we get an equation

100
Αx + Βy + Cz + D = 0, (2.12)
which is called a general equation of a plane.
Remark. For arbitrary values of A, B and C, which are not equal to zero, the
equation (2.11) defines a bundle of planes, i.e. a set of planes which pass
through the given point Μ 0 ( x0 , y0 , z0 ) that is a centre of the bundle.

8.2. Incomplete equations of a plane

The equation (2.12) is called incomplete if at least one of the coefficients


A,B,C ,D is equal to zero. Some cases of the incomplete general equation of a
plane are shown in Table 8.1.
Table 8.1

Condition The equation of a plane Position of a plane

It passes through the origin


D=0 Ax + By + Cz = 0
of coordinates
А=0 By + Cz + D = 0 It is parallel to an axis Ох
А = 0, В = 0 Cz + D = 0 It is parallel to a plane Оху
А = 0, В = 0, D = 0 z=0 It is the plane Оху
А = 0, D = 0 By + Cz = 0 It passes through an axis Ох

Other cases of the incomplete equations may be considered in the similar


manner.

8.3. Equations of a plane passing through three given points

Let three points Μ 1 ( x1 , y1 , z1 ) , Μ 2 ( x2 , y2 , z2 ), Μ 3 ( x3 , y3 , z3 ) be given.


They do not lie on the same straight line (Fig. 2.9). These points unequivocally
determine a plane. We shall find an equation of this plane.
To this end we shall take any point Μ ( x, y, z ) in this plane and we shall
find vectors:
JJJJJG
M2 M 1 M = {x − x1 ; y − y1 ; z − z1 } ,
M3
JJJJJG
M Μ1 Μ 2 = {x2 − x1 ; y2 − y1 ; z2 − z1 }
М1
JJJJJG
Μ 1 Μ 3 = {x3 − x1 ; y3 − y1 ; z3 − z1 } .
Fig. 2.9

101
So as all four points as well as the found vectors lie in one plane, these
vectors are coplanar.
G JJJJJJJG Due to the condition of coplanarity their triple product
JJJJJG JJJJJJJ
M1M ⋅ M1M 2 ⋅ M1M 2 = 0. It may be represented in coordinate form

x − x1 y − y1 z − z1
x2 − x1 y2 − y1 z2 − z1 = 0. (2.13)
x3 − x1 y3 − y1 z3 − z1

The formula (2.13) is the equation of a plane passing through three given
points.
Having expanded this determinant by elements of the first row we can get a
general equation of a plane.

8.4. Equation of a plane in segments on axes

Let a plane intersect coordinate axes at points Α(a; 0; 0) , Β (0; b; 0) and


C (0; 0; c). (Fig. 2.10.) Then the equation (2.13) looks as following:

x−a y z
−a b 0 =0
−a 0 c

It is equivalent to the equation


x y z
+ + = 1. (2.14)
a b c
The latter (2.14) is called a plane equation in segments on axes.

8.5. Normal equation of a plane. Distance from a point to a plane

The normal equation of a plane may be received if the plane is defined by


the length p of the perpendicular ΟP dropped from the origin of coordinates
onto the plane and by the angles α, β, γ formed by the perpendicular ΟP with
coordinate axes (Fig.2.10).
Let us take a point M ( x, y, z ) in the plane. It is easy to see that the
JJJJG G
projection of the vector OM onto the normal vector of the plane n0 =
JJJJG
= {cos α, cos β, cos γ} is equal to p: PrnJJG OM = p .
0

102
So we get
JJJJG G
OM ⋅ n0 x ⋅ cos α + y ⋅ sin β + z ⋅ cos γ
p := G = .
n0 cos 2 α + cos 2 β + cos 2 γ

z
z
C(0, 0, c)

Р
p
М
О y G
γ n0
B(0, b, 0) α β
A(a, 0, 0) О y
x х
Fig. 2.10 Fig. 2.11

So as cos 2 α + cos 2 β + cos 2 γ = 1 we get the final equation


x ⋅ cos α + y ⋅ cos β + z ⋅ cos γ − p = 0. (2.15)
The equation (2.15) is called the normal equation of a plane.
Properties of this equation:
a) the sum of squares of coefficient before x, y, z is equal to 1;
b) the constant term is introduced in the equation with the sign «–».
To reduce the general equation (2.12) to the normal view it is necessary to
multiply it by so called the normalizing factor.
1
μ=± ,
A + B2 + C 2
2

whose sign is chosen opposite to the sign D .


The normal equation (2.15) is applied to find the distance d of the point
Μ 0 ( x0 , y0 , z0 ) to the plane. This distance may be calculated by the formulas

d = x0 cos α + y0 cos β + z0 cos γ − p ,


or
Ax0 + By0 + Cz0 + D
d= . (2.16)
A2 + B 2 + C 2
The value
δ = x0 cos α + y0 cos β + z0 cos γ,

is named a deviation of the point Μ 0 ( x0 , y0 , z0 ) from a plane.

103
If the point Μ 0 and the origin of coordinates lie on the same side out of the
plane, then, δ < 0 and otherwise δ > 0.

8.6. Angle between two planes, conditions of parallelism


and perpendicularity of planes

Given planes
α1: Α1 x + Β1 y + C1 z + D1 = 0 , α2: Α2 x + Β2 y + C2 z + D2 = 0 .
An angle between these planes may be found by the formula
G G
n ⋅n Α1 Α2 + Β1 B2 + C1C2
cos ϕ = G1 G2 = .
| n1|⋅| n1| Α12 + B12 + C12 Α22 + B22 + C22
The condition of parallelism of planes is
A1 B1 C1
= = .
A2 B2 C2

The condition of perpendicularity of planes is


Α1 Α2 + Β1 B2 + C1C2 = 0.

Micromodule 8
EXAMPLES OF PROBLEMS SOLUTION

Example 1. Coordinates of four points are given M 0 (2; 3; 1), M1 (1; 2; −1),
M2 (3;1;−2), M 3 (−2; 3; − 2). Find:
a) an equation of the plane M 1 M 2 M 3 ;
b) an equation of the plane passing through the point M 0 and parallel to the
plane M1 M 2 M 3 ;
c) an equation of the plane passing through the point M 0 perpendicular to
JJJJJJJG
the vector M1M 3 ;
d) distance from the point M 0 to the plane M1 M 2 M 3 .
Solution. a) We can take an arbitrary point M ( x, y, z) on the plane M1 M2 M3
and consider three vectors
JJJJJG JJJJJJJG JJJJJJJG
M 1M = { x − 1, y − 2, z + 1} , M 1M 2 = {2, − 1, − 1} , M 1M 3 = {−3; 1; − 1} .

104
According to the formula (2.13) we get the equation

x −1 y − 2 z + 1
2 −1 −1 = 0,
−3 1 −1

It may be rewritten as a general equation of the plane M 1 M 2 M 3 .

2 x + 5 y − z − 13 = 0.

b) We can write down the equation of a plane passing through the point M 0
and parallel to the plane M1 M 2 M 3 taking into account the fact that parallel
planes have collinear normal vectors. The normal vector of the plane M 1 M 2 M 3
G
is a vector n = {2; 5; − 1} .
By the formula (2.11) the equation of a desired plane looks as follows:
2( x − 2) + 5( y − 3) − ( z −1) = 0 or 2 x + 5 y − z −18 = 0 .

с) To write down the equation of a plane passing through the point M 0


JJJJJJJG
perpendicularly to the vector Ì 1Ì 3 , we take advantage of the equation (2.11)
JJJJJJJG
where coordinates of the vector Ì 1Ì 3 are coordinates of a normal vector:
−3( x − 2) + ( y − 3) − ( z − 1) = 0, or 3x − y + z − 4 = 0.
d) The distance from the point M 0 (2; 3; 1) to the plane M1 M 2 M 3 , given by
the equation 2 x + 5 y − z −13 = 0, may be found by the formula (2.16):

2 ⋅ 2 + 5 ⋅ 3 − 1 − 13 5
d = = .
2 2
2 + 5 + ( − 1) 2 30

Example 2. Find an equation of a plane passing through the point M 0 (1; − 2; 4)


G G
and parallel to two vectors a = {2; − 1; 0} and b = {3; − 1; 3} .
G G
G Solution. So as the plane is parallel to the vectors a and b the normal vector
n of the required plane is perpendicular to these vectors. Therefore weGcan take
G G G G
a cross product of the vectors a and b for the normal vector: n = a × b (recall
the definition of a cross product of two vectors), that is
G G G
G G G i j k G G G
n = a × b = 2 −1 0 = −3i − 6 j + k .
3 −1 3

105
By the formula (2.11) we write down the equation of the required plane
−3( x − 1) − 6( y + 2) + ( z − 4) = 0 , or 3 x + 6 y − z + 13 = 0 .
Example 3. Evaluate the volume of the pyramid bounded by the coordinate
planes and the plane 3x − 5 y + 2 z − 30 = 0 .
Solution. So as the pyramid is rectangular it is easy to find its volume by the
1
formula V = OA ⋅ OB ⋅ OC where А, В, С are intersection points of the plane
6
with coordinate axes. (see Fig. 2.10).
On the axis Оx coordinates y and z are equal to zero, therefore having
substituted y = 0 and z = 0 in the equation of the last plane, we can find
3x − 30 = 0 , or x = 10 . So Α(10; 0; 0) is an intersection point of that plane
with the axis Оx. Similarly we can find intersection points of the plane with the
axes Оу and Оz.
So, | OA |= 10 , | OB |= 6 , | OC |= 15 and
1
V = 1 0 ⋅ 6 ⋅ 1 5 = 1 5 0 (of cubic units).
6

Micromodule 8
CLASS AND HOME ASSIGNMENTS

1. Coordinates of points M 0 (0; −2; 4) , M1 (1; 5; −5) , M 2 (3; 0; −2) ,


M 3 (−1; 3; 2) are given. Find:
а) an equation of the plane M1 M 2 M 3 ;
b) an equation of the plane passing through the point M 0 parallelly to the
plane M1 M 2 M 3 ;
c) an equation of the plane passing through the point M 0 and perpendicular
JJJJJJJG
to the vector M 1M 3 ;
d) distance of the point M 0 to the plane M1 M 2 M 3 .
2. Find an equation of the plane passing through the point M 0 (2; −1; 6)
G G
and parallel to the vectors a = {3; − 2; 1} and b = {−1; − 4; 3} .
3. Find an equation of the plane passing through the points M 1 (2; 3; −1)
G
and M 2 (0; 2; 2) and parallel to the vector a = {4; − 1; 3} .
4. Evaluate the volume of the pyramid bounded by the plane 2 x + 3 y −
− 6 z = 24 and coordinate planes.
5. Find direction cosines of a normal vector of the plane 2 x + 6 y −
− 3 z + 14 = 0 and evaluate the distance from the origin to this plane.

106
6. Show that three planes x − 2 y + z − 7 = 0 , 2 x + y − z + 2 = 0 and x −3y +
+ 2 z −11 = 0 have a common point and find coordinates of this point.
7. Prove that the plane 3x − 4 y − 2 z + 5 = 0 intersects a line segment
connecting two points M 1 (3; −2; 1) and M 2 (−2; 5; 2) .

Answers
1. а) 29 x + 20 y + 14 z − 59 = 0 ; b) 29 x + 20 y + 14 z −16 = 0 ; c) 2 x + 2 y −
16 2
− 7 z + 32 = 0; d) . 2. x + 5 y + 7 z = 39 . 3. 3y + z = 8. 4. 64. 5. cos α = ,
1437 7
6 3
cos β = , cos γ = − ; d = 2 . 6. (1; –2; 2). 7. Hint. You have to show that
7 7
deviations of points M1 and M 2 from the plane have opposite sings.

Micromodule 8
SELF–TEST ASSIGNMENTS

8.1. Coordinates of points M 0 , M 1 , M 2 , M 3 are given. Find:


а) an equation of the plane M1 M 2 M 3 ;
b) an equation of the plane passing through the point M 0 parallel to the
plane M 1 M 2 M 3 ;
в) an equation of the plane passing through the point M 0 and perpendicular
JJJJJJJG
to the vector Ì 1Ì 3 .
8.1.1. M0 (0; –1; 1), M1(1; 0; 1), M2(4; 6; 1), M3(6; –1; 0).
8.1.2. M0 (0; 1; 1), M1(–13; 0; 6), M2(10; 1; –3), M3(–2; 1; 3).
8.1.3. M0 (0; 4; 1), M1(6; –8; –2), M2(–4; 10; –1), M3(0; –2; –3).
8.1.4. M0 (0 ;1; 2), M1(2; 0; 2), M2(8; –1; 7), M3(12; 1; 1).
8.1.5. M0 (0; 1; –2), M1(1; –12; 8), M2(0; 11; –10), M3(0; –1; 2).
8.1.6. M0 (1; –1; 0), M1(7; –5; –1), M2(–3; 13; 0), M3(1; 1; –2).
8.1.7. M0 (1; 3; 1), M1(0; –2; –1), M2(–3; –1; 6), M3(–5; –3; 0).
8.1.8. M0 (1; 2; 3), M1(14; 3; –2), M2(–9; 2; 7), M3(3; 2; 1).
8.1.9. M0 (–3; 1; –1), M1(–7; 0; 5), M2(11; 1; –5), M3(–1; –1; –1).
8.1.10. M0 (0; –1; 1), M1(1; 0; 1), M2(4; 6;1), M3(6; –1; 0).
8.1.11. M0 (1; 0;–1), M1(–2; –1; 4), M2(11; 0; 5), M3(–1; 0;1).
8.1.12. M0 (–2; –2; 3), M1(4; 6; 2), M2(–6; 12; 3), M3(–2; 0; 1).
8.1.13. M 0 (1; −2; −1) , M 1 (2; −1; −1) , M 2 (5; 0; 4) , M 3 (7; −2; −2) .

107
8.1.14. M 0 (2; 0; 0) , M 1 (−4; 5; 1) , M 2 (2; 0; −4) , M 3 (−2; 0; −2) .
8.1.15. M 0 (3; −1; 2) , M 1 (7; 5; 0) , M 2 (−1; −5; 2) , M 3 (1; −1; −2) .
8.1.16. M 0 (2; 1; 0) , M 1 (3; 2; 0) , M 2 (6; 3; 5) , M 3 (8; 1; −1) .
8.1.17. M 0 (3; 5; 1) , M 1 (−3; 9; 2) , M 2 (7; −9; 1) , M 3 (3; 3; 3) .
8.1.18. M 0 (−1; 1; 0) , M 1 (0; 1; 1) , M 2 (1; 6; 4) , M 3 (−1; 0; 6) .
8.1.19. M 0 (4; −2; −6) , M 1 (2; −4; 4) , M 2 (4; −2; 1) , M 3 (0; −2; 2) .
8.1.20. M 0 (−1; −3; 1) , M 1 (5; −7; 0) , M 2 (−5; 1; 1) , M 3 (−1; −1; −1) .
8.1.21. M 0 (−1; 0; 3) , M 1 (0; 1; 3) , M 2 (3; 2; 8) , M 3 (5; 0; 2) .
8.1.22. M 0 (2; −1; −3) , M 1 (−1; −2; 2) , M 2 (2; −1; −7) , M 3 (0; −1; 1) .
8.1.23. M 0 (−2; 3; 2) , M 1 (10; 7; 1) , M 2 (−1; 0; 2) , M 3 (−2; 1; 0) .
8.1.24. M 0 (1; 0; 2) , M 1 (0; 1; 2) , M 2 (−1; 4; 12) , M 3 (1; 6; 0) .
8.1.25. M 0 (3; 2; −2) , M 1 (−4; −9; 0) , M 2 (6; 9; −1) , M 3 (2; −3; 1) .
8.1.26. M 0 (2; −1; −5) , M 1 (−1; 1; 3) , M 2 (3; 2; −6) , M 3 (1; 2; 0) .
8.1.27. M 0 (2; 3; 1) , M 1 (1; 2; 1) , M 2 (−2; 1; −4) , M 3 (−4; 3; 2) .
8.1.28. M 0 (0; −1; 1) , M 1 (−1; 4; 12) , M 2 (0; −5; 1) , M 3 (0; 1; −1) .
8.1.29. M 0 (0; −8; −2) , M 1 (3; −4; −1) , M 2 (−2; 5; −1) , M 3 (0; 4; 6) .
8.1.30. M 0 (0; −2; 1) , M 1 (13; −3; −4) , M 2 (−10; 2; 5) , M 3 (2; − 2; 1) .

8.2. Evaluate the volume of the pyramid bounded by the given plane and
coordinate planes. Sketch the figure.
8.2.1. 4 x − 3 y + 12 z − 60 = 0 . 8.2.2. 5 x − 4 y + 3z + 120 = 0 .
8.2.3. 2 x − 3 y + z −18 = 0 . 8.2.4. 6 x − 2 y + 3 z + 12 = 0 .
8.2.5. 4 x − 5 y + 2 z − 20 = 0 . 8.2.6. 3x + 4 y + 6 z + 24 = 0 .
8.2.7. 2 x − 5 y + 5 z − 20 = 0 . 8.2.8. x − 3 y + 4 z + 12 = 0 .
8.2.9. 2 x − 3 y + 10 z − 30 = 0 . 8.2.10. 5 x − 3 y + z + 15 = 0 .
8.2.11. 4 x − y + 6 z −12 = 0 . 8.2.12. 7 x + 2 y − z + 14 = 0 .
8.2.13. 3x − 2 y + 8 z − 24 = 0 . 8.2.14. 3x + y − 7 z + 21 = 0 .
8.2.15. x − 4 y + 2 z − 8 = 0 . 8.2.16. x − 5 y − 3 z + 15 = 0 .
8.2.17. 6 x − 2 y − 3 z −18 = 0 . 8.2.18. 5 x + y − z + 10 = 0 .
8.2.19. 9 x −15 y + 5 z − 45 = 0 . 8.2.20. 6 x + 6 y − 7 z + 42 = 0 .
8.2.21. 9 x − 4 y + 12 z − 36 = 0 . 8.2.22. 6 x + 5 y −10 z + 30 = 0 .
8.2.23. 11x − 4 y + 11z − 44 = 0 . 8.2.24. 4 x + 7 y −14 z + 28 = 0 .
8.2.25. 12 x − 9 y + 4 z − 36 = 0 . 8.2.26. 2 x + 9 y − 3 z −18 = 0 .
8.2.27. 13x − 2 y + 13z − 26 = 0 . 8.2.28. 2 x − 7 y −14 z −14 = 0 .
8.2.29. 6 x − 4 y + 3 z − 24 = 0 . 8.2.30. x − 3 y − 5 z −15 = 0 .

108
Micromodule 9
BASIC THEORETICAL INFORMATION. STRAIGHT LINE IN SPACE.
MUTUAL POSITION OF A STRAIGHT LINE AND A PLANE

General equation of a straight line in space, symmetric and parametric


equations. Equations of a straight line passing through two given points.
Angle between two straight lines, conditions of parallelism and per-
pendicularity of two straight lines. Intersection point of a straight line and
a plane, an angle between a straight line and a plane, conditions of
parallelism and perpendicularity of a straight line and a plane, conditions
when a straight line lies in a plane.

Literature: [1, chapter 8, item 8.2, 8.3], [4, section 3, item 3.5], [6, chap-
ter 3, § 4—5], [7, chapter2, § 6], [10, chapter2, § 3,4], [11, chapter 2, § 2].

9.1. Symmetric equations of a straight line in space


G
Let a point Μ 0 ( x0 , y0 , z0 ) and a vector a = {l ; m; n} be given in space.
G
Only one straight line passes through this point and is parallel to the vector a
(Fig. 2.12). Its equations are:

x − x0 y − y0 z − z0
= = . (2.17)
l m n

The equations (2.17) are called symmetric equations of a straight line in


space. The proof of these equations is similar to the case of a straight line in a
plane (formula (2.3)).

М0
G
a

Fig. 2.12

9.2. Parametric equations of a straight line

Having denoted each ratio in the equations (2.17) as t:


x − x0 y − y0 z − z0
= = = t,
l m n

109
⎧ x = x0 + lt

we get the equations: ⎨ y = y0 + mt (2.18)
⎪⎩ z = z0 + nt ,

where t is called a parameter of the line. It can take on any real value.
The equations (2.18) are called parametric equations of a straight line in
space.

9.3. Equations of a straight line passing through two given points

Let a straight line L pass through two points Μ1 ( x1; y1 ; z1 ) and


Μ2 ( x2 ; y2 ; z2 ). A vector M 1M 2 = { x2 − x1 ; y2 − y1 ; z2 − z1} lying on the straight
line L is a direction vector of this straight line. Then

x − x1 y − y1 z − z1
= =
x2 − x1 y2 − y1 z2 − z1

is an equation of a straight line passing through two given points.

9.4. General equations of a straight line

Two nonparallel planes intersect each other along a straight line. These
planes are given by the equations Α1 x + Β1 y + C1 z + D1 = 0 and Α2 x + Β2 y +
JJG JJG
+ C2 z + D2 = 0 . Their normal vectors n1 = { A1 , B1 , C1} and n2 = { A2 , B2 , C2 }
JJG JJG
are not collinear, that is, n1 × n2 = 0 Then the system of the equations

⎧ A1 x + B1 y + C2 z + D1 = 0
⎨ A x + B y + C z + D = 0, (2.19)
⎩ 2 2 2 2

determines a straight line in space and this system is called general equations of
the straight line (Fig. 2.13).
Usually for practical application the symmetric equations of a straight line
are more convenient. To reduce the general equation (2.19) to symmetric form
(2.17) it is necessary to find a point Μ 0 ( x0 , y0 , z0 ) on the straight line L and a
G
direction vector a = {l ; m; n} of the straight line.
To find such a point we take one of its coordinates, for example z, which is
equal to zero or z = z0 . Then from the system (2.19) we have to find corres-
ponding values of two other coordinates. If the system is inconsistent then an
arbitrary value may applied to another coordinate. It is possible to take a cross

110
JJG JJG
product of normal vectors n1 = { A1 , B1 , C1} and n2 = { A2 , B2 , C2 } as a direction
G
vector a of the straight line.
Remark. We can reduce the equation (2.19) to the symmetric form by
another way: to find two points Μ 1 ( x1 , y1 , z1 ) and Μ 2 ( x2 , y2 , z2 ) on the straight
line L and to take the vector M 1M 2 as a direction vector of the straight line L.

9.5. An angle between two straight lines.


Conditions of parallelism and perpendicularity of two straight lines

Let two straight lines L1 and L2 are given by symmetric equations:


x − x1 y − y1 z − z1 x − x2 y − y2 z − z2
L1 : = = , L2 : = =
l1 m1 n1 l2 m2 n2
The angle φ between these lines is equal to the angle between their direction
vectors a1 = {l1; m1; n1 } and a2 = {l2 ; m2 ; n2 }, that is,

a1a2 l1l2 + m1m2 + n1n2


cos ϕ = = .
a1 a2 l12 + m12 + n12 l22 + m22 + n22

If two straight lines L1 and L2 are parallel, their direction vectors are
parallel too. It means that their corresponding coordinates are proportional. Thus
a condition of parallelism of two straight lines is:
l1 m n
= 1 = 1.
l2 m2 n2

If two straight lines L1 and L2 are perpendicular, their direction vectors are
JJG JJG
perpendicular too. So their dot product a1 × a2 = 0. Therefore a condition of
perpendicularity of two straight lines is:

l1l2 + m1m2 + n1n2 = 0.

9.6. Angle between a straight line and a plane.


Conditions of parallelism and perpendicularity
of a straight line and a plane

An angle between a straight line and a plane is defined as an angle between a


straight line and its projection onto the plane (Fig. 2.14).
Let a straight line L be given by symmetric equations (2.18), and a plane P
has a general equation (2.12). We denote an angle between the straight line L
111
and the plane P by φ and an angle between a normal vector of the plane P and a direction
vector of the straight line L by ψ. For an acute angle ψ φ = 90˚– ψ and cos ϕ = sin ψ. For
an obtuse angle ψ φ = ψ – 90˚ and sin ϕ = cos ψ. In both cases sin ϕ = cos ψ .
JJG G
n
n2 JG
n1 G G G
a = n1 × n2
ψ
G G G φ
a = n1 × n2

Fig. 2.13 Fig. 2.14

So, the angle between a straight line and a plane may be found by the
formula:
G G
n×a Al + Bm + Cn
sin ϕ = JJG G = .
n×a A + B 2 + C 2 l 2 + m2 + n2
2

9.7. Condition of parallelism of a straight line and a plane

If a straight line is parallel to a plane (Fig. 2.15) vectors n and a are


perpendicular, so their scalar product is equal to zero:
Αl + Βm + Cn = 0.

9.8. Condition of perpendicularity of a straight line and a plane


G G
If a straight line is perpendicular to a plane (Fig. 2.16), vectors n and a are
collinear, thus their coordinates are proportional:

Α Β C
= = .
l m n
G
G G G a
n a n

Fig. 2.15 Fig. 2.16

112
9.9. Intersection point of a straight line and a plane
To find an intersection point of the straight line given by the equations
(Fig.2.16) and a plane, it is necessary to solve the system of equations
⎧ x − x0 y − y0 z − z0
⎪ = =
⎨ l m n
⎪⎩ Ax + By + Cz + D = 0
or
⎧ x = x0 + lt ,
⎨ y = y + mt ,
⎩ 0
⎧ z = z 0 + nt ,
⎨ Al + Bm + Cn t + Ax + By + Cz + D = 0.
⎩( ) 0 0 0

A number of solutions of the latter system is determined by the equation


Rt + S = 0, where R = Al + Bm + Cn, S = Ax0 + By0 + Cz0 + D.
Next cases are possible:
S
1) if R≠0, then the equation Rt + S = 0 has the only root t* = − . So the
R
straight line and the plane have one common point ( x0 + lt*, y0 + mt*, z0 + nt * );
2) if R = 0, S ≠ 0 then the equation Rt + S = 0 has no solution. It means,
that the straight line and the plane have no common points;
3) if R = 0, S = 0, the equation Rt + S = 0 has an infinite set of solutions. In
this case the straight line lies on the plane.
So, if a straight line lies in a plane, two conditions are valid:
⎧ Al + Bm + Cn = 0,
⎨ Ax + By + Cz + D = 0.
⎩ 0 0 0

The first equation means parallelism of a straight line and a plane, and the
second means that the point Μ 0 ( x0 ; y0 ; z0 ) of straight line L lies in the plane.

9.10. Distance between parallel straight lines


A distance between parallel straight lines
x − x1 y − y1 z − z1
L1 : = =
l m n
x − x2 y − y2 z − z2
and L 2 : = = , M2
l m n
equals a height of a parallelogram built on the L2
G d
vectors a and M 1M 2 , where M 1 is an arbit-
M1 a
rary point of the line L1 , and M 2 is an L1
arbitrary point of the line L2 (Fig. 2.17). Fig. 2.17

113
G
a ⋅ M1M 2
Thus, d= G .
a

The distance between parallel straight lines may be found by the next
scheme:
1) to draw a plane through the point M 1 ( x1 ; y1 ; z1 ) , perpendicular to the
straight line L1 ;
2) to find an intersection point О of the plane with the straight line L2 ;
3) to calculate the length of a line segment M1O. The result will be the
required distance between parallel straight lines.

Micromodule 9
EXAMPLES OF PROBLEMS SOLUTION

Example1. Find symmetric equations of a straight line

⎧2 x − 5 y + 2 z − 10 = 0,
⎨ x − 2 y − z + 1 = 0.

Solution. To write down symmetric equations of a straight line you must just
know coordinates of its point and direction vector.
Let us take such a point that its ordinate y = 0 , the other coordinates must
satisfy the left equations:

⎧2 x + 2 z − 10 = 0, or ⎧ x + z = 5,
⎨ x − z + 4 = 0, ⎨ x − z = −1,
⎩ ⎩

Its solution is x = 2, z = 3. So a point Μ (2; 0; 3) belongs to the given


straight line.
We can find a direction vector by the formula:

i j k
G G G
a = n1 × n2 = 2 −5 2 = 9 i + 4 j + k ,
1 −2 −1
G
or a = {9; 4; 1} .
So symmetric equations of the straight line are:
x−2 y z −3
= = .
9 4 1

114
Example 2. Prove that straight lines
x − 3 y −1 z + 2 ⎧2 x + 3 y − 8 z + 5 = 0,
= = and ⎨
1 −2 3 ⎩3x + y − 5 z + 1 = 0
are perpendicular.
Solution. Straight lines are perpendicular if andJJGonly if direction vectors of
these straight lines are perpendicular. A vector a1 = {1; − 2; 3} is a direction
vector of the first straight line. A direction vector of the second straight line may
be found by the formula:
G G G
i j k G G G G G G
G G G
a2 = n1 × n2 = 2 3 −8 = −7i − 14 j − 7k = −7(i + 2 j + k ).
3 1 −5

Let us calculate a dot product


a1a2 = 1 ⋅1 − 2 ⋅ 2 + 3 ⋅1 = 0.
So the straight lines are perpendicular.

Remark. Similar straight lines are perpendicular if a triple product


a1n1n2 = 0.
x +1 y −1 z − 2
Example 3. Prove that the straight line = = lies in the plane
2 4 −1
4 x − 3 y − 4 z + 15 = 0.
Proof. The first way. It is enough to show that any two points of the straight
line lie in the plane.
Actually, points Μ 1 (−1; 1; 2) and Μ 2 (1; 5; 1) lie on the straight line
(explain the choice of these points). Having substituted coordinates of these
points in the equation of the plane, we get two identities.
The second way. We can represent equations of the straight line in the
parametric form:
x = −1 + 2t , y = 1 + 4t , z = 2 − t ,
then we can substitute x, y, z in the equation of the plane:

4(−1 + 2t ) − 3(1 + 4t ) − 4(2 − t ) + 15 = 0,


and receive the identity: 0=0. It means that any point of the straight line lies in
the plane.
Example 4. Find a point P symmetric to a point M (−2; 3; −5) relative to
x − 8 y +1 z +1
the straight line L: = = .
3 1 −1

115
Solution. Let P be a required point (Fig. 2.18). The problem may be solved in
such a sequence.
1) We make an equation of the plane α passing
through the point М and perpendicular to the straight
line L.
М 2) We find a point Q, that is, a projection of the
α
Q point M onto the straight line L.
l
3) We determine coordinates of the point P, taking
G into account that the point Q is a midpoint of the line
a segment MP. G
PP
The vector a = {3; 1; − 1} is a direction vector of
the straight line L and a normal vector of a plane
perpendicular to the straight line L. We write down the
Fig. 2.18 equations of a plane α as following:

3( x + 2) + ( y − 3) − ( z + 5) = 0 or 3x + y − z − 2 = 0.

The point Q belongs both to the plane and to the straight line. To find its
coordinates we have to solve the system of equations

⎧ x − 8 y +1 z +1
⎪ = =
⎨ 3 1 −1
⎪⎩3x + y − z − 2 = 0.

It is convenient to solve such a system by introducing a parameter t we shall


write down the equations of a straight line in a parametric form: x=3t + 8, y = t–
1, z = –t–1. Having substituted them in the equation of the plane we shall
receive:
3 (3 t + 8 ) + (t – 1) – (– t – 1) – 2 = 0, 11 t + 22 = 0, t = –2.

So, xQ = 3(–2) + 8=2, yQ = –2–1= –3, zQ = 2–1=1.


The point Q(2; −3; 1) is a midpoint of the line segment MP, therefore its
coordinates satisfy the equalities

xM + x P y + yP z + zP
xQ = , yQ = M , zQ = M .
2 2 2
That is
−2 + xP 3 + yP −5 + z P
2= , −3 = , 1= .
2 2 2
From here we find coordinates of the point P(6; ─9; 7).

116
Micromodule 9
CLASS AND HOME ASSIGNMENTS

1. Three vertices of a parallelogram M1 (6; 2; −10) , M 2 (9; −5; 6) ,


M 3 (2; −8; 4) are given. Find the equations of its diagonals.
2. Write down equations of a straight line in the symmetric form:
⎧ x − 2 y + 3 z + 15 = 0,
⎨ 2 x + 3 y − 4 z − 12 = 0.

x − 2 y + 3 z −1
3. Find an intersection point of the straight line = = and the
3 5 −1
plane x + 2 y − 3 z − 9 = 0.
4. Determine an acute angle between straight lines
x + 2 y − 3 z −1
x = 11t − 1, y = −8t + 4, z = −7t + 5 and = = .
7 2 −8
x−3 y −5 z + 2
5. Calculate an angle between the straight line = = and the
2 −3 −2
plane 6 x − 9 y − 6 z + 10 = 0 .
x −1 y z
6. Find a projection of the point А(1; 2; 8) onto the straight line = = .
2 −1 1
7. Calculate distance of the point P (2; 3; −1) to the straight line x = t + 1,
y = t + 2, z = 4t + 13.
8. Making sure that straight lines
x+7 y −5 z −9 ⎧2 x + 2 y − z − 10 = 0
= = and ⎨
3 −1 4 ⎩ x − y − z − 22 = 0
are parallel, calculate a distance between them.
Answers

x − 6 y − 2 z + 10 x − 1 y − 2 z + 7 x+3 y−6 z
1. = = , = = . 2. = = . 3. (5;
2 5 −7 5 −2 9 −1 10 7
2; 0). 4. 60˚. 5. 90˚. 6. (3; -1; 1). 7. 6. 8. 25.

Micromodule 9
SELF─TEST ASSIGNMENTS
9.1. Coordinates of three points M 1 , M 2 , M 3 are given(see problem 8.1). Find:
а) symmetric equations of the straight line passing through the points M1 and M2 ;

117
b) parametric equations of the straight line passing through the points M 1
and M 3 ;
c) an angle between straight lines M 1 M 2 and M 2 M 3 .

9.2. Find symmetric equations of a straight line


⎧ x − 4 y + 4 z − 10 = 0 ⎧ x − 2 y + 3z − 6 = 0
9.2.1. ⎨ 9.2.2. ⎨
⎩2 x + y − 2 z − 6 = 0 ⎩2 x + 3 − z − 8 = 0
⎧ x + 4 y + z + 10 = 0 ⎧ x + 5 y + 2 z − 20 = 0
9.2.3. ⎨ 9.2.4. ⎨
⎩2 x − y − 2 z + 5 = 0 ⎩4 x + 2 y − z − 8 = 0
⎧ x − 6 y + 3 z − 12 = 0 ⎧ x − 2 y + 3z + 5 = 0
9.2.5. ⎨ 9.2.6. ⎨
⎩3x + 2 y − 3 z − 6 = 0 ⎩5 x + y − 4 z − 12 = 0
⎧x + 3 y + 2z + 6 = 0 ⎧ x − 6 y + 2 z − 14 = 0
9.2.7. ⎨ 9.2.8. ⎨
⎩2 x + 2 y − 3 z − 6 = 0 ⎩4 x − y − 2 z − 8 = 0
⎧ x − 3 y + 3z − 7 = 0 ⎧2 x − 3 y + 4 z − 15 = 0
9.2.9. ⎨ 9.2.10. ⎨
⎩2 x + y − z − 3 = 0 ⎩2 x + y − 3 z − 4 = 0
⎧3x − 2 y + z − 8 = 0 ⎧ x − 2 y + z − 10 = 0
9.2.11. ⎨ 9.2.12. ⎨
⎩2 x + y − z − 7 = 0 ⎩2 x + 3 y − 2 z − 12 = 0
⎧3 x + y + 4 z − 6 = 0 ⎧2 x − 3 y + 2 z − 5 = 0
9.2.13. ⎨ 9.2.14. ⎨
⎩4 x − y − 2 z − 3 = 0 ⎩2 x + 2 y − 3 z − 4 = 0
⎧ x − 5 y + 3 z − 11 = 0 ⎧x + 6 y + 2z − 2 = 0
9.2.15. ⎨ 9.2.16. ⎨
⎩2 x + 3 y − z − 3 = 0 ⎩3 x + y − 3 z = 0
⎧ x − 7 y + 2 z − 14 = 0 ⎧ x − 4 y − 4 z + 10 = 0
9.2.17. ⎨ 9.2.18. ⎨
⎩2 x + 4 y − z − 6 = 0 ⎩2 x − y + 2 z + 6 = 0
⎧3 x − 4 y + 2 z − 15 = 0 ⎧5 x − y + 2 z − 20 = 0
9.2.19. ⎨ 9.2.20. ⎨
⎩ x + 2 y − 2 z − 10 = 0 ⎩2 x + 2 y − z − 4 = 0
⎧ x + 3 y + 3z − 9 = 0 ⎧x − 8 y + 2z + 6 = 0
9.2.21. ⎨ 9.2.22. ⎨
⎩4 x + 2 y − z − 7 = 0 ⎩2 x + y − z + 4 = 0
⎧ x − 2 y + 4 z + 10 = 0 ⎧x − 2 y + 4 z − 2 = 0
9.2.23. ⎨ 9.2.24. ⎨
⎩5 x + y − 3 z − 16 = 0 ⎩3x + 2 y − z − 36 = 0

118
⎧x − y + z − 5 = 0 ⎧ x − 2 y + 4 z − 12 = 0
9.2.25. ⎨ 9.2.26. ⎨
⎩3x − 2 y − 2 z − 4 = 0 ⎩2 x + y − 2 z − 6 = 0
⎧ x + 3 y + 2 z − 13 = 0 ⎧3 x − y + 2 z − 6 = 0
9.2.27. ⎨ 9.2.28. ⎨
⎩2 x + y + z − 16 = 0 ⎩2 x + y − 4 z − 8 = 0
⎧ x − 3 y + 6 z − 11 = 0 ⎧x − 4 y + 5z = 0
9.2.29. ⎨ 9.2.30. ⎨
⎩x + 2 y − 2z − 5 = 0 ⎩2 x − y − 3 z − 2 = 0

9.3. Find: а) an intersection point of a straight line and a plane;


b) an angle between the straight line and the plane;
c) a point symmetric to the point Р relative to the given plane.
x − 2 y − 3 z +1
9.3.1. = = , x + 2 y + 3 z −14 = 0 , P (1; 3; −6) .
−1 −1 4
x +1 y − 3 z +1
9.3.2. = = , x + 2 y − 5 z + 20 = 0 , P (2; 7; −4) .
3 −4 5
x −1 y + 5 z − 1
9.3.3. = = , x − 3 y + 7 z − 24 = 0 , P (0; 10; −2) .
−1 4 2
x −1 y z + 3
9.3.4. = = , 2 x − y + 4 z = 0 , P (−4; 6; 6) .
1 0 2
x −5 y −3 z − 2
9.3.5. = = , 3x + y − 5 z −12 = 0 , P (7; 2; −5) .
1 −1 0
x +1 y + 2 z − 3
9.3.6. = = , x + 3 y − 5 z + 9 = 0 , P (5; 0; −6) .
−3 2 −2
x −1 y − 2 z + 1
9.3.7. = = , x − 2 y + 5 z + 17 = 0 , P (−12; 4; 6) .
−2 1 −1
x −1 y − 2 z +−4
9.3.8. = = , x − 2 y + 4 z −19 = 0 , P (9; 0; −3) .
2 0 1
x + 2 y −1 z + 4
9.3.9. = = , 2 x − y + 3z + 23 = 0 , P (6; −3; 2) .
−1 1 −1
x + 2 y−2 z +3
9.3.10. = = , 2 x − 3 y − 5 z − 7 = 0 , P (15; 6; 0) .
1 0 0
x − 1 y −1 z + 2
9.3.11. = = , 4 x − 2 y − z −11 = 0 , P (7; 1; −1) .
2 −1 3
119
x −1 y + 1 z − 1
9.3.12. = = , 3 x − 2 y − 4 z − 8 = 0 , P (−4; 0; 8) .
1 0 −1
x + 2 y −1 z + 3
9.3.13. = = , x + 2 y − z − 2 = 0 , P(5; 2; −2) .
−1 1 2
x +3 y−2 z +2
9.3.14. = = , 5 x − y + 4 z + 3 = 0 , P (9; 5; −3) .
1 −5 3
x−2 y−2 z−4
9.3.15. = = , x + 3 y + 5 z − 42 = 0 , P (−2; −4; −6) .
2 −1 3
x −3 y − 4 z − 4
9.3.16. = = , 7 x + y + 4 z − 47 = 0 , P (5; −2; 1) .
−1 5 2
x + 3 y −1 z −1
9.3.17. = = , 2 x + 3 y + 7 z − 52 = 0 , P (0; 6; −8) .
2 3 5
x − 3 y +1 z + 3
9.3.18. = = , 3x + 4 y + 7 z −16 = 0 , P (−5; 1; −3) .
2 3 2
x −5 y − 2 z + 4
9.3.19. = = , 2 x − 5 y + 4 z + 24 = 0 , P (2; 2; −4) .
−2 0 −1
x −1 y − 8 z + 5
9.3.20. = = , x − 2 y − 3 z + 18 = 0 , P (11; 4; −3) .
8 −5 12
x − 3 y −1 z + 5
9.3.21. = = , x − 7 y + 3z + 11 = 0 , P (14; 12; −2) .
1 −1 0
x − 5 y + 3 z −1
9.3.22. = = , 3x + 7 y − 5 z −11 = 0 , P (0; 13; −16) .
−1 5 2
x −1 y − 2 z − 6
9.3.23. = = , 4 x + y − 6 z − 5 = 0 , P (−1; 11; 5) .
7 1 −1
x −3 y + 2 z −8
9.3.24. = = , 5 x + 9 y + 4 z − 25 = 0 , P (7; 0; −4) .
1 −1 0
x +1 y z +1
9.3.25. = = , x + 4 y + 13 z − 23 = 0 , P (−6; 4; −2) .
−2 0 3
x −1 y − 3 z + 5
9.3.26. = = , 3 x − 2 y + 5 z − 3 = 0 , P (11; 0; −1) .
6 1 3

120
x − 2 y −1 z + 3
9.3.27. = = , 3 x − y + 4 z = 0 , P (−6; −3; −2) .
4 −3 −2
x −1 y + 2 z − 3
9.1.28. = = , x + 2 y − 5 z + 16 = 0 , P (1; 3; 7) .
2 −5 −2
x −1 y − 3 z + 2
9.3.29. = = , 3 x − 7 y − 2 z + 7 = 0 , P (2; 4; 8) .
1 0 −2
x +3 y−2 z +5
9.3.30. = = , 5 x + 7 y + 9 z − 32 = 0 , P (7; 5; −3) .
0 −3 11

Micromodule 10
BASIC THEORETICAL INFORMATION
CURVES OF THE SECOND ORDER

Circle, an ellipse, a hyperbola: definition, equations in standard form, an


eccentricity, directrices and their geometrical sense, hyperbola asymp-
totes. Parabola: definition, an equation in standard form, a parameter and
a directrix of a parabola.

Literature: [1, chapter 7], [4, section 3, item 3.4], [6, chapter 3, § 6],
[7,chapter 3, § 8], [10, chapter 2, § 5], [11, chapter 2, § 1].

10.1. Curves of the second order


A set of points in a plane whose coordinates satisfy the equation
ax 2 + by 2 + cxy + dx + ey + f = 0
is called a second order curve provided at least one of numbers a, b, c is other
from zero. A circle, an ellipse, a hyperbola and a parabola belong to curves of
the second order.
A set of points on a plane, whose distances from a given point of the plane (a
center) are equal to a constant value (radius) is called a circle.
The equation
( x − a ) 2 + ( y − b) 2 = R 2
describes a circle of radius R. The center of the circle is at a point К(а,b)
(Fig.2.19). If a circle center is located at the origin of coordinates (Fig.2.20), the
equation of a circle has so called standard form.

x2 + y2 = R2.

121
y y
R
R
b
K
O R x

O а x

Fig. 2.19 Fig. 2.20

10.2. Ellipse

An ellipse is a locus of all points in a plane whose sum of distances from


two fixed points (foci) has a constant magnitude denoted as 2a (Fig.2.21).

Let us consider two points F1 and F2 (foci of an ellipse) in a plane.


We arrange coordinate axes so that the axis Ox passes through given points,
and the axis Оу passes through a midpoint of the line segment F1 F2 per-
pendicularly to Ox. We denote a distance between foci as F1 F2 = 2c , and the
sum of distances from any point of the ellipse to its foci as 2a, 2a > 2c. Then
foci will have coordinates F1 (−c, 0) and F2 (c, 0) .
According to the definition, any point M ( x, y ) lies on the ellipse if and
only if the following equality holds

( x + c ) 2 + y 2 + ( x − c ) 2 + y 2 = 2a .

When we square twice the left and right parts of this equation, we receive

x 2 (a 2 − c 2 ) + y 2 a 2 = a 2 (a 2 − c 2 ) .

Let us denote a difference a 2 − c 2 = b 2 . Then x 2 b 2 + y 2 a 2 = a 2 b 2


or
x2 y2
2
+ = 1.
a b2

The latter is called an equation of an ellipse in standard form.


Line segments A1 A2 = 2a and B1 B2 = 2b are called major and minor axes
of the ellipse.

122
If a = b , the equation looks as y
x + y 2 = a 2 . So a circle is a particu-
2 a a
x=− x=
ε ε
lar case of an ellipse whose foci coincide d1 B1 M d2
at the same point called the center.
The measure of ellipse deviation r1 r2
A1 A2
from a circle is characterized by the
F1 О F2 x
c
value ε = , 0 < ε < 1 and is called an
a
eccentricity of an ellipse. B2
Line segments F1 M and F2 M are
called focal radii of a point М: Fig. 2.21

r1 = F1 M = ( x + c) 2 + y 2 and r2 = F2 M = ( x − c) 2 + y 2 .

a a2
Straight lines x = ± , or x = ± are called directrices of an ellipse.
ε c
a2
Inasmuch as 0 < ε < 1, then > a, i.e. directrices of an ellipse lie outside of it.
c
The following statement is true for directrices.
The ratio of a focal radius of any point of an ellipse to the distance of this
point to the corresponding directrix is a constant magnitude equal to the ellipse
eccentricity, that is,
r1 r
= 2 = ε.
d1 d 2

10.3. Hyperbola

A hyperbola is a locus of all points in a plane a difference module of


whose distances from two fixed points (foci) has a constant magnitude
denoted as 2a (Fig. 2.22).

Let us denote a distance between foci F1 F2 = 2c, and a module of difference


of distances from any point of hyperbola to its foci as 2a, 2a < 2c. Then the
foci will have coordinates F1 (−c, 0) and F2 (c, 0).
By definition, any point M ( x, y ) lies on a hyperbola if and only if the
following equality holds

( x + c ) 2 + y 2 − ( x − c ) 2 + y 2 = 2a.

123
After some transformations we can get a standard equation of the hyperbola

x2 y2
− = 1,
a2 b2
where b 2 = c 2 − a 2 .
A hyperbola consists of two branches and has two asymptotes whose
b
equations are y = ± x.
a
y
d2
d1

r1 r2
A2
F1 A1 F2 x

a a
x= – ε x= ε

Fig. 2.22

A line segment A1 A2 = 2a is called a real axis of a hyperbola, а line


segment B1 B2 = 2b is called an imaginary axis.
y2 x2
The equation − = 1 determines a hyperbola called conjugate to the
b2 a2
x2 y2
given hyperbola − =1 .
a 2 b2
A hyperbola eccentricity is determined by the ratio of a focal distance of a
hyperbola to the length of its real axis:
c
ε= , ε > 1.
a
a
Straight lines x = ± , where a is a real semi-axis of the hyperbola, are
ε
called directrices of the hyperbola. They have the same property as directrices of
an ellipse:
r1 r
= 2 = ε.
d1 d 2

124
10.4. Parabola

A parabola is a set of all points of a plane, equidistant from a given point


(a focus) and a given straight line (a directrix) (Fig. 2.23).

Let us write down an equation of a parabola.


A focus F and a directrix are given on a plane and distance between them is
equal to p. We can choose an axis Ox so that it would pass through the focus
perpendicularly to the directrix, and an axis Оу would halve the distance
between the focus and the directrix.
p
In the chosen coordinate system the focus is a point F ( , 0) , and a
2
p
directrix equation is x = − .
2
Any point M ( x, y ) lies on a parabola if and only if the following equality
holds MB = MF , where
у
p p
MB = x + , MF = ( x − ) 2 + y 2 . M
2 2 K

Then

p p
x+ = ( x − )2 + y 2 , О p
F ( ; 0)
х
2 2 2
whence after some transformations we get a stan- y2 = 2px
dard equation of the parabola x= –
p
2
y 2 = 2 px. Fig. 2.23

An axis of parabola symmetry is called an axis of parabola. An intersection


point of parabola with its axis is called a vertex of parabola. The number p equal
to the distance between its focus and directrix is called a parameter of the
parabola.
The parameter of a parabola characterizes the width of the area bounded by
the parabola (the greater is p the wider the parabola is).

Micromodule 10
EXAMPLES OF PROBLEMS SOLUTION

Example 1. An equation of the second order curve is given 4x2 −5 y2 + 20 = 0.


Determine a type of the curve, find its foci, semi-axes, an eccentricity, equations
of directrices and asymptotes (for a hyperbola). Sketch its graph.

125
Solution. The given equation is reduced to a standard form

x2 y2 x2 y2
− = −1 , or − + = 1.
5 4 5 4
It is a conjugate hyperbola with a real semi-axis b = 2, lying on the axis Оу
and an imaginary semi-axis a = 5 on the axis Ох. A half of focal distance may
be found from the condition

c 2 = a 2 + b 2 = 9; c = 3.
Foci F1 і F2 lie on the axis Оу and have coordinates (0; –3) and (0; 3).
c
The eccentricity is ε = , so ε =1,5.
b
b
Equations of directrices are y = ± , or y = ± 4 / 3.
ε
b 2
Equations of asymptotes are y = ± x, or y = ± x.
a 5
A graph of the hyperbola is given in Fig. 2.24.

y l1
F2
2
D2

–√5 √5 D1 x

–2
F1 l2

Fig. 2.24

Example 2. Determine the type of a curve 4 x 2 + y 2 + 8 x − 2 y + 1 = 0, reduce


the equation to the simplest form, sketch the graph of the equation.
Solution. Having completed perfect squares in х and y, we get

4( x 2 + 2 x) + ( y 2 − 2 y + 1) = 0 , 4( x 2 + 2 x + 1) + ( y 2 − 2 y + 1) = 4 ,
( y − 1) 2
4( x + 1) 2 + ( y − 1) 2 = 4 , ( x + 1) 2 + =1.
4

126
We have received an equation of the ellipse, that follows from an ellipse
y2
x2 + = 1 by parallel displacement along the vector (–1; 1) (Fig. 2.25).
4
Example 3. Determine, what line is defined by the equation y = 2 − x − 2
and sketch its graph..
Solution. It is obvious, that x ≥ 2 and y < 2. At such restrictions we carry out
transformations: y − 2 = − x − 2 , ( y − 2) 2 = x − 2. The graph of the given
equation is the lower branch of a parabola represented in Fig. 2.26.

у
3 у
2

–1
х 2 6 х
–1

Fig. 2.25 Fig. 2.26

Micromodule 10
CLASS AND HOME ASSIGNMENT

1. Write down an equation of the ellipse whose foci are placed on the axis Ox
symmetrically relative to the coordinate origin if its minor axis is equal to 24,
and the distance between its foci is 2c = 10.
2. Write down equations of a hyperbola whose foci are placed on the axis Ox
symmetrically relative to the coordinate origin if a distance between its foci is
2c = 6 and its eccentricity is ε = 1,5.
3. Find a vertex and a parameter р of a parabola x = 4 y 2 − 8 y + 7.
4. Determine a type of a curve 4 x 2 − 32 x − y 2 + 2 y + 59 = 0 and sketch its
graph.

Answers
x2 y2 1
1. + = 1 . 3. p = , (3; 1) is a vertex of the parabola. 4. A hy-
169 144 8
perbola.

127
Micromodule 10
SELF-TEST ASSIGMENTS

10.1. Equation of a second order curve is given. Perform such operations:


а) determine its type using an equation of the curve;
b) in case of an ellipse find lengths of its semi-axes, coordinates of its foci,
an eccentricity , write down equations of directrices;
c) in case of a hyperbola find lengths of its semi-axes, coordinates of its foci,
an eccentricity , write down equations of directrices and asymptotes;
d) in case of a parabola find a value of its parameter, coordinates of its focus,
write down an equation of its directrix;
e) sketch the graph of the curve with representation of foci, directrices and
asymptotes (if it is possible).
10.1.1. x 2 + 4 y 2 − 4 = 0. 10.1.2. 9 x 2 + 4 y 2 − 36 = 0.
10.1.3. 16 x 2 − 25 y 2 − 400 = 0. 10.1.4. −16 x 2 + 25 y 2 − 400 = 0.
10.1.5. x 2 + 10 y = 10. 10.1.6. 9 x 2 −16 y 2 + 144 = 0.
10.1.7. 16 x 2 + 25 y 2 − 400 = 0. 10.1.8. y 2 − 4 x = 4.
10.1.9. 16 x 2 − 36 y 2 − 576 = 0. 10.1.10. 25 x 2 + 16 y 2 − 400 = 0.
10.1.11. x 2 − 4 y 2 − 4 = 0. 10.1.12. 4 x 2 + 25 y 2 −100 = 0.
10.1.13. 9 x 2 − 36 y 2 + 324 = 0. 10.1.14. 4 x 2 + 9 y 2 − 36 = 0.
10.1.15. 5 x 2 + 4 y 2 − 20 = 0. 10.1.16. 25 x 2 + 4 y 2 −100 = 0.
10.1.17. y 2 + 8 x = 16. 10.1.18. 16 x 2 − 9 y 2 + 144 = 0.
10.1.19. x 2 + 9 y 2 − 9 = 0. 10.1.20. 25 x 2 − 36 y 2 − 900 = 0.
10.1.21. 9 x 2 − 4 y 2 − 36 = 0. 10.1.22. 4 x 2 − 9 y 2 + 36 = 0.
10.1.23. x 2 −12 y = 24. 10.1.24. 36 x 2 + 16 y 2 − 576 = 0.
10.1.25. 9 x 2 + 16 y 2 −144 = 0. 10.1.26. 5 x 2 − 4 y 2 + 20 = 0.
10.1.27. 9 x 2 + 36 y 2 − 324 = 0. 10.1.28. x 2 − 4 y 2 + 4 = 0.
10.1.29. 36 x 2 + 25 y 2 − 900 = 0. 10.1.30. 25 x 2 − 36 y 2 + 900 = 0.

10.2.Distinguish, what line is defined by the equation and sketch its graph.
3 4
10.2.1. y = 1 + 16 − x 2 . 10.2.2. y = 2 − 9 − x2 .
4 3
2 5
10.2.3. y = 1− 25 − x 2 . 10.2.4. y = −2 − 9 − x2 .
5 3

128
2 5
10.2.5. x = 1 + 9 − y2 . 10.2.6. x = 2 − 16 − y 2 .
3 4
7 3
10.2.7. x = 3 + 4 − y2 . 10.2.8. x = 3 − 49 − y 2 .
2 7
3 3
10.2.9. y = −1 + 16 + x 2 . 10.2.10. y = 2 − 25 + x 2 .
4 5
5 6
10.2.11. y = 37 + x 2 + 2 x . 10.2.12. y = − 29 + 4 x + x 2 .
6 5
4 7
10.2.13. y = 50 − 2 x + x 2 . 10.2.14. y = − 20 − 4 x + x 2 .
7 4
4 9
10.2.15. y + 1 = 81 + x 2 . 10.2.16. y − 2 = − 25 + x 2 .
9 5
7 3
10.2.17. x = 5 − 2 y + y2 10.2.18. x = − 53 + 4 y + y 2 .
2 7
7 8
10.2.19. x = 25 + 6 y + y 2 . 10.2.20. x = 25 −16 y + y 2 .
4 3
5 4
10.2.21. x − 2 = − 49 + y 2 . 10.2.22. x + 1 = − 9 + y2 .
7 3
10.2.23. y = 1− 3 1− x 2 . 10.2.24. x + 3 = −2 4 − y 2 .
5 3
10.2.25. y −1 = 9 − x2 . 10.2.26. y = − 24 − 2 x − x 2 .
3 5
9 7
10.2.27. x − 2 = 49 + y 2 . 10.2.28. x + 1 = − 81 + y 2 .
7 9
7 2
10.2.29. x = − 80 − 2 y − y 2 . 10.2.30. y = 26 + 2 x + x 2 .
9 5

Micromodule 11
BASIC THEORETICAL INFORMATION
SURFACES OF THE SECOND ORDER

Cylindrical surfaces. Conic surface. A sphere. An ellipsoid. Hyperboloids


of one and two sheets. An elliptic and hyperbolic paraboloid.

Literature: [1, chapter 7], [4, section 3, item 3.4], [6, chapter 3, §7],
[7, chapter 3, §9], [10, chapter 2, §6].

129
11.1. General equation of a second order surface

A set of points whose coordinates satisfy the equation


ax 2 + by 2 + cz 2 + dxy + exz + fyz + gx + hy + kz + l = 0 ,
is called a surface of the second order, where at least one of coefficients
a, b, c, d , e, f is other than zero. Such an equation is called a general equa-
tion of a surface of the second order.
As a geometrical object a surface of the second order will not change after
transition from one coordinate system to another. There exists a system of
coordinates in which the equation of a surface has the simplest (standard) form.
Cylindrical, conic surfaces, rotating surfaces, a sphere, an ellipsoid, hyper-
boloid of one and two sheets, an elliptic and hyperbolic paraboloid are referred
to the surfaces of the second order.

11.2. Cylindrical surfaces

The surface is called cylindrical if it is formed by a set of parallel straight


lines (generatrices) intersecting the given line L (directrix).
The cylindrical surfaces whose directrices lie in a coordinate plane and
generatrices are parallel to the coordinate axis perpendicular to that coordinate
plane are considered most often. In this case an equation of the cylinder
coincides with the equation of its directrix. For example, the equation
f ( x, y ) = 0
decribes a cylindrical surface whose directrix lies in the plane Оху and gene-
ratrices are parallel to the axis Oz .
Cylindrical surfaces whose directrices are curves of the second order are
called cylindrical surfaces of the second order. Their standard equations are:

10. A circular cylinder x 2 + y 2 = R 2 .


x2 y2
20. An elliptic cylinder + = 1 (Fig. 2.27).
a2 b2
x2 y2
30. A hyperbolic cylinder −
= 1 (Fig. 2.28).
a 2 b2
40. A parabolic cylinder y 2 = 2 px (Fig. 2.29).

130
z
z z

О О
y y О
y
x
x
x

Fig. 2.27 Fig. 2.28 Fig. 2.29

11.3. Conic surfaces


The surface is called conic if it is formed by a set of z
straight lines (generatrices) passing through the given point
(vertex) and intersecting the given line L (directrix)
provided the vertex does not belong to the directrix.
A standard equation of an elliptic cone looks as

x2 y2 z2
+ − = 0. (Fig. 2.30) О
a2 b2 c2 y

A section of the given cone by the plane z = z0 ≠ 0 x


is an ellipse, by the plane z = 0 is a point (0; 0; 0) (a
vertex of the cone).
If a = b = c we have a right circular cone
Fig. 2.30
x2 + y 2 = z 2 .

11.4. Sphere
A sphere is a surface formed by rotation of a circle around its diameter. An
equation of a sphere with a centre at the point M 0 ( x0 , y0 , z0 ) and a radius R
looks as follows
( x − x0 )2 + ( y − y0 ) 2 + ( z − z0 ) 2 = R 2 .

If the centre of the sphere is the origin we have the following standard
equation of the sphere
x2 + y2 + z 2 = R2 .

131
11.5. Ellipsoid

A set of all points in space satisfying the equation

x2 y2 z2
+ + = 1.
a2 b2 c2
z is called a standard equation of the ellipsoid
(Fig. 2.31).
Here the positive numbers а, b, с represent
semi-axes of the ellipsoid.
If а = b we have an ellipsod of rotation:
О y
x2 y2 z2
2
+ = 1 , obtained by rotation of the
2
+
a c2 a
x 2 2
Fig. 2.31 ellipse x + z = 1 about the axis Оz.
2 2
a c

11.6. Hyperboloid of one sheet


z A hyperboloid of one sheet (Fig.2.32) is a surface
described by the equation

x2 y2 z2
+ − = 1.
a2 b2 c2

y In case а = b we have a rotation hyperboloid of one sheet


2 2
x formed by rotation of the hyperbola x − z = 1 , that lies in
2 2
a c
the plane Охz about the axis Оz.
A section of one-sheeted hyperboloid by a plane z = z0 is
Fig. 2.32
an ellipse, by planes x = x0 or y = y0 is a hyperbola.
We have to note that each point of any one-sheeted hyperboloid is passes by
a straight line lying in this hyperboloid entirely.

11.7. Hyperboloid of two sheets

A hyperboloid of two sheets (Fig. 2.33) is a surface described by the equation

x2 y2 z2
2
+ 2
− = −1.
a b c2

132
11.8. Elliptic paraboloid

A surface is called an elliptic paraboloid (Fig. 2.34), if its standard equation


looks as follows:

x2 y 2
+ = 2 z , ( p, q > 0).
p q

If p = q we have a circular paraboloid x 2 + y 2 = 2 pz formed by rotation of


the parabola x 2 = 2 pz lying in the plane Охz about the axis Оz.
The point (0; 0; 0) is the vertex of the elliptic paraboloid.
A section of an elliptic paraboloid by the plane z = z0 is an ellipse, by the
planes x = x0 or y = y0 is a parabola.

z z

О y

x О y

x
Fig. 2.33 Fig. 2.34

11.9. Hyperbolic Paraboloid

A surface is called a hyperbolic paraboloid (Fig. 2.35) if its standard


equation is

x2 y2
− = 2 z , ( p, q > 0).
p q

This surface is also called a saddle.


A section of a hyperbolic paraboloid by the plane z = z 0 ≠ 0 is a
hyperbola, by the plane z = 0 is a pair of two parallel straight lines, by the
planes x = x0 or y = y0 is a parabola.

133
z

Fig. 2.35

Micromodule 11
EXAMPLES OF PROBLEMS SOLUTION

Example 1. Determine what surface is defined by the equation:


x2 y2 z2 z2
а) + + =1; b) x 2 − =1 ; в) z 2 − 4 = 0 ;
16 9 16 16
y 2 z 2 x2
c) 4 x 2 + y 2 = 4 z ; d) + − =0.
4 9 9
x2 y2 z2
Solution. а) The equation + + = 1 defines an rotation ellipsoid
16 9 16
x2 y2
formed by rotation of the ellipse + = 1 about the axis Оу.
16 9
b) The given equation defines a hyperbolic cylinder whose generatrices are
parallel to the axis Оу.
c) The equation z 2 − 4 = 0 is equivalent to the pair of equations z = 2 or
z = −2 , whose geometric image is a pair of two parallel planes.
y2
d) We can rewrite the given equation as z = x 2 + , but it is an equation of
4
an elliptic paraboloid whose symmetry axis is Оz.
y2 z 2 x2
e) The equation + − = 0 describes a circular cone whose symmetry
4 9 9
axis is Ох.
Example 2. Transform the equation of a surface 3x 2 + 4 y 2 − 8 z 2 − 18 x +
+ 8 y + 32 z − 1 = 0 to the standard form and determine which surface it
describes.
Solution. We can transform the left member of the equation by completing
perfect squares:

134
3x 2 + 4 y 2 − 8 z 2 −18 x + 8 y + 32 z −1 =
3( x 2 − 6 x + 9) + 4( y 2 + 2 y + 1) − 8( z 2 − 4 z + 4) −1− 27 − 4 + 32 =
= 3( x − 3) 2 + 4( y + 1) 2 − 8( z − 2) 2 .
So the given equation looks as follows:
3( x − 3) 2 + 4( y + 1) 2 − 8( z − 2) 2 = 0 ,
or
( x − 3) 2 ( y + 1) 2 ( z − 2) 2
+ − = 0.
8 6 3
After introducing new variables x = x − 3, y = y + 1, z = z − 2 , we can get
an equation
x2 y2 z 2
+ − = 0,
8 6 3
that is a standard equation of a cone whose vertex is a point P(3; –1; 2) (in the
system Оxyz).

Micromodule 11
CLASS AND HOME ASSINMENTS

1. Determine what surface is defined by the following equation:


x2 y2 z2 y2 z2 y2 z2
а) + + =1; b) x 2 + + = −1 ; c) x 2 + − = −1 ;
5 7 25 4 9 4 9
y2
d) 9 z 2 + 4 y 2 = 36 x ; e) x 2 + z 2 − = 0 ; f) x 2 + z 2 −1 = 0 ;
4
g) z 2 − y 2 = x ; h) x 2 − z 2 = 0 ; i) x 2 + y 2 + z 2 = 9 .
2. Determine what surface is defined by the following equation:
a) x 2 + y 2 + z 2 − 2 x + 4 y + 6 z − 2 = 0 ;
b) 5 x 2 + 4 y 2 + 3 z 2 + 10 x − 8 y − 6 z = 0 ;
c) y 2 + 2 x − 4 y + 16 = 0 .
x y z+2
3. Find an intersection point of the straight line = = and the
4 −3 4
x2 y2 z2
surface + − =1.
16 9 4
4. Form an equatin of a sphere whose points А(5; 0; –2) and В(1; 4; 0) are
endpoints of its diameter.

135
5. Form an equation of a revolution paraboloid obtained by rotation of the
⎧ x 2 = 12 y,
parabola ⎨ about the axis Оz.
⎩ y=0
Answers

2. а)a sphere; b)an ellipsoid; c)a parabolic cylinder. 3. (4;–3; 2). 4. ( x − 3) 2 +


+ ( y − 2)2 + ( z + 1)2 = 9 . 5. x 2 + y 2 = 12 z.

Micromodule 11
SELF-TEST ASSINMENTS
11.1. General equation of a second order surface is given. Determine what
surface is defined by that equation and reduce it to standard form.
11.1.1. 36 x 2 + 4 y 2 − 8 y + 9 z 2 − 32 = 0.
11.1.2. x 2 + 2 y 2 − 4 x + 4 y − 4 z + 6 = 0.
11.1.3. x 2 + 16 y 2 − 4 z 2 − 4 x + 8 z = 0.
11.1.4. 3x 2 + y 2 − 6 x + 4 y − 2 z + 7 = 0.
11.1.5. x 2 + 4 y 2 − 4 x − 24 y − 4 z + 40 = 0.
11.1.6. x 2 + y 2 − z 2 − 2 x + 4 y + 6 z − 4 = 0.
11.1.7. x 2 + y 2 − z 2 + 4 x + 2 y + 4 z + 1 = 0.
11.1.8. x 2 + y 2 − z 2 − 6 x − 2 y − 2 z + 9 = 0.
11.1.9. x 2 + y 2 − z 2 − 4 x − 4 y − 4 z + 4 = 0.
11.1.10. 4 x 2 + y 2 − 2 z 2 − 8 x + 4 z + 2 = 0.
11.1.11. 12 x 2 + 3 y 2 + 4 z 2 − 24 x + 12 y + 8 z + 16 = 0.
11.1.12. 4 x 2 + 2 y 2 + z 2 − 8 x − 4 y − 4 z + 6 = 0.
11.1.13. 4 x 2 + 9 y 2 + 36 z 2 − 36 y − 72 z + 36 = 0.
11.1.14. 3x 2 + y 2 + 9 z 2 + 12 x − 2 y + 4 = 0.
11.1.15. x 2 + 5 y 2 + 5 z 2 − 4 x − 20 z − 1 = 0.
11.1.16. 3x 2 + 4 y 2 − 4 z 2 − 8 y + 8 z − 12 = 0.
11.1.17. 3x 2 + 2 y 2 − 6 z 2 − 6 x + 4 y − 1 = 0.
11.1.18. x 2 + 3 y 2 − z 2 − 4 x + 4 z + 1 = 0.

136
11.1.19. 4 x 2 + 9 y 2 − 36 z 2 + 8 x − 18 y − 23 = 0.
11.1.20. 2 x 2 + 5 y 2 − 10 z 2 + 8 x + 20 z − 12 = 0.
11.1.21. 4 x 2 + y 2 − 4 z 2 − 16 x − 2 y + 21 = 0.
11.1.22. 9 x 2 + y 2 − 9 z 2 + 18 x + 2 y + 19 = 0.
11.1.23. 2 x 2 + y 2 − 2 z 2 − 12 x − 4 y + 22 = 0.
11.1.24. 4 x 2 + 4 y 2 − z 2 + 16 x + 8 y + 2 z + 23 = 0.
11.1.25. 4 x 2 + 2 y 2 − z 2 − 8 y − 2 z + 11 = 0.
11.1.26. 2 x 2 − y 2 − 4 x − 2 y − 6 z + 1 = 0.
11.1.27. 5 x 2 − 4 y 2 + 10 x + 8 y − 20 z + 1 = 0.
11.1.28. 9 x 2 − 4 y 2 − 16 y − 36 z − 52 = 0.
11.1.29. 4 x 2 − y 2 + 4 y − 4 z + 4 = 0.
11.1.30. 4 x 2 − y 2 − 16 x − 2 y − 4 z + 7 = 0.

137
Моdule INTRODUCTION TO MATHEMATICAL ANALYSIS.

3
DERIVATIVES AND DIFFERENTIALS
OF A FUNCTION OF ONE VARIABLE.
APPLICATIONS OF DERIVATIVES

MODULE STRUCTURE
Micromodule 12. Sequence. The limit of a numerical sequence. Theo-
rems about limits.
Micromodule 13. The Concept of a function. Classification of functions.
Limits of function. Theorems about limits.
Micromodule 14. Honorable limits.
Micromodule 15. Comparison of infinitesimals. Equivalent infinitesi-
mals. Their application in calculation of limits.
Micromodule 16. Continuity of a function.
Micromodule 17. The derivative. Geometrical, mechanical and physical
interpretation. Differentiability and continuity. The differentiation rules.
The derivatives of the elementary functions. The derivatives of composite
functions.
Micromodule 18. The derivative of an inverse function. The derivative of
an implicit function and parametric function. Logarithmic differentiation.
Micromodule 19. The differential of a function. Geometrical interpre-
tation of a differential. Applications of the differentials. A tangent line and a
normal.
Micromodule 20. Higher order derivatives and differentials. Leibniz’s
formula.
Micromodule 21. Basic theorems of differentiable functions (Fermat’s,
Rail’s, Lagrange’s, Cauchy’s theorems). Taylor’s and Maclaurin’s formula.
L’Hospital’s rule.
Micromodule 22. The usage of derivative for function investigation.
Function monotony. Extremum. Concavity intervals, inflection points. Asym-
ptotes. Minimum and maximum values of a function. General investigation of
a function. General plan of graph construction.

Basic concepts. 1. A sequence. 2. A function. 3. A limit. 4. A continuity.


5. A derivative. 6. A differential. 7. A tangent line. 8. Function increase and
decrease. 9. Relative extremum. 10. Concavity intervals. 11. Inflection points.
12. Asymptotes.

Key words: sequence — послідовність, limit — границя, continuity —


неперервність, discontinuity — розривність, removable discontinuity — усув-

138
на розривність, infinitesimals — нескінченно малі, infinitely large — нескін-
ченно велика, indeterminate form — невизначений вираз, indeterminacy —
невизначеність, domain of definition — область визначення, a range —
множина значень, inverse function — обернена функція, еven function —
парна функція, odd function — непарна функція, periodic function —
періодична функція, honorable limit — важлива границя, derivative —
похідна, differential — диференціал, relative extremum — відносний екстре-
мум, asymptote — асимптота, inflection point — точка перегину, convexity —
опуклість, concavity –вгнутість.

Main tasks: 1. Evaluation of limits. 2. Investigation of continuity. 3. Evalua-


tion of the first and higher order derivatives. 4. Evaluation of the derivatives of
the implicit functions and the parametric functions. Logarithmic differentiation.
5. Evaluation of the differentials of functions. 6. The usage of derivative. 7. The
usage of the differentials. 8. Functions investigation. 9. Graph construction.

THE STUDENT MUST BE READY TO DO


THE FOLLOWING ASSIGNMENTS

1. Concepts, definitions, formulations:


1. Sets. Classification of numerical sets. Operations on sets. The modules of
a real number.
2. A sequence.
3. A function. Classification of functions. The elementary functions. An
inverse function. A composite function.
4. The Limit of a numerical sequence. The Limit of a function. Infinitesi-
mals.
5. Continuity. Continuity of a function at a point and on an interval. Pro-
perties of continuous functions. Points of discontinuity and its classification.
6. Definition of a derivative. Geometrical and physical interpretation.
7. A table of derivatives.
8. Rules of differentiation.
9. A connection between continuity and differentiability.
10. A differential. Geometrical interpretation of a differential.
11. The usage of the differentials.
12. Evaluation of the first and higher order derivatives.
13. Leibniz’s formula.
14. Lagrange’s formula.

15. L’Hospital’s rule for expansion of indeterminate forms ⎡⎢ ⎤⎥ or ⎡⎢ ⎤⎥ .
0
⎣0⎦ ⎣∞⎦
16. Taylor’s formula.
17. Maclaurin’s formula.

139
18. Investigation for function increase and decrease on the given interval.
19. Investigation of a function for extremum.
20. Minimum and maximum values on the interval.
21. Concavity intervals. Inflection points.
22. Asymptotes.
23. Plan of graph construction.

2. Proofs and conclusions


1. Theorems about limits.
2. The first and the second honorable limits.
3. Theorems about equivalent infinitesimals.
4. The derivatives of elementary functions.
5. The first order and higher order derivatives of the parametric functions.
6. Theorem about continuity of differentiable functions.
7. Geometrical interpretation of the first order differential.
8. Equation of a tangent line and a normal to the curve.
9. Lagrange’s and Fermat’s theorems.
10. L’Hospital’s rule for expanding of indeterminate form ⎡⎢ ⎤⎥ .
0
⎣0⎦
11. The necessary monotony conditions.
12. The necessary and sufficient extremum conditions.
13. Curve asymptotes seeking rule.

3. Assignments
1. Find the derivatives of functions.
2. Find the derivatives of composite functions, implicit functions and para-
metric functions.
3. Find the differentials of functions.
4. Find the derivatives and the differentials of higher order.
5. Solve tasks for geometrical and physical interpretation of a derivative.
6. Investigate elementary functions.
7. Sketch the graphs of elementary functions.
8. Evaluate the limits.
9. Evaluate the limits using the equivalent infinitesimals.
10. Investigate functions for continuity.
11. Find different limits with the help of L’Hospital’s rule.
12. Find intervals of function increase and decrease, local extremum.
13. Find concavity intervals.
14. Find graph asymptotes.
15. Construct the graph.

140
Micromodule 12
BASIC THEORETICAL INFORMATION
SEQUENCE. THE LIMIT OF A NUMERICAL SEQUENCE.
THEOREMS ABOUT LIMITS

Sequence. The limit of a numerical sequence. Theorems about limits.


Number e. Determinate and indeterminate forms.

Literature: [2, chapter 1], [3, chapter 3, §§ 3.1—3.8], [4, part 4, §§ 3.1—
3.3, 4.2—4.3], [6, chapter 4, § 3], [7, chapter 4, § 11], [10, chapter 3,
§ 2], [11, chapter 3, § 1], [12 chapter 2, § 1], [13].

12.1. Sequence

Definition 3.1. If the numbers xn correspond to each natural number accor-


ding to the definite rule then the set of numbers {x1 , x2 , … , xn , …} is called the
sequence of numbers.
{ }
We denote a sequence as x n ; xn is the term of sequence.

12.2. The Limit of a Numerical Sequence

Definition 3.2. Number a is called a limit of a sequence { x n } if for any


positive number ε > 0 such number N ( ε ) may be found, that for any
n > N ( ε ) the inequality xn − a < ε will hold.

In this case we write down: lim xn = a or xn → a for n → ∞.


n →∞
Definition 3.3. The sequence { x n } is said to be infinitesimal, if

lim xn = 0 .
n →∞

Definition 3.4. The sequence { x n } is said to be infinitely large if for any


number M > 0 there exists such number N so that for n > N the inequality
xn > M holds. In this case we write down: lim xn = ∞.
n →∞

141
12.3. Theorems about Limits

We shall formulate properties of limits as theorems.


Theorem 3.1 The sequence can have only one limit.

Theorem 3.2 The sequence which has a finite limit, is bounded.

Theorem 3.3 If {xn } and { yn } have limits the following equalities are valid:

1) lim ( xn ± yn ) = lim xn ± lim yn ; 2) lim xn yn = lim xn ⋅ lim yn ;


n →∞ n →∞ n →∞ n →∞ n →∞ n →∞
3) lim Cyn = C lim yn , where С is constant;
n →∞ n →∞

( lim y ≠ 0).
lim xn
xn n→∞
4) lim = n
n →∞ yn lim yn n →∞
n →∞

Theorem 3.4 If xn ≥ 0, then lim xn = a ≥ 0.


n →∞

Theorem 3.5 If for sequences {xn}, {yn } and {zn}: xn ≤ yn ≤ zn and lim xn = a,
n→∞
lim zn = a , then lim yn exists and lim yn = a .
n →∞ n →∞ n →∞

Theorem 3.6 Any monotone bounded sequence has a limit.

Theorem 3.7 For all elementary functions at any point where they are deter-

x→a
( )
mined, the equality lim f ( x ) = f lim x is valid.
x→a
n
Using Theorem 3.6. we can prove that there exists lim ⎛⎜ 1 + ⎞⎟ . We denote
1
n →∞ ⎝ n⎠
this limit as e = 2, 71828… .
That is,
n
lim ⎛⎜ 1 + ⎞⎟ = e.
1
n →∞ ⎝ n⎠

12.4. The Determinate and Indeterminate Expressions

While calculating the limits of variables it is necessary to take into account


the following:

142
1) a sum and a product of the finite number of infinitesimal magnitudes, and
also a product of an infinitesimal by a bounded magnitude are infinitesimal
magnitudes;
2) a sum and a product of infinitely large magnitudes, and also a product of
infinitely large magnitude by a nonzero constant are infinitely large magnitudes;
3) a quotient of a constant by an infinitely large magnitude is an infinitesi-
mal, a quotient of nonzero constant by an infinitesimal is an infinitely large
magnitude.
In the examples of finding the limits sometimes indeterminate expressions
occur: the ratio of two infinitesimal magnitudes; the ratio of two infinitely large
magnitudes; a difference of two infinitely large magnitudes; a product of an
infinitesimal by an infinitely large magnitude; an infinitesimal or infinitely large
magnitude in an infinitesimal degree; a magnitude which tends to unit in an infinitely
large degree. Symbolically the indeterminate expressions can be written down as:

0 ∞
, , ∞ − ∞, 0 ⋅ ∞, 00 , ∞0 , 1∞.
0 ∞

Micromodule 12
EXAMPLES OF PROBLEMS SOLUTION

n2 + 1
Еxample 1. Prove that lim = 1.
n →∞ n
Solution. Let us find such number N ( ε ) so that xn − 1 < ε for n > N ( ε ) .
For this purpose we should solve the inequality

n2 + 1
− 1 < ε. (3.1)
n

As far as n ∈ N we get from the inequality (3.1):

n2 + 1 n2 + 1 1 1
< ( ε + 1) ⇒ 2 < ε 2 + 2ε ⇒ n >
2
−1 < ε ⇒ .
n n 2
n 2
ε + 2ε
If to take N ( ε ) so that it would be equal to the nearest but greater than
1
natural number then xn − 1 < ε for all n > N ( ε ) . From here it
2
ε + 2ε
n2 + 1
follows that lim = 1.
n →∞ n
7n 4 − 5n 2 + 4n
Еxample 2. Find lim .
n →∞ 10 + 2n − 3n 4

143

Solution. We have an indeterminacy of a kind ⎡⎢ ⎤⎥ . We evaluate it by
⎣∞⎦
dividing the numerator and the denominator by the greatest power of n:

5 4
7− 2 + 3
7n 4 − 5n 2 + 4n n n
lim = lim =
n →∞ 10 + 2n − 3n 4 n →∞ 10 2
+ − 3
n 4 n3
5 4
lim 7 − lim + lim 3
=
n →∞ n 2
n →∞ n →∞ n = 7−0+0 = −7.
10 2 0+0−3 3
lim + lim 3 − lim 3
n →∞ n 4 n →∞ n n →∞

By means of division of the numerator and the denominator by the greatest


power of n it is possible to prove the validity of such formula:

⎧a0 / b0 , if m = k;
a0 n m + a1n m −1 + … + am ⎪
lim = ⎨ 0, if m < k;
n →∞ b0 n k + b1n k −1 + … + bk ⎪⎩ ∞, if m > k.

This formula is valid if we consider in the similar way limits of sequences


where there is a value n and n → ∞ instead of x.

2 1
n5 − n3 + 4
Еxample 3. Find lim 3
.
n →∞
(n − 2) 7 −5

Solution. We have an indeterminacy of a kind ⎡⎢ ⎤⎥ . Then
⎣∞⎦

2 1 2 1 2 2
− −
n5 − n3 + 4 n 5 (1 − n 15 + 4n 5)
n5
lim 3
= lim 3
= lim 3
=
n →∞ n →∞ 3 3 n →∞ 3
(n − 2) 7 − 5 n7 ⎛ 1 − 2 ⎞ 7 − 5n − 7 n7 ⎛1 − 2 ⎞ 7
⎜ n⎟ ⎜ n⎟
⎝ ⎠ ⎝ ⎠
2
n5 1 1 1
= lim = lim = lim = =0.
n →∞ 3 n →∞ 3 2
− n →∞ 1 ∞
n7 n7 5 n 35

144
Here we have used limits

lim n p = 0, if p < 0 ;
n →∞

lim n p = ∞, if p > 0.
n →∞

n !− (n + 1)!
Еxample 4. Find lim .
n →∞ (n + 2)!− n !

Solution. We have an indeterminacy of a kind ⎡⎢ ⎤⎥ . Then
⎣∞⎦
n !− (n + 1)! n !− n !(n + 1)
lim = lim =
n →∞ ( n + 2)!− n ! n →∞ n !( n + 1)( n + 2) − n !

n !(1 − n − 1) −n −n
= lim = lim = lim 2 =0
n →∞ n !(( n + 1)( n + 2) − 1) n →∞ ( n + 1)( n + 2) − 1 n →∞ n + 3n + 1
.

1 + 3 + 5 + … + (2n − 1)
Еxample 5. Find lim .
n →∞ 1 + 4 + 7 + … + (3n − 2)

Solution. We have an indeterminacy of a kind ⎡⎢ ⎤⎥ . We use the formula
⎣∞⎦
a +a
Sn = 1 n n for sum of n terms of arithmetical progression:
2
1 + 2n − 1
1 + 3 + 5 + … + (2n − 1) = n = n2 ,
2
1 + 3n − 2 n(3n − 1)
1 + 4 + 7 + … + (3n − 2) = n= .
2 2
Then
1 + 3 + 5 + … + (2n − 1) 2n 2 2n 2
lim = lim = lim = .
n →∞ 1 + 4 + 7 + … + (3n − 2) n →∞ n(3n − 1) n →∞ 3n − 1 3

Еxample 6. lim ( n 2 − 2n − 3 − n) .
n →∞
Solution. We have an indeterminacy of a kind (∞ − ∞). Let us multiply the
numerator and the denominator of the given fraction by the expression conjugate
to the numerator n 2 − 2n − 3 + n . Then

( n 2 − 2n − 3 − n)( n 2 − 2n − 3 + n)
lim ( n 2 − 2n − 3 − n) = lim =
n →∞ n →∞
n 2 − 2n − 3 + n

145
n 2 − 2n − 3 − n 2 −2n − 3
= lim = lim =
n →∞ 2 n →∞ 2
n − 2n − 3 + n n − 2n − 3 + n
3
−2 −
⎛∞⎞ n −2
= ⎜ ⎟ = lim = = −1
⎝ ∞ ⎠ n→∞ 2 3 1+1
1− − +1
n n2 .

Micromodule 12
CLASS AND HOME ASSIGNMENT

Using the definition of a limit prove that lim xn = a .


n →∞

3n − 5 3 n2 + 5 6n 2 + 7 6
1. lim = . 2. lim =1. 3. lim = .
n →∞ 2n − 1 2 n →∞ n n →∞ 5n 2 + 6 5
2n + 3 2
4. lim a n = 0 , if a < 1 . 5. lim = .
n →∞ n →∞ n + 2
3 3

Evaluate limits of sequences:

3n − 2 n3 + 1 100n 2 + n + 1
6. а) lim ; b) lim ; c) lim .
n →∞ n + 1000 n →∞ 2n + 11 n →∞ 4n3 + 5
13 14
4n 4 − 5n 2 + 4 n 3 + 3n 4 + 2
7. lim 3 . 8. lim 16
.
n →∞ ( n − n + 2)(2n + 3) n →∞
n5 −1

n !⋅ (n 2 + 1) ( n + 2 )!+ n !
9. lim . 10. lim .
n →∞ ( n + 2 ) ! n →∞ ( n + 2 )!− (n + 1)!
3 + 5 + … + (2n + 1) 3n + 1
11. lim . 12. lim .
n →∞ 2n 2 − 3n + 4 n →∞ 3n +1 + 1

1
3n + 2n 2n −1
13. lim n . 14. lim .
n→∞ 3 + 4 n n →∞ 1
4n −1

15. lim ( 4n 2 − n − 2 − 2n) .


n →∞

146
Answers
6. а) 3; b) ∞; c) 0. 7. 2. 8. ∞. 9. 1. 10. 1. 11. 1/2. 12. 1/3. 13. 0. 14. 1/2.
15. –1/4.

Micromodule 12
SELF-TEST ASSIGNMENTS

Evaluate limits of sequences:


10 13
(2 + n)3 − (3 + n)3 + 1 n 3 + 2n 4 + 1
12.1. а) lim ; b) lim ;
n →∞ (2 + n) 2 + (3 + n) 2 n →∞ 15
n4 −2
( n + 2 )!+ ( n + 1)! n
5 −2 n
c) lim ; d) lim .
n →∞ ( n + 3) ! n→∞ 5n + 2n
4 3
3n3 − 2(n + 1) 2 + 2 n7 − n5 + 3
12.2. а) lim ; b) lim ;
n →∞ (2n + 1)3 + n − 1 n →∞ 5
(n − 1) 8
1 + 5 + … + (4n − 3)
c) lim ; d) lim ( n 2 − 4n + 3 − n 2 + 1) .
n →∞ 1 + 7 + … + (6n − 5) n →∞
2 3
5n 4 − 3n + 1 n 3 + 2n 4 + 1
12.3. а) lim ; b) lim ;
n →∞ 2 − 7 n − 2n 4 n →∞ 7
(n + 3) 8

( )
3
2 + 4 + 6 + ... + 2n
c) lim ; d) lim n 2 n3 + 3 − n3 − 2 .
n →∞ 1 + 3 + 5 + ... + ( 2n + 1) n →∞

1000n 4 − 3n + 1 ⎛1 1 n −1 1 ⎞
12.4. а) lim ; b) lim ⎜ − + ... + ( −1) ⎟;
n →∞ 2 − 7 n − n5 n →∞ ⎝ 5 25 5n ⎠
6 7
n 5 + 2n 6 + 1
c) lim 13
; d) lim ( n 2 − n + 3 − n 2 + n ) .
n →∞ n →∞
n10 −4
4 5
(2n) 4 − 3n3 + n n 3 + 2n 4 + 5
12.5. а) lim ; b) lim ;
n →∞ 100 + 4n + n3 n →∞ 11
n8 −2
1
( n + 2 )!+ ( n + 1)! 5n −1
c) lim ; d) lim .
n →∞ ( n + 3) ! n →∞ 1
5n +1

147
(3n) 4 + 5n3 − 2n n 4 + 5 + 3 7 − 2n3
12.6. а) lim ; b) lim ;
n →∞ (4n)3 − n − 6 n →∞ 3
3 − 8n6 + n − 1
2
⎛ 1 1 1 ⎞ 8n −1
c) lim ⎜ 1 + + ... + n ⎟; d) lim .
n→∞ ⎝ 3 9 3 ⎠ n →∞ 2
8n +1

(2n − 1) 2 (n − 3) + 6n3 − n n3 − 3 + 3 5n5


12.7. а) lim ; b) lim ;
n →∞ (n − 1)3 + n + 2 n →∞ 4 7
2 n 5 − n8 + 5
⎛ 1 1 1 ⎞
⎜ 1 + 2 + 4 ... + n ⎟ ( n + 2 )!+ ( n + 1)!
c) lim ⎝ 2 ⎠ ; d) lim .
n →∞ ⎛ 1 1 1 ⎞ n →∞ ( n + 2 )!− ( n + 1)!
⎜ 1 + 4 + 16 ... + n ⎟
⎝ 4 ⎠

(3n + 2) 2 (n − 1) + 3n 2 + 1 n4 − 2 + 3 n7
12.8. а) lim ; b) lim ;
n →∞ (n − 2)3 + 2n + 5 n →∞ 4
2n5 − 7 n16 + 1
⎛ 1 1 1 ⎞
⎜ 1 + 3 + 9 ... + n ⎟ ( n + 3)!+ ( n + 2 )!
c) lim ⎝ 3 ⎠ ; d) lim .
n →∞ ⎛ 1 1 1 ⎞ n →∞ ( n + 3 ) !− ( n + 2 ) !
⎜ 1 + 5 + 25 ... + n ⎟
⎝ 5 ⎠

(4n − 1) 2 (n + 2) + 2n 2 − 1 n 5 + 1 + 4 n9
12.9. а) lim ; b) lim ;
n →∞ (2n − 1)3 + n 2 n →∞ 4 7
2n11 − n9 + 1
⎛ 1 1 (−1) n ⎞
⎜1 − 2 + 4 ... + n ⎟
2 ⎠ ( n − 1)!− ( n − 2 )!
c) lim ⎝ ; d) lim .
n →∞ ⎛ 1 1 (−1)n ⎞ n →∞ ( n − 1) !+ ( n − 2 ) !
⎜1 − 4 + 16 ... + n ⎟
⎝ 4 ⎠

(3n − 1) 2 (n − 2) − 5n3 n − 3 + 3 3n 4
12.10. а) lim ; b) lim ;
n →∞ 8(n − 1)3 + 4n + 2 n →∞ 4
2n3 − 3 6n 4
⎛ 1 1 1 ⎞
⎜ 1 + 3 + 9 ... + n ⎟ ( n + 2 )!+ n !
c) lim ⎝ 3 ⎠ ; d) lim .
n →∞ ⎛ 1 1 (−1) n ⎞ n →∞ ( n + 2 )!− n !
⎜ 1 − 3 + 9 ... + n ⎟
⎝ 3 ⎠

148
(2n − 1) 2 (n − 1) − 5n3 3
n 4 + 7 − 6 5n7
12.11. а) lim ; b) lim ;
n →∞ 8(n + 1)5 + n + 1 n →∞ 5
n7 + 3 + n
⎛ 1 1 1 ⎞
⎜ 1 + 6 + 36 ... + n ⎟ 2 ( n + 2 ) !+ 3 ⋅ n !
c) lim ⎝ 6 ⎠ ; d) lim .
n →∞ ⎛ 1 1 (−1) n ⎞ n →∞ ( n + 2 )!− n !
⎜ 1 − 4 + 16 ... + n ⎟
⎝ 4 ⎠

(2n + 1) 2 (n + 2) + 3n3 3
2n 4 + 1 + 5 n 7
12.12. а) lim ; b) lim ;
n →∞ 5(n − 3) 4 n →∞ 5
n8 + 2 + n
⎛ 1 1 1 ⎞
⎜1 + 7 + 49 ... + n ⎟ 3 ( n + 1) !+ (n − 1)!
c) lim ⎝ 7 ⎠ ; d) lim .
n →∞ ⎛ 1 1 (−1) n ⎞ n →∞ ( n + 1) !− ( n − 1)!
⎜ 1 − 7 + 49 ... + n ⎟
⎝ 7 ⎠

(3n − 1)(n − 1) 2 + 2n3 3


n7 + 5 + 4 3n9
12.13. а) lim ; b) lim ;
n →∞ 10(n + 1)3 + 1 n →∞ 5
n7 + 2 + n
1 + 2 + 3 − 4 + ... − 2n ( n + 2 )!+ 5 ⋅ n !
c) lim ; d) lim .
n →∞ 2 n →∞ 2 ( n + 2 ) !− n !
n +1
(2n − 1)(n − 3) 2 + 7 n3 5
n7 + 4 2n11
12.14. а) lim ; b) lim ;
n →∞ 14(n − 1)3 + 2 n →∞ 5
n 7 + 1 + n5
1 + 3 + 5 + ... + (2n − 1) ( n + 3)!+ 5 ⋅ (n + 2)!
c) lim ; d) lim .
n →∞
2n + 32 n →∞ 3 ( n + 3) !− (n + 2)!

(2n − 1)(n − 3) 2 − n3 4
n11 + 4 3n13
12.15. а) lim ; b) lim ;
n →∞ 11(n − 2)3 + 22 n →∞ 5
n16 + 1 + n
1 + 4 + 7 + ... + (3n − 2) ( n + 3)!− 4 ⋅ (n + 2)!
c) lim ; d) lim .
n →∞ 2
2n + 1 n →∞ 2 ( n + 3) !+ (n + 2)!

(4n − 1)(n + 1) 2 + 3n3 3


n10 + 4 2n13
12.16. а) lim ; b) lim ;
n →∞ 6(n + 2)3 + 5 n →∞ 5
n16 + 1 + 2
1 + 5 + 9 + ... + (4n − 3) ( n + 3)!− 5 ⋅ (n + 2)!
c) lim ; d) lim .
n →∞ 2
3n + 1 n →∞ 3 ( n + 3) !+ (n + 2)!

149
( n + 2 )3 − ( n − 1)3 3
n7 + 4 5n9
12.17. а) lim ; b) lim ;
n →∞ 3n 2 + 2n − 1 n →∞ 7
n16 − 1 + 3
1 + 6 + 11 + ... + (5n − 4) ( n − 1)!− 2 ⋅ (n − 2)!
c) lim ; d) lim .
n →∞ n →∞ 3 ( n − 1) !+ ( n − 2)!
n 4n 2 + 1
( n + 3 )3 − ( n − 2 )3 3
n5 + 4 3n7
12.18. а) lim ; b) lim ;
n →∞ 5n 2 + 2n − 6 n →∞ 5
n8 − 1 + 1
1 + 7 + 13 + ... + (6n − 5) n !− 2 ⋅ (n − 2)!
c) lim ; d) lim .
n →∞ n →∞ n !+ ( n − 2)!
n 5n 2 + 1
( n + 1)4 − ( n − 1)4 3
n 4 + 5 6n 6
12.19. а) lim ; b) lim ;
n →∞ 2n3 + n + 1 n →∞ 5
n6 − 1 + 2
⎛ 1 2 n −1 ⎞ ( n + 1)!− 2 ⋅ (n − 1)!
c) lim ⎜ 2 + 2 + ... + 2 ⎟ ; d) lim .
n →∞ ⎝ n n n ⎠ n →∞ 4 ( n + 1) !+ ( n − 1)!

( n + 2 )4 − ( n − 2 )4 5
n 4 + 5 3n3
12.20. а) lim ; b) lim ;
n →∞ 4n3 + 1 n →∞ 7
n6 + 1 − 1
⎛ 1 4 3n − 2 ⎞ ( n + 1)!− 4 ⋅ (n − 1)!
c) lim ⎜ 2 + 2 + ... + 2 ⎟ ; d) lim .
n →∞ ⎝ n n n ⎠ n →∞ 2 ( n + 1) !+ ( n − 1)!

( )
3
5n(n − 1)3 − 2n
12.21. а) lim ; b) lim n 2 n 3 + 2 − n3 − 4 ;
n →∞ 2 − 8n 2 − 3n 4 n →∞
2 3
2 + 6 + 10 + ... + (4n − 2) n 7 + 4n 8 + 2
c) lim ; d) lim .
n →∞ 1 + 3 + 5 + ... + ( 2n + 1) n →∞ 7
(n + 1)16
(2n + 1)(2n − 1) 2 + 5n3 4
n7 + 5 4n11
12.22. а) lim ; b) lim ;
n →∞ 3(n − 2)3 + 1 n →∞ 3
n7 + 1 + n7
1 + 3 + 5 + ... + (2n − 1) 1 + 4n + 2
c) lim ; d) lim .
n →∞ (2n + 3)(n + 1) n →∞ 5 − 3 ⋅ 4n

(2n − 3) 2 (n − 3) − 6n3 n + 2 + 7 3n 4
12.23. а) lim 3
; b) lim ;
n →∞ 2(2n − 1) n →∞ 4 9
n3 − 3n 4
⎛ 1 1 1 ⎞
⎜ 1 + 4 + 16 ... + n ⎟ ( n + 4 )!+ (n + 2)!
c) lim ⎝ 4 ⎠ ; d) lim .
n →∞ ⎛ 1 1 (−1) n ⎞ n →∞ ( n + 4 ) !− ( n + 3)!
⎜ 1 − 4 + 16 ... + n ⎟
⎝ 4 ⎠

150
(2n − 3)3 + 7 n3 n + 1 + 6 n5
12.24. а) lim ; b) lim ;
n →∞ 3(3n − 1)3 n →∞ 4 7
n3 + 3n 4
2 + 5 + 8 + ... + (3n − 1) ( n + 5)!+ (n + 4)!
c) lim ; d) lim .
n →∞ n (4n + 2)(n + 11) n →∞ ( n + 5 ) !− ( n + 3)!

3
6
(2n − 5)(n + 1)(3n − 1) n7 + n 2
12.25. а) lim ; b) lim ;
n →∞ (4n − 1)3 n →∞ 4 3
n3 + 4n 4
3 + 5 + 7 + ... + (2n + 1) ( n + 6 )!+ (n + 4)!
c) lim ; d) lim .
n →∞ (n + 1) 2n + 3 n →∞ ( n + 6 ) !− ( n + 5)!

4
3
(3n + 1)(n − 1)(2n − 3) n2 + n 3
12.26. а) lim ; b) lim ;
n →∞ (n + 2)3 n →∞ 5 3
n3 + 8n 4
3 + 6 + 9 + ... + 3n ( n + 2 )!+ (n + 1)!
c) lim ; d) lim .
n →∞ n n+3 n →∞ ( n + 2 )!+ n !
7
3
(n + 2)(2n − 1)(2n + 3) n5 + n 4
12.27. а) lim ; b) lim ;
n →∞ (2n + 5)3 n →∞ 5 3
n8 + 27n 4
3 + 7 + 11 + ... + (4n − 1) 2 ( n + 2 ) !+ (n + 1)!
c) lim ; d) lim .
n →∞ 2 + 6 + 10 + … + (4n − 2) n →∞ 3 ( n + 2 ) !− n !
11
4
(n + 4)(3n − 1)(2n + 1) n9 + n 5
12.28. а) lim ; b) lim ;
n →∞ (n + 1)(2n + 3) 2 n →∞ 5 6
n8 + n13
1 + 5 + 9 + ... + (4n − 3) ( n + 3)!+ (n + 2)!
c) lim ; d) lim .
n →∞ 2 + 5 + 8 + … + (3n − 1) n →∞ 10 ( n + 2 ) !− ( n + 1)!

1
(n − 3)(2n − 3)(n + 2) 4
n + n5
12.29. а) lim ; b) lim ;
n →∞ (n + 4)(2n + 7) 2 n →∞ 5
n2 + n2
7

3 + 5 + 7 + ... + (2n + 1) ( n + 4 )!+ (n + 3)!


c) lim ; d) lim .
n →∞ 2 + 4 + 6 + … + 2n n →∞ n ( n + 3) !− ( n + 2)!

(3n − 1)(2n − 1)(n − 1) (n + 1)(n + 1)!+ (n + 2)!


12.30. а) lim ; b) lim ;
n →∞ (n + 2)(2n + 3) 2 n →∞ n(n + 1)!− (n + 2)!
3
5
⎛ 1 5 4n − 3 ⎞ n2 + n 8
c) lim ⎜ 2 + 2 + ... + 2 ⎟ ; d) lim .
n →∞ ⎝ n n n ⎠ n →∞ 5 7
n2 + n4

151
Micromodule 13
BASIC THEORETICAL INFORMATION
THE CONCEPT OF A FUNCTION. CLASSIFICATION
OF FUNCTIONS. LIMIT OF A FUNCTION.
THEOREMS ABOUT LIMITS

The Concept of a function.


Elementary functions and their graphs.

13.1. The concept of a function

Definition 3.5. If to each value of a variable x which belongs to a set of real


numbers D there corresponds by a certain rule the definite value of y which
belongs to a set of real numbers E then y is said to be a function of x. Thus,
we write y = f ( x) or f : D → E. The element x ∈ D is said to be an argument
of the function f , the element y ∈ E is the value of the function f correspon-
ding to the element x ∈ D .
Definition 3.6. The set D is called a domain of definition of the function
f ( x) , and the set E is a range (or the set of values) of the function f ( x ) .
Different ways of functions representation are possible:
1. analytical method (by means of the formula);
2. tabular method (by means of a table where numerical values of an inde-
pendent variable and values of a function corresponding to them) had written;
3. graphic method.
For the graphic method we consider variables x and y as Cartesian coordi-
nates of a point on a plane. In this case the graph of a function is a set of points
on a coordinate plane x0 y , whose abscissas are independent variables, and
ordinates are corresponding values of the function.
Definition 3.7. If the function reflects the set D into the set E so that for
any x1 and x2 ∈ D provide x1 ≠ x2 , f ( x1 ) ≠ f ( x2 ) then a function of x = ϕ( y )
which reflects the set E into the set D is said to be an inverse to the function
f ( x). To find a function inverse to the given y = f ( x ), it is necessary to express
x from this equality through y . By tradition, a function inverse to y is denoted
so: y = ϕ( x).
If the functions y = f ( x ) and x = ϕ( y ) are mutually inversed, their graphs
are expressed by the same curve. But, if we denote again an argument of the
inverse function through x, and a function through y the graphs of the func-
tions y = f ( x ) and y = ϕ( x) will be symmetric relatively bisectors of the first
and the third coordinate angles.

152
The domain of definition D of a function y = f ( x) is a set of those numbers
that the argument of the function can take on and for which the analytical
expression of the function is meaningful.
Those are real values of the argument for which:
1. an expression standing in the denominator of a fraction is not equal to
zero;
2. an expression under a square root or a root of an even degree is positive;
3. an expression under the sign of a logarithm is positive;
4. an expression under the sign of arcsine or arccosine does not exceed a unit
by the absolute value;
5. the argument of a cotangent is not equal to n π , n ∈ Z .
π
6. the argument of a tangent is not equal to + n π, n ∈Z .
2
If the given function is the sum of two or the greater number of various
functions we find separately a domain of definition of each function, and a
common part (intersection) of domains of definition of the functions is the
domain of definition of the given function.
Definition 3.8. A function y = f ( x) is said to be even if for any value of an
argument from a domain of definition the equality f (− x) = f ( x) holds. The
graph of an even function is symmetric relatively to the axis of ordinates.
Definition 3.9. A function y = f ( x) is said to be odd if for any value of an
argument from the domain of definition the equality f (− x) = − f ( x) holds. The
graph of an odd function is symmetric relatively to the origin of coordinate
system.
The function which is neither even, nor odd, is said to be as a general view
function.

13.2. Elementary functions and their graphs

The basic elementary functions are the following analytically given functions:
1) a power function y = x α , where α is a real number;
2) an exponential function y = a x , where a is the positive number different
from a unit;
3) a logarithmic function y = log a x , where a is a positive number different
from a unit;
4) trigonometric functions y = sin x, y = cos x, y = tgx, y = ctgx, y = sec x,
y = cosecx.
5) inverse trigonometric functions: y = arc sin x, y = arccos x, y = arctgx,
y = arcctgx, y = arc sec x, y = arccosecx.

153
Let us consider domains of definition and graphs of the basic elementary
functions.
m
1. A power function is y = x α . If α∈Q, i.e. α = , where m ∈ N , n ∈ N .
n
The integer α can be considered as a special case of a fraction if n = 1.
For α > 0 and even n a domain of definition of the function y = x α will be
an infinite interval [ 0; + ∞ ) , for odd n the domain of definition of the function
is an infinite interval ( − ∞; + ∞ ) .
For even m a power function is even, for m and n simultaneously odd a
power function is odd, and for even n the power function is neither even
nor odd.
Graphs of power functions for some values of α > 1 are represented on
4 3 5
Fig. 3.1 (a) y = x 3 , b) y = x 2 , c) y = x 3 ).
Graphs of the power function y = x α for 0 < α < 1 are represented on
3 3 2
Fig. 3.2 (a) y = x 4 , b) y = x 5 , c) y = x 3 ).
If α < 0, x = 0 does not enter into a domain of definition of a power function
y = xα .
2

Graphs of function y = x α at α < 0 are represented on Fig. 3.3 (a) y = x 5,

3 3
− −
b) y = x 5 , c) y = x 2 ).
If a power function is even for α > 0 the range of this function is an infinite
interval [ 0; + ∞ ) , and for α < 0 is an interval ( 0; + ∞ ) .

4 3 4

3 3
1
2 2

1 –3 –1 1 3
–1 1

–3 –1 1 3 –1 1 3
–3
a b c
Fig. 3.1

154
3 2 4
1 3
2
2
1 –3 –1 1 3
–1 1
–2
–1 1 3 5 –3 –1 1 3
a b c
Fig. 3.2
3
3

2 1

–3 –1 1 3
1 –1

–1 0 1 –3
a b
4

–1 0 1 2 3 4
c
Fig. 3.3

If a power function is odd for α > 0 the range of this function is an interval
( −∞; ∞ ) , and for α < 0 is a union of intervals ( −∞; 0 ) ∪ ( 0; + ∞ ) .
If a power function is neither even nor odd for α > 0 the range of the
function is the interval [ 0; + ∞ ) , for α < 0 is ( 0; + ∞ ) .
2. Let’s consider the exponential function y = a x , where а is a positive
number different from unit. The domain of definition of an exponential function

155
y = a x is the set of all real numbers, i.e. x ∈ R, and the range of this function is
the set of positive numbers, i.e. y∈( 0; + ∞ ) . Graphs of exponential function for α > 1

( ) ( ) ).
x
1
( a = 2 ) and for 0 < a <1 a=
2
are represented on Fig. 3.4 (a) y = 2 x , b) y =
1
2

4 4

3 3

2 2

1 1

–3 –2 –1 0 1 2 3 –3 –2 –1 0 1 2 3
a b
Fig. 3.4

3. A logarithmic function is y = log a x, where a is a positive number


different from unit. The domain of definition of the logarithmic function
y = log a x is the set of positive numbers ( 0; + ∞ ) , and the range of the function
is the set of all real numbers. Graphs of logarithmic function are represented on
Fig. 3.5 (a) y = log 2 x , b) y = log 1 x ).
2
3 3

1 1

0 0,5 1 1,5 2 0 0,5 1 1,5 2


–1 –1

–3 –3
a b
Fig. 3.5

4. Trigonometric functions. The domain of definition of functions y = cos x


and y = sin x is the set of all real numbers, the domain of functions y = tgx and
π
y = sec x is the set of real numbers which are not equal to + k π, k ∈ Z and
2
the domain of definition of functions y = ctg x and y = cos ec x is the set of real
numbers which are not equal to kπ, k ∈ Z .

156
The range of functions y = cos x and y = sin x is the interval [ −1; 1] , the
range of functions y = tgx and y = ctg x is the set of all real numbers, and the
set of values of functions y = sec x and y = cos ec x is unions of intervals
( −∞; − 1] ∪ [1, + ∞ ) . Graphs of trigonometric functions: y = sin x; y = cos x;
y = tg x; y = ctg x; y = cos ecx; y = sec x are represented on Fig. 3.6 (a) y = sin x,
b) y = cos x , c) y = tgx , d) y = ctg x, e) y = cos ecx , f) y = sec x ).

1 1

1 1

a b
5 5

0 0

5 5
c d
3 3

1 1

0 0
1 1

3 3
e f
Fig. 3.6

157
5. Inverse trigonometric functions. The domain of definition of the functions
y = arcsin x and y = arccos x is the interval [ −1; 1] , the domain of definition of
functions y = arctgx and y = arcctgx is the set of all real numbers, the domain of
definition of functions y = arcsecx and y = arccosecx is the union of intervals
( − ∞ ; − 1] ∪ [1; + ∞ ).
π π
The range of the function y = arcsin x is the interval ⎡⎢ − ; ⎤⎥ , the range of
⎣ 2 2⎦
y = arccos x is the interval [ 0, π] , the range of y = arctgx is the interval
⎛ π π⎞
⎜ − 2 ; 2 ⎟ , the range of y = arcctgx is the interval ( 0; π ) , the range of
⎝ ⎠
y = arccosecx is the set of intervals ⎡ − ; 0 ⎞⎟ ∪ ⎛⎜ 0; ⎤ , and the range of
π π
⎣⎢ 2 ⎠ ⎝ 2 ⎦⎥

y = arcsecx is the set of intervals ⎡ 0 ; ⎞⎟ ∪ ⎛⎜ ; π ⎤⎥ .


π π
⎢⎣ 2 ⎠ ⎝ 2 ⎦
Graphs of inverse trigonometric functions y = arcsin x (а); y = arccos x; y =
= arctg x (в); y = arcctg x (c); y = arccosec x (d); y = arcsec x (е) are represented
on Fig. 3.7.
Definition 3.10. A function is said to be periodic with the period T (where
T is a number which is not equal to zero) if for any argument x from the
domain of definition of the function x + T and x − T belong to the domain of
the function and f ( x + T ) = f ( x ) or f ( x − T ) = f ( x ) .
The period of a function is the least of all positive periods (if such exists).
In this case all periods of a function are multiples to the least period T = kT0 ,
where T0 is the least positive period of the function, and k is an integer not
equal to zero.
The period of the functions y = sin x, y = cos x, y = sec x, y = cosec x is the

number 2π, and the period of the function y = sin ax is the number T = .
a
Definition 3.11. The function received from the basic elementary functions
by means of a finite number of algebraic operations and a finite number of
formation of composite functions is said to be an elementary function.
Functions may be algebraic and transcendental.
Algebraic functions consist of:
— the whole rational function or a polynomial
y = a0 x n + a1 x n −1 + ... + an −1 x + an ,
where n is a whole positive number;
158
— a rational function which is expressed by a ratio of two polynomials:
a x n + a x n −1 + ... + an −1 x + an
y = 0 m 1 m −1 ;
b0 x + b1 x + ... + bm −1 x + bm
— an irrational function which is written as an expression containing the
variable x under the sign of radicals.

–2 –1 0 1 2 –2 –1 0 1 2

a b

–2 0 2 –2 0 2

c d
2

–3 –2 –1 0 1 2 3

–2
–3 –2 –1 0 1 2 3
e f
Fig. 3.7

Definition 3.12. A function y = f ( x ) is said to be algebraic if it satisfies the


equation
A0 ( x ) y n + A1 ( x ) y n −1 + ... + An −1 ( x ) y + An ( x ) = 0 ,

where A0 ( x ) , A1 ( x ) , ..., An ( x ) are whole rational functions of x; A0 ( x ) ≠ 0; n


is a whole positive number.

159
Functions which are not algebraic are said to be transcendental. For
example, y = log a x, y = tgx, y = arcsin x.
Transformations of graphs of functions. If the graph of a function y = f ( x ) is
known: then
а) the graph of the function y 1= − f ( x ) is a mirror reflection of the graph of
the function y = f ( x ) with respect to the axis Ox;
b) the graph of the function y 2 = f ( − x ) is the mirror reflection of the graph
of the function with respect to the axis Oy;
c) the graph of the function y3 = f ( x − a ) is a graph of the function
y = f ( x ) which moves on a units to the right along the axis Ox (for a < 0 a
removal occurs to the left along the axis Ox );
d) ordinates of the graph of the function y4 = A f ( x ) for identical values x
are increased ordinates of the function y = f ( x ) in A times (if A < 1 length
of ordinates are reduced in A times in comparison with the graph of y = f ( x ) );
e) the graph of the function y5 = f ( a x ) is compressed for a > 1 and
stretched for a <1 along the axis Ox in a times. If a function y = f ( x ) is
periodic with the period T , the period of a function y = f ( a x ) is decreased in a
times for a > 1 and increased for 0 < a < 1;
f) the graph of function y6 = f ( x) + B moves along the axis Oy on B units
of scale (for B > 0 upward and for B < 0 downward).

13.3. Limit of a function

Suppose a function y = f ( x) is defined at the some neighbourhood of a


point а, except may be, the point а.

Number A is called a limit of a function y = f(x) if x→ a, if for any very


small number ε > 0 such number δ = δ(ε) > 0 may be found, that for any x
such as ⏐х–a⏐<δ, х≠a, the following inequality is fulfilled
f ( x) − A < ε.

We denote it as:
lim f ( x) = A or f ( x) → A if x → a .
x→a

160
Number A is called a limit of a function y = f ( x) if x → ∞, if for any
small number ε > 0 such number M = M (ε) > 0 may be found, that for any x
such as x > M (ε), the following inequality is fulfilled f ( x) − A < ε.
We denote it as:
lim f ( x) = A or f ( x) → A if x → ∞.
x→∞
A function y = f ( x) is called infinitely large if x → a, if for any number
M > 0 such number δ = δ( M ) > 0 may be found, that for any x such as
⏐х–a⏐< δ, х ≠ a, the following inequality is fulfilled
f ( x) > M .
We denote it as:
A function α ( x) is called infinitesimal if x → a, if
lim α ( x) = 0.
x→a

lim f ( x) = ∞.
x →∞

1
For example, the function y = is infinitely large if х → 0, and is infi-
x
nitesimal if х→±∞ .

13.4. One-sided limits

Number B is called a limit from the right of a function f ( x ) if x → a, if for


any very small number ε > 0 such number δ = δ(ε) > 0 may be found, that for
any x such as 0 < х – a < δ, the following inequality is fulfilled
f ( x) − B < ε.
We denote it as:
lim f ( x) = B, or f(a + 0) = B.
x →a + 0

Number C is called a limit from the left of a function f ( x ) if x→ a, if for


any small number ε > 0 such number δ = δ(ε) > 0 may be found, that for any x
such as 0 < a – x < δ, the following inequality is fulfilled
f ( x) − C < ε.

161
We denote it as:
lim f ( x) = C , or f(a – 0) = C.
x →a −0

A limit from the right and a limit from the left are called one-sided limits of
a function.
If a function f ( x) has a limit at point a, then
lim f ( x ) = f ( a − 0 ) = f ( a + 0 ).
x→ a

13.5. Properties of limits

Property 1. Suppose each function f ( x) and g ( x) has a finite limit at


point а, then:

1) lim( f ( x) ± g ( x)) = lim f ( x) ± lim g ( x) ;


x→ a x→ a x→ a
2) lim f ( x ) g ( x ) = lim f ( x) lim g ( x) ;
x→ a x→ a x→ a

f ( x ) x→ a lim f ( x)
3) lim = (if lim g ( x) ≠ 0 );
x→ a g ( x) lim g ( x) x→ a
x→ a

( )
lim g ( x )
g ( x)
4) lim ( f ( x)) = lim f ( x) x→ a ;
x→ a x→ a

5) lim cf ( x) = c lim f ( x ) ( с is a constant)..


x→ a x→ a
Property 2. Suppose the functions g ( x), f ( x ) and h( x) are defined at
some neighbourhood of point x0 , except may be, the point x0 .
Suppose lim g ( x) = lim h( x) = A and g ( x) ≤ f ( x) ≤ h( x) , then
x → x0 x → x0

lim f ( x) = A .
x→ x0

Property 3. (about limit of monotone function). If a function f ( x ) is mo-


notone and bounded for x < x0 or for x > x0 , then it has limit from the right or
limit from the left.
That is, lim f ( x) = f ( x0 − 0) or lim f ( x) = f ( x0 + 0).
x → x0 − 0 x → x0 + 0

162
Micromodule 13
EXAMPLES OF PROBLEMS SOLUTION

e x − e− x
Example 1. Find a function inverse to the function y = .
2
Solution. We can solve the given equation with respect to x :

e x − e − x = 2 y , i.e. e 2 x − 2 ye x − 1 = 0.

This equation is square with respect to e x : e x = y + y 2 + 1 .


We accept only one value of a root, because e x > 0. After taking logarithms
of both parts we have x = ln ( y + y 2 + 1). This function is inverse to the initial
one. We can rewrite it in a usual form, denoting the argument as x, and the
function as y : y = ln ( x + x 2 + 1).
Example 2. Find a domain of definition of a function

y = sin (2 x − 1) + 20 − x − x 2 .
Solution. The given function is the sum of two functions. Therefore we find
the domain of definition for each function separately: y = sin(2 x −1) and
y = 20 + x − x 2 . For the first function it must be sin (2 x − 1) ≥ 0, i.e. 2 x − 1 ∈

∈[ 2k π; (2k + 1)π] , whence x ∈ ⎡ k π + ; k π + + ⎤ . For the second function the


1 π 1
⎢⎣ 2 2 2 ⎥⎦
domain of definition is the set of numbers x∈[ −4; 5] . Hence, the domain of
definition of the given function is the set of intervals
⎡ k π + 1 ; k π + π + 1 ⎤ ∩ −4; 5 , k ∈ Z ,
[ ]
⎣⎢ 2 2 2 ⎦⎥
π 1 1 π 1
x ∈ ⎡⎢ −π + ; − + ⎤⎥ ∪ ⎡⎢ ; + ⎤⎥ ∪ ⎡⎢ π + ; 5⎤⎥ .
1 1
⎣ 2 2 2⎦ ⎣2 2 2⎦ ⎣ 2 ⎦
Example 3. Find out which of the given functions is even, odd or neither
even, nor odd.
2+ x
а) f ( x) = x sin 2 x; b) f ( x) = ln ; c) f ( x) = x 2 − 3 x + 8 .
2− x
Solution. а) We obtain:
f (− x) = − x sin(−2 x) = ( − x) ( − sin 2 x) = x sin 2 x ; f (− x) = f ( x) ,

hence, the given function is even;

163
b) Substituting − x into the formula b) we get:

2 + (− x) 2− x 2+ x
f (− x) = ln = ln = ln(2 − x) − ln(2 + x) = − ln = − f ( x),
2 − (− x) 2+ x 2− x

i.e. the function is odd;


c) Substituting − x into the formula c) we get:

f (− x) = (− x) 2 − 3(− x) + 8 = x 2 + 3 x + 8.
The analytical expression of the function has changed, i.e. it is neither even,
nor odd.
Example 4. Sketch the graph of the function
y = 2sin ( 3 x + 6 ) + 1 .
Solution. We can write down the given function as
y = 2sin 3 ( x + 2 ) + 1 .

It is easy to find out that the period of this function is equal to .
3
In comparison with the graph of a function y = sin x the graph of the given
function is removed two units to the left along the axis Ox, the amplitude of the
graph of the given function is twice greater and the graph moves one unit
upward along the axis Oy (Fig. 3.8).
As far as the function is periodic the graph can be extended both sides along
the axis Ox.
3

⋅ (3x
2sin ⋅ +6)+1

⋅ (3x
2sin ⋅ +6) 1

⋅ (3x
2sin ⋅)

⋅)
sin(3x

sin(x)

2
x

Fig. 3.8

164
Micromodule 13
CLASS AND HOME ASSIGNMENT
Find a domain of definition of the functions:
3x − 2 x+4
1. y = . 2. y = 3x − 2 − x 2 . 3. y = .
x +1 x −1

2x − 1 x−2
4. y = lg . 5. y = . 6. y = lg(lg( x 2 + 3 x + 3)) .
x+3 lg x

⎛ ⎞
(
7. y = log 1 x 2 − 4 x + 3 . ) 8. y = lg ⎜ log 1 ( log2 ( x −1) ) ⎟ .
2 ⎝ 3 ⎠
9. y = x − x . 10. y = x −x. 11. y = arcsin ( 3 x − 1) .

Evaluate limits.

3x5 + 2 x − 1 5 x3 + 5 x + 3
12. lim . 13. lim .
x →∞ 8 x − 2( x − 1) 4 − 2
5 x →∞ 4 x 2 − 2 x 4 + 3

7 x2 + 4 x + 1 2 x2 − 5x + 2
14. lim . 15. lim .
x →∞ −2 x 6 − x 4 + 1 x→
1 8 x3 − 4 x 2
2

x3 − 5 x 2 + 2 x + 2 5 − x − 2x + 2
16. lim . 17. lim .
x →1 x2 + 5x − 6 x →1 x2 − 4 x + 3
3 2 3 3
7 + x − 1+ 7x x+6 − x+2
18. lim . 19. lim .
x →1 x2 − 1 x→2 x + 14 − 4
20. lim ( x 2 + 2 x − 3 − x) . 21. lim ( 3 x3 + x − 3 x3 − 1) .
x →∞ x →∞

Sketch graphs of the functions:


x−2 1
22. y = . 23. y = x 2 − 4 x . 24. y = x 2 + x + 2 . 25. y = .
x−3 2
x +4
26. y = 4 − 2 x . 27. y = 4 − 2 x . 28. y = 3 − x 2 + 2 x .
−x
29. y = log 2 (1 − x) . 30. y = 3 ⋅ 2 x . 31. y = 10lg sin x .
32. y = 1/ ctg x . 33. y = arcsin x + 2 arccos x . 34. y = cos 3x .

Answers
12. 3/8. 2. ∞ . 13. 0. 14. –1,5. 15. –5/7. 16. 0,125. 17. –5/24. 18. –4/3.
19. ∞ , if x → −∞ ; 2, if x → +∞ . 20. 0.

165
Micromodule 13
SELF-TEST ASSIGNMENTS

13.1. Find the limits of functions

2 x 2 − x3 2 x2 + 9x − 5
13.1.1. а) lim ; b) lim ;
x →∞ 3 x →−5 x 2 + 3 x − 10
8 x9 − 7 x 7 + 5 x
4x − 3 − 2x + 3
c) lim .
x →3 2x2 − 5x − 3

4 x2 − 2 x − x + 1 3x 2 − 7 x + 2
13.1.2. а) lim ; b) lim ;
x →∞ 3x − 5 2
x→ 3x + 2 x − 1
1
3

4 x 2 + 3x − 1
c) lim .
x →−1 5 x + 6 − 3x + 4
3x3 − 2 x 2 + 5 2 x 2 − 3x − 2
13.1.3. а) lim ; b) lim ;
x →∞ 4 + x − 2 x 2 x→2 3x 2 − 7 x + 2

2x + 2 − 1
c) lim .
2
1 4x − 1
x→−
2

3 3
8 x3 + 3 x 2 − 4 x 2 + 7 x x2 + 2x − 3
13.1.4. а) lim ; b) lim ;
x →∞ 3x + 5 x →−3 2 x 2 + 5 x − 3

6 x − 1 − 12 x − 3
c) lim .
x→
1 3x 2 − 4 x + 1
3

12 x − 7 x 2 + x − 12
13.1.5. а) lim ; b) lim ;
x →∞ 4 3 x →3 2 x 2 − 7 x + 3
16 x 4 − 3 x3 + 2 x 2 − 1
4x − 5 − x +1
c) lim .
x→2 2x2 − 5x + 2
3x 2 − 5 x + 2 6 x2 − 5x − 4
13.1.6. а) lim ; b) lim ;
x →∞ 1 + 4 x − 7 x 3 2
x →− 2 x + 3 x + 1
1
2

3x − 4 − x
c) lim .
x→2 2x2 − x − 6

166
7
5x 2 − 3 + 4 x 4 − 2 x 2 + 3 x2 − 6 x + 5
13.1.7. а) lim ; b) lim 2 ;
x→∞ 3x − 2 x →5 2 x − 9 x − 5

3 ( x + 1) − 1
c) lim .
x →−
2 27 x3 + 8
3
3
2 x 4 − 5 x3 + 1 2 x2 + x − 3
13.1.8. а) lim ; b) lim ;
x →∞ 4 3 4x2 − 9
x5 − 3 x 4 + 7 x3 x →−
2

2 x − 3x − 8
c) lim .
x →8 x2 − 9x + 8
5 + x − 3x3 5x2 + 8x + 3
13.1.9. а) lim ; b) lim ;
x →∞ 7 x 2 − x 3 2
x →− 5 x − 7 x − 6
3
5

3x + 1 − 2 x + 3
c) lim .
x→2 x2 + 2 x − 8
2
7x −x+5 x3 − 1
13.1.10. а) lim ; b) lim 2
;
3 2 x →1 5 x − 4 x − 1
x → ∞ 2x − x

3 − 2x + 9
c) lim .
x →0 4 x 2 − 3x
3
x5 − 2 x 2 2 x 2 − 3x − 2
13.1.11. а) lim ; b) lim ;
x →∞ 4
3x7 + 5 x4 − 2 x x →−
1 8 x3 + 1
2

x2 + 3 − 5x − 1
c) lim .
x →1 x2 + x − 2
3
x2 − 3x + 5 − 2 x x3 − 1
13.1.12. а) lim ; b) lim 2
;
x →∞ 3 x →1 5 x − 2 x − 3
x3 + 3x 2
12 x 2 − 1 − 4 x 2 + 1
c) lim .
x→
1 2x −1
2
5
3 x7 − 2 x5 3 x 2 + 11x − 4
13.1.13. а) lim ; b) lim ;
x →∞ 4 x →−4 x 2 + 21x + 68
2 x5 − 4 x 2 + x
3x + 4 − 6 x + 6
c) lim .
x →−
2 9 x2 − 4
3

167
3
x5 − 2 x 2 2 x 2 − 3x − 2
13.1.14. а) lim ; b) lim ;
x →∞ 4
3x7 + 5 x 4 − 2 x x →−
1 8 x3 + 1
2
2
x + 3 − 5x − 1
c) lim .
x →1 x2 + x − 2
3
x2 − 3x + 5 − 2 x x3 − 1
13.1.15. а) lim ; b) lim ;
x →∞ 3 x →1 5 x 2 − 2 x − 3
x3 + 3x 2
12 x 2 − 1 − 4 x 2 + 1
c) lim ;
x→
1 2x −1
2
5
3 x7 − 2 x5 3 x 2 + 11x − 4
13.1.16. а) lim ; b) lim ;
x →∞ 4 x →−4 x 2 + 21x + 68
2 x5 − 4 x 2 + x
3x + 4 − 6 x + 6
c) lim .
x →−
2 9 x2 − 4
3

x2 − 2 x − x + 2 2 x2 − x − 3
13.1.17. а) lim ; b) lim ;
x →∞ 3x − 4 12
x → 8 x − 10 x − 3
3
2

2x + 5 − 4x +1
c) lim .
x→2 x 2 + 3 x − 10
x 4 − 2 x3 + 3 2 x2 + 5x + 3
13.1.18. а) lim ; b) lim ;
x →∞ 5 x 4 + x 2 x →−1 4 x 2 + 3 x − 1

16 x 2 − 1
c) lim .
x →−
1 12 x + 4 − 4 x + 2
4

3
x6 − 3 x + 3 x 4 − x3 2 x2 + 5x + 3
13.1.19. а) lim ; b) lim ;
x →∞ x 2 + 7 x − 11 x →−1 4 x 2 + 3 x − 1

5x + 3 − 3 − 2x
c) lim .
x →0 5x2 − 6 x
3
x 2 − 3x + x 2 + 3x 6 x2 − x − 2
13.1.20. а) lim ; b) lim 2
;
x→ 3x + x − 2
x →∞ 3 2
8 x3 + 3 x 2 + 2 3

10 x − 1 − 5 x + 4
c) lim .
x →1 2 x2 + x − 3

168
3
3 − x3 + 2 x + 5 4 x 2 − 4 x − 15
13.1.21. а) lim ; b) lim 2
;
x →∞ 4 x→ 2 x − 7 x + 5
5
16 x 4 − 7 x3 + x 2 − 4 2

x + 3 − 5x − 1
c) lim .
x →1 x3 − 1
2 x2 − 4 x + 5 2 x2 + 7 x − 4
13.1.22. а) lim ; b) lim ;
x →∞ x →−4 3 x 2 + 10 x − 8
x 4 + 3x3 − x 2 + 3
4 x2 + 3 − 2 x + 3
c) lim .
x→
1 2 x 2 + 3x − 2
2

5x − 4 − 2 x + 3 5x2 − 9 x − 2
13.1.23. а) lim ; b) lim ;
x →∞ 5 x2 − 3x + 5 x→2 x3 − 8
6x + 7 − 2x + 3
c) lim .
x →−1 2 x2 + x −1
4 3
x12 − 4 x5 + x3 + 5 x 2 4 x2 + 4 x − 3
13.1.24. а) lim 3 2
; b) lim 2
;
x − 3x + 7 x →− 2 x + 7 x + 6
x →∞ 3
2

x +1 − 2x − 2
c) lim .
x →3 x3 − 27
3 x3 − 8 x 2 + 7 3x 2 − 8 x − 3
lim 2
lim
x →3 2 x 2 − 9 x + 9
13.1.25. а) x→∞ 20 x − 4 x + 3 ; b) ;
2
16 x − 1
lim
x →−
1 8x + 3 − 1
c) 4 ;
3
8 x3 − 3 x + x 2 + 7 x 5 x 2 − 11x + 2
13.1.26. а) lim ; b) lim ;
x →∞ 3x − 5 2
x → 10 x + 3 x − 1
1
5

3x − 4 − 4 − x
c) lim .
x→2 2 x2 − 3x − 2
5
x 4 − 3 x3 + 5 + x − 1 3x 2 − 8 x + 5
13.1.27. а) lim ; b) lim 2
;
x → 3 x + x − 10
x →∞ 3 5
2 x 2 − 3x 3

3x + 4 − 2 x + 3
c) lim .
x →−1 11x 2 + 10 x − 1

169
x5 − 17 x 2 3x2 − 5 x − 2
13.1.28. а) lim ; b) lim ;
x →∞ x + 2 x 2 + 7
3 2
x →− 3 x + 4 x + 1
1
3

3x + 2 − 6 x
c) lim .
x→
2 3x 2 − 5 x + 2
3

3
x 2 − 7 x + 4 x3 + 2 x 2 2 x2 + 9x + 9
13.1.29. а) lim ; b) lim ;
x →∞ 5 x →−3 x 2 + x − 6
x4 + 5x2 − 1
x2 + 2 − 6 x − 3
c) lim .
x →5 x2 − 4 x − 5
3 x11 − 5 x10 + 7 x3 2 x2 + 5x − 3
13.1.30. а) lim ; b) lim ;
x →∞ 7 x 6 + 12 x10 + 5 x11 2
x→ 6 x − x − 1
1
2

7 x + 2 − 3x + 6
c) lim .
x →1 x2 + 4 x − 5

13.2. Sketch graphs of the functions.


1
x 1 −
13.2.1. а) y = ; b) y = 10lg cos 2 x ; c) y = ; d) y = 2 x .
x−2 2
x + 4x + 6
1
x 1 2
13.2.2. а) y = ; b) y = 6 − 4 x ; c) y = ; d) y = 2 x .
x +1 x2 − 2 x + 3
1
x +1
13.2.3. а) y = ln(e − x) ; b) y = sin x + cos x ; c) y = ; d) y = 2 x .
x −1
x+2
13.2.4. а) y = log 2 x 2 ; b) y = sin 2 x ; ; d) y = 2arcsin x .
c) y =
x +1
x+2
13.2.5. а) y = log3 ( x − 1) ; b) y = tg 2 x ; c) y = ; d) y = 2arc tg x .
x−4
1 x−2
13.2.6. а) y = 2 ; b) y = ctg3 x ; c) y = ; d) y = arccos 2 x .
x +2 x + 1
1 x
13.2.7. а) y = ; b) y = sin(2 x − 1) ; c) y = ; d) y = 2| x| .
x −12 x+3
x −3
13.2.8. а) y = log 2 x 4 ; b) y = 2 cos 2 x ; c) y = ; d) y = 3sin x .
x+2
1− x
13.2.9. а) y = log 2 x + log3 x ; b) y = sin x ; c) y = ; d) y = 5cos x .
x +1
170
x+2 100
13.2.10. а) y = 4 x + 4− x ; b) y = cos x ; c) y = ; d) y = lg
.
2− x x2
x 10
13.2.11. а) y = 3x + 3− x ; b) y = tg x ; c) y = ; d) y = lg .
3− x x
x −1 1
13.2.12. а) y = 2 x − 2− x ; b) y = − sin 2 x ; c) y = ; d) y = lg .
2− x x
1 x −| x |
13.2.13. а) y= 2 ; b) y = sin 3 x ; c) y = ; d) y = 4 .
x −4 2− x
−x 4
13.2.14. а) y = x + 3x ; b) y = − − sin 2 x ; c) y = ; d) y = log 2 .
1− x x
1
1 −
x x2
13.2.15. а) y = ; b) y = 1 − x 2 ; c) y = ; d) y = 2
.
−x + 4 x2 + 3
x 2
13.2.16. а) y = ; b) y = − x 2 + 6 x ; c) y = ; d) y = log x 4 .
3− x sin x
1
13.2.17. а) y = lg( x + 1) ; b) y = arctg 2 x ; c) y = 2 ; d) y = −2− x .
2

x
1
x+2 2
13.2.18. а) y = ; b) y = −x − 2 x ; c) y = 2 cos x ; d) y = log x 9 .
3− x
1
|x|
13.2.19. а) y = ; b) y = − 9 − x 2 ; c) y = 2 sin x ; d) y = 2 x + 5 x .
1+ | x |
|x|
13.2.20. а) y = ; b) y = − 4 − x 2 ; c) y = 2 tg x ; d) y = 2− x + 3− x .
1+ x
−2 x 9
13.2.21. а) y = x + 4 x ; b) y = − cos 2 x ; c) y = ; d) y = log3 .
1+ x x2
1 x −1
13.2.22. а) y = 2 ; b) y = ctg 2 x ; c) y = ; d) y = arcsin 2 x .
x −4 x−4
x x +1 10
13.2.23. а) y = 5 x + 5− x ; b) y = tg ; c) y = ; d) y = lg .
3 2− x x3
x+2 1
13.2.24. а) y = ; b) y = − x 2 − 6 x ; c) y = ; d) y = log x 3 .
−x cos x
2
x2 −1 1 −
13.2.25. а) y = ; b) y = 25 − x 2 ; c) y = ; d) y = 3 x .
1− x arcsin x
x x−2 1
13.2.26. а) y = 6 x − 6− x ; b) y = sin ; c) y = ; d) y = lg 5 .
4 x x

171
5 x | x | −1 1
13.2.27. а) y = ; b) y = ctg ; c) y = ; d) y = arccos .
x +52
2 x +1 x
4− x |1− x|
13.2.28. а) y = log 2 x − log3 x ; b) y = tg 3x ; c) y = ; d) y = 2 .
x +1
1 x +1
13.2.29. а) y = 2 ; b) y = tg 3x ; c) y = ; d) y = arcsin 2− x .
x −9 x+3
x+2 log x
13.2.30. а) y = ; b) y = − x 2 + 2 x ; c) y = 3ctg x ; d) y = 2 2 .
1− x

Micromodule 14
BASIC THEORETICAL INFORMATION
HONORABLE LIMITS

The first honorable limit. Number е. The second honorable limit. Se-
quences.

Literature: [3, chapter 3, §§ 3.1—3.8], [4, part 4, §§ 4.2—4.3], [5], [6,


chapter 4, §§ 3, 4], [10, chapter 3, § 4], [12, chapter 2, §§ 6—8], [13].

14.1. The first honorable limit


sin x ⎛0⎞
A function for x = 0 has an indeterminacy of a kind ⎜ ⎟ .
x ⎝0⎠
sin x
Definition 3.13. The limit lim =1 (3.2)
x→0
x
is called the first honorable limit.
It is widely used for calculation of limits of functions which contain
trigonometric and inverse trigonometric functions.
By means of the limit (3.2) the following limits may be found
tgx arctgx arcsin x
lim =1 , lim =1 , lim =1 .
x →0 x x →0 x x →0 x

14.2. The second honorable limit. Sequences


1
A function (1 + x ) x at the point x = 0 has an indeterminacy 1∞ , but there
exists the limit of this function if x → 0 which is called the second honorable limit
1 ⎛ ⎛ 1⎞
x ⎞
lim (1 + x ) x = e , ⎜ lim ⎜ 1 + ⎟ = e ⎟ . (3.3)
x →0 ⎜ x →∞ ⎝ x⎠ ⎟
⎝ ⎠

172
1 v
The graph of the function y = lim (1 + x ) x is
x →0
in the Fig. 3.9. e
Number e is transcendental number.
l
e ≈ 2,718281828459045.
O x
The second honorable limit is connected
with indeterminate expression ⎡⎣1∞⎤⎦ . ( u ( x)v ( x ) , Fig. 3.9

u ( x) → 1, v( x) → ∞, if x → x0 ).

log a (1 + x )
1) lim = log a e;
x →0 x
ax −1 (1 + x )k − 1
2) lim = ln a; 3) lim = k.
x →0 x x →0 x

Using the limit (3.3), we can find the limits


If а = е then
ln (1 + x ) ax −1 ex −1
lim = 1 , lim = ln a , lim = 1.
x →0 x x →0 x x →0 x

Micromodule 14
EXAMPLES OF PROBLEMS SOLUTION
πx
Example1. Evaluate lim ( x − 4 ) tg .
x →4 8
Solution. At x = 4 we have an indeterminacy [ 0 ⋅ ∞ ] .
Let us put x − 4 = t. For x → 4, t → 0 and we obtain :
πx π(4 + t ) ⎛ π πt ⎞
lim ( x − 4 ) tg = lim ttg = lim t tg ⎜ + ⎟ =
x→4 8 t →0 8 t →0 ⎝ 2 8⎠
πt π πt
t cos t cos
⎛ πt ⎞ 8 8 8 =
= lim t ⎜ − ctg ⎟ = − lim = − lim
t →0 ⎝ 8⎠ t →0 πt t →0 π πt
sin sin
8 8 8
π
t
πt 1 8
= − lim 8 ⋅ lim cos = − ⋅1 = − .
t →0 π sin πt t →0 8 π π
8 8 8

173
sin ⎛⎜ x −
π⎞

Example 2. Evaluate lim ⎝ 3⎠
.
x → π 1 − 2 cos x
3

Solution. We have an indeterminacy ⎡⎢ ⎤⎥ . Let us represent the


0
⎣0⎦
denominator of the fraction as:
π π
+x x−
⎛ 1 ⎞ ⎛ π ⎞ 3 3 .
1 − 2 cos x = 2 ⎜ − cos x ⎟ = 2 ⎜ cos − cos x ⎟ = 2 ⋅ 2sin ⋅ sin
⎝2 ⎠ ⎝ 3 ⎠ 2 2
π π
x− x−
⎛ π⎞ 3 3 .
The numerator will be sin ⎜ x − ⎟ = 2sin cos
⎝ 3⎠ 2 2
π π π
π x− x− x−
sin ⎛⎜ x − ⎞⎟ 2sin 3 cos 3 cos 3
⎝ 3⎠ 2 2 = lim 2 = 1 .
Then lim = lim
x → 1 − 2 cos x π π x→ π π
π π 3
3
x→
3 x− x+ 3 x+
4sin 3 sin 3 2sin 3
2 2 2
x −4
3x − 2 ⎞
Example 3. Evaluate lim ⎛⎜ ⎟ .
x →∞ ⎝ 3 x + 1 ⎠

Solution. We have an indeterminacy ⎡⎣1∞ ⎤⎦ . To open it we shall take the


1
second honorable limit (3.3), i.e. we shall represent this limit as (1 + α ( x ) ) α ( x ) ,
α ( x ) → 0 . Then
3( x − 4 )
3 x +1 − 3 x +1

)

( ) ( ) (
x−4 x −4 −
3x − 2 3x + 1 − 3 3 3
= = ⎢ 1− ⎥ .
3x + 1 3x + 1 ⎢⎣ 3x + 1 ⎥⎦

− 3 x +1
( )
x −4 lim − 3( x − 4 )
3x − 2
As lim ⎛⎜1 −
3 ⎞ 3 1
= e, then lim = e →∞ 3 x +1
x = e −1 = .
x →∞ ⎝ 3 x + 1 ⎟⎠ x→∞ 3x + 1 e
2
3x + 6 ⎞ x +1
Example 4. Evaluate lim ⎛⎜ ⎟ .
x →−1 ⎝ x + 4 ⎠

Solution. For x = −1 we have an indeterminacy 1 . We can take x + 1 = t , [ ]
then x = t − 1, and if x → −1, t → 0 , we obtain:

174
( ) ( ) ( )
2 2 2 2 2
3x + 6 3t − 3 + 6 3 + 3t 3 + t + 2t ⎞ t ⎛
= ⎛⎜
x +1 t t 2t ⎞ t
= = = 1+ .
x+4 t −1+ 4 3+t ⎝ 3 + t ⎟⎠ ⎜⎝ 3 + t ⎟⎠
2 4
lim 4
3x + 6 ⎞ x +1 t →0 3+t
Then lim ⎛⎜ ⎟ =e = e 3 = 3 e4 .
x →−1 ⎝ x + 4 ⎠

Example 5. Evaluate lim u v .


u →1
v →∞

1
Solution. We shall denote u = 1 + for u → 1 y → ∞ . Then
y
v
lim lim v ( u −1)
v v →∞ y v →∞
⎡ v y ⎤ vlim
→∞ y
⎛ 1⎞ ⎛ 1⎞ y →∞ u →1
lim u v =. lim ⎜ 1 + ⎟ = ⎢ lim ⎜ 1 + ⎟ ⎥ y →∞ = e =e .
u →1 v →∞ ⎝ y⎠ y →∞ ⎝ y⎠ ⎥
v →∞ y →∞ ⎣⎢ ⎦

Thus, considering the indeterminacy such as ⎡⎣1∞ ⎤⎦ it is possible to use the


lim v ( u −1)
v →∞
u →1
formula lim u v = e . (3.4).
u →1
v →∞
1

Example 5. Evaluate lim ⎛⎜


cos x ⎞ x2
⎟ .
x →0 ⎝ cos 2 x ⎠

Solution. We have an indeterminacy ⎡⎣1∞ ⎤⎦ . In this case we can use the


formula (3.4).
We find:
3x x
⎛ cos x − cos 2 x ⎞ 2sin sin
⎛ cos x ⎞ 1 2 2
lim ⎜ − 1⎟ ⋅ 2 = lim ⎜ ⎟ = lim =
x →0 ⎝ cos 2 x ⎠ x x →0 ⎝ x 2 cos 2 x ⎠ x →0 x 2 cos 2 x
3x x
3 sin sin
= lim 2 ⋅ lim 2 ⋅ lim 1 = 3 .
x →0 3x x →0 x x → 0 cos 2 x 2
2
2 2

( )
lim ⎛⎜ −1⎞⎟
1 cos x 1 3
cos x 2 ⎝ cos 2 x ⎠ x 2
Thus, lim x = e x → 0 = e 2 = e3 .
x →0 cos 2x

175
Micromodule 14
CLASS AND HOME ASSIGNMENT

Evaluate limits.
πx
cos
sin 8 x + sin 6 x sin 2 x 6 .
1. lim . 2. lim . 3. lim
x →0 sin 9 x − sin 5 x x →0 tg 7 x x →3 3 − x

πx
1 − 2 cos 3x+2
πx 5x − 1 ⎞
4. lim 6 . 5. lim(9 − x2 ) tg . 6. lim ⎛⎜ ⎟ .
x→2 x−2 x→3 2 x →∞ ⎝ 5 x + 1 ⎠
sec x
ln(cos x)
8. lim ⎛⎜ tg ⎞⎟
x
7. lim (1 − tg 3 x )
ctg 2 x
. . 9. lim .
x →0 x→
π⎝ 2 ⎠ x →0 ln(cos 2 x )
2

32 x − 3x
10. lim ( 2 x + 3) ⎡⎣ ln ( 4 x − 5 ) − ln ( 4 x − 6 ) ⎤⎦ . 11. lim .
x →∞ x→0 5tg x − 1

ln ( 5 x − 19 ) (1 + 2 x)7 − 2 x
12. lim . 13. lim .
x→4 x2 − 6 x + 8 x →0 x
(1 + x)8 − (1 − x) −8
14. lim . 15. lim( x 2 − 1) log 2.
x →0 ln(1 + x) x →1 x2

Answers

π π 3 12
1. 3,5. 2. 2/7. 3. − . 4. . 5. . 6. e−6 / 5 . 7. e−3/ 2 . 8. e−1 . 9. 0,25.
3 6 π
10. 0,5. 11. log5 3 . 12. 2,5. 13. 14 − ln 2 . 14. 0. 15. ln 2 .

Micromodule 14
SELF–TEST ASSIGNMENTS

14.1. Evaluate limits of functions using the first honorable limit:


cos 2 x − cos8 x πx
14.1.1. а) lim ; b) lim ( 5 − x ) tg .
x →0 x tg x x →5 10
sin 3x + sin 7 x cos 2 x + 1
14.1.2. а) lim ; b) lim .
x →0 sin 3x − sin 5 x x→
π 1 − sin x
2

sin 3 x − sin x 1
14.1.3. а) lim ; b) lim x tg .
x →0 sin 5 x x →∞ x

176
sin 2 3x 1 + x sin 2 x − cos 2 x
14.1.4. а) lim ; b) lim .
x →0 tg 6 x 2 x →0 x2
x 2 − π2 1 − cos 4 x
14.1.5. а) lim ; b) lim .
x →π tg 5 x x →0 2 x sin 5 x
cos 2 x − cos3 2 x sin 2 x
14.1.6. а) lim ; b) lim .
x →0 sin 2 8 x x →π sin 7 x

2− x 2 − 3cos 2 x + cos 2 2 x
14.1.7. а) lim ; b) lim .
x → 2 sin 3πx x →0 x2
cos 3 x + 1 tg 3x − tg x
14.1.8. а) lim ; b) lim .
x →π tg 2 x x →0 tg 2 x
lim ( x + 2) ctg 3πx cos 2 4 x − 5cos 4 x + 4
14.1.9. а) x →−2 ; b) lim .
x →0 x sin x
2x + 1 sin 8 x − sin 3x
14.1.10. а) lim ; b) lim .
x →−
1 cos 3πx x →0 sin 5 x + sin 2 x
2

tg 2 3x
14.1.11. а) lim ; b) lim (2 x − π) tg x .
x →0 2 x sin x x→
π
2

arctg 5 x tg 2 4 x − tg 2 2 x
14.1.12. а) lim ; b) lim .
x →0 sin 3 x x →0 x2
2 cos 2 x − 1 tg 2 6 x − tg 2 3 x
14.1.13. а) lim ; b) lim .
x→
π sin 4 x x →0 sin x 2
4

cos x − cos 3 x 1
14.1.14. а) lim ; b) lim x 2 sin .
x →0 sin 2 4 x x →∞ x +12

arcsin 2 2 x sin 2 x − 3sin x + 2


14.1.15. а) lim ; b) lim .
x →0 tg 2 3 x x→
π 2x − π
2

sin 2 x
14.1.16. а) lim( x − 3) ctg πx ; b) lim .
x →3 x →0 π
sin( − arccos x)
2
1 − cos 2 x sin 2 x + tg 3 x
14.1.17. а) lim ; b) lim .
x →π 1 + cos 3 x x →0 sin x − arcsin 2 x

tg 3 x − sin 3 x 1
14.1.18. а) lim ; b) lim x 2 tg .
x →0 sin 3 x x →∞ 2
x −4
177
tg 2 2 x 1
14.1.19. а) lim ; b) lim ( x + 3) sin .
x →π cos 5 x + 1 x →∞ x−2
sin πx cos 2 x − 6 cos x + 5
14.1.20. а) lim ; b) lim .
x →1 sin 2πx x →0 x2
cos 5πx + 1 π
14.1.21. а) lim ; b) lim ctg5 xctg ⎛⎜ − x ⎞⎟ .
x →3 cos 4πx − 1 x →0 ⎝ 2 ⎠
5x 1 − cos 3 x
14.1.22. а) lim ( x − π) tg ; b) lim .
x →π 2 x →0 xtg3 x

cos 5 x − cos 4 x 1 + cos 5 x


14.1.23. а) lim ; b) lim .
x →0 tg 2 3x x →π − cos 6 x
1

x−π ⎛ 1 1 ⎞
14.1.24. г) lim ; b) lim ⎜ − ⎟.
x →π tg 5 x x →0 ⎝ sin x tgx ⎠

sin 7 x − sin 3 x x − 3 πx
14.1.25. а) lim ; b) lim sin tg .
x →0 sin 4 x x →3 2 6
2− x sin 5 x − sin 3x
14.1.26. а) lim ; b) lim .
x→2 πx x →π sin 2 x
cos
4
sin 2 5 x cos x − sin x
14.1.27. а) lim ; b) lim .
x →π tg 2 3 x x→
π cos 2 x
4

1 + cos 3 x cos 2 x − 7 cos x + 6


14.1.28. а) lim ; b) lim .
x →π sin 2 x x →0 tg 2 2 x
1 − cos 4 x 1
14.1.29. а) lim ; b) lim (2 x + 1) sin .
x →π tg 2 3x x →∞ 3x − 1

π+ x
14.1.30. а) lim ; b) lim ( x − 1)ctgπx .
x →−π tg 2 x x →1

14.2. Evaluate limits of functions using the second honorable limit and its
sequences:
3x − 1 ⎞
x+2
ln ( cos 2 x )
14.2.1. а) lim ⎛⎜ ⎟ ; b) lim .
x →∞ ⎝ 3 x + 1 ⎠ x →0 ln ( cos 4 x )

2
x2 − 4 e3 x − e x
14.2.2. а) lim ( 2 x + 5 ) ; b) lim .
x →−2 x →0 lncos x

178
7 − x2
ln ( 4 x − 3) ⎛ 5 − 2 x2 ⎞
14.2.3. а) lim ; b) lim ⎜ ⎟ .
x →1 x −1 x →∞ ⎝ 3 − 2 x 2 ⎠

14.2.4. а) lim ⎛⎜
x + 5 ⎞ x+4
;
3

b) lim
( ).
ln 1 + 3x

x →−4 ⎝ 2 x + 9 ⎠ x→−∞ ln (1 + 2 ) x

2 x 2 −1
x+3 ⎞
14.2.5. а) lim ( 2x −1) ⎣⎡ln ( x − 2) − ln ( x +1)⎦⎤ ; b) lim ⎛⎜
x
⎟ .
x→∞ x →∞ ⎝ 2 x − 1 ⎠
1
x −1
2x + 5 ⎞
b) lim ⎛⎜
sin x ⎞ x − 2
14.2.6. а) lim ⎛⎜ ⎟ ; ⎟ .
x →∞ ⎝ 2 x + 3 ⎠ x → 2 ⎝ sin 2 ⎠
5
e x +3 − 1 3x + 7 ⎞ x +1
14.2.7. а) lim ; b) lim ⎛⎜ ⎟ .
2
x →−3 x + 2 x − 3 x →−1 ⎝ 2 x + 6 ⎠
3
x+2 5x − 4 x
14.2.8. а) lim ( 2 x + 5 ) ; b) lim .
x →−2 x →0 sin 3 x
2
3 x − 2 ⎞ x2 −1
14.2.9. а) lim ⎛⎜ ⎟ ; b) lim( x sin x log e) .
x →1 ⎝ 2 − x ⎠ x →0 1− x 2

3x − 8 x
14.2.10. а) lim ( x − 2) ⎡⎣ln ( 2x −1) − ln ( 2x + 3)⎤⎦ ; b) lim .
x→∞ x →0 tg 2 x
4
ln ( x − 4 ) x2 − 4
14.2.11. а) lim ; b) lim ( 3x − 5 ) .
x →5 x2 − 4 x − 5 x→2

14.2.12. а) lim ⎛⎜
x + 7 ⎞ x+2
;
3

b) lim
( ).
ln 1 + 5 x

x →−2 ⎝ 2 x + 9 ⎠ x→−∞ ln (1 + 3 )x

2
x 2 −1 1
14.2.13. а) lim ( 3 x − 2 ) ; b) lim .
x →1 x →1 ( x 2 − 1) log x 2
2 x +3
e5 x + 2 − 1 3x + 5 ⎞
14.2.14. а) lim ; b) lim ⎛⎜ ⎟ .
2
x →− 5 x + 7 x + 2
2 x →∞ ⎝ 3 x − 1 ⎠
5
2
x + 3 ⎞ x2 − 4
14.2.15. а) lim ⎛⎜ ⎟ ; b) lim log cos x (1 + x 2 ) .
x→2 ⎝ 2 x + 1 ⎠ x →0

179
3x +5
2x + 3 ⎞
14.2.16. а) lim ⎛⎜ ⎟ ; b) lim log cos 2 x (1 − x 2 ) .
x →∞ ⎝ 2 x + 1 ⎠ x →0

6 x+2
4x − 5 ⎞
14.2.17. а) lim ( 3 x − 2 ) ⎡⎣ ln ( 2 x + 1) − ln ( 2 x + 5 ) ⎤⎦ ; b) lim ⎛⎜ ⎟ .
x →∞ x →∞ ⎝ 4 x + 1 ⎠

2
x2 −9 ln ( 4 x − 23)
14.2.18. а) lim ( 2 x − 5 ) ; b) lim .
x →3 x →6 x 2 − 8 x + 12
4
x + 5 ⎞ x+3 5 x − 2− x
14.2.19. а) lim ⎛⎜ ⎟ ; b) lim .
x →−3 ⎝ 2 x + 8 ⎠ x →0 sin 4 x

3 2
x −7 4 x −1 − 1
14.2.20. а) lim ( 2 x − 13) ; b) lim 2 .
x →7 x →1 x + 2 x − 3

3
2x + 7 ⎞ x+2
14.2.21. а) lim ( x − 4 ) ⎡⎣ ln ( 3 x − 1) − ln ( 3 x − 2 ) ⎤⎦ ; b) lim ⎛⎜ ⎟ .
x →∞ x →−2 ⎝ 3 x + 9 ⎠

2 x +3
3x + 2 ⎞
14.2.22. а) lim ⎛⎜ ⎟ ; b) lim log (1 − x 2 ) .
x →∞ ⎝ 3 x − 1 ⎠ x →0 1− x 4

ln ( 5 x − 14 ) 6x + 5 ⎞
4 x +3
14.2.23. а) lim ; b) lim ⎛⎜ ⎟ .
x →3 x2 − 9 x →∞ ⎝ 6 x − 1 ⎠

14.2.24. а) lim ⎛⎜
3 x + 2 ⎞ x 2 −1
5

; b) lim
( ).
ln 1 + 7 x

x →1 ⎝ x + 4 ⎠ x→−∞ ln (1 + 5 ) x

36 x − 3−2 x
14.2.25. а) lim ( x − 7 ) ⎡⎣ln ( 4 x + 3) − ln ( 4 x + 1) ⎤⎦ ; b) lim .
x →∞ x →0 sin x
3x
25 x − 2 − x 2 x − 3 ⎞ 2 x2 +3
14.2.26. а) lim ; b) lim ⎛⎜ ⎟ .
x →0 3x x →∞ ⎝ 2 x − 1 ⎠

x +1
ln ( 3 x − 5 ) ⎛ 2 x +1⎞ 3
14.2.27. а) lim ; b) lim ⎜ ⎟ .
x→2 x2 − 4 x →∞ ⎝ 2 x − 1 ⎠

5 x3
e 4 x −3 − 1 5 x + 2 ⎞ x 2 −1
14.2.28. а) lim 2 ; b) lim ⎛⎜ ⎟ .
x→ 4 x − 7 x + 3
3 x →∞ ⎝ 5 x + 4 ⎠
4

180
x +3
3x + 1 ⎞
14.2.29. а) lim ⎛⎜
2
⎟ ; b) lim log cos3 x cos 2 x .
x →∞ ⎝ 3 x − 1 ⎠ x →0
3
x2 − 4 ln ( 5 x − 19 )
14.2.30. а) lim ( 3 x + 7 ) ; b) lim .
x →−2 x→4 x2 − 5x + 4

Micromodule 15
BASIC THEORETICAL INFORMATION. COMPARISON
OF INFINITESIMALS. EQUIVALENT INFINITESIMALS.
THEIR APPLICATION IN CALCULATION OF LIMITS

Literature: [3, chapter 3, §§3.1—3.8], [4, section 4, §§4.2—4.3], [6, chap-


ter 4, § 4], [11, chapter 2, § 11], [12].

15.1. Comparison of infinitesimals. Equivalent infinitesimals


Definition 3.14. If the ratio of two infinitesimals α and β has a finite limit
α
(if x → x0 ) which is different from zero: lim = q ≠ 0 , then α and β are
x → x0 β
said to be infinitesimals of the same order if x → x0 .
We denote it as α( x) = O(β( x)) if x → x0 .
Definition 3.15. Two infinitesimals α and β are said to be equivalent if
α
lim β = 1 . Then we write down: α ~ β .
x → x0

Definition 3.16. If the ratio of two infinitesimals has a limit equal to the
α β
number 0: lim = 0 i.e. lim = ∞ the infinitesimal α is said to be one of
x → x0 β x → x0 α

higher order than β , and the infinitesimal β is said to be one of lower order
than α.
We denote it as α( x) = o(β( x)) if x → x0 .
Definition 3.17. An infinitesimal α is said to be an infinitesimal of the k-th
order relative to an infinitesimal β if α and βk are the infinitesimals of the
α
same order(if x → x0 ), i.e. lim = А ≠ 0.
x → x0 βk

181
For example, 1 − cos x is the infinitesimal of the second order relative to x
x x
2sin 2 sin 2
1 − cos x 2 1 2 1
because lim = lim = lim = , x3 and x are
x →0 x2 x →0 x2 2 x →0 ⎛ x ⎞ 2 2
⎜2⎟
⎝ ⎠
infinitesimals of the third and 0.5 orders relative to x correspondingly.
The remark. The comparison of infinitely large magnitudes is performed
similarly to comparison of infinitesimals. Slight difference exists in terms so, if
u v
lim = 0 i.e. lim = ∞, where magnitudes u and v are infinitely large
v u
magnitudes we say that u is of lower order than v, and v is of higher order than u .
In practical of calculation of limits two theorems about equivalent infini-
tesimals and infinitely large magnitudes are widely applied.

Theorem 3.8 The limit of a ratio of two infinitesimals will not change by
substitution of these infinitesimals by their equivalents.
The similar statement is true for infinitely large magnitudes.

Theorem 3.9 The sum of infinitesimals of different orders is equivalent to its


term of the lowest order.

The sum of infinitely large magnitudes of different orders is equivalent to its


term of the highest order.
The most important equivalences which follow from formulas (3.2) and (3.3) are:

x ∼ sin x ∼ tg x ∼ arcsin x ∼ arctg x ∼ e x − 1 ∼ ln (1 + x ) ,


1 (3.5)
1 − cos x ∼ x 2 , a x − 1 ∼ x ln a, (1 + x ) − 1 ∼ kx.
k
2

Micromodule 15
EXAMPLES OF PROBLEMS SOLUTION
3
ln cos 4 x +e 5 x − 1−2 x 4
Example 1. Evaluate lim .
x →0
arctg 2 3 x −73 sin 8 x

Solution. We have an indeterminacy ⎡ ⎤ .


0
⎣⎢ 0 ⎦⎥
Using Theorems 3.8 and 3.9, and equivalences (3.5) we get:
( )
lncos 4 x = ln (1 + cos 4 x − 1) = ln 1 − 2 sin 2 2 x ~ − 2 sin 2 2 x ~ − 8x 2 ,

182
8
3
e5 x − 1 ~ 5x3 ; arctg 2 3x ~ 9x ,
2 3
sin 8 x ~ x 3 ,
3
ln cos 4 x + e5 x − 1 − 2 x 4 −8 x 2 + 5 x3 − 2 x 4 −8 x 2 8
i.e. lim = lim = lim =− .
x →0
arctg 2 3 x − 7 3 sin 8 x x →0 8 x →0 9 x 2 9
9 x2 − 7 x 3
4
3n3 − 4n 2 + 3 5n 2 − 8n3
Example 2. Evaluate lim .
n →∞ 5
9n 2 + 7 n − n 4 + 6 n 3

Solution. We have an indeterminacy ⎛⎜ ⎞⎟ .
⎝∞⎠
Let us open it by means of Theorems 3.8 and 3.9.
3 4
4
As 3n − 4n ~3 2 4
3n 4 ; 3 5n2 3
− 8n ~ − 2n; 2
9n + 7n ~ 3n; 5 4
n + 6n ~3 5
n ,
then
3
4
4 3 2 3 2 3 4
3n − 4n + 5n − 8n 3n − 2n 2
lim = lim 4
=− .
n→∞
9n 2 + 7 n − 5 n 4 + 6n 3 n→∞ 3
3n − n 5
Example 3. Compare infinitesimals α ( x) = x 2 + 3 x + 2 and β( x) = tg( x + 1)
if x → −1.
Solution. Let us evaluate limit of quotient of the given functions:

x2 + 5x + 4 ⎛ 0 ⎞ x2 + 5x + 4
lim = ⎜ ⎟ = lim =
x →−1 tg( x + 1) ⎝ 0 ⎠ x →−1 x +1
( x + 1)( x + 4)
= lim = lim ( x + 4) = 3 ,
x →−1 x +1 x →−1

that is, the functions α( x) і β( x) if x → −1 are infinitesimals of the same order.


Example 4. Compare infinitesimals α( x) = x arctg x −2 and β( x) = ( x − 3) −1/ 3
if x → ∞ .
Solution. Let us evaluate limit of quotient of the given functions:

x arctg x −2 x arctg x −2
lim = |х–3~х| = lim =
x →∞ ( x − 3) −1/ 3 x →∞ x −1/ 3
arctg x −2 arctg t t
= lim −4 / 3
= x −2 = t = lim 2/3
= lim = lim t1/ 3 = 0 ,
x →∞ x t →0 t t →0 t 2 / 3 t →0

that is, α( x) = O(β( x)) if x → ∞ .

183
Micromodule 15
CLASS AND HOME ASSIGNMENT
Evaluate limits using equivalents.
ln cos x + e x − cos x − x 2 ln(10 − x 2 )
1. lim . 2. lim .
x →0 arcsin2x +arctg 2 3 x − x 4 x →3 sin 2π x

cos x ⋅ (e3 x − 1) x2 − 4 x + 1 − 1
3. lim . 4. lim .
x →0 4
1+ x −1 x →0 tg π x

Compare infinitesimals.
5. α ( x ) = x 2 + 6 x + 8 and β( x) = tg( x + 2) if x → −2 .
arcsin 2 ( x − 3)
6. α( x) = and β( x) = x3 − 7 x − 6 if x → 3.
x +1
7. α ( x) = x 4 − 5 x 2 + 4 and β( x) = x 2 − 8 x + 7 if x → 1 .
8. α( x) = 3 − 3 x + 20 and β( x) = 3 2 x + 13 − 3 if x → 7 .
9. α( x) = x 2 − 1 and β( x) = 2 ln x if x → 1 .
10. α( x) = 3tg x − 3− tg x ; β( x) = sin 2 x if x → 0 .

Answers
3 2
1. –0,5. 2. − . 3. 12. 4. − . 5. α ( x) = О (β( x)) . 6. α ( x) = о (β( x)) .
π π
7. α ( x ) ~ β( x) . 8. α ( x) = О (β( x)) . 9. α ( x ) ~ β( x) . 10. β( x) =о( α ( x ) ).

Micromodule 15
SELF–TEST ASSIGNMENTS

15.1. Evaluate limits using equivalents.


1 − cos π x (1 + 2 x)5 − 1
15.1.1. a) lim ; b) lim .
x →0 ln(1 − x ) tg π x x →0 e3 x − 1
tg 3 2 x − sin x 4 x + ln(1 + x )
15.1.2. a) lim 3
; b) lim .
x →0 x x →0 sin x
ln(1 + sin 3 x ) 2 x3 − 2 x + 3x
15.1.3. a) lim ; b) lim .
x →0 tg(2 3 x ) x →0 ex −1

184
1 − cos (1 − cos x ) x2 + e x
−1
15.1.4. a) lim 2
; b) lim .
x →0 arcsin x x →0 x
x +1
2 − 1 + cos x 2 −1
15.1.5. a) lim 2
; b) lim .
x →0 sin x x →−1 arcsin 3 x +1

arcsin 5 x − arcsin 3 x ( 2 + x )3 − 8
15.1.6. a) lim ; b) lim .
arctg2 x + arctgx ln (1 + x )
x →0 x →0 6

x + tg x − 2 sin x e x − e −2 x
15.1.7. a) lim ; b) lim .
x →0 x + x 2 + tg x x →0 tg x

1 + arcsin x − 1 − sin x e x − e + ln(2 − x 2 )


15.1.8. a) lim ; b) lim .
x →0 tg x x →1 2 x 2 − 3x + 1
ln(e x + x sin x) sin(tg5 x − tgx)
15.1.9. a) lim ; b) lim
x →0 ln(1 + tgx) x →0 arcsin tg2 x

sin 2 x − x arcsin(tg x − x)
15.1.10. a) lim ; b) lim .
x →0 tg x + x x →0 ln(1 + sin x)

1 + x sin x − 1 e x − 2− x
15.1.11. a) lim ; b) lim .
x →0 2 x x →0 sin x
tg
2
sin ( tg5 x − tgx ) 2 x
− cos x
15.1.12. a) lim ; b) lim .
x →0 arcsin tg2 x x →0 sin x
sin x + sin 2 x − sin 3x 2 x −1 − 1
15.1.13. a) lim ; b) lim .
x →0 ⎛ e3 x2 − 1⎞ ln 1 + x x →1 x 2 − 3x + 2
⎜ ⎟ ( )
⎝ ⎠
sin 2 ( x − 1) sin 2 x + sin 2 2 x − sin 2 3 x
15.1.14. a) lim ; b) lim .
x →1 x 2 − 3x + 2 x →0 (e− x − 1) ln (1 + sin x )

3x2 − 5 x 1 − cos10 x
15.1.15. a) lim ; b) lim 2
.
x →0 sin 3 x x →0
ex −1
tg3 x + tgx − sin 2 x 3x +1 − 3
15.1.16. a) lim ; b) lim .
x →0 sin x − sin 5 x x →0
(
ln 1 + x 1 + xe x )
185
15.1.17. a) lim
tg x + ln 1 + 3 x 2( ); b) lim
35 x −3 − 32 x
.
2

x →0
sin 3x + 23 x − 1
2 x →1 tg πx

4 x +1 − 4 ln(1 + 3x) − 2 sin 2 x


15.1.18. a) lim ; b) lim .
x →0
(
ln 1 − x 1 + x 2 x ) x →0 arctg2 x

2 sin 2 x − tg 4 x ln ( 5 x − 19 )
15.1.19. a) lim ; b) lim .
x →0 x2 x →4 x2 − 5x + 4
e − 1 + x3

15.1.20. a) lim
1+ x −1
; b) lim
(
ln 1 + x 2 ).
x →0 sin π ( x + 2 ) x →0
1 − x2 + 1
2
3tgx − 2 x 2 + x 4 2cos x − 1
15.1.21. a) lim ; b) lim .
x →0 arcsin 6 x x→
π ln sin x
2

1 − cos 2 x 2(e πx − 1)
15.1.22. a) lim ; b) lim .
x →0 cos 7 x − cos 3 x x →0 3( 3 1 + x − 1)
2 x3 − x 2 x2 − 9
15.1.23. a) lim . b) lim .
x →0 ln cos x x →3 esin π x −1
1 − cos 4 x 3x +1 − 3
15.1.24. a) lim ; b) lim .
x →0 cos 5 x − cos 4 x x →0
ln(1 + x 1 + xe x )
ln(e x + x 2 ) x 2 (e x − e − x )
15.1.25. a) lim ; b) lim 3
.
x →0 arcsin 2 x x →0
ex −1
1 + x sin x − cos 2 x 2x + 4 x
−2
15.1.26. a) lim 2
; b) lim .
x →0 sin x x →0 arctg x

15.1.27. a) lim
(
ln 9 − 2 x 2 ); b) lim
2(e πx − 1)
.
x→2 sin 2πx x →0 3 8+ x −2
x 2 − 3x + 3 − 1 ln(e x + x sin x)
15.1.28. a) lim ; b) lim .
x →1 sin πx x →0 ln(1 + sin x )

15.1.29. a) lim
(
cos x + 5π
2 ) tg x ; b) lim
2x − 1
.
x →0 arcsin 2 x x →0 ln (1 + 2 x )

186
(1 + 2 x)6 − (1 + x)7 e πx − 1
15.1.30. a) lim ; b) lim .
x →0 sin 2 x x →0 ln (1 + arcsin x )

15.2. Compare infinitesimals α( x) and β( x) .


15.2.1. α( x) = x − x 2 − x , β( x) = x + x3 − 3 ⋅ 3 x if x → 0 .
15.2.2. α( x) = x3 − 3 x + 2 , β( x) = x − 2 if x → 2.
1 1
15.2.3. α( x) = , β( x) = if x → +∞ .
2 3
x + 3x x − 2x
2
15.2.4. α ( x) = x − 2 , β( x) = x − 16 if x → 4 .
3
15.2.5. α( x) = x + x − 2 , β( x) = x − 1 if x → 1.
15.2.6. α ( x) = 1 − 2 cos x , β( x) = 4 x − π if x → π / 4.
3 2
15.2.7. α( x) = x + 2 x − 12 , β( x) = x − 4 if x → 2.
15.2.8. α ( x) = ctg x , β( x) = π − 2 x if x → π / 2.
15.2.9. α ( x) = sin( x − 2) , β( x ) = x − 4 if x → 4 .
3
15.2.10. α( x) = tg( x + x) , β( x) = x if x → 0 .
3 −1 10 −1/ 3
15.2.11. α( x) = ( x + 2) , β( x ) = ( x − 15) if x → ∞.
15.2.12. α( x) = sin( x − x ) , β( x) = 2 x if x → 0 .
15.2.13. α( x) = arcsin(2 − x ) , β( x) = 4 − x if x → 4 .
x 2
15.2.14. α ( x) = e − 1 , β( x ) = x if x → 0 .
3
15.2.15. α ( x) = ln(1 + x ) , β( x) = 1 + x − 1 if x → 0 .
3
15.2.16. α( x) = x − 3 , β( x) = 27 − x if x → 27.
x −2 x−4
15.2.17. α( x) = , β( x) = if x → 4 .
x +2 x+4
15.2.18. α( x) = x sin x , β( x) = x3 if x → 0 .
15.2.19. α( x) = x − sin x , β( x) = x if x → 0 .
15.2.20. α( x) = x sin(2 / x) , β( x) = x if x → 0 .
15.2.21. α( x) = x 2 cos(1 / x ) , β( x) = x if x → 0 .
arctg x 1
15.2.22. α( x) = , β( x) = if x → ∞ .
x x−2
π
15.2.23. α( x) = arctg x − , β( x) = 1 / x if x → +∞ .
2

187
15.2.24. α ( x) = x 2 sin(1 / x) , β( x ) = x − x if x → 0 .
15.2.25. α( x) = arcctg(1/ x) , β( x) = x if x → 0 .
x3 − 8 x2 − 4
15.2.26. α ( x) = , β( x) = if x → 2 .
x x+2
arcsin( x − 2)
15.2.27. α( x) = , β( x) = x3 − 8 if x → 2 .
x
15.2.28. α ( x) = x + 2 x 2 − 3 x , β( x) = 1 − 1 − x if x → 0 .
2 2
15.2.29. α( x) = x + 5 x + 4 , β( x) = x + 3 x + 2 if x → −1 .
2
15.2.30. α( x) = 1 − 2 cos x , β( x) = (3 x − π) if x → π / 3 .

Micromodule 16
BASIC THEORETICAL INFORMATION
CONTINUITY OF A FUNCTION
Continuity of a function at a point. Classification of points of discontinuity.
Literature: [3, chapter 2, п.п. 2.1—2.5], [4, section 4, п. 4.3], [6, chapter
4, § 5], [7, chapter 4, § 13], [10, chapter 3, § 5], [11, chapter 3, § 4], [12,
chapter 1, §§ 6—8, chapter 2, §§ 9—10], [13].

16.1. Continuity of a function at a point


Definition 3.18. A function f ( x ) is said to be continuous at point x = c if
it satisfies three conditions:
1) the function f ( x ) is determined at this point and in its neighbourhood;
2) there exists a limit of the function if x → c;
3) this limit is equal to the value of the function at the point c , i.e.
lim f ( x ) = f ( c ) (3.6)
x →c
If the condition (1) is performed, the conditions
у (2) and (3) are equivalent to such an expression as:
f(x) y = f(x) lim Δy = 0 , (3.7)
Δx → 0
∆y
f(x0) where Δy = f ( x 0 + Δx )− f ( x 0 ) .
∆x
Functions continuity at a point possesses the
О х0 х х following properties:
1) a sum, a difference, a product of a finite
Fig. 3.10 number of functions, continuous at a point c , is a
function, continuous at this point;

188
f ( x)
2) The ratio of two functions , continuous at a point c , is a function,
ϕ( x)
continuous at this point provided ϕ ( c ) ≠ 0 ;
3) a composite function formed from a finite number of continuous functions
is continuous;
4) a function inverse to a monotone continuous function is continuous.
The basic elementary functions are continuous at all points where they are
determined. So from properties of continuous functions the next important
conclusion follows: any elementary function is continuous at each point where it
is determined.
If one of conditions (1—3) of continuity of a function is not carried out at a
point c then the point c is said to be a point of discontinuity of the function
y = f ( x).

16.2. Classification of points of discontinuity

We have to distinguish three cases:


i) the limit of a function at point c exists, but the function is not determined
at this point or lim f ( x ) ≠ f ( c ) then c is said to be a point of removable
x →c
discontinuity;
ii) there exist finite limits of the function from the left and from the right but
f ( c − 0 ) ≠ f ( c + 0 ) then this point c is said to be a point of essential discon-
tinuity or of the first kind;
iii) there do not exist finite limits of the function from the left or from the
right at the point x = c (in particular, c may be equal to infinity) then c is said
to be a point of infinite discontinuity or of the second kind.

Micromodule 16
EXAMPLES OF PROBLEMS SOLUTION

Example 1. Investigate the function for continuity:


x+2
f (x ) = 2
.
3x + 5 x − 2
Solution. The function is determined on all the numerical axis, except the
points where a denominator is equal to zero, i.e.

3x 2 + 5 x − 2 = 0 .

189
Solving this equation we obtain:

− 5 ± 25 + 24 −5 ± 7
x= = ,
6 6
1
i.e. x1 = −2 x 2 = . They are points where the function is not determined i.e. it
3
has discontinuities. We have to find the limits of the function at these points:

x+2 x+2 1
lim = lim = − ,
x → −2 3 x 2
+ 5x − 2 x→−2 (3 x − 1)( x + 2 ) 7

i.e. the point x = −2 is a point of removable discontinuity.


1
Having accepted f ( −2 ) = − we shall get a continuous function.
7
1
For x2 = we have:
3
x+2
lim = lim 1 = + ∞ ;
x→ +0 3x − 1
2
1
x→ +0 3x + 5 x − 2 1
3 3

x+2 1
lim = lim = −∞ ,
1
x→ −0 3x + 5 x − 2 x → −0 − 1
2 1 3 x
3 3

1
i.e., the point x = is a point of infinite discontinuity. The graph of this func-
3
tion is represented on Fig. 3.11.

–3 – 1 –1 1 3

–3

–5

Fig. 3.11

190
Example 2. Find points of discontinuity of the function
3
f (x )= 1
.
2+ 4 x +1
Solution. The given function is determined on all the numerical axis except
for a point x = −1. We find the left — and the right-sided limits at this point:

3 3 3
f ( −1 − 0 ) = lim = −∞
= ;
x →−1− 0 1
2+4 2
2 + 4 x +1
3 3
f ( −1 + 0 ) = lim = = 0.
x →−1+ 0 1
2 + 4+∞
2 + 4 x +1
Thus, x = −1 is a point of essential dis- 3
continuity (of the first kind).
3 3
lim f (x ) = lim = = 1.
x → ±∞ x →±∞ 1
2 + 40 1
2 + 4 x +1

The graph of the function f ( x ) is rep- –2 –1 0 1 2 3


resented on Fig. 3.12. –1
Example 3. Find points of discontinuity
of the function and sketch its graph: Fig. 3.12

⎧ 2 x if 0 ≤ x ≤1

f ( x ) = ⎨ 4 − 2 x, if 1 < x < 2,5
⎪2 x − 7, if 2,5 ≤ x ≤ ∞

Solution. A function f ( x ) is determined for all values x ≥ 0. But it does
not mean that it is continuous for x ≥ 0, as far as this function is not elementary.
It is given by three different formulas for different intervals of changing the
argument x and can have discontinuities at points where its analytical
expression varies, i.e. at x = 1, x = 2,5. At all other points of the domain of
definition the function f ( x ) is continuous so as any of three formulas represents
an elementary function, continuous in the interval of changing the argument x.
Let us study the behavior of the function at the points x = 1 and x = 2,5 :

lim f (x ) = lim 2 x = 2 . lim f (x ) = lim (4 − 2 x ) = 2 .


x →1−0 x →1−0 x →1+ 0 x →1+ 0

191
The value of the function f ( x ) for x = 1 is f (1) = 2 1 = 2 , i.e. at the point
x = 1 function f ( x ) is continuous.
2
lim f (x ) = lim (4 − 2 x ) = −1 ,
x → 2, 5− 0 x → 2 ,5− 0

lim f (x ) = lim (2 x − 7) = −2 ,
x → 2,5 + 0 x → 2 ,5 + 0
0 2 4
f (2,5) = −2 .

At the point x = 2,5 the function has essen-


2 tial discontinuity. The graph of the function is
Fig. 3.13 represented on Fig. 3.13.

Micromodule 16
CLASS AND HOME ASSIGNMENT

Find the points of discontinuity of functions, investigate their character, in


case of removable discontinuity predetermine the function to be continuous.
Sketch graphs of non-elementary functions:
x −1 x+2 4x + 1
1. f (x ) = 2 . 2. f (x ) = 3 . 3. f (x ) = 2 .
3x − x − 2 x +8 4x − 7x + 2
3
2x + 5 2x 2 + 7x + 6
4. f (x ) = . 5. f (x )
=2 7
x − . 6. f (x ) = .
2x + 5 3x 2 + 5 x − 2
2x + 1 sin x cos x
7. f (x ) = . 8. f (x ) = . 9. f (x ) = .
x 2 (x + 1) x x −1
x−2
3 2
10. f (x ) = 5 x 11. f (x ) = . 12. f (x ) =
2
−1 . .
1 x
2+ 2
4 x− 1+ 2 +1
x

1 1 1

1 4 x +3 −1
13. f (x) = (x + 2) arctg . 14. f ( x ) = 1 . 15. f (x ) = x x + 1 .
x 1 1
4 x +3 +1 −
x −1 x
x
x + 15 − 3 x −1
16. f (x ) = 3 x 17. f (x) = . 18. f (x ) =
2
−4 . .
2
x − 36 arctg(x − 1)

192
⎧ − 1 x 2 , x ≤ 2;
⎧ 1 − x, x ≤ −1; ⎪
19. f ( x ) = ⎨ 20. f ( x ) = ⎨ 2
⎩2 x − 1, x > −1. ⎪⎩ x , x > 2.

⎧ 2 x + 5, x < −1; ⎧ − x, x ≤ 0;
⎪ ⎪
21. f ( x ) = ⎨ 1 22. f ( x ) = ⎨ 2
⎪⎩ x , x ≥ −1. ⎪⎩ x − 2 , x > 0.

⎧ 2x , x ≤ 1; ⎧ 1 − x2 , x < 1;
⎪ ⎪
23. f ( x ) = ⎨ x − 1, 1 < x < 2; 24. f ( x ) = ⎨( x − 1) , 0 ≤ x ≤ 2;
2

⎪ − x2 + 6 x − 7, x ≥ 2. ⎪ 4 − x, x > 2.
⎩ ⎩
1 1+ x
25. f (x ) = ln .
x 1− x

Micromodule 16
SELF-TEST ASSIGNMENTS

Investigate the function f ( x) for continuity.


2 x2 − x − 1 3
16.1. а) f ( x) = ; b) f ( x) = .
3x 2 − x − 2 1
2+5 2
x −

5 x 2 − 3x − 2 x −1
16.2. а) f ( x) = ; b) f ( x) = arctg .
x2 + x − 2 x +1
2
5x2 − x − 4
16.3. а) f ( x) = ; b) f ( x) = 7 x +3 .
x2 + 2x − 3
3x 2 + 5 x − 8 5
16.4. а) f ( x) = ; b) f ( x) = .
x2 + 4 x − 5 lg x + 1
1
2x2 − 5x + 3 5 x −1 − 1
16.5. а) f ( x) = ; b) f ( x) = .
2x2 + x − 3 1
+1
5 x −1
1 1

x2 − 4 x + 3
16.6. а) f ( x) = ; b) f ( x) = x x + 1 .
2x2 + 5x − 7 1 1

x −1 x

193
5x2 − 4 x − 1 6
16.7. а) f ( x) = 2
; b) f ( x) = 1
.
x + 5x − 6
3 + 2 x+4
2x2 + 7 x − 9 1 1+ x
16.8. а) f ( x) = ; b) f ( x) = ln .
2
x + 3x − 4 x 1− x
4 x 2 − 3x − 1 1
16.9. а) f ( x) = 2
; b) f ( x) = ( x + 1) arctg .
2 x + 3x − 5 x
2x2 − 7 x + 5 1
16.10. а) f ( x) = 2
; b) f ( x) = 1 1
.
x − 3x + 2
3 x+2 + 3 x−2
3
3x 2 − 7 x + 4
16.11. а) f ( x) = ; b) f ( x) = x
2 7− .
x2 − 4 x + 3
4x2 − 7 x + 3 6
16.12. а) f ( x) = 2
; b) f ( x) = 1
.
x − 5x + 4
6 + 6 −6
x

5x2 − 7 x + 2 1
16.13. а) f ( x) = 2
; b) f ( x) = 1 1
.
x − 6x + 5
4 x +1 + 4 x −1
x
2x2 − 9 x + 7 2
16.14. а) f ( x) = ; b) f ( x) = 2 x −9 .
x2 − 7 x + 6
3x 2 − 5 x + 2 2
16.15. а) f ( x) = 2
; b) f ( x) = (2 x + 3) arctg .
x + 3x − 4 x
2x2 − x − 3 x
16.16. а) f ( x) = ; b) f ( x) = .
2
x −x−2 lg (1 + x )

2x2 − 7 x − 9 2
16.17. а) f ( x) = 2
; b) f ( x ) = 1
.
x − 3x − 4
3 + 4 x −5
x
2x2 − 5x − 7 2
16.18. а) f ( x) = ; b) f ( x) = 3 x −4 .
x2 − 4 x − 5
3x 2 − 4 x − 7 1
16.19. а) f ( x) = 2
; b) f ( x) = ( x + 4) arctg .
x − 2x − 3 x

194
x−2
4 x2 − x − 5
16.20. а) f ( x) = ; b) f ( x) = 5 x −1 .
x2 − 4 x − 5
1
2
5x − x − 6 4 +3
x −1
16.21. а) f ( x) = 2
; b) f ( x) = 1
.
x − 5x − 6
4 +3
x +1
1
6x2 − x − 7 3 x+2 −1
16.22. а) f ( x) = 2
; b) f ( x) = 1
.
x − 5x − 6
3 x+2 +1
1
4 x 2 − 3x − 7 7x
16.23. а) f ( x) = ; b) f ( x ) = .
x2 − 6 x − 7 x
4+2 2
x −

4 x2 − 5x − 9 x
16.24. а) f ( x) = ; b) f ( x) = .
2
x − 7x − 8 lg ( 2 − x )

6x2 − x − 7 3
16.25. а) f ( x) = 2
; b) f ( x) = 1
.
x −x−2
2+4 x
2 −

7 x2 − x − 8 x −1
16.26. а) f ( x) = ; b) f ( x) = .
2
x − 7x − 8 arctg ( x − 1)
1
4 x 2 − 7 x − 11 3x
16.27. а) f ( x) = ; b) f ( x) = x
.
x 2 − 3x − 4
4 + 2 x −5
1
4 x2 + x − 3 2 x−2
16.28. а) f ( x) = ; b) f ( x) = .
x2 + 4x + 3 x
1 + 3 x +1
3
3x 2 + x − 2 2
16.29. а) f ( x) = ; b) f ( x) = 6 x −1 .
x 2 + 3x + 2
5x2 + x − 4 1 2− x
16.30. а) f ( x) = ; b) f ( x) = ln .
2
x + 5x + 4 x 2+ x

195
Micromodule 17
BASIC THEORETICAL INFORMATION
DERIVATIVE
The derivative, its geometrical and physical concept. Differentiability and
continuity. The rules of differentiation. The derivatives of the elementary
functions. The derivative of composite function (The chain rule for dif-
ferentiation).

Literature: [2, chapter 4], [3, chapter 4, items 4.1—4.2], [4, part 5], [6,
chapter 5, §§ 1, 2], [7, chapter 6, § 16], [9, chapter 4, § 2, 4, 5], [10,
chapter 4, § 1], [11, chapter 3, §§ 1—15], [8].

17.1. Some problems leading to understanding


of the derivative definition
1. The instantaneous velocity of non-uniform motion. Suppose that a body
begins to move at the moment of time t = 0 along the straight line. Let the path
traveled by the body over the time t is defined by the formula S = f (t ) . A
function S = f (t ) is called the law of body motion. Let us consider a path
traveled by the body over the time [t ; t + Δt ] ; it is equal to

ΔS = f (t + Δt ) − f (t ) .
If the body is moving uniformly, the ratio of the covered path to the time
ΔS f (t + Δt ) − f (t )
=
Δt Δt
is velocity. It does not depend on t and Δt . In case of non-linear motion this
ratio depends both on the chosen time t and on increment Δt and expresses the
average velocity of motion over the time interval [t ; t + Δt ] . The less is the time
interval Δt , the more reasons we may have to consider the motion over the time
t and t + Δt to be uniform.
The limit
ΔS f (t + Δt ) − f (t )
lim = lim = v(t ) ,
Δt →0 Δt Δt →0 Δt

if it exists, is called the instantaneous velocity at the instant of time t .


2. The rate of the chemical reaction. Suppose the chemical reaction begins
at an instant of time t . Let us designate the quantity of substance, which reacted
till the instant of time t is c (t ) . The quantity of substance, which started to react
during time interval [t ; t + Δt ] , equals c(t + Δt ) − c(t ). The ratio

196
c(t + Δt ) − c(t )
Δt
defines the average rate of the reaction and the limit
c(t + Δt ) − c(t )
lim
Δt →0 Δt

is called the rate of the chemical reaction at an instant of time t .

17.2. The definition of the derivative

Let any function y = f ( x) be defined at the interval (a, b) . We take any


meaning of x within this interval and apply it to increment Δx . The difference

Δy = f ( x + Δx) − f ( x)
is called the increment of a function at point x. The increment of argument
Δx ≠ 0 may gain both positive and negative meanings but the meaning x + Δx
does not go beyond the limits of the domain of the function f ( x) .
Definition 3.19. The limit of the ratio of the increment of a function
Δy = f ( x + Δx) − f ( x) to the increment of argument Δx , when the later tends
to zero, is called the derivative of a function y = f ( x) at point x , that is,

f ( x + Δx ) − f ( x )
y ′( x ) = lim
Δx→0
Δx

Definition 3.20. A function, which has the finite derivative at point x , is


called differentiable at this point.
dy df
Notations for derivative: y ′( x), f ′( x) (Lagrange’s notation) or ,
dx dx
(Leibniz’s notation). From the definition of the derivative it follows that the
'
derivative y ( x ) at point x is a number. However, if such a number exists for
each inner point of the interval (a, b) , the derivative, then, can be considered as
a function of a point x from the given interval.
If lim f ( x + Δx) − f ( x)
= ±∞ , the function f ( x) has the infinite derivative
Δx → 0 Δx

at point x .

197
17.3. The geometrical, physical and mechanical
concept of the derivative

Let us try to give the geometrical interpretation of the derivative. We’ll


consider the graph of a function y = f ( x) at the neighbourhood of point x0 (see
Fig. 3.14). Let P0 be a point of the curve with components ( x0 ; f ( x0 )) , and P
be a point of the graph with components ( x0 + Δx; f ( x0 + Δx)). A straight line,
drawn trough the points P0 and P , is called the secant line. If, at the indefinite
approach of point P along the graph of a function y = f ( x) to point P0 , the
secant line P0 P approaches some boundary position ( straight line P0 K ), then
this boundary position of the secant line is called the tangent line to the curve
y = f ( x) at the point P0 .
Let α be the angle, which the tangent line forms with the positive direction
of x-axis and β be the angle between the secant line P0 P and x-axis. From the
right triangle P0 QP it follows that

PQ Δy f ( x0 + Δx) − f ( x0 )
tg β = = = .
P0 Q Δx Δx
There exists such a limit that

f ( x0 + Δx) − f ( x)
y ′( x0 ) = lim = lim tg β = tg α.
Δx →0 Δx ( p → p0 )
Δx → 0

The geometrical concept of the derivative is follows: the derivative of a


function y = f ( x) at pointі x0 equals the slope of the tangent line to the graph
of a given function at the respective point, that is,

f ′( x0 ) = tg α,

where α is the angle, which is formed by the tangent line to the graph of the
function with positive direction of x-axis at point x0 .
The equation of the tangent line drawn to the graph of a function y = f ( x)
at point P0 ( x0 , y0 ) looks as follows

y − y0 = f ′( x0 )( x − x0 ), (3.8)

where y0 = f ( x0 ) .

198
The straight line that passes through the point of contact, perpendicular to the
tangent line is called the normal line (it is the straight line P0 N in Fig. 3.14).
The equation of the normal line is

1
y − y0 = − ( x − x0 ). (3.9)
f '( x0 )

If a function y = f ( x) describes some physical process, the derivative y ′,


then, is the rate of change of this process. This is the way the physical concept
of derivative may be interpreted.
In other words, whatever dependence a function y = f ( x) could express,
Δy
the ratio can be considered as the average rate of change of a function y
Δx
with respect to the argument x, and the derivative f ′( x ) — as the instantaneous
rate of change of this function.

у
N y = f(x)
P
f(x0 + Δx)
К

P0
f(x0) Q

α β Δx
О x0 x0 + Δx х

Fig. 3.14

The mechanical concept of the derivative. If S = S (t ) is the law of motion


of a material point (it means that the dependence of the traversed path S on t is
given), the derivative S ′(t ) , then, is velocity v of a point at moment t ; the
second derivative S ′′(t ) is the instantaneous acceleration a of a point at moment t
and the third derivative S ′′′(t ) is a jerk of a point at moment t , that is,

v = S ′(t ), a = S ′′(t ) = v′(t ), j = S ′′′(t ) = v′′(t ) = a ′(t ).

199
17.4. The main rules of differentiation
Let functions u(x), v(x), w(x) be differentiable at point х and С is a constant. Then

1. (u + v) ′ = u ′ + v ′ . 2. (u − v) ′ = u ′ − v ′ .
3. (uv) ′ = u ′v + uv ′ . 4. (Cu ) ′ = Cu ′ .

⎛ u ⎞ u ′v − uv ′
5. ⎜ ⎟ = ( v ≠ 0) . 6. (uvw) ′ = u ′vw + uv ′w + uvw′ .
⎝v⎠ v2

17.5. The table of derivatives


(differentiation of the elementary functions table)
Table 17.1
1. (C )′ = 0 2. ( x n )′ = nx n −1
1 ′
3. ( x )′ = 4. ⎛⎜ ⎞⎟ = − 2
1 1
2 x x
⎝ ⎠ x
5. (a x )′ = a x ln a 6. (e x )′ = e x
1 1
7. (log a x)′ = 8. (ln x)′ =
x ln a x
9. (sin x)′ = cos x 10. (cos x)′ = − sin x
1 1
11. (tg x)′ = 12. (ctg x)′ = − 2
cos 2 x sin x
1 1
13. (arcsin x)′ = 14. (arccos x)′ = −
2
1− x 1 − x2
1 1
15. (arctg x)′ = 16. (arcctg x)′ = −
1 + x2 1 + x2
17. (sh x)′ = ch x 18. (ch x)′ = sh x
1 1
19. (th x)′ = 20. (cth x)′ = −
ch 2 x sh 2 x

17.6. The derivative of the composite function


(The chain rule for differentiation). Table of derivatives
If a function y = f (u ) has the derivative at point u , and a function u = g ( x)
has the derivative at point x , a composite function y = f ( g ( x)) then is
differentiable at point x . In this case

200
y ′ = f ′(u ) ⋅ g ′( x).

In other words, the derivative of the composite function y = f (u ) where


u = g ( x) equals the product of the derivative of external function taken with
respect to internal argument u , and that of internal function taken with respect
to independent variable x.
If u (x) is the function differentiable at point x, the following rules are
fulfilled:
Table 17.2

1 ′
1. (u n )′ = nu n −1u ′ 2. ( u )′ = u
2 u
3. (a u )′ = au ln a ⋅ u ′ 4. (eu )′ = eu ⋅ u ′
1 1 ′
5. (log a u )′ = u′ 6. (ln u )′ = u
u ln a u
7. (sin u )′ = cos u ⋅ u ′ 8. (cos u )′ = − sin u ⋅ u ′

1 1
9. (tg u )′ = u′ 10. (ctg u )′ = − u′
cos 2 u sin 2 u
1 1
11. (arcsin u )′ = u′ 12. (arccos u )′ = − u′
2
1− u 1− u2
1 1
13. (arctg u )′ = u′ 14. (arcctg u )′ = − u′
1+ u2 1+ u2
15. (sh u )′ = ch u ⋅ u ′ 16. (ch u )′ = sh u ⋅ u ′

1 ′ 1
17. (th u )′ = u 18. (cth u )′ = − u′
ch 2 u sh 2u

Micromodule 17
EXAMPLES OF PROBLEMS SOLUTION

Example 1. Using the definition of the derivative, find the derivative of the
function y = x 2 .
Solution. According to the definition of the derivative, we have

201
f ( x + Δx) − f ( x) ( x + Δx ) 2 − x 2
( x 2 )' = lim Δx →0 = lim Δx →0 =
Δx Δx
x 2 + 2 xΔx + Δx 2 − x 2
= lim Δx →0 = lim Δx →0 (2 x + Δx ) = 2 x
Δx
Example 2. Find the derivative of the function y = cos x by the definition.
Solution. We write the increment of function
Δy = cos( x + Δx) − cos x.

Remember the first honorable limit


sin Δx
lim Δx →0 =1
Δx
as well as formula
x− y x+ y
cos x − cos y = −2sin sin .
2 2
Then
Δx Δx
cos( x + Δx ) − cos x −2sin sin( x + )
'
(cos x) = lim Δx →0 = lim Δx →0 2 2 =
Δx Δx
Δx
− sin
2 ⋅ lim Δx
= lim Δx →0 Δx →0 sin( x + ) = −1 ⋅ sin x = − sin x.
Δx 2
2
Example 3. Find the derivative of the function y = 3 x at point x = 0 .
Solution. We have
3
x + Δx − 3 x 3
0 + Δx −3 0
lim = lim =
Δx → 0 Δx Δx →0 Δx
1
= lim 2
= ∞.
Δx →0
( Δx ) 3

So the limit increases infinitely when Δx → 0. Therefore the derivative of the


function y = 3 x at point x = 0 does not exist. It might be said, as well, that
there exists no derivative of continuous function y =| x | within the complete
number scale at point x = 0. Consequently, if a function is discontinuous at
pointі x, it does not follow necessarily that it has the derivative at this point.

202
However, if the function is differentiable (if it has the derivative) at point x , it is
continuous at this point then.

Find the derivatives of the functions:


Example 4. y = 4 x5 − 3 x 4 + 1 .
Solution.
y ′ = (4 x5 − 3 x 4 + 1)′ = (4 x5 )′ − (3x 4 )′ + (1)′ = 4( x5 )′ − 3( x 4 )′ =
4 ⋅ 5 x 4 − 3 ⋅ 4 x3 = 20 x 4 − 12 x3 .
3 1
Example 5. y = 2
− .
x x
Solution.
⎛ 3 1 ⎞ −2
−1
−2 −1/ 2
y' = ⎜ 2 − ⎟ = (3x − x )′ = 3( x )′ − ( x )′ =
2
⎝x x⎠
6 1
= 3 ⋅ (−2) x −2 −1 − (−1 / 2) x −1/ 2−1 = = −6 x −3 + (1/ 2) x −3/ 2 = − 3 + .
x 2 x3
x2
Example 6. y = .
sin x
Solution.
⎛ x 2 ⎞′ ( x 2 )′ sin x − x 2 (sin x)′ 2 x sin x − x 2 cos x
y' = ⎜ ⎟= = .
⎝ sin x ⎠ (sin x) 2 sin 2 x

Example 7. y = arcsin x ⋅ tg x.
Solution.
y ′ = (arcsin x ⋅ tgx)′ = (arcsin x )′tgx +
1 1
+ arcsin x ⋅ (tg x)′ = ⋅ tg x + arcsin x ⋅ .
1− x 2 cos 2 x

Example 8. Find y ' (0), if y = e x ⋅ x .


Solution. y ′ = (e x ) x + e x ; y '(o) = e0 0 + e0 = 1 .
Applying the rule of differentiation of composite function, find the
derivatives of the functions:
Example 9. y = ( x 2 + 1)3 .
Solution. Let us make a designation u = x 2 + 1 , then y = (u ( x))3 . By the
rule of differentiation of the composite function, it follows
y ′ = (u 3 )′ = 3u 2 u ′ = 3( x 2 + 1) 2 ( x 2 + 1)′ = 3( x 2 + 1) 2 ⋅ 2 x = = 6 x( x 2 + 1) 2 .

203
Example 10. y = sin 3 x .
Solution. Let us designate u = sin x , then y = (u ( x))3 . Hence,

y ′ = (u 3 )′ = 3u 2 u ′ = 3sin 2 x ⋅ (sin x )′ = 3sin 2 x cos x.

Example 11. y = x 4 + x3 + 1 .
Solution. In this case y = u where u = x 4 + x3 + 1 . Then

1 ( x 4 + x3 + 1) ' 4 x3 + 3 x 2
y' = ( u)' = u' = = .
2 u 2 x 4 + x3 + 1 2 x 4 + x3 + 1
1
Example 12. y = .
cos 2 x
Solution. We write the given function as follows y = cos−2 x . Then

u = cos x , y = u−2 ,
y ′ = (u −2 )′ = −2u −3u ′ = −2 cos −3 x(cos x)′ = 2 cos −3 x sin x.

Example 13. y = 5sin x .


Solution. We have the composite function y = 5u where u = sin x . Then

y ′ = (5u )′ = 5u ln 5 ⋅ u ′ = 5sin x ln 5(sin x)′ = 5sin x ln 5cos x .

Example 14. y = tg e x .
Solution. We have the composite function y = tg u , where u = e x . Then

1 1 ex
y ′ = (tg u )′ = 2
u′ = 2 x
(e x ) ′ = .
cos u cos e cos 2 e x

Example 15. y = arcsin x arcsin x .


Solution. Having applied the product rule, we get

y = (arcsin x )′ arcsin x + arcsin x ( arcsin x )′ =


( x )′ 1
⋅ arcsin x + arcsin x (arcsin x)′ =
1− ( x) 2 2 arcsin x
1 1 1
arcsin x + arcsin x .
2 x 1− x 2 arcsin x 1 − x 2

204
Example 16. y = ln( x 2 + 1) .
Solution. We have the composite function y = ln u , where u = x 2 + 1 .
Then
1 ′ 1 2x
y ′ = (ln u )′ = u = 2 ( x 2 + 1)′ = 2 .
u x +1 x +1

Example 17. y = arctg(log 2 ( x + cos x)) .


Solution. We have the composite function y = arctg u , where u = log 2 ( x +
+ cos x) is also composite, that is, u = log 2 w , w = x + cos x . In this case, we
find the derivative as follows:

1 1 1
y ′ = (arctg u )′ = 2
u ′ = 1 2 (log 2 w)′ = 2 w ln 2
w′ =
1+ u 1+ u 1+ u
( x + cos x)′ 1 − sin x
= 2
= .
(1 + (log 2 ( x + cos x)) )( x + cos x) ln 2 (1 + log 22 ( x + cos x))( x + cos x) ln 2

Example 18. y = arcctg 2 ( x3 − 2) .


Solution.
6 x 2 arcctg( x3 − 2)
y ′ = 2 arcctg( x3 − 2) ⋅ (arcctg( x3 − 2))′ = − .
1 + ( x3 − 2) 2

Micromodule 17
CLASS AND HOME ASSIGNMENT

Find the derivatives of the functions:


2
1. y = 2 + x − x 2 . 2. y = 3 ⋅ 4 x . 3. y = 3 + − x3 .
x
1 3
4. y = + − x ln x . 5. а) y = arccos x ⋅ 3x ; b) y = 10 x / x .
x sin x
cos x sin x
6. y = 4 tg x ⋅ 6 x5 . 7. а) y = ; b) y = .
x 3
x2
arcsin x
8. y = . 9. y = 2 x sin x ⋅ x 4 . 10. y = arctg x + arcctg x .
arccos x
1 − x5 2 − 7 x2
11. y = (3 x + x )( x − x) . 12. а) y = ; b) y = .
x5 + 1 7
x2 + 2

205
13. y = ( x 2 − 1)( x 2 + 1)( x 4 + 1) . 14. y = (1 + x )(1 + 3 x ) .
3
x x x
15. а) y = ; b) y = . 16. y = ( x 2 − 3 x + 2)( x 2 − 5 x + 3) .
1+ x 1+ 4 x
3 x3 − 2 x + 4 3
x +1
17. y = 3 4
. 18. а) y = ; b) y = .
( x −1)(1− 2 x ) x+ x 3
x2 − 1
19. y = log 2 x + 1/ ln x . 20. y = tg x ⋅ arctg x − 3arccos x .

Find the derivatives of the composite functions:


4
21. y = (3 x + 2)5 . 22. y = cos x . 23. y = tg log 3 x .

24. y = arcsin(ln x) 25. y = arccos x . 26. y = 3x + x


.
27. y = 2arctg x . 28. y = x ln sin 2 x . 29. y = e x ⋅ tg( 3 x + 1) .

30. y = 4 ctg x . 31. y = sin 6 (cos 3 x) . 32. y = cos5 (log32 x) .

1− x 1− x arctg x −1
33. y = arccos 34. а) y = arctg ; b) y = .
1+ x 1+ x arctg x + 1
x
35. y = sh(cos ). 36. y = earccos(ln x ) .
1+ x
1 x2 +1
37. а) y = ( x 2 −1) arccos ; b) y = . 38. y = ln(ln 2 x) .
x arccos x
ex −1 sin x tg x − x
39. y = cos ln x
. 40. y = . 41. y = .
3 4
e +1 cos x x
ln( x + arctg x) 3 sin 3 x
42. y = . 43. y = . 44. y = .
arctg ln x 3
arccos 4 x cos 4 x
1 1
45. y = 3sin 5x cos4 x . 46. y = 4 sin cos 2 .
x x

47. y = sin(cos x) cos(sin x). 48. y = ln( x + x 2 −1) .

x2 − 4
49. y = 3 + 4 log53 cos 2 x. 50. y = ln(e3x + e3x −1) + arccos e−3x .
4 3
x −1

51. y = (1 + x 2 ) 1− x 2 .

206
Answers

3 1 1 cos x 12x + 5 sin 2x


1. 1 − 2x. 2. . 3. − − 3x2 . 4. − 2
− 3 2 − ln x − 1 . 6. .
4 x
4 3 x x x sin x 3 cos2 x6 x
2x sin x + cos x π
7. а) − . 8. . 9. 2 x x3 (ln 2 ⋅ x sin x + x cos x + 4sin x) .
x x 2 1 − x arccos2 x
2

− 10 x 4
10. а) 0. 11. 3 x − 6 x + 1 . 12. а) . 13. 8x 7 . 14. 4 x 3 − 24 x 2 + 40 x − 19 .
( x 5 + 1) 2
1 1 1
19. − . 21. 15(3 x + 2) 4 . 22. −4cos3 x sin x. 23. .
x ln 2 x ln 2 x x ln 3 ⋅ cos 2 (log 3 x)
1 1 2arctg x ln 2
24. . 25. − . 27. . 28. ln sin 2 x + 2 xctg 2 x .
x 1 − ln 2 x 2 x − x2 1 + x2
1 −1 x
31. −18sin 5 (cos3x)cos(cos3x)sin 3x. 33. . 34. а) . 35. − chcos ×
x (1 + x) 2 1− x2 x +1

x −earccosln x e x − 1 2e2 x sin2 x


× sin /(1 + x) 2 . 36. . 39. sin ln x ⋅ . 44. (3 + sin2 x) .
x +1 x 1 − ln 2 x e + 1 1 − e2 x cos5 x
1
48. .
2
x −1

Micromodule 17
SELF-TEST ASSIGNMENTS
17.1. Find the derivatives of the first order y = f ( x ) .
17.1.1. а) y = cos 2 x + sin(tg x) ; b) y = ln 2 arcsin x ;
2x
c) y = 2sin x + cos ; d) y = 5 (2 x + 1) arcctg x .
x
17.1.2. а) y = 3 ctg x + tg x 2 ; b) y = log3 ;
1− x 2
c) y = 10 ln x
⋅ 3 tan x ; d) y = 5 arctg(ln 2 x) − 1 .
1
17.1.3. а) y = sin x − 2 sin 3 x ; b) y = ln arccos ;
x
c) y = earctg x cos 2 x ; d) y = 4 log 3 sin( x3 + 1) .
x−2
17.1.4. а) y = x 2 /(1 + cos 2 2 x) ; b) y = 3 ln cos ;
5
x
b) y = ln sin(3x x 2 ) ; d) y = 3 cos3 (tg x) .

207
17.1.5. a) y = cos 5 (sin 3 x) ; b) y = (1 + cos 2 x)5 sin 4 x ;
1
c) y = ln( x + arccos 1− x 2 ) ; d) y = ln 5 arctg .
x
2 arctg ln x
17.1.6. а) y = e x x / sin x 2 ; b) y = ;
ln arctg x
4x
c) y = 102 − tg ; d) y = 6 e − x + 1 ⋅ sin(4 x + 1) .
17.1.7. а) y = tg 2 x − 2 ctg x 2 ; b) y = (2 + ln 2 sin x)3 ;
x
c) y = 2 5 ⋅ 4 cos x ; d) y = log 32 arcsin( x 2 ) .

17.1.8. а) y = x cos 2 x − ctg 4 x ; b) y = arcsin 3 ln sin 2 x ;

1 + ln 2 x
c) y = arctg 5 (e2 x x ) ; d) y = .
x3 + 2
17.1.9. а) y = x3 /(1 + sin 4 x) ; b) y = log 2 arctg(1− x 2 ) ;

2
sin 5 x 1 + x3
c) y = 3ln + 2x ; d) y = 1 + sh .
1− x3

1+ x x + x2 − 2
17.1.10. а) y = sin ; b) y = lg ;
1− x 3x
2
c) y = e− x sin(3 x − 2) ; d) y = cos(sin 3 ( x tg x)) .

1+ x2 1− arctg x
17.1.11. а) y = 2 + sin ; b) y = ;
1− x 2
ln 2 x
ln x
c) y = 2 ⋅ x3 ; d) y = tg 4 (ch x) − ch(tg x 2 ) .
5 ctg 2 (5 + 1/ x)
17.1.12. а) y = ; b) y = ln arccos(2 x − 5) ;
x
c) y = tg(2cos x ) ln( x3x ) ; d) y = c h 2 ( x 2 − 1) − c h x .
sec 2 (1 + x 2 )
17.1.13. а) y = −1 ; b) y = log 42 arcsin(3x3 ) ;
cos x
1−tg 2 x
c) y = 5 ctg 3x ; d) y = 4 2− x + 1 ⋅ cos 4 x .

208
17.1.14. а) y = tg(cos(5 ctg x)) ; b) y = 3 ln arctg x ;
log 2 ( x + 1 / x)
c) y = 3sin x sin 3 x + 3 ; d) y = .
2 x3
x +1
17.1.15. а) y = cos(sin x tg x ) ; b) y = ln 5 arctg ;
x −1
x −1
c) y = 6arctg x
− 2 tg x ; d) y = 4 .
x +1
1
17.1.16. а) y = tg 2 − 5 ctg 3x ; b) y = log34 sin 1 + x3 ;
x2
2 /2
c) y = x3 e− x − cos 2 x ; d) y = arc tg 2 ( x ln x) .
17.1.17. a) y = (tg 3x ) / sin x ; b) y = arcsin 4 ln ln x ;

c) y = 2arccos x cos 2 x ; d) y = ln( x + ln( x + 1− x 2 ) .


17.1.18. а) y = 4 tg( x / 4) + ctg 4 x ; b) y = arccos 2 ln sin x ;
2 − arcctg x
c) y = tg(4ln x + 3tgx ) ; d) y = .
log 34 x
17.1.19. а) y = cos3 5 x − 8 sin 3 4 x ; b) y = ln ln cos ln tg x ;
x2
c) y = 2 / x2 ; d) y = 7 log33 sin 1 + x .

ln(1 + ln 2 x)
17.1.20. а) y = 5 sin 4 x − cos x ; b) y = ;
log 2 x
sh 2 (1 + x 2 )
c) y = 2 ln arcsin x
; d) y = − 2 th 2 x .
ch x
arcsin ln x
17.1.21. а) y = ctg 3 ( 6 2 − x tg 2 x ) ; b) y = ;
ln( x 2 + 1)
3
sin 5 x−5 cos2 x
c) y = e ; d) y = tg(4ln x + 7ctg x ) .
x
17.1.22. а) y = cos ; b) y = log x 3 + log34 x ;
sin x
4 x−1/ x x −1
c) y = cos e ; d) y = ln 5 arctg .
x +1

209
x2
17.1.23. а) y = 2 2
; b) y = x ln( x + 1− x 2 ) ;
sin x − cos( x )
3
−2) / sin x 1
c) y = 2( x ; d) y = tg 2 2
− 5 ctg 4 2 x .
x +1
sin x x
17.1.24. а) y = cos ; b) y = sin ln tg x − ln ctg ;
x +1 2
2 1 + ln 4 x
c) y = cos 2 x /(e x + 3x ) ; d) y = .
log 5 x
sin x
17.1.25. а) y = 3
; b) y = x 2 (cos ln x − sin ln x) ;
3 cos x + cos x
x x
c) y = 54 + 45 ; d) y = x ln( 4 − x 2 ) .

17.1.26. а) y = cos 1 + x3 + cos x ; b) y = log 32 (ln x − log cos x 2) ;


x −1
c) y = 10 3 x − 4 ⋅ (3 x − 4)10 ; d) y = arccos .
x
17.1.27. а) y = 3 tg( x / 3) + cos sin x ; b) y = log 33 log 22 ln(5 x − 2) ;

2 3 cos x arcsin ln x
c) y = 3sin 3x
+ ; d) y = .
x x2
3 4
17.1.28. а) y = cos 6 + ; b) y = ln(sin 2 x + sin 3 ln x ) ;
x x cos x
1− x 2
c) y = x cos(2 arctan ); d) 10 2 x −5 ⋅ (7 x 2 − 1) 8 .
cos 3 ( x 2 − 1 / x 2 )
17.1.29. a) y = ; b) y = ln cos log 7 ctg ln x ;
cos x
5
c) y = e arctan x
/ ln x ; d) y = tg sin 2 x + x3 .

17.1.30. а) y = x sin 3 5 x + cos ec 2 x ; b) y = ln 3 sin( x cos x) ;


tan x
c) y = arcsin 8sin x / 2cos x ; d) y = 3 .
x ln x

210
Micromodule 18
BASIC THEORETICAL INFORMATION
DERIVATIVE AND ITS CALCULATION (CONTINUED)

Derivative of inverse function. Derivative of implicit function and func-


tion in parametric form. Logarithmic differentiation.

Literature: [3, chapter 3, §§ 3.1—3.8], [4, part 5], [6, chapter 5, § 2], [9],
[10, chapter 4, § 5], [11, chapter 4, § 2], [12, chapter 3, §§ 16—18].

18.1. Derivative of inverse function


Let y = f ( x) and x = g ( y ) be a pair of inter-inverse functions (remember,
that the plots of such functions coinside).
Let’s prove a theorem about connection between derivatives of these
functons.

Theorem 3.10 If functon y = f ( x) is monotone on the interval (a; b) and


has non-zero derivative f ′( x) at any point of this interval, then
there exists inverse function x = g ( y ), which has derivative x′ = g ( y ), and

1 1
g ′( y ) = , or y ′ = .
f ′( x) x′
This formula has geometrical interpritation.
The curve is set by the function y = f ( x) or inverse function x = g ( y ) (Fig. 3.15).
Then f ′( x) = tg α ( α is the angle between the tangent line and x-axis),
g ′( y ) = tg β ( β is the angle between the tangent line and y-axis). Since α + β = π .
2
Then, tg α = tg ( π − β) = ctg β = 1 and y′ = 1 .
2 tg β x′

y
y = f(x)
x = g(x)
M(x; y)

α
О x
β

Fig. 3.15

211
18.2. Differentiation of implicit function

Let the implicit function y = f ( x) is set by equation F ( x, y ) = 0. In order


to find the derivative y ′( x) , we need to differentiate both parts of equation
F ( x, y ) = 0 with respect to x , and not to forget that y is a function of
variable x . The obtained equation may be solved with respect to y ′( x) . So, we
find derivative from the condition
d
F ( x, y ) = 0.
dx
Derivative of implicit function is expressed through the independent variable
x and function y itself.

18.3. Differentiation of functions given parametrically

The derivative of function y = f ( x), given by parametrical equation x = x(t ),


y = y (t ) (where x(t ) and y (t ) are differentiable at the point t and y ′(t ) = 0, is
found by the formula
dy
d ψ ′(t ) y′
= dt = or y ′x = t .
dx dx ϕ′(t ) xt′
dt

18.4. Logarithmic differentiation


In some cases while finding derivative we should at first take the logarithm
of given function and then find the derivative as in the case of implicit functon.
This operation is called logarithmic differentiation.
It is particularly convenient to use logarithmic differentiation if the function
is represented as:
u k1 ( x) ⋅ u 2 k2 ( x) ⋅ … ⋅ u m km ( x ) ( x)
а) y = 1 l l2 ln ( x )
; b) or y = u ( x) v ( x ) .
v1 ( x) ⋅ v2 ( x) ⋅ … ⋅ vm
1
( x)
Let’s show how to find the derivative of the function y = u ( x )v ( x ) , where
u ( x), v( x) are differentiable functions with respect to x, u ( x ) > 0 .
Applying the logarithm, we receive
1 1
ln y = ln(u v ) = v ln u ; (ln y )′ = (v ln y )′ ; y ′ = v′ ln u + v u ′ ;
y u
⎛ 1 ⎞
y ′ = u v ⎜ v′ ln u + v u ′ ⎟ = u v ln u ⋅ v′ + vu v −1 ⋅ u ′
⎝ u ⎠

212
Micromodule 18
EXAMPLES OF PROBLEMS SOLUTION

Example 1. Find the derivative y ′ , if x = y 3 + 3 y .


Solution. We receive
1 1
x′ = 3 y 2 + 3 ; y x′ = = .
x ′y 3( y 2 + 1)

Example 2. Find the derivative x ′y , if y = x + e x .


Solution. The functon y = x + e x is monotone for x ∈ R. y ′ = 1 + e x > 0 .
That is why for function y ( x) there exists inverse functon x = x( y ) and its
1 1
derivative is x′ = = .

y 1+ ex
Example 3. Prove that
1
(arcsin x)′ = .
1− x2
Solution. The function y = arcsin x (where x ∈ [−1; 1] ), is inverse to
function x = sin y , y ∈ ⎡ −π , π ⎤ . On interval ⎡ −π , π ⎤ the function
⎣ 2 2⎦ ⎣ 2 2⎦
x = sin y increases and the derivative x′ = cos y > 0. That is, all conditions of
theorem are fullfiled. We receive

1 1 1
(arcsin x )′ = = = .
cos y 2
1 − sin y 1 − x2

Example 4. Find the derivative y ′, if x 2 + y 2 = 1.


x
Solution. We have 2 x + 2 yy ′ = 0 , and we get y ′ = − .
y
Example 5. Find the derivative y ′, if x 2 + 2 xy − y 2 = 2 x.
Solution. Let’s differentiate both sides of equation with respect to х keeping
in mind that у is a function of х:

2 x + 2( y + xy ′) − 2 yy ′ = 2 , or x + y + xy ′ − yy ′ = 1 .

1− x − y
We find y ′( x − y ) = 1 − x − y. That is, y ′ = .
x− y

213
y
Example 6. Find y ′ , if arctg = ln x 2 + y 2 .
x
Solution. We have

⎛ y ⎞′
d ⎛
dx ⎜⎝
y⎞ d 1
arctg ⎟ = ⎛⎜ ln( x 2 + y 2 ) ⎞⎟ ,
1
2 ⎜ x⎟
=
1
(x 2
) ′
+ y2 ,
x ⎠ dx ⎝ 2 ⎠ ⎛ y⎞ ⎝ ⎠ 2 x + y
1+ ⎜ ⎟
2
(2
)
⎝x⎠
x2 y′ − y 2 x + 2 yy ′
= , y ′x − y = x + yy ′, y ′( x − y ) = x + y,
2
x +y 2
x 2
(
2 x2 + y2 )
so
x+ y
y′ = .
x− y

⎧ y = a sin t
Example 7. Find y ′x , if ⎨ .
⎩ x = b cos t
Solution. y ′(t ) = a cos t , x′(t ) = −b sin t , then

a cos t a
y ′x = = − ctg (t ) .
− b sin t b

⎧⎪ y = t 2 − 2t 3
Example 8. Find y ′x , if ⎨ .
⎪⎩ x = 2t + t 2

Solution. Let’s find the derivatives

y ′(t ) = 2t − 6t 2 , x′(t ) = 2 + 2t ,
2t − 6t 2 t − 3t 2
then y ′x = = .
2 + 2t 1+ t

⎧⎪ y = arctg (t )
Example 9. Find y ′x , if ⎨
⎪⎩ x = ln 1 + t (2 .
)
1 2t
Soluton. We have y ′ = , x ′(t ) = . Then
1+ t2 1+ t2

1 1+ t2 1
y′ = ⋅ = .
1 + t 2 2t 2t

214
x 2 (3 x − 2 )e x
Example 10. Find the derivative of function y = .
sin x ⋅ 2 x + 1
Solution. We use logarithmic differentiation and receive:

⎛ x 2 (3x − 2)e x ⎞
ln y = ln⎜⎜ ⎟ ; ln y = 2 ln x + ln (3 x − 2) + x − ln sin x − 1 ln (2 x + 1) ;

⎝ sin x ⋅ 2 x + 1 ⎠ 2
1
ln y = 2 ln x + ln (3 x − 2) + x − ln sin x −
ln (2 x + 1) ;
2
1 2 3 1 1
y′ = + +1− cos x − ;
y x 3x − 2 sin x 2x +1
⎛2 3 1 1 ⎞
y′ = y ⎜ + +1− cos x − ,
⎝ x 3x − 2 sin x 2 x + 1 ⎟⎠
or
x 2 (3 x − 2)e x ⎛ 2 3 1 ⎞
y′ = ⎜ + + 1 − ctg x − .
sin x 2 x + 1 ⎝ x 3x − 2 2 x + 1 ⎟⎠

Example 11. Find the derivative of function y = x cos x .


Solution. First method. Let’s use logarithmic differentiation. We have
ln y = ln x cos x ; ln y = cos x ln x; (ln y )′ = (cos x ln x)′;
y′
= − sin x ln x + cos x ; y ′ = y ⎛⎜ − sin x ln x + cos x ⎞⎟ .
1 1
y x ⎝ x⎠
That is,

y ′ = x cos x ⎛⎜ − sin x ln x + cos x ⎞⎟ .


1
⎝ x⎠

Second method. Using basic logarithmic equality a log a b = b, let's write


the given function as

( )
cos x
y = x cos x = eln x = ecos x ln x .
Then

(
y ′ = ecos x ln x ) = ecos x ln x (cos x ln x)′ =

= ecos x ln x ⎛⎜ − sin x ln x + cos x ⎞⎟ = x cos x ⎛⎜ − sin x ln x + cos x ⎞⎟ .


1 1
⎝ x⎠ ⎝ x⎠

215
Micromodule 18
CLASS AND HOME ASSIGNMENT

dy
Find using the rule of differentiation of inverse function, if:
dx
1. x = y 2 + y 2 + 1 . 2. x = y ln y + sin y.
3. x = lg cos y + cos ln y. 4. x = earccos y .

Find the derivative y ′ of implicit functions:

y
5. 3x+ y = 3x − 3 y . 6. arctg= x2 + y 2 . 7. x y = y x .
x
x2 y2
8. x3 + y 3 = 3 xy . 9. 2 + 2 = 1 .
a b

Find the derivative y ′x of functions given parametrically:

10. x = a cos 2 t , y = b sin 2 t . 11. x = t , y = 3 t .


3at 3at 2
12. x = 3
, y= 3
. 13. x = et , y = e 2t .
1+ t 1+ t
14. x = et cos t , y = et sin t . 15. x = a(t − sin t ) , y = a(1− cos t ) .
3
t
16. x = t 2 , y = −t . 17. x = e 2t cos 2 t , y = e 2t sin 2 t .
3
cos3 t sin 3 t
18. x = t cos t , y = t sin t . 19. x = ,y= .
cos 2t cos 2t

Find the derivative y ′ , using logarithmic differentiation:

2 x tg x ⋅ 5 x
20. y = (ln x) x . 21. y = (2 x + 1) 2 x−1 . 22. y = .
3x − 4

23. y =
( x − 4) ⋅ x 3 + 3 . 24. y = ( x5 + 5 x )arctg x .
(x − 2)3 ⋅ x
25. y = x ln x + (ln x) x . 26. y = (sin x)cos x (cos x)sin xx .
x
27. y = ( x5 + 5 x )arctg x . 28. y = x x .

216
Answers

y2 + 1 1 3x (1 − 3y ) x x2 + y2 + y ln y − y / x
1. . 2. . 5. y . 6. . 7. .
y(2 y 2 + 1 + 1) ln y + 1 + cos y 3 (1 + 3 )x 2
x− y x + y 2 ln x − x / y

y − x2 b2 x 2 t (2 − t 3 ) sin t + cos t
8. . 9. − 2 . 10. −b / a . 11. 6 . 12. . 13 2e t . 14. .
2
y −x a y 3 t 1 − 2t 3 cos t − sin t
t 2sin 2 t + sin 2t
15 ctg . 16. .
2 2cos 2 t − sin 2t

Micromodule 18
SELF-TEST ASSIGNMENT
dy
18.1. Find the derivative of implicit function.
dx
18.1.1. x 2 y + y 2 x = x3 y 3 . 18.1.2. y = arctg x − arctg y .
18.1.3. sin( xy ) = x 2 + y 2 . 18.1.4. y cos x = sin( x − y ) .
x y x+ y
18.1.5. 3 + 3 = 3 . 18.1.6. x3 + y 3 − 4axy = 0 .
18.1.7. ln( x + y ) + x 2 y = 1 . 18.1.8. x sin y = x 2 + y 2 .
18.1.9. x = y 3 − 4 y + 1 . 18.1.10. sin x − cos y = x − y .
18.1.11. cos( xy ) + sin( xy ) = y . 18.1.12. ctg 2 y = 2 ctg x .
18.1.13. y 3 + x3 y + xy 2 = 1 . 18.1.14. x 4 + y 4 = x 3 y 3 .
18.1.15. y = x − arcsin y . 18.1.16. x3 y + y 3 x = x − y 2 .
18.1.17. x 2 y 2 + 2 xy + x3 = y 3 . 18.1.18. sin( x + y ) = x − y .
3 4 3
18.1.19. x + y x = x − 2 y . 18.1.20. x tg y − y tg x = yx .
3 3 3
18.1.21. x y − y x = ( x − y ) . 18.1.22. arctg( x + y ) = x − y 2 .
18.1.23. 5 x − 5 y = 5 x+ y . 18.1.24. y sin x + x sin y = y .
2
18.1.25. 3 y ln y = x ( y + 5) . 18.1.26. y 3 − 5 y + 6ax = 0 .
18.1.27. x3 y 2 + 2 x− y = y . 18.1.28. 3x+ y + 3x− y = y 3 .
18.1.29. y = x + e1+ xy . 18.1.30. arcsin( x / y ) + yx = y .

dy
18.2. Find the derivative of function given parametrically.
dx
18.2.1. y = arccos t , x = arcsin t. 18.2.2. y = 1 / cos 2 t , x = ln tg t.
18.2.3. y = b sin 3 t , x = a cos3 t. 18.2.4. y = 1 / sin 2 t , x = ln ctg t.

217
1− t 2 2
18.2.5. y = ln 2
, x = arctg t 2 . 18.2.6. y = e−t , x = e−t .
1+ t
t
18.2.7. y = , x = (arcsin t )2 . 18.2.8. y = et sin t , x = et cos t .
2
1− t
1 2at 2 bt
18.2.9. y = , x = ln cos t . 18.2.10. y = 3
, x= .
sin t 2
1+ t 1+ t3
18.2.11. y = a (1− cos 2t ) , x = a(2t − sin 2t ) .
18.2.12. y = a (sin t − t cos t ) , x = a (t sin t + cos t ) .

18.2.13. y = arcsin(t −1) , x = 2t − t 2 .

18.2.14. y = t t 2 −1 , x = ln(t + t 2 −1) .


18.2.15. y = 3 ln tg t , x = arctg t . 18.2.16. y = t − arctg t , x = ln ctg t .
cos t tg t ctg 2t
18.2.17. y = 2
, x = (1 + cos t ) 2 . 18.2.18. y = 2
,x= .
sin t 1− t 3
2
1 + 1− t 2 1 at t
18.2.19. y = ln , x = 2 . 18.2.20. y = , x= .
t ln t 1+ t 2
1+ t 2
18.2.21. y = 2sin 2 t + sin 2t , x = 3 tg 2t .
18.2.22. y = ln ctg et , x = tg(2e −t ) .

18.2.23. y = arcsin 1 − t 2 , x = arctg(t 2 − 1) .

18.2.24. y = 1 + t 2 −1 , x = 3 1− ln t .
1 t
18.2.25. y = arccos , x = arcsin .
1+ t 2 1+ t 2
1 1
18.2.26. y = t − arctg , x = t 3 − arcctg 2 .
t t
18.2.27. y = (2 + 3 ln t ) / t , x = a cos3 t .

18.2.28. y = arctg 1 + t 2 , x = arccos1/ t 2 .


18.2.29. y = tg 3 2t + ctg 3 2t , x = cos 2t − sin 2t .

18.2.30. y = arcsin 1 − t , x = arccos 1− t 2 .

218
dy
18.3. Find the derivative of functions using the rule of logarithmic diffe-
dx
rentiation.
18.3.1. y = x arcsin x . 18.3.2. y = (lg x) x / 2 .
18.3.3. y = ( x3 + 1)sin x . 18.3.4. y = (cos 2 x)ln tg x / 2 .
x
18.3.5. y = (sin x )1/ x . 18.3.6. y = x e .
18.3.7. y = (ctg 5 x)5 x−1 . 18.3.8. y = ( x5 + 1)ctg x .
− tg x
18.3.9. y = x e . 18.3.10. y = ( x8 + 1) tg x .
3x (2 x − 1)( x + 1) 4
18.3.11. y = (x − 5) ⋅ x + 1 .
2 3 2
18.3.12. y = .
e (x + 2)
x 5
x ( x − 3)6
2 x ( x − 5)( x + 1)3
18.3.13. y = (x − 1) ⋅ x + x .
3 4 2
18.3.14. y = .
4 (3x − 2)
x 3
x (4 x − 3)5
2 +1
18.3.15. y = ( x3 − x) x . 18.3.16. y = (2 x − 3)cos x .
2
18.3.17. y = x arctg x . 18.3.18. y = ( x sin x) x .
x
18.3.19. y = ( x cos x)ln x . 18.3.20. y = x 2 .
18.3.21. y = (tg x )ctg x . 18.3.22. y = (arcsin x)sin x .
x
18.3.23. y = (cos x) tg x . 18.3.24. y = x 4 .
2x
18.3.25. y = (4 x − 3)arccos x . 18.3.26. y = (ln( x + 1))ln .
ctg x sin x
18.3.27. y = (ctg 2 x) . 18.3.28. y = x + (sin x) x .
18.3.29. y = (5 x + 2)sin x . 18.3.30. y = x arctg x
.

Micromodule 19
BASIC THEORETICAL INFORMATION
DIFFERENTIAL OF FUNCTION. TANGENT

Differential of function. Geometrical interpretation of differential. The


application of differentials in calculus of approximation. Tangent and
normal.

Literature: [3, chapter 3, §§ 3.1—3.8], [4, section 4], [6, chapter 5, § 3],
[7, chapter 6, § 17], [9], [10, chapter 4, § 14], [11, chapter 4, § 3], [12,
chapter 3, §§ 20, 26].

219
19.1. Definition and geometrical interpretation of differential

Let function y = f ( x) be differentiable at the point x, so that it has the


derivative at this point:
Δy
f ' ( x) = lim .
Δx →0 Δx
In general case f ' ( x) ≠ 0 . Then
Δy
= f ' ( x) + α , where α → 0 if Δx → 0 ,
Δx
therefore the increment of the function is
Δy = f '( x)Δx + α ⋅ Δx.
The first term is linear with respect to Δx , the second one is infinitesimal
of the higher order than Δx . Therefore the first term forms the main part of the
increment of function, which is called the differential of function.
Definition 3.21. The main, linear with respect to Δx , part of the increment
of function f ( x) is called the differential dy of function y = f ( x) :

dy = f ′( x)Δx .

Differential dy is also termed as the first-order differential or the first


differential.
If y = x then dy = dx = x' Δx = Δx , so the differential of the independent
variable x is equal to its increment. Therefore:

dy = f ′( x)dx .

The geometrical interpretation of the differential is clear from Fig.3.16.


у
P
f(x + Δx) •
Q
y = f(x)
Δу
M dу
f(x) •Δxα
N

α A
О x x + Δx х

Fig. 3.16

220
We have

PN = Δy , QN = MN ⋅ tg α = Δx ⋅ f ′( x) = f ′( x) dx = dy.

So, the differential of function f (x) under given values of х and Δx equals
the increment QN of secant MQ ordinate, which is drawn to the curve y = f ( x) at
point M , when argument receives increment Δ x.

19.2. General properties of the differential

Let u ( x ) , v( x) be differentiable functions. Then the following equalities


are true:

1. dC = 0 ( C = const ). 2. d (u + v) = du + dv .
3. d (uv) = udv + vdu . 4. d (Cu ) = Cdu .
u vdu − udv
5. d ( ) = , v ≠ 0. 6. df (u ) = f '(u )du , u = u ( x).
v v2

19.3. The application of differentials in calculus


of approximation and in error theory

At small values of Δx the following formula is true:

Δy ≈ dy

or

f ( x + Δx) ≈ f ( x) + f ' ( x)Δx

Let y = f ( x) , and the value х is defined approximately, in other words with


some absolute error Δx , then a value of function у will have its own absolute
error Δy too. The relative error of computation of the value of function y can be
approximately calculated with the help of differential:
Δy dy
≈ .
y y

221
19.4. Tangent and normal

We remember that tangent equation, constructed to the graph of function


y = f ( x) at point P0 ( x0 , y0 ) is the following:

y − y0 = f ′( x0 )( x − x0 ) , (3.10)

where y0 = f ( x0 ) . Equation of normal is


1
y − y0 = − ( x − x0 ) . (3.11)
f ' ( x0 )

Micromodule 19
EXAMPLES OF PROBLEMS SOLUTION

Example 1. Find the differential of function y = x 2 + 2 x at point x = 2 .


Solution. The first method. As the differential is the main, linear with respect
to Δx , part of increment of function at point x , let’s find the increment of given
function at point x = 2 . Therefore

Δy = y (2 + Δx) − y (2) = (2 + Δx) 2 + 2(2 + Δx) − 8 = 6Δx + (Δx) 2 .

The linear part of increment is the expression 6Δx . Finally, dy (2) = 6Δx .
The second method. As dy = f ' ( x)dx , then we have

y ' = 2 x + 2 ; y ' (2) = 6 ; dy (2) = 6dx.

Example 2. Find the differential of function y = 1 − x 2 + ln sin x.


Solution. We have

dy = d ( 1− x 2 + ln sin x) = ( 1 − x 2 + ln sin x)' dx =


−x
= ( + ctg x)dx .
1 − x2

Example 3. Calculate approximate value ( with the help of differential) of 15 .


Solution. Let’s consider a function f ( x) = x . Let x = 16 , x + Δx = 15 ,
1 1 31
Δx = −1 . Then x + Δx ≈ x + Δx , or 15 ≈ 4 + (−1) = = 3.875 .
2 x 2⋅4 8

222
Example 4. Calculate approximate value of tg 54° .
Solution. Let’s consider a function f ( x) = tg x . Then we receive:

1
tg( x + Δx) ≈ tg x + (tg x) ′Δx ; tg( x + Δx) ≈ tg x + ⋅ Δx .
cos 2 x

54π 3π
Let’s convert degrees to radians: 54° =
= .
180 10
π 3π 3π π π
Let x = , x + Δx = , Δx = − = , then
4 10 10 4 20

3π π 1 π π
tg ≈ tg + ⋅ = 1 + ≈ 1,314 .
10 4 π 20 10
cos 2
4
Finally, tg 54° ≈ 1,314.
Example 5. Find tangent equation and equation of normal, drawn to the
curve y = x ln x + 1 at point with ordinate 1.
Solution. By conditions y0 = 1 . Let’s find an abscissa of touching point.
We have
1 = x ln x + 1 , x ln x = 0 ; x ≠ 0 . Therefore ln x = 0 , then x0 = 1 .

Let’s find derivative


y ′ = ( x ln x + 1) ′ = ln x + (ln x) ′ ⋅ x = ln x + 1 .

Then y ′(1) = ln1 + 1 = 1 . Substituting values of x0 , y0 and y ′( x0 ) in


formula of tangent (3.10) and normal (3.11), we will obtain tangent equation
1
y − 1 = 1 ⋅ ( x − 1) , then y = x , and equation of normal y − 1 = − ( x − 1) , or
1
y = 2 − x.
Example 6. Find an angle between the tangent, constructed to the curve
y = x3 − 2 x 2 − 5 x at point x = 1 , and positive direction of Ox .
Solution. Let’s find derivative y ′ = 3 x 2 − 4 x − 5 . Then y ′(2) = 12 − 8 − 5 = −1 .
According to the geometrical interpretation of derivative, solve the equation

tg α = −1 . We obtain α = (let’s point out that 0 ≤ α < π ).
4

223
Example 7. Write the tangent equation and equation of normal to the curve
x + 2 y 3 = 5 xy at point M (2; 1) .
3

Solution. Substituting coordinates of point M in equation of the curve you


will obtain right equality 10=10. Then point М will belong to this curve and
x0 = 2, y0 = 1 .
Let’s find derivative of function y at point x0 . We have:

( x3 + 2 y 3 )′ = (5 xy )′ , 3x 2 + 6 y 2 y ' = 5( y + xy ') , (6 y 2 − 5 x) y ' = 5 y − 3x 2 ,


5 y − 3x 3 5 ⋅1 − 3 ⋅ 8 19
y' = , y ' ( 2) = = .
2
6 y − 5x 6 ⋅1 − 5 ⋅ 2 4
Then:
19
y −1 = ( x − 2) , or 19 x − 4 y − 34 = 0 is equation of tangent,
4
4
y − 1 = − ( x − 2) , or 4 x + 19 y − 27 = 0 is equation of normal.
19

Example 8. Compose tangent equation for an ellipse, given by equations


x = 2 sin t , y = 3cos t , at point M 0 (1; y0 ) , where y0 > 0.
Solution. Equation of tangent is y − y 0 = f ' ( x 0 )( x − x 0 ) . By conditions
π 5π
x0 = 1 . Let’s find an ordinate of touching point y0 . t = and t = are
6 6
solutions of the equation 1 = 2 sin t if t ∈ [ 0; 2π ) . Taking into account the

π π 3
condition y0 > 0, that is, cos t > 0, we obtain t = . Then y0 = cos = .
6 6 2
Next we shall find the derivative f ′( x0 ) of the parametrically given function:

y ' (t ) = −3 sin t , y '(t ) = 2 cos t.


Therefore
−3 sin t 3 3 π 3
y x' = = − tgt ; f ' (1) = − tg = − .
2 cos t 2 2 6 2
Tangent equation is
3 3
y− =− ( x − 1) ,
2 2
3
or y=− x + 3.
2

224
Micromodule 19
CLASS AND HOME ASSIGNMENT

1. Find the increment and the differential of the function y = x 2 − 4 x + 3 , if:


а) x = 1 , Δx = 0,1 ; b) x = 3 , Δx = 0,05 .
Find the differential of a function:
2. y = (4 − x 2 )2 x . 3. y = sin x + tg 2 x . 4. y = ln arcsin x .
arccos x 1
5. y = x 2 x+1 . 6. y = + arctg . 7. y = 4 x + x log 2 tg x .
x2 x
Find the differential of an implicit function at the point M 0 ( x0 ; y0 ) :
8. x3 + y 3 + 3xy −15 = 0 , M 0 (1; 2) .
x
9. ln x 2 + y 2 = arctg , M 0 (0; 1) .
y
Calculate an approximate value with the help of the differential:
10. 3 131 . 11. (0, 95)6 . 12. sin 9° . 13. cos155° .
14. arctg1, 05 . 15. arccos 0, 02 . 16. ctg 85° .
Compose an equation of a tangent line and a normal to a curve, which is
given by a parametric equation:
π
17. x = t − sin t , y = 1− cos t , at the point t = .
2
π
18. x = cos3 t , y = sin 3 t , at the point t = .
4
π
19. x = 2 cos t − cos 2t , y = 2 sin t − sin 2t , at the point t = .
2
t −t
20. x = te , y = te , at the point x0 = e .
Compose an equation of a tangent line and a normal to a curve at point М:
21. 4 x 4 + 6 xy − y 4 = 0 , M (1; 2) .
22. x 2 (2 x − y ) = 2 x − y 3 , M (1; 1) .
23. x 2 / 3 + y 2 / 3 = 8 , M (8; 8) .

Answers
1. a) Δy = −0,19 , dy = −0, 2 ; b) dy = 0,1 , Δy = 0,1025. 2. 2 x [(4 − x 2 ) ln 2 − 2 x]dx .
5. x 2 x (2 x ln x + 2 x + 1) dx . 8. −3dx / 5 . 9. dx. 10. 5,08. 11. 0,7. 12. 0,157. 13. – 0,908.
14. 0,81. 15. 1,55. 16. 0,087. 17. y = x + 2 − π / 2 ; y = − x + π / 2 . 18. y + x = 2 / 2 ;

225
y = x . 19. y + x = 3 ; y = x + 1 . 20. y = 1 / e ; x = e . 21. 14 x − 13 y + 12 = 0 ;
13x + 14 y − 41 = 0 . 22. y = x ; y = − x + 2 . 23. y + x = 16; y = x .

Micromodule 19
SELF-TEST ASSIGNMENT

19.1. Calculate an approximate value with the help of the first differential:
19.1.1. cos 61° . 19.1.2. e0,2 . 19.1.3. sin 33° .
3
19.1.4. arctg1, 05 . 19.1.5. 120 . 19.1.6. 340 .
3 5 6
19.1.7. 66 . 19.1.8. 33 . 19.1.9. 70 .
19.1.10. cos 85° . 19.1.11. sin 8° . 9.1.12. sin 28° .
19.1.13. arctg 0, 95 . 19.1.14. arctg 0, 9 . 19.1.15. e0,3 .
19.1.16. ln 1, 05 . 19.1.17. ln 0, 97 . 19.1.18. ln 1, 08 .
19.1.19. tg 47° . 19.1.20. ctg 50° . 19.1.21. (1, 02)5 .
19.1.22. arccos 0, 45 . 19.1.23. arcsin 0, 52 . 19.1.24. (1, 97)6 .
19.1.25. (2, 04) 4 . 19.1.26. ln tg 48° . 19.1.27. ln tg 43° .
19.1.28. cos 86° . 19.1.29. sin 26° . 19.1.30. tg 40° .

19.2. Solve the tasks for composition of an equation of the normal and the
tangent to a curve.
19.2.1. Write the equation of a normal and a tangent to the curve
1 1 5
y = x3 − x 2 + at the points, where the slope of the tangent is equal to 2.
3 2 6
2
x2 y
19.2.2. Write the equation of a normal and a tangent to the ellipse + =1
9 4
1
at the points, where the slope of the normal line is equal to .
2
19.2.3. Write the equation of a normal and a tangent to the ellipse
x = 3cos t , y = 2sin t at the points, where the tangent is parallel to the line
2
y =− x+4.
3
19.2.4. Write the equation of a normal and a tangent to the curve
y = 3 x 3 + 5 x 2 + 5 x + 4 at the points, where the tangents are parallel to the line
y = 4x − 2 .

226
19.2.5. Write the equation of a normal and a tangent to the curve
2 2
y = x3 + 5 x 2 + 9 x + at the points, where the tangents are parallel to the
3 3
line y = x − 2 .
19.2.6. Write the equation of a normal and a tangent to the curve
1 1
y = x3 + 7 x 2 + 15 x + at the points, where the slope of the tangent line is
3 3
equal to 2.
19.2.7. Write the equation of a normal and a tangent to the curve
8
y = x3 + 7 x 2 + 11x + where the tangent is parallel to the line y = 3 x − 4 .
27
19.2.8. Write the equation of a normal and a tangent to the curve
5 8
y = x3 + 11x 2 + 13 x + at the points, where the slope of the normal line is
3 75
1
equal to − .
5
19.2.9. Write the equation of a normal and a tangent to the curve
4 4
y = x3 + 5 x 2 + 3 x + at the points, where tangents and the x-axis form an
3 3
angle of 1350 .
19.2.10. Write the equation of a normal and a tangent to the curve
1
= x + 6 x 2 + x + 5 at the points, where the slope of the normal line is equal
3
2
1
to .
2
19.2.11. Write the equation of a normal and a tangent to the curve
1
y = x3 + 7 x 2 + 10 x + 130 at the points, where the tangents are parallel to the
3
line y = −3 x + 2 .
19.2.12. Write the equation of a normal and a tangent to the curve
2 8
y = x3 + 6 x 2 + x + at the points, where the tangents are parallel to the line
3 27
y = −3 x + 1 .
19.2.13. Write the equation of a normal and a tangent to the curve
1
y = 3x3 + 6 x 2 − x + at the points, where the slope of the tangent lines is equal
9
to −4 .
19.2.14. Write the equation of a normal and a tangent to the curve
25
y = 3x 2 + 6 x 2 − 6 x + at the points, where the tangents and the x- axis form
9
0
the angle 135 .

227
19.2.15. Write the equation of a normal and a tangent to the line
x 2 − 3x + 6
y= at the point with the abscissa x0 = 3 .
x2
19.2.16. Write the equation of a normal and a tangent to the curve
y = ( x + 1)3 3 − x at the point with the abscissa x0 = 2 .
19.2.17. Write the equation of a normal and a tangent to the
π
astroid x = 2 2 cos3 t , y = 2 2 sin 3 t , if t = .
4
19.2.18 Write the equation of a normal and a tangent to the cycloid
π
x = 2(t − sin t ), y = 2(1− cos t ) , if t = .
2
19.2.19. Write the equation of a normal and a tangent to the parabola
2
y − y + 2 x − 4 = 0 at the point with the abscissa x0 = −4 .
19.2.20. Write the equation of a normal and a tangent to the curve
x + 2 xy + 2 y 4 = 5 at the point M 0 (1; 1) .
2

19.2.21. Write the equation of a normal and a tangent to the cycloid



x = 3(t − sin t ), y = 3(1− cos t ) , if t = .
2
19.2.22. Write the equation of a normal and a tangent to the curve
7
y = 3x3 − 6 x 2 − 6 x + at the points, where the tangents and the x- axis form
9
0
the angle 135 .
19.2.23. Write the equation of a normal and a tangent to the curve
y = 3x3 − 6 x 2 − x + 1 at the points, where the tangents are parallel to the line
y = −4 x + 5 .
19.2.24. Write the equation of a normal and a tangent to the curve
x 4 + 3xy 2 + 3 y 4 = 1 at the point M 0 (−1; 1) .
19.2.25. Write the equation of a normal and a tangent to the curve
2 4
y = x3 − 5 x 2 + 9 x + at the points, where the tangents are parallel to the
3 3
bisectrix of the first coordinate quarter (angle).
19.2.26. Write the equation of a normal and a tangent to the curve
1 1
y = x3 − 3 x 2 + 6 x − at the points, where the tangents are parallel to the
3 3
line y = −2 x + 3 .
19.2.27. Write the equation of a normal and a tangent to the
curve x = t 2 , y = t 3 , if t = 2 .

228
19.2.28. Write the equation of a normal and a tangent to the curve
4 1
y = x3 − 5 x 2 + 3 x + at the points, where the tangents are parallel to the
3 3
bisectrix of the second coordinate quarter (angle).
19.2.29. Write the equation of a normal and a tangent to the curve
x3 − 3 xy 2 + y 3 = −3 at the point M 0 (1; 2) .
19.2.30. Write the equation of a normal and a tangent to the curve
1
y = x3 − 5 x 2 + x − at the points, where the normal is parallel to the line
27
1
y = x +1 .
2

Micromodule 20
BASIC THEORETICAL INFORMATION.
HIGHER ORDER DERIVATIVES AND DIFFERENTIALS

Higher order derivatives of functions given explicitly, implicitly, or


parametrically. Leibniz's formula. Higher order differentials.

Literature: [3, chapter 4, items 4.1—4.2], [4, part 5], [6, chapter 5, § 4],
[7, chapter 6, § 18], [9], [10, chapter 4, § 12, 20], [12, chapter 3, §§ 22—24].

20.1. Higher order derivatives

Suppose differentiable function y = f ( x) is defined on an interval (a; b),


then its derivative f ′( x) , called also the first-order derivative or the first
derivative, is a function of x too. The function f ′( x) may have derivative at the
interval (a; b) too.
Definition 3.22. The later derivative is called the second derivative (or the
second-order derivative).
It is denoted with one of the following symbols:
d 2 y d 2 f d ⎛ dy ⎞
y '' , f '' ( x), 2 , 2 , ⎜ ⎟ .
dx dx dx ⎝ dx ⎠
The second derivative has the following mechanical matter. If a mass point
occurs in accordance with the law S = f (t ) , then derivative S ′ is a velocity at
the given time point and S ′′ is an acceleration in the same time point t.
Definition 3.23. Derivative of the second-order derivative, if it is existed, is
called the third-order derivative.

229
d ⎛d y⎞
2
So, according to definition we write y ′′′ = ⎜ ⎟.
dx ⎜⎝ dx 2 ⎟⎠
The first derivative of the (n – 1)-th derivative, if it is existed, is called the n-
th derivative:

d ⎛ d n −1 y ⎞′
y (n)
= (y ( n −1)
) ′ , or y (n) = ⎜⎜ n −1 ⎟⎟
dx ⎝ dx ⎠

Derivatives having the order higher than one are called the higher order
derivatives.

20.2 Leibniz’s formula

Suppose y = uv , where u(x) and v(x) are the n times differentiated functions.
Then

n
(uv)( n ) = ∑ Cnk u ( n −k ) v( k ) ,
k =0

k n!
where Cn =
(n − k)!k! , n ! = 1 ⋅ 2 ⋅ 3 ⋅ k ⋅ ( n − 1) ⋅ n , 0 ! = 1 .
Particularly,
( u v ) ' = u ' v + u v ' (n = 1) ,
( u v ) ′′ = u ′′v + 2 u ′v ′ + u v ′′ (n = 2),
( u v ) ′′ = u ′′′v + 3 u ′′v ′ + 3 u ′v ′′ + u v ′′′ (n = 3).

20.3. Calculation of the higher order derivatives


of parametrically given functions

If some function is given parametrically by equations x = x(t ) , y = y (t ) ,


then derivatives are calculated by the following formulas:

⎛ dy ⎞′
dy yt′ d y ⎜⎝ dx ⎟⎠
2
= , = ,
dx xt′ dx 2 xt′

230
⎛ d 2 y ⎞′ ⎛ d n −1 y ⎞′
3 ⎜ 2 ⎟ n ⎜ n −1 ⎟
d y
= ⎝ d x ⎠ , d y = ⎝ dx ⎠ .
d x 3 x t′ dxn x t′

20.4. Higher order derivatives of implicit function


Assume that a function y = f ( x) is given implicitly with equality F ( x, y ) = 0.
Executing differentiation with respect to x and solving the obtained equation
relatively to y ′ it is possible to determine the first derivative.
To find the second derivative it is necessary to differentiate the first
derivative with respect to x and to put its values into obtained relation. While
continuing differentiation the any order derivatives can be determined one after
another. All of them will be expressed through the independent variable x and
the function y itself.

20.4. Higher order differentials

Definition 3.24. The differential of the first-order differential of function


f ( x) is called the second-order differential of twice differentiable function
y = f ( x) i.e.
d 2 y = d (dy ).
In general case
d n y = d (d n −1 y).

The differentials of the function y = f ( x) are calculated by the formulas:

d 2 y = f ′′( x)(dx) 2 , d 3 y = f ′′′( x)(dx )3 , … , d n y = f ( n ) ( x)(dx) n .


If x is some function of a variable t, then
d 2 y = f ′′( x)(dx) 2 + f ′( x) x ′′(t )(dt ) 2
and so on. Thus the higher order differentials don’t have invariance property.

Micromodule 20
EXAMPLES OF PROBLEMS SOLUTION

Example 1. Determine the third-order derivative of the function y = x 4 −


− 2 x 2 + 3 x − 5.

231
Solution. Let’s find the third-order derivative step by step
y ′ = 4 x 3 − 4 x + 3 , y ′′ = 12 x 2 − 4 , y ′′′ = 24 x .
Example 2. Determine the second-order derivative of the function

x a2
y= x2 − a2 − ln( x + x 2 − a 2 ) , a = const .
2 2
Solution. We receive

y' =
1⎛ '
⎜x
2⎝
x2 − a2 + x ( x 2 − a 2 ⎞⎟ −
'


a2
2
)
ln x + ( ( x2 − a2 )) =
'

=
1
2


⎜ x2 − a2 + x
x −a
1
2 2
⎞ a2
2 x ⎟⎟ −
2 x+
1
x2 − a2
(x + (
'
x2 − a2 )) =
⎝ 2 ⎠
1⎛ x2 ⎞ a2 ⎛ ⎞ x
( )
= ⎜ x2 − a2 + ⎟− ⎜1 + ⎟=
2 ⎜⎝ ⎟ ⎜ ⎟
x2 − a2 ⎠ 2 x + x2 − a2 ⎝ x −a ⎠
2 2

x2 − a2 + x2 a 2 ( x 2 − a 2 + x)
=
2 x2 − a2

(
2x+ x2 − a2 ) x2 − a2
=

2 x2 − a2 a2 x2 − a2
= − = = x2 − a2 ;
2 2 2 2 2 2
2 x −a 2 x −a x −a

y '' = ( x2 − a2 )=2
' 1
x2 − a2
(x 2
− a2 ) '
=
1
2 x2 − a2
2x =
x
x2 − a2
.

Example 3. Determine the n-order derivative of the functions:


1
а) y = e x ; b) y = a x ; c) y = sin x ; d) y = .
x
Solution.
а) y ′ = e x , y ′′ = e x , y ( n ) = e x ;
b) y ′ = a x ln a , y ′′ = a x ln 2 a , y ( n ) = a x ln n a ;
π π
c) y ′ = cos x = sin( x + ) , y ′′ = − sin x = sin( x + 2 ) ,
2 2
π π
y ′′′ = − cos x = sin( x + 3 ) , y ( n ) = sin( x + n ) ;
2 2
d) y ′ = − x −2 , y ′′ = 2 x −3 , y ′′′ = −2 ⋅ 3 x −4 , y (4) = 2 ⋅ 3 ⋅ 4 x −5 , …,
y ( n ) = (−1) n 2 ⋅ 3 ⋅ 4 ⋅… ⋅ nx − ( n +1) = (−1)n n ! x − ( n +1) .

232
Example 4. Determine the n-order derivative of the function
3x +1
y= 2
.
x + 2x − 3
Solution. Let’s decompose this function into partial fractions:
3x +1 3x + 1 A B A( x + 3) + B( x − 1)
= = + = .
2
x + 2x − 3 ( x −1)( x + 3) x −1 x + 3 ( x − 1)( x + 3)
So,
3x + 1 = A( x + 3) + B ( x −1)
Suppose x = 1, then 4 = 4A, whence A = 1. If x = – 3, then – 8 = – 4B, B = 2.
Thus
1 2
y= + .
x −1 x + 3
Finally according to case d) of the previous example, we obtain
1 1
y ( n ) = (−1) n n! n +1
+ 2(−1) n n ! .
( x − 1) ( x + 3) n+1

Example 5. Determine the fifth-order derivative y(5) of the function

y = ( x 2 − 3 x + 4)e 2 x .

Solution. Let’s designate u = e2x , v = x 2 − 3x + 4 . Applying Leibniz's


formula we can obtaine
y (5) = (( x 2 − 3 x + 4)e 2 x )(5) = (e 2 x )(5) ( x 2 − 3 x + 4) +
+ C51 (e2 x )(4) (2 x − 3) + C52 (e2 x )(3) ⋅ 2 .

Here we consider that ( x 2 − 3 x + 4)( n ) = 0 if n > 2. Then we get

y (5) = 32e 2 x ( x 2 − 3 x + 4) + 5 ⋅ 16e 2 x (2 x − 3) + 10 ⋅ 8e 2 x ⋅ 2 =


= e 2 x [32( x 2 − 3 x + 4) + 80(2 x − 3) + 160] = e 2 x (32 x 2 + 64 x + 48) .
d2 y
Example 6. Find , if x = 2 cos t , y = 3 sin t .
dx 2
Solution. We have

dy (3 sin t )t′ 3 cos t 3


= = = − ctg t ;
dx (2 cos t )t′ −2 sin t 2

233
3 3
2 (−ctg t )t′
d y 2 3
= 2 = 2 sin t = − .
dx 2 (2 cos t )t′ −2 sin t 4 sin 3 t
Example 7. Find d3y, if y = cos 3 x .
Solution. Let’s determine derivatives
y ′ = −3sin 3 x , y ′′ = −9 cos 3x , y ′′′ = 27 sin 3x .
Therefore
d 3 y = 27 sin 3 x(dx)3 .
2
Example 8. Find d2y(0), if y = 4− x .
Solution. Let’s determine the second-order derivative in the point x = 0. We
obtain
2
y ′ = 4− x ln 4(−2 x) ,
2 2 2
y ′′ = −2 ln 4[(4− x ) ′ x + 4− x ] = −2 ln 4 ⋅ 4− x [−2 x 2 ln 4 + 1] .
Thus
d 2 y (0) = y ′′(0)dx 2 = −2 ln 4dx 2 .

Micromodule 20
CLASS AND HOME ASSIGNMENT

d2 y
Find :
dx 2
1. y = ( x3 − 2) 4 . 2. y = x 4 + 1 . 3. y = e x sin 2 x . 4. y = 4 x ( x + 1) .
x x x4
5. y = x . 6. y = ln tg x . 7. y = . 8. y = .
x2 + 1 2x −1
9. y = (ln x ) x . 10. y = sin 3 x . 11. y = 4 cos3 x − 3cos x .

Determine the n-order derivative of function:

12. y = 3− x . 13. y = ln x . 14. y = sin kx . 15. y = sin x sin 2 x .

xn 1 x−5
16. y = 2
− x n tg 2 x . 17. y = 2
. 18. y = 2
.
cos x x + 3x + 2 x − 4x + 3

234
Determine the n-order derivative of function applying Leibniz's formula:
19. y = ( x3 − 2 x + 5) sin x , n = 10 .
20. y = ( x 2 + 4 x − 3)2 x , n = 8 .
21. y = x ln( x 2 − 3x + 2) , n = 6 .

d2 y
Find of implicit function:
dx 2
22. x3 + y 3 = 3 xy . 23. e x + x = e y + y . 24. cos( x + y ) = x .
2
25. y = 2 px . 26. y = sin( x + y ) . 27. ln( x + y ) = y − x .
y
28. x y = y . 29. tg = y .
x

d2 y
Find of parametric function:
dx 2
30. y = 3t − t 3 , x = 2t − t 2 . 31. y = et sin t , x = et cos t .
32. y = ln t , x = t 6 . 33. y = t 2 / 2 , x = arctg t .
34. y = cos 2t , x = sin 2 t . 35. y = ln(1 + t 2 ) , x = arcctg t .

Answers

1. 12 x( x3 − 2) 2 (11x3 − 4). 2. 2 x 2 ( x 4 + 3)( x 4 + 1) −3 / 2 . 5. x x (ln x + 1)2 + x x −1.


6. −4cos 2 x / sin 2 2 x . 11. − 9cos3x . 12. (−1)n 3− x ln n 3 . 13. (−1) n −1 ( n − 1)!/ x n .
1
14. k n sin(kx + nπ / 2). 15. [cos( x + nπ / 2) − 3n cos(3 x + nπ / 2)] . 16. n! .
2
17. (−1) n n![1/( x + 1)n +1 − 1/( x + 2)n+1] . 18. (−1)n n![2/( x − 1)n+1 − 1/( x − 3)n +1] .
19. (30x + 700)cos x − ( x − 272x + 5)sin x. 20. 2 ln 2[ln2 2( x2 + 4x − 3) + 16ln 2( x + 2) + 42] .
2 3 x 6

24(6 x − 11) 24(6 x − 23) 2 xy (3 xy − x 3 − y 3 − 1) (e y − e x )(e x + y − 1)


21. + . 22. . 23. .
( x − 1)6 ( x − 3)6 ( y 2 − x) 3 (e y + 1) 3
x+ y
− cos
− cos( x + y ) p2 2 . 27.
24. . 25. − . 26. − 4( x + y ) /( x + y − 1) 3 .
3
sin ( x + y ) y 3 x+ y
4 sin
2
30. 3/(4(1 − t )) . 31. 2e −t /(cos t − sin t )3 . 32. − 1 /(6t ) . 33. 3t 4 + 4t 2 + 1 . 34. 0.
12

35. 2(1 + t 2 ) .

235
Micromodule 20
SELF-TEST ASSIGNMENT

20.1. Find d2y at point x0 .


20.1.1. y = x x − 3, x0 = 12. 20.1.2. y = x 2 ⋅ x − 5, x0 = 6.
2 2
20.1.3. y = x ⋅ 2 x + 3, x0 = 11. 20.1.4. y = (2 x − 1) ⋅ x + 2, x0 = 7.
20.1.5. y = (ln x ) ⋅ 2 x − 1, x0 = 5. 20.1.6. y = (ln x) ⋅ 2 x + 1, x0 = 12.
π π
20.1.7. y = sin 3 x ⋅ cos5 x, x0 = . 20.1.8. y = sin 2 x ⋅ cos 4 x, x0 = .
3 6
π π
20.1.9. y = sin 3 x ⋅ cos7 x, x0 = . 20.1.10. y = sin 4 x ⋅ cos6 x, x0 = .
4 6
π π
20.1.11. y = sin 3 x ⋅ tg 5 x, x0 = . 20.1.12. y = sin 2 x ⋅ tg 4 x, x0 = .
4 3
20.1.13. y = ( x − 1) ⋅ 3 5 x + 2, x0 = 5. 20.1.14. y = (2 x − 1) ⋅ 4 3 x + 4, x0 = 4.
2 3
20.1.15. y = ( x + 1) ⋅ 3 x + 5, x0 = 1. 20.1.16. y = (3 x − 1)3 ⋅ 4 7 x + 2, x0 = 2.
20.1.17. y = ( x − 1)5 ⋅ 3 2 x − 2 , x0 = 5. 20.1.18. y = (4 x − 1)3 ⋅ 5 x − 2, x0 = 3.
3
3x + 2 x+5
20.1.19. y = 5
, x0 = 2. 20.1.20. y = , x0 = 4.
x ( x − 2)3
5 4
x2 − 4 x2 − 9
20.1.21. y = , x0 = 6. 20.1.22. y = , x0 = 5.
x x2
sin 3 x π cos 4 x π
20.1.23. y = , x0 = . 20.1.24. y = , x0 = .
cos x 4 sin x 3
4
sin x π π
20.1.25. y = 2
, x0 = . 20.1.26. y = tg 4 x, x0 = .
cos x 4 6
5 π
20.1.27. y = ctg x, x0 = . 20.1.28. y = ( x + 1)3 ⋅ 3 x + 2, x0 = 6.
4
4
20.1.29. y = x ⋅ 2 x + 7 , x0 = 9. 20.1.30. y = (2 x − 3) 2 ⋅ x + 3, x0 = 1.

d2 y
20.2. Find of parametric function:
dx 2
20.2.1. x = 5(2t − sin 2t ), y = 10 sin 2 t.
1
20.2.2. x = , y = tg t − t.
cos t

236
1
20.2.3. x = , y = ctg t + t.
sin t
20.2.4. x = lg sin t , y = lg cos t.
20.2.5. x = sin lg t , y = tg lg t.
20.2.6. x = cos t + t sin t , y = sin t − t cos t.
20.2.7. x = cos 2t + 2t sin 2t , y = sin 2t − 2t cos 2t.
20.2.8. x = ln(1 + t 2 ), y = t − arctg t.
20.2.9. x = arcsin et , y = 1 − e2t .
20.2.10. x = sin et , y = cos et .
1
20.2.11. x = ln ctg t , y = .
sin 2t
t −1
20.2.12. x = ln t , y = .
t +1
20.2.13. x = ln(1 + t ), y = arctg t .
1
20.2.14. x = arctg et , y = 2t
.
e +1
20.2.15. x = ln(1 + 4t 2 ), y = 2t − arctg 2t.
20.2.16. x = sin 3 et , y = cos3 et .
20.2.17. x = 3t cos t , y = 3t sin t.
20.2.18. x = arc ctg t , y = log 3 (t 2 + 1).
20.2.19. x = cos 2t − ln ctg t , y = sin 2t.
20.2.20. x = tg 2t , y = ln cos3 2t .

20.2.21. x = arccos 2t , y = 1 − 4t 2 .

20.2.22. x = arcsin t , y = 1 − t 2 .
1 2 1
20.2.23. x = tg t , y = .
2 cos t
1
20.2.24. x = ctg 2 et , y = .
sin et
20.2.25. x = t 2 + 1, y = ln(t + t 2 + 1).
20.2.26. x = ln(1 + t 4 ), y = arctg(t 2 ).

237
1
20.2.27. x = ln ctg(1 + t ), y = .
sin(1 + t )
20.2.28. x = tg et , y = ln cos 2 et .
20.2.29. x = arctg t , y = log 2 (t 2 + 1).
20.2.30. x = ln(1 + t 6 ), y = arctg(t 3 ).

20.3. Determine the n-order derivative of function applying Leibniz's formula:


20.3.1. y = e 2 x −1 ( x3 − 3 x + 4) , n=10.
20.3.2. y = ( x3 − 2 x + 1) sin 2 x , n=12 .
20.3.3. y = 2 x (3x 3 − 7 x + 5) , n=15.
20.3.4. y = ( x3 + 2 x 2 + 3) ln x , n=8.
20.3.5. y = ( x 2 + 4 x − 5) ln x , n=10.
20.3.6. y = ( x 2 + 5 x − 12) ⋅ 2 x , n=9.
20.3.7. y = ( x3 + 5 x 2 − 2) sin x , n=11.
20.3.8. y = ( x3 − 2 x 2 − 3) cos x , n=9.
20.3.9. y = ( x 2 − 4 x 2 − x + 3) cos 2 x , n=8.
20.3.10. y = ( x3 + 2 x + 3) ln x , n=7.
20.3.11. y = ( x 2 − 5 x) ln( x + 1) , n=8.
20.3.12. y = (2 x 2 − 7 x) ln( x − 2) , n=10.
20.3.13. y = (4 x 2 − 9 x) ln( x − 2) , n=6.
20.3.14. y = (2 x 2 − 3x − 11) ⋅ 3x , n=9.
20.3.15. y = (4 x 2 − x + 8) ⋅ 4− x , n=10.
20.3.16. y = (2 x3 − 4 x 2 − 1) ⋅ 5− x , n=7.
20.3.17. y = ( x 2 + 3 x − 7) ⋅ 6− x , n=8.
20.3.18. y = e − x −1 ( x3 − x 2 + 2) , n=10.
20.3.19. y = 2− x −1 ( x3 − 6 x + 3) , n=7.
20.3.20. y = 3− x (2 x 2 + x + 3) , n=8.
20.3.21. y = (4 x 3 − x 2 − 1) cos 2 x , n=10.
20.3.22. y = (3 x 2 − 4 x + 1) cos( x + 1) , n=12.
20.3.23. y = (5 x 2 − 3 x + 2) sin( x − 2) , n=11.

238
20.3.24. y = (6 x 3 − 1) sin( x + 3) , n=15.
20.3.25. y = (2 x 3 − 4 x − 1) ln( x − 3) , n=10.
20.3.26. y = (3 x3 + 2 x + 3) ln( x + 3) , n=8.
20.3.27. y = ( x 2 − 2 x − 1) ln(2 x − 1) , n=11.
20.3.28. y = ( x3 + 2 x − 3) ln(2 x + 1) , n=10.
20.3.29. y = e −2 x ( x3 − 6) , n=8.
20.3.30. y = 2− x ( x3 − 4) , n=10.

Micromodule 21
BASIC THEORETICAL INFORMATION
BASIC THEOREMS OF DIFFERENTIAL CALCULUS

Fermat’s, Rolle’s, Lagrange’s, Cauchy’s theorems. Taylor’s, Maclaurin’s


formulas. L’Hospital’s rule.

Literature: [4, part 5], [6, chapter 5, §5], [7, chapter 6, §19], [9], [10,
chapter 4, §24], [11, chapter 4, §4], [12, chapter 4, §§1-7] .

21.1. Fermat’s, Rolle’s, Lagrange’s, Cauchy’s theorems


Those functions, which have the derivative on the current interval, are
distinguished by the properties , which help to investigate the function behavior
on the interval of differentiability.

Theorem 3.11 (Fermat’s theorem). Let the function f ( x) be continuous on


the interval (a; b) and reaches it’s minimum and maximum values at the point on
this interval. Therefore, if there’s a derivative f ′(c) at the point с, then f ′(c) = 0 .

This theorem has trivial geometric interpretation. If at the point x = c


function f ( x) reaches minimum or maximum values (Fig. 3.17 and 3.18), then
the tangent to the graph of this function at the point (c; f (c)) is parallel to the x-axis.

Theorem 3.12 (Rolle’s theorem). If the function f ( x) is continuous on the


segment [a; b] , has the derivative at the each point on the interval (a; b) and at
the ends of the interval reaches the same values f (a) = f (b) , then there’s at
least one point c ∈ (a; b) such as f ′(c) = 0 .

239
The geometric interpretation of this theorem is understood from the Fig. 3.19–3.21.
y y

О a с b x О a с b x

Fig. 3.17 Fig. 3.18


y y y

О a с b x О a с b x a с1 с2 b x

Fig. 3.19 Fig. 3.20 Fig. 3.21

Theorem 3.13 (Lagrange’s theorem). If the function f ( x) is continuous on


the segment [a; b] and differentiable at the interval (a; b) ,
then there’s at least one point on this interval c ∈ (a; b) , where the following
statement takes place
f (b) − f (a )
= f ′(c).
b−a

This formula also known as Lagrange’s formula. It can be transform


as: f ( x + Δx) − f ( x) = f ′( x + θΔx)Δx , 0 < θ < 1 .
The geometric interpretation of Lagrange’s theorem. If the function f ( x ) is
satisfied conditions of Lagrange’s theorem, then there’s at least one point on this
graph, where the tangent to the graph is parallel
y
В to the bisecant, which unites the ends of the curve
A(a; f (a )) and B (b; f (b)) (Fig. 3.22).
Some useful consequence follow Lagrange’s
theorem:
1) if the derivative f ′( x) = 0 at the each point
А of the interval, then f ( x) = const;
О a c1 c2 b x
2) if f ′( x) = c at each point of the interval,
Fig. 3.22 then f ( x) = cx + d , i.e. the function is linear;

240
3) if the derivative at some point is positive (negative), then in the
neighbourhood of this point the function increases (decreases);
4) if the functions f1 ( x) and f 2 ( x) are differentiable on the interval (a; b) ,

f ( x) = f ′ ( x) , at the points a and b functions are continuous, then the
1 2
difference between these functions is
f1 ( x) − f 2 ( x) = const.

Theorem 3.14 (Cauchy’s theorem). If the functions f ( x) and g ( x) are


continuous on the segment [a; b] , differentiable at the
interval (a; b) , g ′( x) ≠ 0 , x ∈ (a; b) , then there’s at least one point c ∈ (a; b) ,
where the following statement takes place :
f (b) − f (a) f ′(c)
= .
g (b) − g (a) g ′(c)

21.2. Taylor’s and Maclaurin’s formulas

In some cases Taylor’s formula is used for function investigation (calculation


the value of a function, boundaries of a function).

Theorem 3.15 Let the function f ( x) at the point x0 has the derivatives to
the (n + 1) -th order inclusively and x is arbitrary value from
the given neighbourhood ( x ≠ x0 ) . Then, there’s point c between points x0 and
x , so that Taylor’s formula is true

f ′( x0 ) f ′′( x0 )
f ( x) = f ( x0 ) + ( x − x0 ) + ( x − x0 ) 2 +…
1! 2!
f ( n ) ( x0 )
... + ( x − x0 ) n + Rn ( x) ,
n!

f ( n +1) (c)
where Rn ( x) = ( x − x0 ) n +1 is remainder term in the Lagrange’s form,
(n + 1)!
c = x0 + θ( x − x0 ) , 0 < θ < 1 .
The expression
f ′( x0 ) f ( n ) ( x0 )
Pn ( x) = f ( x0 ) + ( x − x0 ) + … + ( x − x0 ) n
1! n!
is called Taylor’s polynomial.

241
If x 0 = 0 then Taylor’s formula is called Maclaurin’s formula:

f ′(0) f ′′(0) 2 f ( n ) (0) n f ( n +1) (c) n +1


f ( x) = f (0) + x+ x +…+ x + x
1! 2! n! (n + 1)!

where c = θ ⋅ x, 0 ≤ θ ≤ 1.
Let us consider some examples of expansions of elementary functions using
Maclaurin’s formula:

x x 2 x3 xn
ex = 1 + + + +…+ + Rn ( x) ;
1! 2! 3! n!
x3 x5 x 2 n +1
sin x = x − + − … + (−1)n + R2 n +1 ( x) ;
3! 5! (2n + 1)!
x2 x4 x 2n
cos x = 1 − + − … + (−1) n + R2 n ( x ) ;
2! 4! (2n)!

x 2 x3 xn
ln(1 + x) = x − + − … + (−1) n −1 + Rn ( x) ; (3.12)
2 3 n
x3 x5 x 2 n +1
arctg x = x − + − … + (−1) n + R2 n +1 ( x) ;
3 5 2n + 1
m m(m − 1) 2 m(m − 1)(m − 2) 3
(1 + x) m = 1 + x+ x + x + … + (3.13)
1! 2! 3!
m(m − 1)(m − 2) (m − (n − 1)) n
+ x + Rn ( x) ,
n!
If m = −1 then
1
= 1− x + x2 − x3 +…+ (−1)n xn + Rn (x) , (3.14)
1+ x
1
= 1 + x + x 2 + x3 + … + x n + Rn ( x) . (3.15)
1− x

21.3. L’Hospital’s rule

We use L’Hospital’s rule to disclosing indeterminate form of such kind as


⎡0⎤ ⎡∞⎤
⎢⎣ 0 ⎥⎦ or ⎢⎣ ∞ ⎥⎦ .

242
Theorem 3.16 (L’Hospital’s rule). Let functions ƒ(x) and g(x) be:
1) determined and differentiable in a neighbourhood of a point x0 , except,
probably, the point x0 , and g ′( x) ≠ 0 in this neighbourhood;
2) lim f ( x) = lim g ( x) = 0 or lim f ( x) = lim g ( x) = ∞ ; i.e. ƒ(x), g(x)
x→ x0 x→ x0 x→ x0 x→ x0
are simultaneously infinitesimals or infinites as x → x0 ;
f ′( x)
3) there is a finite lim .
x → x 0 g ′( x )
f ( x)
Then there is a limit of the quotient of functions lim and
x → x0 g ( x)

f ( x) f ′( x)
lim = lim .
x → x0 g ( x) x → x0 ′( x)
g

Let’s note, that the theorem is true and in that case, when x0 = ∞.
It should be noted, that sometimes students do a gross error, to search instead
f ′( x) ⎛ f ( x ) ⎞′
of lim for lim ⎜ ⎟.
x → x0 g ′( x ) x → x0 ⎝ g ( x ) ⎠
L’Hospital’s rule is named after the mathematician which for the first time
has published it. But this rule was proved for the first time by I. Bernulli,
therefore L’Hospital’s rule is still named Bernulli-L’Hospital’s rule.
L’Hospital’s rule directly apply to disclosing indeterminate forms of a kind
⎡ 0 ⎤ or ⎡ ∞ ⎤ which are called the basic. Other indeterminate forms 0 ⋅ ∞ ,
[ ]
⎣⎢ 0 ⎦⎥ ⎣⎢ ∞ ⎦⎥
[∞ − ∞] , ⎡1∞ ⎤ , ⎡ 00 ⎤ , ⎡ ∞ 0 ⎤ are reduced to the basic.
⎣ ⎦ ⎣ ⎦ ⎣ ⎦
10. Indeterminate form [0 ⋅∞] ( lim f (x)g(x) , when lim f (x) = 0 , lim g( x) = ∞)
x→x0 x→x0 x→x0


is reduced to indeterminate forms ⎡⎢ ⎤⎥ or ⎡⎢ ⎤⎥ . Thus,
0
0
⎣ ⎦ ⎣∞⎦
f ( x) ⎛ 0 ⎞ g ( x) ⎛ ∞ ⎞
f ( x) g ( x) = or f ( x) g ( x) =
1 ⎜⎝ 0 ⎟⎠ 1 ⎜⎝ ∞ ⎟⎠
g ( x) f ( x)

20. Indeterminate form [ ∞ − ∞ ] ( lim ( f ( x ) − g ( x)) , when lim f ( x) = ∞


x → x0 x → x0

and lim g ( x) = ∞) is reduced to the indeterminate form ⎡⎢ ⎤⎥ so:


0
x → x0 ⎣0⎦

243
1 1

g ( x) f ( x)
f ( x) − g ( x) = .
1
f ( x) g ( x)

30. Indeterminate forms ⎡⎣1∞ ⎤⎦ , ⎡⎣ 00 ⎤⎦ , ⎡⎣ ∞ 0 ⎤⎦ are reduced to the indeterminate


form 0 ⋅ ∞ with the help of taking previous logarithm or representation of
function ⎡ f ( x) g ( x) ⎤ as ⎡ e g ( x) ln f ( x) ⎤ (using the basic logarithmic identity).
⎣⎢ ⎦⎥ ⎣⎢ ⎦⎥

Micromodule 21
EXAMPLES OF PROBLEMS SOLUTION

Example 1. Expand functions using the Maclaurin’s formula:


1
a) f ( x ) = 2
; b) f ( x ) = ln(2 x 2 + 7 x + 3).
4+ x
1 1
Solution. а) we shall write down f ( x ) = .
4 ⎛ x ⎞2
1+ ⎜ ⎟
⎝2⎠
Then using the formula (3.14) we have
1 ⎡ ⎛ x⎞ ⎛ x⎞ ⎤
2 4 2n
⎛ x⎞
f ( x) = ⎢1 − ⎜ ⎟ + ⎜ ⎟ − … + (−1) n ⎜ ⎟ + Rn ( x) ⎥ .
4 ⎢⎣ ⎝ 2 ⎠ ⎝ 2 ⎠ ⎝ 2⎠ ⎥⎦
b) We use transformation
ln(2 x 2 + 7 x + 3) = ln(1 + 2 x)( x + 3) = ln(1 + 2 x) + ln( x + 3) =
x
= ln(1 + 2 x) + ln(1 + ) + ln 3 .
3
Using the formula (3.12) twice, we shall get

(2 x) 2 (2 x)3 (2 x) n
2x − + − … + (−1) n −1 + Rn1 ( x) ,
ln(1 + 2 x) = 2 3 n
2 3 n
x x 1⎛ x⎞ 1⎛ x⎞ 1⎛ x⎞
ln(1 + ) = − ⎜ ⎟ + ⎜ ⎟ − … + (−1) n −1 ⎜ ⎟ + Rn2 ( x) ,
3 3 2⎝3⎠ 3⎝3⎠ n⎝3⎠
(−1) k −1
n
⎛ k 1 ⎞ k
2 ∑
ln(2 x + 7 x + 3) = ln 3 + k =1 k
⎜2 + k ⎟ x
+ Rn ( x ) .
⎝ 3 ⎠

244
Example 2. Calculate limit, using L’Hospital’s rule.
e3 x − e x e3 x − e x
have indeterminate form ⎡⎢ ⎤⎥ , then lim
0
Solution. We lim =
x →0 x ⎣0⎦ x →0 x
3e3 x − e x
= lim = 3 −1 = 2 .
x →0 1

Example 3. Calculate lim x 2 ln x.


x → +0
Solution. In this case we have indeterminate form 0 ⋅ ∞ . We shall reduce to

indeterminate form ⎡⎢ ⎤⎥ and use L’Hospital’s rule:
⎣∞⎦
ln x (ln x) ′ 1/ x
lim x 2 ln x = lim = xlim 2
= lim = − 1 lim x 2 = 0 .
x →+0 x →+0 1 / x 2 →+0 (1/ x ) ′ x →+0 −2 / x3 2 x →+0
x + sin x
Example 4. Calculate lim .
x →∞ x

Solution. We have indeterminate form ⎡⎢ ⎤⎥ . We shall find
⎣∞⎦
x + sin x sin x
lim = lim (1 + ) = 1 + 0 = 1.
x →∞ x x →∞ x

Example 5. Calculate lim (tgx − sec x) .


x→π
2
Solution. Here indeterminate form [∞ − ∞] . We shall reduce it to

indeterminate form ⎡⎢ ⎤⎥ then use L’Hospital’s rule:


0
⎣0⎦
1 sin x − 1 ⎡ 0 ⎤
lim (tg x − sec x) = lim (tg x − ) = lim =⎢ ⎥=
x →π / 2 x →π / 2 cos x x →π / 2 cos x ⎣0⎦
(sin x − 1) ′
= lim = lim cos x = 0 .
x →π / 2 (cos x) ′ x →π / 2 − sin x

Example 6. Calculate lim (π − 2 x)ños x .


x→π
2
π
Solution. Substituting in expression the value x =
, we shall receive
2
0
indeterminate form 0 . For convenience purposes we execute replacement
π − 2 x = t , then t → 0 . So,

245
lim (π − 2 x)cos x = lim t sin(t / 2) = lim esin(t / 2) ln t = lim sin(t / 2) ln t
x →π / 2 t →0 t →0 et → 0 = eA ,
1 1 ln t
A = lim sin(t / 2) ln t = [0 ⋅ ∞] = lim t ln t = [0 ⋅ ∞] = lim =
t →0 2 t →0 2 t → 0 1/ t
⎡∞⎤ 1 (ln t ) ′ 1 1/ t
= ⎢ ⎥ = lim = lim = 1
⎣∞⎦ 2 t → 0 (1 / t ) ′ 2 t →0 −1 / t 2 − 2 lim t →0
t =0.

Then
lim ( π − 2 x )
cos x
= e0 = 1.
x →π / 2

Micromodule 21
CLASS AND HOME ASSIGNMENTS

1. Expand a polynomial P4(x)=x4 – 5x3 + 5x2 + x + 2 in powers of binomial


x – 2.
Expand functions using Maclaurin’s formula:
1 1 1
2. . 3. . 4. .
1 − 2x 2+ x ( x − 1)( x + 2)
x2 + 1 1− 2x
5. . 6. x cos 2 x . 7. ln .
x−2 1+ x
1
8. x sin x . 9. . 10. ln( x 2 − 3 x + 2) .
1 + 4x

Calculate limits, using L’Hospital’s rule:


7x − 3
11. lim 2 . 12. lim ( x 2 + 1)e− x . 13. lim(tg 2 x) x .
x →∞ x + 2 x →∞ x →0

1 1
x cos 2 x − sin x x 4 2
14. lim . 15. lim(e + x) x . 16. lim x ⋅3x .
x →0 x3 x →0 x →0

ln(cos 3 x) ln 2 x
17. lim ln(7 − 2 x) ⋅ ln(6 − 2 x) . 18. lim . 19. lim .
x →3 x →0 ln(cos 8 x ) x →∞ x
1
2 ⎛ 1 1 ⎞ ⎛ arctg x ⎞ x
20. lim tg x ⋅ ln x . 21. lim ⎜ − x ⎟ . 22. lim ⎜ .
x →0 x →0 ⎝ sin x e −1 ⎠ x →0 ⎝ x ⎟⎠
e x − esin x 2 − (e x + e − x ) cos x x + x2
23. lim . 24. lim . 25. lim .
x →0 x − sin x x →0 x4 x →∞
e x

246
Answers
4 3
1. ( x − 2) + 3( x − 2) − 7( x − 2) . 3. The instruction. Write down expression as
1 1 5
⋅ . 5. The instruction. To reduce expression of a kind x + 2 − .
2 1+ x / 2 2(1 − x / 2)
11. ∞ . 12. 0. 13. 1. 14. –11/6. 15. e 2 . 16. ∞ . 17. 0. 18. 3/8. 19. 0. 20. 0. 23. 1. 24. 1/3.
25. 0.

Micromodule 21
SELF-TEST ASSIGNMENTS

3x
21.1. а) lim x ⋅ ln 2 x ; b) lim ; c) lim (sin x) x .
x→0 x→∞ x 2 − 5x + 2 x→0

2x
21.2. а) lim sin x ⋅ ln 2 x ; b) lim 2 ; c) lim(tg x) x .
x→0 x→∞ x + 3 x + 1 x →0
tg x − sin x
21.3. а) lim x 2 e − x ; b) lim ; c) lim (tg x) 2 x − π .
x→∞ x →0 x − sin x x→
π
2
2
ln( x − 8)
; c) lim ( ctg x )
sin x
21.4. а) lim ( x3 + 1)4− x ; b) lim 2
.
x →∞ x →3 2 x − 5x − 3 x →0
1
e x − e− x ln x
3
21.5. а) lim ( x − x − 2) ⋅ 3 −x
; b) lim ; c) lim ( ctg x ) .
x →∞ x →0 sin x cos x x →0
πx
⎛ 1 x ⎞
21.6. а) lim ⎡⎣(1 − cos x ) ctg x ⎤⎦ ; ; c) lim (1 − x ) 2 .
cos
b) lim ⎜ − ⎟
x →0 x →1 ⎝ ln x ln x ⎠ x →1

x cos x − sin x
21.7. а) lim ⎡⎣ ln x ln ( x − 1) ⎤⎦ ; b) lim ; c) lim(1 − cos x) x .
x →1 x→0 x3 x →0
1 1
21.8. а) lim
x →0
x2 e x
2
; b) lim
x →0
x − arctg x
x3
;
x →0
(
c) lim e x + x ) x .
1
ex −cos x − x cos x
; c) lim ( sin x )
4 2 tg x
21.9. а) lim x ⋅2x ; b) lim 2
.
x →0 x→0 x x →0
1

ln cos x ctg 2 x
21.10. а) lim x −2
⋅2 x2 ; b) lim ; c) lim ( cos x ) .
x →0 x →0 ln cos 4 x x →0
1

ln 2 x x −2
21.11. а) lim x −4
⋅5 x2 ; b) lim ; c) lim ( cos 2 x ) .
x →0 x →∞ 3 x x →0

247
1

ln 2 x
x4 c) lim ( arcsin x ) .
x
21.12. а) lim x −4 ⋅ 6 ; b) lim .;
x →0 x →∞ 5 4 x →0
x
πx
ln x tg
21.13. а) lim ln(5 − 2x) ⋅ ln(4 − 2x) ; b) lim ; c) lim(2 − x) 2 .
x→2 x →0 ctg x x →1

ln x
21.14. а) lim ln( x − 3) ⋅ ln(7 − 2 x) ; b) lim ; c) lim (tg x ) tg 2 x .
x →3 x →0 ctg x x →π / 4
1
ln(sin 2 x)
21.15. а) lim ln(2 x − 1) ⋅ ln(2 − 2 x) ; b) lim ; c) lim x 1− x .
x →1 x →0 ln(sin 5 x ) x →1

ln(sin 3 x) 2
21.16. а) lim (1 − cos x ) ctg 2 x ; b) lim ; c) lim ( arctg x) x .
x →0 x →0 ln(sin 7 x ) x →+∞ π
1
2 2 ln(tg3 x)
21.17. а) lim(cos x − 3cos x + 2) ⋅ e x ; b) lim ; c) lim x tg x .
x→0 x →0 ln(sin 4 x ) x →+0
1
2 ⎛1 1 ⎞
21.18. а) lim(cos 2
x − 1) ⋅ e x ; b) lim ⎜ − x lim (tg x) 2 x −π .
⎟ ; c) x →
x →0 x →0 ⎝ x e −1 ⎠ π /2

1 1
2 x2 ln 3 x ⎛ arctg x ⎞ x2
21.19. а) lim(1 − cos x) ⋅ e ; b) lim ; c) lim ⎜ .
x →0 x →∞ x x →0 ⎝ x ⎟⎠
1
2 ⎛ 1 1 ⎞ ⎛ sin x ⎞ x2
21.20. а) lim arcsin x ⋅ ln (2 x) ; b) lim ⎜ − ⎟ ; c) lim ⎜ ⎟ .
x →0 x →1 ⎝ ln x x −1 ⎠ x →0 ⎝ x ⎠

2 x + 3x
21.21. а) lim x3 ⋅ ln 2 x ; b) lim ; c) lim x arcsin x .
x →0 x →∞ x3 − x + 1 x →0
x x
5 −3
21.22. а) lim x ⋅ ln 2 x ; b) lim 2
; c) lim (− ln x) x .
x →0 x →∞ x − 6x + 4 x →+0

3x 2 + 4 x − 6
21.23. а) lim 4 x ⋅ ln 2 x ; b) lim x
; c) lim(ctg x)sin x .
x →0 x →∞ (1,1) x →0

1
3 2 6 x2 − x − 2 ⎛ tg x ⎞ x2
21.24. а) lim x ⋅ ln x ; b) lim ; c) lim ⎜ ⎟ .
x →0 x →∞ (1, 5) x x →0 ⎝ x ⎠

1 1
4 2x2 +10x − 20
21.25. а) lim x8 ⋅ 2 x ; b) lim ; c) lim( x − 1) 2− x .
x →0 x→∞ (2,5)x x→2

248
ln( x 2 + 3x + 1)
21.26. а) lim ( x 2 + 4 x + 5) ⋅ 3− x ; b) lim 2
; c) lim x arc tg 2 x .
x →∞ x →∞ ln(2 x − 1) x →0

ln( x 2 + e x )
21.27. а) lim (5 x 2 + 3 x + 2) ⋅ 6− x ; b) lim ; c) lim sin xarc tg x .
x →∞ x →∞ x x →0

ln(3x 2 + 2 x )
21.28. а) lim x ⋅ ln 3 x ; b) lim ; c) lim x x −sin x .
x →0 x →∞ x x →0

ln(4 x 2 + 3x )
21.29. а) lim( x 2 − 1) ⋅ ln 2 ( x − 1) ; b) lim ; c) lim ln x ln x .
x →1 x →∞ x x →1
1
− e− x
21.30. а) lim x −6 ⋅ 2
x →0
x6 ;
⎛ 1
b) lim ⎜ −
1 ⎞
⎟ ; c) xlim
x→0 ⎝ sin x etg x −1 ⎠ →∞
e− x ( ) .

Micromodule 22
BASIC THEORETICAL INFORMATION.
THE USAGE OF DERIVATIVE IN INVESTIGATION OF A FUNCTION

Monotony of a function. Extremum. Intervals of convexity and concavity,


inflection points. Asymptotes. The greatest and the least meanings of the
function. General scheme of investigation of a function and constructing
of its graph.

Literature: [2, chapter 5], [4, part 5], [6, chapter 5, § 6], [7, chapter 6,
§20, 21], [9], [10, chapter 4, §§ 27—31], [11, chapter 5, §1], [12, chapter 5,
§ 3, 4].

22.1. Increasing and decreasing functions.


Relative extremum of a function

Definition 3.25. Function f ( x ) is called increasing (decreasing) on an


interval (a; b) , if for any two points x1 and x2 from the given interval
( x1 < x2 ), the following inequality is fulfilled f ( x1 ) < f ( x2 ) ( f ( x1 ) > f ( x2 )) .
Properties of increasing and decreasing functions:
1) if f ′( x) > 0 for all x ∈ (a; b) , then function f ( x) increases on ( a; b);
2) if f ′( x) < 0 for all x ∈ (a; b) , then function f ( x) decreases on ( a; b);
3) if f ′( x ) = 0 for all x ∈ (a; b) ,then function f ( x) is constant on (à; b) .

249
y y
f(x0)
f(x)

f(x)
f(x0)

О x0–δ x x0 x0+δ x О x0–δ x x0 x0+δ x


x0 — point of maximum x0 — is a point of minimum

Fig. 3.23 Fig. 3.24

Definition 3.26. Point x0 is called the point of relative maximum (or


minimum) of function f ( x) , if there exists such neighbourhood 0 < x − x0 < δ
of point x0 , which belongs to the domain of the function, and for all x from
this neighbourhood the following inequality is fulfilled
f ( x) < f ( x0 ) ( f ( x) > f ( x0 )) .
Geometrical interpretation of the definition is clear from the Fig. 3.23 and 3.24.
Let’s define the conditions of relative extreme.

Theorem 3.17 (necessary condition of relative extreme). If a function


f ( x) has at point x0 relative extremum and differentiable
at this point, then f ′( x0 ) = 0 .

Geometrical interpretation of the theorem 3.17. If function f ( x) has in


point x0 relative extremum and differentiable at this point, then in this point
there exists the tangent line to the graph of function y = f ( x) and this point
belongs to the x-axis.
Condition f ′( x0 ) = 0 is necessary, but is not enough for the function, which
is differentiable at the point x0 , to have relative extremum. For example, the
derivative of the function y = x3 at point x = 0 is equal to zero, but it does not
have relative extremum at this point. So, function y =| x | has at point x = 0
minimum, but does not have the derivative at this point. Function y = 3 x is not
differentiable at point x = 0 and does not have extremum at this point.
Point, in which the derivative is equal to zero, is called stationary. Point, in
which the derivative is equal to zero or does not exist are called critical. Critical
point is the point of possible extremum.

250
Conclusion. If a function has the relative extremum at the point, then this
point is critical. But not every critical point is a point of extremum.

Theorem 3.18 (first sufficient condition of relative extremum). Let x0 be


critical point of the function f ( x) . Suppose there exists neigh-
bourhood ( x0 − δ; x0 + δ) of point x0 , in which a function has a derivative
f ′( x) except, probably, point x0 . Then:
1) if on the interval ( x0 − δ; x0 ) the derivative f ′( x) > 0 and on the interval
( x0 ; x0 + δ) the derivative f ′( x ) < 0 then the point x 0 is the point of relative
maximum of the function f ( x) ;
2) if on the interval ( x0 − δ; x0 ) the derivative f ′( x) < 0 and on the interval
( x0 ; x0 + δ) the derivative f ′( x) > 0 , then the point x0 is the point of relative
minimum of function f ( x) ;
3) if on both intervals ( x0 − δ; x0 ) and ( x0 ; x0 + δ) the derivative f ′( x) has
the same sign, then the point x0 is not the extreme point of function f ( x) .
In other words, if when passing the critical point x0 from right to left the
sing of the derivative f ′( x) has changed from plus to minus, then x0 is the
point of relative maximum, if the sing of the derivative changes from minus to
plus, then point x0 is point of relative minimum, if the derivative does not
change the sing, then in point x0 extremum is absent.

The rule for investigation of extremum of the function


For finding relative extremum of function f ( x) it is needed:
1) to find the critical points of the function f ( x) . For this it is necessary
to solve the equation f ′( x) = 0 and from its solutions we must to select that,
which belongs to the domain of existence of function; find points in which the
derivative does not exist;
2) if the critical point exists, it is needed to investigate the sing of
derivative at all intervals, on which the domain of existence is divided by
these critical points. For this purpose it is enough to determine the sign at any
point of this interval, because the derivative can change its sign only when
passing the critical point;
3) after changing the sign f ′( x) when passing the critical points from left
to right to define the points of maximum and minimum. When it is necessary
to define the meaning of the function f ( x) in these points.

251
Theorem 3.19 (the second sufficient condition for relative extremum). Let
x0 be fixed point of the function f ( x) , so f ′( x0 ) = 0 and in
neighbourhood of point x0 there exists second derivative, so that f ′′( x0 ) ≠ 0 . If
f ′′( x0 ) > 0 , then x0 is the point of relative minimum, if f ′′( x0 ) < 0 , then x0 is
the point of relative maximum.

22.2 The greatest and the least values of the function


The most important is not to confuse the local maximum (minimum) with the
greatest (least) value of the function, which reaches on a definite segment. The
function may have several local maximum and minimum , when the greatest
value (it can also be named as absolute maximum), if it exists, unique. The same
is true about the least value (absolute minimum) of the function.

To find the greatest and the least values of the function y = f ( x) on the
interval [a; b] it is necessary:
1) to find the derivative f ′( x) and determine the critical points of the
given function;
2) to calculate the value of the function on those critical points, which
belong to the interval (a; b) , and also in points a and b ;
3) to select between the obtained values the greatest and the least values.

22.3. The concavity of the curve. The inflection points


Definition 3.27. Curve y = f ( x) is called concave down on the interval
(a; b) , if all its points, besides the tangent point, lie below its any tangent lines
on the interval (Fig.3.25).
Definition 3.28. The curve y = f ( x) is called concave up on the interval
(a; b) , if all its points, besides the tangent point, lie above its any tangent lines
on the interval (Fig.3.26).
The inflection point is such a point of the curve, which marks out its parts of
concavity (Fig.3.27).
y y y
М
М М

О а b x О а b x О а b x
Fig. 3.25 Fig. 3.26 Fig. 3.27

252
For investigation of the graph of a function on the intervals for concavity the
second derivative of a function is used.

Theorem 3.20 Let function y = f ( x) be twice differentiable on interval


(a; b) . Then:
1) if f ′′( x ) < 0 , x ∈ ( a; b) , then the graph of function y = f ( x) is concave
down on interval (a; b) ;
2) if f ′′( x ) > 0 , x ∈ ( a; b) , then the curve y = f ( x) is concave up on
interval (a; b) .

From the theorem it comes, that at the inflection point the second derivative
is equal to zero, if it exists. But the inflection points of the curve y = f ( x) may
be the points, in which the second derivative f ′(x ) does not exist (for example
point x = 0 of the curve f ( x) = 5 x ).
Points in which the second derivative f ′(x ) is equal to zero or does not
exist, are called the critical points of the second type of the function y = f ( x) .
So, if x0 is abscissa of inflection point, then x0 is critical point of the second
type of this function. The opposite statement is incorrect.
Let’s formulate sufficient conditions of existence of inflection point.

Theorem 3.21 Let x0 be the critical point of the second type of function
f ( x) . If passing the point x0 f ′′(x ) changes its sign, then
point ( x0 ; f ( x0 )) is the inflection point of the curve f ( x ) .

22.4. Asymptotes of the curve

There exist three types of asymptotes: vertical, inclined, horizontal.


The straight line x = c is vertical asymptote (Fig. 3.28), if

lim f ( x ) = ±∞ or lim f ( x ) = ±∞ .
x → c+0 x → c−0

The straight line y = kx + b is inclined asymptote (fig.3.29), if the following


finite limits exist
f ( x)
lim = k ( k ≠ 0), lim ( f ( x ) − kx) = b .
x → ∞
x x → ∞
Remember, if it is necessary to consider the case as x → +∞ so and
x → −∞ .
253
When lim f ( x ) = b ( lim f ( x ) = b) , then straight line y = b is called
x → +∞ x → −∞
a horizontal asymptote of graph of a function y = f ( x) (Fig.3.30).
It is clear, that horizontal asymptote is separate case of inclined asymptote
( k = 0 ).

y y y
x

b y=b
y = kx + b

О с х О х О х

Fig.3.28 Fig.3.29 Fig.3.30

22.5. Scheme of investigation of the graph.


Constructing the graphs of functions

For investigation of the function and constructing the graph it is necessary:


1) to find the domain of existence of a function;
2) to find (if it is possible) the points of intersection of a graph with the
coordinate axes;
3) to test a function on periodic, odd and even check. It should be noted, that
graph of even function is symmetric relatively to axis of ordinates, and graph of
odd function is symmetric relatively to the origin;
4) to find the points of discontinuity and set there characteristics;
5) to find the monotony intervals, the local extremum points and the value of
a function at these points;
6) to find the intervals of convexity and concavity, and the inflection points;
7) to find the asymptote of the curve;
8) to investigate the behavior of a function in infinite remote points;
9) to calculate if it is necessary the value of a function in several control
points;
10) to sketch the graph of a function.

Micromodule 22
EXAMPLES OF PROBLEMS SOLUTION

Example 1. Find intervals of increasing and decreasing of the function

x2 +1
f ( x) = .
( x −1) 2

254
Solution. The domain is ( −∞;1) ∪ (1;+∞) . Find the derivative
2 2
2 x ( x − 1) − 2( x − 1)( x + 1) 2( x + 1)
f ′( x ) = 4
=− .
( x − 1) ( x −1)3

The derivative f ′(x ) is equal to zero at the point x = −1 and it doesn’t exist
if x = 1 . So, x = −1 ; 1 are critical points of a given function.
Let’s denote these points at the numerical axis (and we should remember
about the domain of a function) and define the sign of derivative at each interval
(Fig. 3.31):

f ′( x ) : – + –
f ( x) : -1 1 x

Fig. 3.31

In such case, the function decreases, if x ∈ ( −∞;1) ∪ (1;+∞ ) , and increases at


the interval (−1; 1) .
Example 2. Find the local extremums of the function
f ( x) = x − x5 / 5 .
Solution. The domain is (−∞; ∞) . Let’s find the critical points:
4 2
f ′( x ) = 1 − x , f ′( x ) = 0 if (1 + x )(1 − x )(1 + x ) = 0 ⇔ x = ±1.
So, points x = ±1 are the critical (stationary) points. Define signs of a
derivative at the intervals (Fig. 3.32):

f ′( x ) : – + –
f ( x) : -1 1 x

Fig. 3.32

As we can see on Fig. 3.32 within the intervals (−∞;−1) , (1;+∞ ) the function
decreases but at the interval (−1; 1) it increases. By the theorem 3.18 we can
make a conclusion that x = −1 is a point of relative minimum; x = 1 is a point
of relative maximum, and ymin = y (−1) = −4 / 5 , ymax = y (1) = 4 / 5 .
3
x2
Example 3. Find the relative extremums of the function f ( x) = .
x+2

255
Solution. The domain is ( −∞;−2) ∪ ( −2;+∞ ) . Let’s find the critical points:
2 −1 / 3
x ( x + 2) − x 2 / 3
3 2 x −1 / 3 ( x + 2) − 3x 2 / 3
f ′( x) = = =
( x + 2) 2 3( x + 2) 2
2 x + 4 − 3x x−4
= =− .
33 x ( x + 2) 2 3 x ( x + 2) 2
3

Equation f ′( x) = 0 has a unique root x = 4 . The derivative doesn’t exist at


the points x = −2 and x = 0. Thus, at the point x = −2 the function is
undefined, but at the point x = 0 it is defined. Let’s define signs of derivative at
each interval (Fig. 3.33):

f ′( x ) :
– – + –
f ( x) : –2 0 4 х

Fig. 3.33

With conversion through the point x = 0 from the left to the right, the
derivative changes the sign from minus to plus. On the interval (−2; 0) the
function decreases, but on the interval (0; 4) the function increases. Therefore
x = 0 is the point of relative minimum. By analogy we are sure, that x = 4 is the
point of relative maximum. The point x = −2 isn’t the critical point (at this
point the function isn’t defined).
Example 4. Investigate the function y = 3x 4 − 4 x3 −12 x 2 + 2 on extremum.
Solution. The domain is ( −∞;+∞ ) . The derivative of the given function is
3 2
y ′ = 12 x − 12 x − 24 x. Solve the equation f ′( x ) = 0 :
12 x 3 −12 x 2 − 24 x = 0 ; 12 x( x 2 − x − 2) = 0 ;
x1 = −1 , x2 = 0 , x3 = 2 are stationary points.
2 2
The second derivative is y ′′ = 36 x − 24 x − 24 = 12(3 x − 2 x − 2). We
define the sing y ′′ at the stationary points:
y ′′( −1) = 12(2 + 2 − 2) = 36 > 0 , y ′′(0) = −24 < 0, y ′′( 2) = 72 > 0.
By the theorem 3.19 we can make a conclusion that x1 = −1 and x3 = 2 are
points of relative minimum and x2 = 0 is the point of relative maximum.

Example 5. Investigate the function f ( x) = sin 2 x − x 2 + 1 on extremum at


the point x = 0 .

256
Solution. We have:
f ′( x ) = sin 2 x − 2 x, f ′( x ) = 0 ;
f ′′( x ) = 2 cos 2 x − 2, f ′( x ) = 0 ;
f ′′′( x ) = −4 sin 2 x, f ′( x ) = 0 ;
f (4) ( x) = −8cos 2 x, f (4) (0) = −8 < 0 .
So, the given function at the point x = 0 has the relative maximum.
Example 6. Find the greatest and the least values of the function y = x 2 ln x
on the interval [1; e] .
Solution. Let’s find the derivative
y ′ = 2 x ln x + x = x ( 2 ln x + 1).
As far as the function is defined when x > 0 , so the critical point we find
1
from the condition 2 ln x + 1 = 0 , so x = e−0.5 = . This point doesn’t belong
e
to the interval [1; e] . So let’s evaluate only the values of the function at the ends
of interval. We have: y (1) = 0, y (e) = e 2 .
Therefore, max f ( x) = f (e) = e2 , min f ( x) = f (1) = 0.
x∈[1; e] x∈[1; e]

x +1
Example 7. Find the greatest and the least values of the function y =
x2 +1
on the interval [0; 3] .
Solution. Let’s find the critical points:
2 2
x + 3 − 2 x ( x + 1) − x − 2x + 3
y′ = 2 2
= 2 2
.
( x + 1) ( x + 1)
2
−x − 2 x + 3 = 0 , x1 = −3 , x2 = 1 .

The point x1 = −3 is not within the interval [0; 3] . We count the values
1 1 1
f ( x2 ) = f (1) = ; f (0) = ; f (3) = . That is,
2 3 3
1 1
max f ( x) = f (1) = , min f ( x) = f (0) = f (3) = .
x∈[0;3] 2 x∈[0;3] 3

Example 8. Find intervals of convexity and concavity and inflection points


of the curve f ( x) = 3x 4 − 4 x3 + 1 .
Solution. Let’s find derivatives:
3 2
f ′( x ) = 12 x − 12 x ,

257
2 ⎛
f ′′( x ) = 36 x − 24 x = 36 x⎜ x −
2⎞
⎟.
⎝ 3⎠
2 2
Solve the equation f ′′( x) = 0, 36 x( x − ) = 0 . So, x1 = 0 ; x2 = are
3 3
critical points of the second type. We define the sign of the second derivative: if
x < 0 , then f ′( x ) > 0 and the curve is concave up; if x is within the interval
(0, 2/3), then f ′( x ) < 0 and the curve is concave down; if x > 2 / 3 , then
f ′( x ) > 0 and the curve is concave up. At transition through points x1 = 0 and
2
x2 = the second derivative changes its sign. It follows that points (0; f (0))
3
and (2 / 3; f (2 / 3)) ,( so (0; 1) and (2 / 3; 11/ 27) ) are the inflection points of the
given curve.
x3 + x 2 +1
Example 9. Find the asymptotes of the curve y = .
x2
Solution. Let’s write the equation of the given curve as following:
1
y = x +1+ .
x2
We find the equation of inclined asymptote in such a form y = kx + b .
We receive
f ( x) x + 1 + 1 x2 1 1
k = lim = lim = lim (1 + + 3 ) = 1;
x →∞ x x →∞ x x →∞ x x

b = lim ( f ( x) − kx) = lim ( x + 1 + 1 x 2 − x) = lim ( x + 1 x 2 ) = 1.


x →∞ x →∞ x →∞

So, y = x + 1 is the equation of inclined asymptote. Further, as the function


1
y = x +1+ at the point x = 0 has the discontinuity of the second type, then
x2
the straight line x = 0 is the vertical asymptote of the given curve. The curve
doesn’t have the horizontal asymptote ( lim ( x + 1 + 1 x ) = ∞).
2
x →∞

x2 +1
Example 10. Investigate the function y = and construct its graph.
x −1
Solution. 1) The domain of existence is the whole numerical axis, except the
point x = 1 , so ( −∞;1) ∪ (1;+∞). .
2) The graph of the function y = f ( x) intersects the axis of ordinates (if it’s
possible) at the point (0; f (0)) . We find y (0) = −1 , so A(0; −1) is the point of

258
intersection of the curve with the axis Oy . To find points of intersection of the
x2 +1
graph with the axis Ox , it is necessary to solve the equation y = 0 , =0.
x −1
This equation doesn’t have real roots, so given function doesn’t intersect the
abscissa axis.
3) The function is none-periodical. Let’s consider the expression

(−x) 2 + 1 x2 +1
f (−x) = = .
− x −1 − x −1
Therefore, f (− x) ≠ f ( x) and f (− x) ≠ − f ( x) . It means that the given
function is neither even nor odd.
4) The function at the point x = 1 has discontinuity of the second type, and
x2 + 1 x2 + 1
lim = −∞, lim = = +∞. In all other points the function is
x →1− 0 x − 1 x →1+ 0 x −1
continuous.
2 x( x − 1) − ( x 2 + 1) x 2 − 2 x − 1
5) We find the derivative y ′ = = and solve
( x − 1) 2 ( x − 1) 2
the equation y ′ = 0 , or x 2 − 2 x −1 = 0 . We get stationary points x1 = 1 − 2
and x2 = 1 + 2 . Besides, the derivative is undefined when x = 1 . So,
x1 = 1 − 2 , x2 = 1 + 2 , x3 = 1 are critical points or points of probable
extremum. These points dissect a numerical axis on four intervals (−∞; 1 − 2) ,
(1− 2; 1) , (1; 1 + 2) , (1 + 2; ∞) . On each of these intervals the
derivative y ′ has particular sign, which we can put by the method of intervals or
calculation of values of the derivative in different points (in one point from each
interval). On intervals (−∞; 1 − 2) , (1 + 2; ∞) the derivative is positive, so
the function increases; if x ∈ (1 − 2; 1) ∪ (1; 1 + 2), then the function
decreases, because on these intervals the derivative is negative. At transition
through the point x1 = 1 − 2 (from the left to the right) the derivative changes
the sign from plus to minus, so, at this point the function has the relative
maximum. Then

(1 − 2) 2 + 1
ymax = y (1 − 2) = = 2−2 2 .
1 − 2 −1

At transition through the point x2 = 1 + 2 the derivative changes the sign


from minus to plus, so, at this point there is relative minimum, and

259
(1 + 2) 2 + 1
ymin = y (1 + 2) = = 2+2 2 .
1 + 2 −1
The point x = 1 isn’t the point of extremum (at this point the function isn’t
defined).
6) Find the second derivative

⎛ x2 − 2 x − 1 ⎞ (2 x − 2)( x − 1) 2 − ( x 2 − 2 x − 1) ⋅ 2( x − 1) 4
′′
y = ⎜⎜ 2 ⎟ ⎟ = 4
= .
⎝ ( x − 1) ⎠ ( x − 1) ( x − 1)3
On the interval (−∞; 1) , y ′′ < 0 , sо, on this interval the curve is concave
down; if x ∈ (1; ∞) , then y ′′ > 0 and the curve is concave up. At the point x = 1
the function isn’t defined, so this point isn’t the inflection point.
7) From the results of item 4 comes up, that the straight line x = 1 is vertical
asymptote of the curve.
x2 + 1 x2 + 1
As lim = = +∞, lim = −∞,
x →+∞ x −1 x →−∞ x − 1

then horizontal asymptotes are missed.


Let’s find the limits
f ( x) x2 + 1 f ( x) x2 + 1
lim = lim 2 = 1, lim = lim 2 = 1.
x →+∞ x x →+∞ x − x x →−∞ x x →−∞ x − x
So,
x2 + 1 x +1
k = 1, b = lim ( f ( x) − kx) = lim ( − x) = lim = 1.
x →±∞ x →±∞ x −1 x →±∞ x − 1

In such a way, the straight y = x + 1


y is the inclined asymptote of the given
В• curve. There are no other asymptotes.
2+2 2 • y=x+1
8) Let’s calculate a few values of
the function: y (3) = 5 , y (−1) = −1 .
So, points В(3;5), С(–1;–1) belongs to
the graph.
9) Taking into account the spent
1 examinations, we draw the graph (see
Fig. 3.34).
–1 1 1+ 2 x Example 11. Investigate the func-
•• •А
С tion y = 3 x3 − 2 x 2 and draw the graph
of this function.
Solution. 1) The domain is whole
Fig. 3.34
axis, so D ( y ) = (−∞; +∞).

260
2) Points of intersection with the coordinate axis: if x = 0 , then y = 0 ; if y = 0 ,
then x = 0 or x = 2 . So, the curve passes through the points (0; 0) and (2; 0).
3) Function is neither even nor odd.
4) The points of discontinuity and vertical asymptotes do not exist.
1 3x 2 − 4 x x( x − 4 / 3)
5) Let’s find the derivative y ′ = = .
33 3
(x − 2x 2 ) 2
3 4
x ( x − 2) 2
Critical points are x = 0 ; 4/3; 2. These points break the numerical axis into
four intervals ( −∞;0) , (0; 4 / 3) , (4 / 3; 2) , ( 2; ∞ ) . On each of these intervals the
derivative y ′ has constant sign, so: if x ∈ (−∞;0) , then y ′ > 0 and the function
4 4
increases; if x ∈ (0; ) , then y′ < 0 and the function decreases; if x ∈ ( ;2) ∪ (2; ∞) ,
3 3
then y ′ > 0 and the function increases. During transition through the point
x = 0 derivative changes the sign from plus to minus, so, x = 0 is the point of
maximum, and ymax = y (0) = 0 .
During transition through the point x = 4 / 3 derivative changes the sign
from minus to plus, so, at this point it is minimum:

23 4
ymin = y (4 3) = 3 (4 3) 2 (4 3 − 2) = − 3 32 27 = − >> – 1.1.
3
During transition through the point x = 2 derivative does not change the
sign, so, this point is not the point of extremum.
6) Let’s find the second derivative
⎛ x( x − 4 3) ⎞ ⎛ x−4 3 ⎞
y ′′ = ⎜ ⎟=⎜ ⎟=
⎜ 3 x 4 ( x − 2) 2
⎟ ⎜ 3 x( x − 2) 2 ⎟
⎝ ⎠ ⎝ ⎠
2 ( x − 2) 2 + 2 x( x − 2)
3
x( x − 2) − ( x − 4 3)
3 3 x 2 ( x − 2) 4
=
3
x 2 ( x − 2)4
9 x( x − 2) 2 − (3x − 4)( x − 2)(3 x − 2) 8 ( x − 2)
=− .
4
9 x ( x − 2)
3 8 9 3
x 4 ( x − 2)8

Second derivative doesn’t exist if x = 0 or x = 2 . So, points x = 0; 2 are


the critical points of the second type. Let’s consider the intervals (−∞; 0) , (0; 2)
and (2; ∞) . On the intervals (−∞; 0) and (0; 2) y ′′ > 0 and the curve is
concave; if x ∈ (2; ∞) , then y ′′ < 0 and the curve is convex. The inflection point
has the coordinates (2; 0).

261
7) Let’s find the inclined asymptote:
f ( x) 3
x3 − 3x 2
k = lim = lim = 1,
x →∞ x x →∞ x
3
b = lim ( f ( x ) − kx) = lim ( x 3 − 2 x 2 − x) =
x→∞ x →∞

x3 − 2 x 2 − x3 2
= lim =− .
x →∞ 3
( x3 − 2 x 2 ) 2 + x ⋅ 3 x3 − 2 x 2 + x 2 3

In such a way, straight line y = x − 2 / 3 is inclined asymptote of our curve.


Other asymptotes don’t exist.
8) Taking into account the spent investigations, we can draw the graph of
our function (Fig.3.35).
у

4/3
О 1 2 х

Fig. 3.35

Micromodule 22
CLASS AND HOME ASSIGNMENTS

Let’s find the intervals of increasing and decreasing of the functions:


1. y = 6 − 3 x 2 − x3 . 2. y = x 4 − 2 x 2 . 3. y = x ln x .
x2 + 2 x
4. y = x 2 e− x . 5. y = .
x −1
Investigate the function on extremum:
6. y = x3 − 9 x 2 + 15 x −10 . 7. y = x5 − 5 x 4 + 5 x3 + 5 .
8. y = ( x −1) 2 ( x − 2) 2 . 9. y = x( x −1) 2 ( x + 1)3 .
1− x + x 2 1 2x
10. y = 2
. 11. y = 3x + 3
. 12. y = .
1+ x − x x 1+ x2

262
Let’s find maximum and minimum values of the functions:
ln 2 x
13. y = xe− x . 14. y = x − ln(1 + x) . 15. y = .
x
Investigate the behavior of the function in neighbourhood of the giving
points with the help of the derivatives of higher order:
16. y = 6e x − x3 − 3x 2 − 6 x − 5 , x0 = 0 .
17. y = x sin x − x 2 , x0 = 0 .

Let’s find the intervals of the concavity and convexity, and the inflection
points of the curves.
18. y = x 2 − 2 x + 1 . 19. y = x 3 − 1 .
20. y = x 3 − 3 x 2 + 9 x + 6 . 21. y = x 2 ln x .
− x2
22. y = e . 23. y = xe x . 24. y = ln x + 2 x 2 .

Let’s find the asymptotes of the curves:


6 x 4 + 3x3 1
25. y = 3
. 26. y = 2
. 27. y = x ⋅ e 2 / x +1.
5x +1 x − 3x + 2
Investigate the functions and sketch the graphs:
x3 + 4
28. y = . 29. y = (2x + 3)e−2 /( x+1) . 30. y = 3 ( x + 3) x 2 .
x2
3x 4 + 1 x
31. y = arctg(sin x) . 32. y = . 33. y = 3 ln −1 .
x 3 x −3
e x+1 4x2 x3
34. y = . 35. y = . . 36. y =
x +1 3 + x2 x2 − 4
2 ln x
37. y = 3 1− x 2 . 38. y = x 2 + . 39. y = 16 x( x −1)3 . 40. y = .
x x
Answers
1. ( −∞;−2) і (0; ∞) – decreases; ( −2;0) – increases. 2. ( −∞;−1) and (0;1) –
decreases; ( −1;0) and (1; ∞) – increases. 3. (0;1 / e) – decreases; (1 / e; ∞) – increases.
4. ( −∞;0) and (2; ∞) – decreases; (0;2) – increases. 5. ( −∞;1 − 3 ) and (1 + 3 ; ∞) –
increases; (1 − 3 ;1) and (1;1 + 3 ) – decreases. 6.
x = 2 – maximum, x = 3 –
minimum. 7. x = 1 – maximum, x = 3 – minimum. 8. x = 1;2 – minimum, x = 3 / 2 –

263
maximum. 9. x = −4 / 3;1 – minimum, x = 1 / 2 – maximum. 10. x = 1 / 2 – minimum.
11. x = −1 – maximum, x = 1– minimum. 12. x = 1 – maximum. 13. y max = y(1) = 1 / e .
14. y min = y (0) = 0 . 15. y min = y (1) = 0 , y max = y (e 2 ) = 4 / e 2 .16. Minimum.
17. Maximum. 20. (−∞;1) – convex, (1; ∞) – concave, (1;13) is inflection point.
21. (0; e −3 / 2 ) – convex, (e −3 / 2 ; ∞) – concave, (e −3 / 2 ;−3 /( 2e 3 )) is inflection point.
−1 / 2
22. ( −∞;−1 / 2 ) ∪ (1 / 2 ; ∞) – concave, (−1 / 2 ;1 / 2 ) – convex, (±1 / 2; e )
are inflection points. 24. (0;1 / 2) – convex, (1 / 2; ∞) – concave, (1 / 2;1 / 2 − ln 2) – point
of bend. 25. 6 x − 5 y + 3 = 0 – inclined asymptote, x = −1 / 3 5 is a vertical asymptote.
26. x = 1;2 are vertical asymptotes, y = 0 is a horizontal asymptote. 27. y = x + 3 .
28. y = x , x = 0 are asymptotes; ( −∞;0) ∪ ( 2; ∞) – increases, (0;2) – decreases;
x min = 2 , y min = 3 ; (−∞;0) ∪ (0; ∞) – concave. 29. Designated everywhere, except
x = −1 ; extremum doesn’t exist; function increases; (−2;−e 2 ) – inflection point;
x = −1 – asymptote. 32. Odd; (−∞;−1) ∪ (1; ∞) – increases, (−1;0) ∪ (0;1) – decreases;
y max = −4 if x = −1 , y min = 4 if x = 1 ; y = 3x , x = 0 – asymptotes; (−∞;0) –
convex, (0; ∞) – concave. 34. Designated everywhere, except x = −1 ; y min = e if
x = 0 ; x = −1 – asymptote; (−∞;−1) – concave down, (−1; ∞) – concave up.
35. Even; y max = 0 if x = 0 ; y = 4 – asymptote; ( ±1; 1) is inflection point;
(−∞;−1) ∪ (1; ∞) – convex, (−1;1) – concave. 39. y min = −27 / 16 if x = 1 / 4 ;
(−∞;1 / 2) ∪ (1; ∞) – concave, (1 / 2;1) – convex; (1 / 2;−1) , (1;0) are inflection points;
asymptotes don’t exist.

Micromodule 22
SELF-TEST ASSIGNMENTS

22.1. Find the intervals of increasing and decreasing of the functions:


22.1.1. y = x 2 − ln x 2 . 22.1.2. y = x − 1( x − 2) .
2 3
22.1.3. y = ( x − 1) ( x + 1) . 22.1.4. y = arcsin(1 + x) .
x x −1
22.1.5. y = . 22.1.6. y = .
x + 100 x + 24
x2
22.1.7. y = . 22.1.8. y = ln x − arctgx .
2x
x5
22.1.9. y = . 22.1.10. y = x 4 ⋅ ln x .
3x

264
x3
22.1.11. y = ln( x 2 + 1) − x . 22.1.12. y = .
ex
22.1.13. y = 9− x − 3− x . 22.1.14. y = x ⋅ ln 3 x .
1 2 4
22.1.15. y = + − . 22.1.16. y = ( x − 5)3 ( x + 4) 2 .
2
x x x3
22.1.17. y = x 2 ⋅ ln x . 22.1.18. y = x ⋅ ln 2 x .
2
22.1.19. y = x 2 ⋅ e − x . 22.1.20. y = 2 x − 4 x .
x 2 + 3x − 4
22.1.21. y = . 22.1.22. y = ( x − 4)3 ( x + 5) 2 .
x−5
22.1.23. y = 2 x − x 2 . 22.1.24. y = x ln x .
x 2 − 3x + 2 1+ x
22.1.25. y = . 22.1.26. y = ln .
( x + 1) 2 1− x
2
x − 7x + 6 x2
22.1.27. y = . 22.1.28. y = .
x − 10 x4 + 4
1
22.1.29. y = e − x − e −2 x . 22.1.30. y = x + .
x
22.2. Find the intervals of the concavity and convexity, and the inflection
points of the curves:

22.2.1. y = x 2 ⋅ x + 1 . 22.2.2. y = x 2 + 1 .
22.2.3. y = ( x − 4) 4 ( x + 7)3 . 22.2.4. y = x x .
22.2.5. y = ln(1 + x 2 ) . 22.2.6. y = x + sin x .
2
22.2.7. y = 3 x − x . 3
22.2.8. y = xe − x .
22.2.9. y = x + x5 / 3 . 22.2.10. y = 3 x 2 − 4 x x .
22.2.11. y = ln( x 4 + 1) . 22.2.12. y = ln x + ln 2 x .
4
22.2.13. y = e − x . 22.2.14. y = x 4 + 8 x3 + 18 x 2 + 8 .
22.2.15. y = ( x − 1) 2 ⋅ x . 22.2.16. y = x3 − 3 x 2 + 6 x + 7 .
x +1 x
22.2.17. y = 22.2.18. y = .
x2 + 1 x2 − 2 x + 2
x 1− x (1 − x) x
22.2.19. y = . 22.2.20. y = .
1+ x 2− x

265
x2 x2 + 2 x + 1
22.2.21. y = . 22.2.22. y = .
( x − 1)3 x3
22.2.23. y = 3 (1 − x)( x − 2) 2 . 22.2.24. y = x x ⋅ (4 − x) −1/ 2 .
x3 x3 − x 2 − 1
22.2.25. y = . 22.2.26. y = .
x2 + 1 x2
22.2.27. y = x + 7 / x − 3 / x 2 . 22.2.28. y = x 4 + 6 x 3 + 12 x 2 .
x3
22.2.29. y = . 22.2.30. y = 3 x( x + 1) 2 .
4x2 + 1
22.3. Investigate the functions and sketch the graphs.
2 x2 − 4x + 3 16
22.3.1. y = x 2 + . 22.3.2. y = . 22.3.3. y = 2 .
x x−2 x ( x − 4)
x x3 x2 − 5
22.3.4. y = . 22.3.5. y = . 22.3.6. y = .
1 + x2 3 − x2 x −3
( x − 1)3 x2 4 x − 12
22.3.7. y = . 22.3.8. y = 2 . 22.3.9. y = .
( x − 2) 2
x −1 ( x − 2 )2
3x 4 + 1 x x4
22.3.10. y = . 22.3.11. y = . 22.3.12. y = .
x3 x2 − 4 x3 − 1
2
x − 2x 4 x2 7 3
22.3.13. y = . 22.3.14. y = . 22.3.15. y = x + − 2 .
x −1 3− x x x
8 x2 − 1 x4
22.3.16. y = . 22.3.17. y = . 22.3.18. y = .
x2 − 4 x2 + 1 (1 + x )3
3 1 x2 − 2 x + 2 x2 − 4
22.3.19. y = − . 22.3.20. y = . 22.3.21. y = .
x x3 x −1 x −1
x3 4x x2 + 1
22.3.22. y = . 22.3.23. y = . 22.3.24. y = .
( x + 1) 2 4 + x2 x2 − 1
x3 2x −1 4 x3
22.3.25. y = . 22.3.26. y = . 22.3.27. y = .
x2 + 1 ( x − 1) 2 x3 − 1
2x2 + 1 x2 − 1 2 − 4 x2
22.3.28. y = . 22.3.29. y = . 22.3.30. y = .
x −1 x 1 − 4x2

266
LITERATURE

1. Бахвалов Н.С., Жидков Н.П., Кобельков Г.М. Численные методы. —


М.: Наука, 1987. — 600 с.
2. Валєєв К.Г., Джалладова І.Л. Вища математика: Навчальний посіб-
ник: У 2-х ч.— К.: КНЕУ, 2001.— Ч.2. — 451 с.
3. Дубовик В.П., Юрик І.І. Вища математика. — К.: А.С.К., 2001. — 648 с.
4. Заварыкин В.М., Житомирский В.Г., Лапчик М.П. Численные мето-
ды. — М.: Просвещение, 1991. — 172 с.
5. Краснов М.Л., Киселев А.И., Макаренко Г.И., Шикин Е.В., Заляпин В. И.,
Соболев С.К. Вся высшая математика: Учебник, Т.4. — М.: Эдиториал УРСС,
2001. — 352 с.
6. Овчинников П.П., Яремчук Ф.П., Михайленко В.М. Вища математика:
Підручник. У 2 ч. — Ч. 1: Лінійна і векторна алгебра. Аналітична геометрія.
Вступ до математичного аналізу. Диференціальне і інтегральне числення. —
К.: Техніка, 2000. — 592 с.
7. Овчинников П.П. Вища математика: Підручник. У 2 ч. — Ч. 2: Дифе-
ренціальні рівняння. Операційне числення. Ряди та їх застосування. Стій-
кість за Ляпуновим. Рівняння математичної фізики. Оптимізація і керування.
Теорія ймовірностей. Числові методи. — К.: Техніка, 2000. — 792 с.
8. Пак В.В., Носенко Ю.Л. Вища математика: Підручник.— Д.: «Видав-
ництво Сталкер», 2003. — 496 с.
9. Пискунов Н.С. Дифференциальное и интегральное исчисления. — М.:
Наука, 1985. — Т. 2. — 456 с.
10. Письменный Д.Т. Конспект лекций по высшей математике. 2 часть. —
2-е изд., испр.— М.: Айрис-пресс, 2003. — 256 с.
11. Фильчаков П.Ф. Численные и графические методы прикладной мате-
матики. — К.: Наукова думка, 1970. — 792 с.
12. Денисюк В. П., Репета В. К. Вища математика: Навч. посіб.: У 4 ч. —
Ч. 1. — К.: Вид-во Нац. авіа. ун-ту «Нау-Друк», 2009. — 296 с.

267
CONTENTS

Introduction . . . . . . . . . . . . . . . . . . . . . . . . . . . . . . . . . . . . . . . . . . . . . . . . 3
Module 1. ELEMENTS OF LINEAR AND VECTOR ALGEBRA . . . . . . 4
Micromodule 1. Determinants . . . . . . . . . . . . . . . . . . . . . . . . . . . . . . . . . . . 6
Micromodule 2. Matrices . . . . . . . . . . . . . . . . . . . . . . . . . . . . . . . . . . . . . 22
Micromodule 3. Systems of linear algebraic equations . . . . . . . . . . . . . . . . . 42
Micromodule 4. Vectors . . . . . . . . . . . . . . . . . . . . . . . . . . . . . . . . . . . . . . 58
Micromodule 5. Dot product of two vectors . . . . . . . . . . . . . . . . . . . . . . . . 69
Micromodule 6. Cross and triple products . . . . . . . . . . . . . . . . . . . . . . . . . . 77
Module 2. ELEMENTS OF ANALYTICAL GEOMETRY . . . . . . . . . . . 85
Micromodule 7. Straight line on a plane . . . . . . . . . . . . . . . . . . . . . . . . . . . 87
Micromodule 8. Plane . . . . . . . . . . . . . . . . . . . . . . . . . . . . . . . . . . . . . . . 100
Micromodule 9. Straight line in space. Mutual position of straight line
and plane . . . . . . . . . . . . . . . . . . . . . . . . . . . . . . . . . . . . 109
Micromodule 10. Curves of the second order . . . . . . . . . . . . . . . . . . . . . . 121
Micromodule 11. Surfaces of the second order . . . . . . . . . . . . . . . . . . . . . 129
Module 3. INTRODUCTION TO MATHEMATICAL ANALYSIS.
DERIVATIVES AND DIFFERENTIALS OF A FUNCTION
OF ONE VARIABLE. APPLICATIONS OF DERIVATIVES 138
Micromodule 12. Sequence. The limit of a numerical sequence. Theorems
about limits . . . . . . . . . . . . . . . . . . . . . . . . . . . . . . . . . 141
Micromodule 13. The concept of a function. Classification of functions.
Limit of function. Theorems about limits . . . . . . . . . . . . 152
Micromodule 14. Honorable limits . . . . . . . . . . . . . . . . . . . . . . . . . . . . . . 172
Micromodule 15. Comparison of infinitesimals. Equivalent infinitesimals.
Their application in calculation of limits . . . . . . . . . . . . 181
Micromodule 16. Continuity of a function . . . . . . . . . . . . . . . . . . . . . . . . 188
Micromodule 17. Derivative . . . . . . . . . . . . . . . . . . . . . . . . . . . . . . . . . . 196
Micromodule 18. Derivative and its calculation (continued) . . . . . . . . . . . . 211
Micromodule 19. Differential of function. Tangent . . . . . . . . . . . . . . . . . . 219
Micromodule 20. Higher order derivatives and differentials . . . . . . . . . . . . 229
Micromodule 21. Basic theorems of differential calculus . . . . . . . . . . . . . . 239
Micromodule 22. The usage of derivative in investigation of a function . . . 249
Literature . . . . . . . . . . . . . . . . . . . . . . . . . . . . . . . . . . . . . . . . . . . . . . . . 267

268
Навчальне видання

ДЕНИСЮК Володимир Петрович


ГРИШИНА Людмила Іллівна
КАРУПУ Олена Вальтерівна
ОЛЕШКО Тетяна Анатоліївна
ПАХНЕНКО Валерія Валеріївна
РЕПЕТА Віктор Кузьмич

ÂÈÙÀ ÌÀÒÅÌÀÒÈÊÀ
У чотирьох частинах
Частина 1

Навчальний посібник

(Англійською мовою)

В авторській редакції

Художник обкладинки Т. Зябліцева


Верстка О. Іваненко
Підп. до друку 22.12.09. Формат 60·84/16. Папір офс.
Офс. друк. Ум. друк. арк. 15,81. Обл.-вид. арк. 17,0.
Тираж 300 пр. Замовлення №

Видавництво Національного авіаційного університету «НАУ-друк»


03680, Київ-58, просп. Космонавта Комарова, 1
Свідоцтво про внесення до Державного реєстру ДК № 977 від 05.07.2002
Тел. (044) 406-78-28. Тел./факс: (044) 406-71-33
E-mail: publish@nau.edu.ua
Вища математика: навч. посіб. У 4 ч. Ч. 1 / В. П. Денисюк, Л. І. Гри-
Н 65 шина, О. В. Карупу та ін. — 2-ге вид., стер. — К.: Вид-во Нац. авіац. ун-ту
«НАУ-друк», 2009. — 272 с. (Англ. мовою).
ISBN 978–966–598–612–6
ISBN 978–966–598–614–0 (Частина 1)

У посібнику запропоновано модульну технологію вивчення вищої мате-


матики.
Викладено основні розділи курсу вищої математики (елементи лінійної
та векторної алгебри, аналітичної геометрії, математичного аналізу), які вив-
чаються в першому семестрі.
Кожна тема містить стислі теоретичні відомості, практичну частину, за-
вдання для аудиторної та самостійної роботи, а також індивідуальні тестові
завдання.
Для студентів вищих технічних навчальних закладів.

UDK 513.123:517.2=111(075.8)
ББК В10я7
For remarks

You might also like